Текст
                    СБОРНИК ЗАДАЧ
ПО ТЕОРИИ
АВТОМАТИЧЕСКОГО
РЕГУЛИРОВАНИЯ
И УПРАВЛЕНИЯ
Под редакцией
В. А. БЕСЕКЕРСКОГО
ИЗДАНИЕ ЧЕТВЕРТОЕ, СТЕРЕОТИПНОЕ
Допущено Министерством
высшего и среднего специального образования СССР
i е качестве учебного пособия
' Сля студентов высших технических
учебных заведений
ИЗДАТЕЛЬСТВО «НАУКА»
ГЛАВНАЯ РЕДАКЦИЯ
ФИЗИКО-МАТЕМАТИЧЕСКОЙ ЛИТЕРАТУРЫ
МОСКВА 1972


6Ф6.5 С 23 УДК 621.078(076.1) Авторы: В. А. БЕСЕКЕРСКИЙ, А. Н. ГЕРАСИМОВ, Л. Ф. ПОРФИРЬЕВ, Е. А. ФАБРИКАНТ, С. М. ФЕДОРОВ, В. И. ЦВЕТКОВ. Сборник задач по теории автоматического регулировании и управления, под редакцией В. А. Б е с е к е р- с к о г о, издание четвертое, стереотипиое, издательство «Наука», Главная редакция физико-математической литературы, М., 1972. Сборник рассчитан на студентов, специализирующихся в области теории автоматического регулирования и следящих систем, а также на студентов других специальностей, изучающих теорию автоматического регулирования, и управления. Сборник, содержащий 491 задачу,-'ч?ри/ентируется на ряд используемых в вузах книг. В,- ^а'честве основной принята книга В. А. Бесе- керскогс) 'и Е. П. Попова «Теория систем автоматического регулирования». Построение сборника, содержание, терминология и методика изложения в основном соответствуем этой книге. Табл.г32. Илл. 376. Библ. 43 назв. 154-72
Предисловие ОГЛАВЛЕНИЕ 9 Р аз д ел I ЛИНЕЙНЫЕ СИСТЕМЫ АВТОМАТИЧЕСКОГО РЕГУЛИРОВАНИЯ Глава 1. Дифференциальные уравнения и передаточные функций звеньев и автоматических систем 11 § 1.1. Дифференциальные уравнения и передаточные функции звеньев И' § 1.2. Типовые динамические звенья 32 § 1.3. Дифференциальные уравнения и передаточные функции автоматических систем 36 § 1.4. Структурные схемы и их преобразование 46 Глава 2. Частотные характеристики динамических звеньев и систем автоматического регулировании 51 § 2.1. Характеристики динамических звеньев 51 § 2.2. Амплитудно-фазовые характеристики разомкнутых систем автоматического регулирования 63 § 2,3. Вещественные частотные характеристики замкнутых систем автоматического регулирования 73 § 2.4. Логарифмические характеристики систем автоматического регулирования 79 Глава 3. Устойчивость линейных систем 89 § 3.1. Алгебраические критерии устойчивости ' 89 § 3.2. Критерий устойчивости Михайлова 101 § 3.3. Критерий устойчивости Найквиста 109 § 3.4. Определение устойчивости по логарифмическим частотным характеристикам разомкнутой системы . . . 120 § 3.5. Построение областей устойчивости ■. . . 125 Глава 4. Построение переходных процессов в системах автоматического регулирования 130 • §4.1. Классический метод решения дифференциальных уравнений 130 § 4.2. Применение изображений Лапласа и Карсона—Хеви- сайда 140
4 ОГЛАВЛЕНИЕ § 4.3. Приближенные методы расчета переходных процессов 158 A. Использование вещественных частотрых характеристик ...'.......; 158 Б. Использование сопрягающих частот л. а. х. ... 165 B. Использование нормированных кривых для мини- малыю-фазовых систем с типовыми л. а. х. ... 167 Г. Построение кривой переходного процесса графическим методом 172 Глава 5. Оценка качества регулирования 176 § 5.1. Определение точности при наличии задающего воздействия 176 § 5.2. Определение точности при наличии возмущающего воздействия 191 § 5-3. Корневые методы оценки динамических свойств . . . 194 § 5.4. Оценка по кривой переходного процесса ...._.. 198 § 5.5. Интегральные оценки 201 § 5.6. Частотные оценки динамических.свойств 206 Глава 6. Синтез линейных систем .( 211 § 6.1 Выбор параметров САР по требуемой точности . . .211 § 6.2. Алгебраические методы выбора параметров САР . .■ 224 § 6.3. Частотные методы выбора параметров САР. Расчет ■ последовательных корректирующих устройств .... 231 § 6.4. Расчет дополнительных обратных связей н прямых параллельных корректирующих связей 249 § 6.5. Расчет систем комбинированного управления .... 263 § 6.6. Расчет последовательных корректирующих контуров, работающих на несущей частоте 27д Глава 7. Случайные процессы в линейных системах .... 273 § 7.1. Вычисление корреляционных функций и спектральных плотностей \ 273 § 7.2. Прохождение случайного стационарного сигнала через линейную систему 286 § 7.3. Оптимальные системы 295 Глава 8. Системы с переменными параметрами 302 § 8.1. Построение переходных процессов . ........ 302 § 8.2. Оценка устойчивости н качества регулирования . . . 309 Глава 9. Системы с запаздыванием . и с распределенными параметрами .- 314 § 9.1. Системы с временным запаздыванием .314 . § 9.2. Системы с распределенными параметрами .,..,.. '321 Глава 10. Импульсные системы 324 § 10.1. Дискретные функции и уравнения импульсных систем 324 § 10.2. Устойчивость и качество импульсных систем 336
ОГЛАВЛЕНИЕ" 5 :'У Раздел 1} ■ .-.нелинейные системы автоматического регулирования Глава 11. Составление уравнений нелинейных систем . . . 343 §11.1 Уравнении нелинейных следящих систем 843 § 11.2. Уравнения нелинейных систем стабилизации 350 Глава 12. Точные методы исследования устойчивости и ав- .токолебаний 35в § 12.1. Метод фазовых траекторий 356 § 12.2. Метод А. М. Ляпунова - А. И. Лурье : . 369 § 12.3. Частотный метод В. М. Попова 374 .§ 12.4. Метод припасовывання 378 Глава 13. Приближенные методы исследования устойчиво- ; сти и автоколебаний ..... ...V . 383 §13.1. Алгебраические способы определения устойчивости и автоколебаний 383 §.13.2. Частотный метод определения автоколебаний .... 392 Глава 14. Оценка качества нелинейных систем 402 :§ 14.1. Исследование колебательных переходных процессов аналитическими методами 402 i§ 14.2. Исследование колебательных переходных процессов частотными методами 412 Глава 15. Вынужденные колебания в нелинейных системах 417 § 15.1.' Отыскание симметричных одночастотных вынуж- ! '■"-■ денных колебаний графическим методом 417 § 15.2. Отыскание симметричных одночастотных вынужденных колебаний частотным методом 423 Г да в а 16; Прохождение случайных процессов через иелиней- ' ные системы . ; 431 . ,§.16.1. Определение функций и моментов распределения ,.!'.. случайного процесса на выходе нелинейной системы 431 § 16.2. Расчет нелинейных систем с помощью статистичес- .,.,- кой линеаризации» 438 ... i • ■ ■ Ра зде л III ОПТИМАЛЬНЫЕ, ЦИФРОВЫЕ, САМОНАСТРАИВАЮЩИЕСЯ СИСТЕМЫ И МОДЕЛИРОВАНИЕ Гл'ава 17. Синтез оптимальных систем управления 448 '§' 17.1. Синтез оптимальных систем с использованием прни- t. . ципа максимума 448 § 17.2. Синтез оптимальных систем методом динамического ■;; программирования н; классического вариационного - '■' ■ исчисления .■■ . . ■:■■. '.' .■ : .'• . ■..-.■....... 455
6 ОГЛАВЛЕНИЕ Глава 18. Системы с цифровыми вычислительными машинами (ЦВМ) 460 § 18.1. Передаточные функции систем с ЦВМ при учете' квантования по времени 4.60 § 18.2. Устойчивость и оценка качества 466 . § 18.3. Синтез систем- с ЦВМ 471 Глава 1-9. Экстремальные и самонастраивающиеся системы 480 § 19.1. Составление структурных схем экстремальных и самонастраивающихся систем и исследование их устойчивости ..... 480 § 19.2. Качество систем экстремального управления .... 489 Глава 20. Составление схем для моделирования автоматических систем на непрерывных вычислительных машинах 499 § 20.1. Составление схем для моделирования элементов автоматических систем на электронных вычислитель- . . ных машинах . « 493 § 20.2.' Составление схем' для моделирования дифференциальных уравнений и автоматических систем на электронных вычислительных машинах 513 Приложение 522 ' 1. Изображения по Лапласу и Карсону—Хевисайду -фун- . кций времени . \. 522 2. Z-преобразования функций времени 524 3. Нормированная' логарифмическая частотная фазовая характеристика апериодического звена первого порядка 525 4. Нормированные логарифмические амплитудная и фазо- . вая характеристики колебательного звена ....... 525 5. Отклонение асимптотической л. а. х. колебательного звена от точной . . . . • 526 6. Критерий устойчивости Гурвица 527 7. Диаграмма Вышнеградского с линиями равного затухания в процентах за один период 528 8. Диаграмма Вьшшеградского с линиями равной норми- рованиой степени устойчивости h0 = hy ай1аъ ....... 529 9. Эквивалентные начальные условия в системе регулирования после воздействия на нее единичной ступенчатой функции . . . . , 529 10. Решения, однородных дифференциальных уравнений первого, второго и третьего порядков 530
ОГЛАВЛЕНИЕ / П. Номограмма для построения вещественной частотной характеристики замкнутой системы по амплитудно-фазовой характеристике разомкнутой системы (вещественная круговая диаграмма) 531 12. Кривые для определения времени переходного процесса и перерегулирования по коэффициенту наклона трапецеидальной вещественной частотной характеристики . . 531 13. Кривые для определения времени переходного процесса и перерегулирования для вещественной частотной ха- , рактеристики, имеющей максимум 532 14. Зависимость требуемого запаса по фазе' от модуля в децибелах при различных показателях колебательности . . ,■ 532 15. Номограмма для построения вещественной частотной характеристики по заданным качественным п показателям . .• • . . •. . 533 16. Нахождение квадратичной интегральной оценки .... 534 17. Формулы для интегрирования спектральной плотности 535 18. Нормированные стандартные передаточные функции разомкнутой системы , 535 19. Типовые л, а. х., соответствующие заданному запасу устойчивости , 53S 20. Нормированные кривые переходных процессов для типовых л. а. х ". . . . . . . 541 21. Корректирующие звенья постоянного тока 542 22.Жорректиругощее звено переменного тока (двойное Т-образиое звено) 544 23. Приведение дифференциальных уравнений к канонической форме „ 547 24. Достаточные условия устойчивости нелинейных систем второго, третьего и четвертого порядков . ; 550 25. Переход от л. а. х. вида 20 lg| I + W (/со) | к л. а. х. вида 20 lg | W (/со) | . . 551 26. Типовые л. а. х. систем с ЦВМ при учете квантования по времени 551 27. Типовые последовательные дискретные корректирующие звенья 55ft 28. Коэффициенты гармонической линеаризации основных нелинейных характеристик. -. . . 558 29. Нормированные амплитудно-фазовые частотные характеристики релейных исполнительных механизмов. . . -. 562
8 ОГЛАВЛЕНИЕ ■ 30. Коэффициенты статистической линеаризации некоторых типовых нелинейностей 565 31. Нормированный интеграл плотности вероятности нормального закона распределения ..... 571 32. Нормированная плотность вероятности нормального закона распределения 573 33. Моделирование элементов структурных схем автоматических систем на операционных усилителях 574 84. Моделирование нелинейных статических характеристик . иа- операционных усилителях 579 85. Таблица функций h {tD) - 582 Литература. 586
ПРЕДИСЛОВИЕ Настоящий сборник рассчитан на студентов, специализирующихся в области автоматического регулирования и управления, но может быть использован и студентами других специальностей при изучении ими теории автоматического регулирования. Круг вопросов, охватываемых сборником, включает в себя темы, входящие, как правило, в программы по теории автоматического регулирования или теории автоматического управления многих' вузов. Сборник ориентируется на ряд используемых в вузах книг, список которых приведен в конце сборника. В качестве основной принята книга В. А. Бесекерского и Е. П. Попова «Теория систем автоматического регулирования», выходящая вторым изданием в 1972 г. Построение сборника, содержание, терминология и методика изложения в основном соответствуют этой книге. Аналогично указанной книге в сборнике применяется одинаковый символ для обозначения оператора дифференцирования, используемого при операторной форме записи дифференциальных уравнений, в которые входят функции времени (оригиналы), и'комплексной переменной в преобразованиях Лапласа и Карсона—Хевисайда. При этом предполагается, что читатель самостоятельно может разобраться в значении этого символа в зависимости от того, используются ли в уравнениях функции времени или их изображения, и знает правила.перехода от оригиналов к изображениям с учетом начальных условий.
10 ПРЕДИСЛОВИЕ Сборник задач написан В. А. Бесекерским (главы 5, 7, 8 и 10), А. Н. Герасимовым (главы 3 и 9), С. В. Лучко (главы 11—15), Л. Ф. Порфирьевым (главы 16, 17 и- 19), Е. А. Фабрикантом (главы 2 и 4)," С. М. Федоровым (главы 6 и 18) и В. И. Цветковым (главы 1 и 20). Авторы приносят благодарность А. В. Нетушилу и Б. П. Рязанову за полезные замечания, сделанные при просмотре рукописи третьего издания. В четвертом издании исправлены замеченные опечатки и неточности. Авторы
РАЗДЕЛ I ЛИНЕЙНЫЕ СИСТЕМЫ АВТОМАТИЧЕСКОГО РЕГУЛИРОВАНИЯ ГЛАВА 1 ДИФФЕРЕНЦИАЛЬНЫЕ УРАВНЕНИЯ ' И ПЕРЕДАТОЧНЫЕ ФУНКЦИИ ЗВЕНЬЕВ И АВТОМАТИЧЕСКИХ СИСТЕМ § 1.1. Дифференциальные уравнения и передаточные функции звеньев 1. Составить в общем виде дифференциальное уравнение электромагнита с пружиной и демпфером (рис. 1, а), если-за входную величину прлнять напряжение и, а за выходную — перемещение якоря х и считать известными приведенные к точке А силы пружины Fr, демпфера FR, электромагнита гэ и инерционную силу F„. Влиянием сил сухого трения пренебречь. Решение. Выберем начало отсчета, как показано на рис. 1, а. Составим уравнение равновесия сил, приведенных к точке- Л, тх. + с{х + с2х = Fa ('. х) ' (О и уравнение равновесия напряжений и = iR + L (6, 0 ■§ + ^, (2) где тх — F„ — инерционная сила, пропорциональная ускорению х и приведенной массе подвижных частей tn; ctx = FA — сила демпфера, пропорциональная ско-: рости х и коэффициенту демпфирования с,; с2х = Ftl — сила пружины, пропорциональная перемещению х и коэффициенту упругости или жесткости пружины с2;
12 ГЛ.'-!,' УРАВНЕНИЯ И ПЕРЕДАТОЧНЫЕ ФУНКЦИИ [1 и, / — напряжение и ток обмотки электромагнита; L = L(b, /) —индуктивность обмотки электромагнита, зависящая в общем виде от рабочего зазора б и тока / (при насыщении мащитопровода); ^—активное сопротивление обмотки электромагнита; F3 = F3{i, x) — си- , ■ л а электромагнита, являющаяся функцией двух переменных. Предположим, что всегда остается рабочий зазор бот^О й такой, что справедливо соотношение Fa (i, х) = czfx~2 при 6 ^ бв, (3) где с3 — постоянный коэффициент. Наличие воздушного зазора (6>60) и рабочих (ограниченных) значений тока i исключает насыщение магнитопровода. Поэтому индуктивность не зависит от тока, а только от перемещения L = L(x). На основании гипотезы малых отклонений будем считать, что L = L0=^ const в окрестности выбранного постоянного значения х = хе. Тогда нелинейное уравнение (2) становится линейным: -'*'■> W(p) б) X YW. 1. Электромагнит с пружи- ,;. ной и демпфером. ■ iR + L0 di di (4) Б уравнениях (1), (3) и (4) только член в правой части уравнения (1) или его выражение (3) являются нелинейными. Линеаризуем его, для чего запишем в виде F (F3, i, x) — F3 — c3i2x~2 = 0. (5) Тогда линеаризованное уравнение в малых отклонениях Ьт-иосительно установившегося' статического значения
2j § 1.1. УРАВНЕНИЯ И ПЕРЕДАТОЧНЫЕ ФУНКЦИИ ЗВЕНЬЕВ {3 переменных (/ = 'о. x = x0l F3 = FaQ) примет вид (£)V. + (£)V + (E)*A,-0. (6, тт „ dF " dF dF ,^. Найдя частные производные -те;, -^р-, -т- из (5) и подставив в них установившиеся значения переменных, получим " * AF3 — kx Д/ + k2Ax = 0 или &.F3 = kiM — k2Ax,(7) где kx = 2c3i0x~2, k2 = 2csi2lx~'K Знак минус в (7) означает, что при увеличении Ах сила А/%, уменьшается. Коэффициенты передачи kt и k2 можно найти графически из статических характеристик F3 = c3Xq2 • i2n F3=c3il • х~2 нахождением тангенса угла наклона касательных, проведенных в точках (г'0> F30) и (-^о. ^эо) соответственно. Выразив А/ из (7) и; подставив в (4), а полученный результат, разрешенный .относительно. AF3, —в (1) и сделав преобразования, получим {Тэр + 1)(Г2Р2 + Tip.+ 1) *■(*) = ku (t), где T3 = -j~ — постоянная времени обмотки электромагнита, т — Cl т —л/ m ' k = fe' ! п = л~- ±l- c2 + k2' 1г \ c2 + ft2* R %(c2 + k2)' P dt — оператор или символ дифференцирования.. 2. Найти передаточную функцию гидравлического исполнительного устройства (рис. 2, а), применявшегося совместно с центробежными.измерителями угловой скорости ЦИС для регулирования скорости вращения тепловых двигателей. За входную величину принять перемещение х муфты ЦИС 3, а' за выходную — перемещение у заслонки или регулирующего органа {РО) теплового двигателя (рис. 2,. б). ' ■' Решение. Гидравлический двигатель (золотник 2 с силовым поршнем 1) вместе с'изодромом (пружина 5 с демпфером 6) могут находиться в покое только при одном определенном положении рычага 4, когда пружина.,,-
1.4 ГЛ. 1. УРАВНЕНИЯ И. ПЕРЕДАТОЧНЫЕ ФУНКЦИИ . [2' находится в ненапряженном состоянии и золотник 2 — в среднем положении (как показано на рис. 2). При этом муфта 3 ЦИС занимает положение, соответствующее заданной угловой скорости Q. При отклонении Q от заданной муфта 3 сдвинется, сместит золотник 2 и вся система придет в движение, до тех пор, пока скорость Q вновь не станет заданной. 1. Уравнение гидравлического двигателя. Усилия', развиваемые силовым поршнем, значительно превосходят силы сопротивления и. инерционные силы, поэтому их влиянием можно пренебречь. Тогда, если не учитывать сжимаемость жидкости и считать, что площадь окна, открываемого золотником, пропорциональна его перемещению г, уравнение гидродвигателя будет dt = k\Z ИЛИ ру = kxZ, (1) где k{— коэффициент передачи. 2. Уравнение рычага, связанного с муфтой, изо- дромом и золотником. Перемещение муфты х вызывает перемещение золотника z и силового поршня, который перемещает поршень демпфера хос в сторону, обратную перемещению муфты. Следовательно, имеем уравнение z = k2(x-ksxoc), (2) Рис. "2. Гидравлическое исполнительное устройство. где къ = '+/,"» ^з. = коэффициенты передачи; a, b — длины плеч рычага, (см. рис. 2). 3. Уравнение цепи обратной связи. В цепь обратной связи входит демпфер, пружина и рычаг 4,- Составим
8j § 1.1. УРАВНЕНИЯ .И .ПЕРЕДАТОЧНЫЕ ФУНКЦИИ ЗВЕНЬЕВ ,}5 уравнение равновесия сил + с2хос = с3у, _ (3) где 0^00 = FR —сила демпфера, пропорциональная скорости перемещения поршня демпфера хос; c2x0C = FB — сила пружины; с3у = Fc — сила, развиваемая силовым поршнем; съ, с2, с3 — постоянные коэффициенты. После -преобразования уравнения (3) получим (ТосР + I) хос = кфу. (4) где 7'ос = —— постоянная времени цепи обратной связи, /г4 = —— коэффициент передачи. ■Найдя хос из (2) и подставив в его выражение z из (1), получим Подставив (5) в (4),' найдем дифференциальное уравнение гидравлического исполнительного устройства (Тр + 1) ру (t) = k (Тоср + 1) а; (0, (6) где откуда искомая передаточная функция WKP) P(Tp + l) • 3. Найти передаточную функцию и дифференциальное уравнение пассивной электрической цепи (рис. 3) относительно напряжений щ и и2. Решение. Для нахождения передаточных функций электрических цепей, подобных изображенной на рис. 3, удобно пользоваться операторной "формой записи сопротивлений: индуктивного — pL, емкостного рг и активного — JR, где :P = -jt — символ или оператор дифференцирования.
16 ГЛ. 1,--УРАВНЕНИЯ II-ПЕРЕДАТОЧНЫЕ ФУНКЦИИ [-1 Преобразуем электрическую цепь рис. 3 в эквивалентную ей (рис. 4), где Г - Rl(ry+TlLP+l) pCi ■Я| Zi(p)= , рС, R2L2p ZAP) ТиР + 1 R2(tIp2 + t2lP + i) R2 + L2p C2p p{'f2c+T22p) T\—\/C\L\, Тц — -^-, 7ic = /?1C1, TV VC2L2, 2C R2C 2»-2> (1) (2) (3) l'll~ R* Размерность всех постоянных-времен (3) [Т] — сек. Так как падение напряжения на последовательно соединенных сопротивлениях пропорционально величине нь *, уХ ПГЧ~\ 0- U,tp) УгШ Рис. 3. Схема к задаче 3. -йГ Рис. 4. Эквивалентная схема. сопротивлений, то передаточная функция эквивалентной цепочки (рис. 4) находится как отношение W КР) ■ «Л (Р) Ар) ZAP) (4) Zbx'(P) Z, (p) + Z2(p) ' Подставив (1), (2) в (4), получим искомую переда точную функцию электрической цепи /?2(fcoP3 + fciP2 + fe2p + fc3) W (р) - Rz (60р3 + fc.p2 + fc2p + Ьл) + R, (d0p1+d1p*+dip1+dsp) , (5) l, bo = T2T\e, .b\—,T2 + T2lT\c b2 = T2l + Tic, < dc = TiT2, di—.TiTx + T.2Tin, ё2~Т[цТ2С + Т2, с1з = Т2с-
4) § 1.1. УРАВНЕНИЯ И''ПЕРЕДАТОЧНЫЕ ■ ФУНКЦИИ' ЗВЕНЬЕВ.. П Дифференциальное уравнение рассматриваемой электрической цепи относительно напряжений имеет: вид ШЬ0р3+ ... +Ь3) + ЯЛс1»р4+ .... +d3p)]u2(t) = ^Я2(Ь0Р3 + ... + b3)ul(t). (6) 4. Составить дифференциальное уравнение и найти передаточную функцию трансформатора (рис. 5) относительно напряжений щ и и2. Электрические параметры трансформатора приведены на рис. 5. £^_ J^ _^£г Решение. Дифференциаль- 0- ^ (■ ■£ *г ные уравнения равновесия напря- ц R иг Ц! жении цепей первичной и вто- L, ■ ричной обмоток трансформатора & имеют вид w> Wz Щ = /■,£, + Lipli + Mpi2, (1) Рис" 5- Схема траисфор- 1 1 1 ■ да' 1 ■ г ^ \ I , мат0ра к задаче-4. 0 = г42 + L2pi2 + Мр^ + щ, (2) где г,, I,, ^ — сопротивление, индуктивность и ток первичной обмотки; r2, L2, i2 — ro же для вторичной обмотки; ^ — сопротивление нагрузки; иь «2~вх°Дное и выходное напряжения трансформатора; М — коэффициент взаимоиндукции обмоток. ■ Найдя выражение для тока ix из уравнения (1) и подставив в (2), получим дифференциальное уравнение трансформатора или. {(Г,Г2 - Tl)p2 + (Г, + Т2) р + 1] и2 (*) = - кх1Рщ (*), (4) где 7, = —, Г2 = ^^-, т, = —, Г3 = У ТЛ^ТТ^' /? + гв " Размерность коэффициента т, и всех постоянных времени [Т{] = сек (i—l, 2, 3). .Так как коэффициент связи MIYL\L2k трансформаторе со. стальным сердечником
18 ГЛ. I. УРАВНЕНИЯ И ПЕРЕДАТОЧНЫЕ ФУНКЦИИ [S близок к единице, то М «= YlxL2, a LXL2 — ЛГ2*»0 или Т\Т% — Тъ «* 0. Тогда уравнение трансформатора (4) упростится КП + ТДр+l] щ (t) = - kxtftitf). (5) Для режима холостого хода (/? = оо, 7V=0) имеем (TlP+l)u2(t) = — т,ри,(0. На . основании дифференциального уравнения (5) можно записать передаточную функцию-трансформатора по напряжению, КР} их[р) (Г, + Г2)р+Г из которой видно, что трансформатор является инерционным дифференцирующим звеном. Знак минус в дифференциальных уравнениях трансформатора означает, что фаза выходного напряжения изменяется на 180° относительно входного. 5. Составить дифференциальное уравнение трансформатора (рис. 5), если входной величиной является ток 1и а выходной — напряжение и2. Решение. Запишем дифференциальное ураввениеД 1) задачи 4 в виде щ = гМ1 + т1р) + мРЩ-. (1) Подставив Н] из (1) в уравнение (4) задачи 4 и сделав преобразования, получим (7,2р+1)и2(0 = -Шр/,(0, (2) где коэффициенты Т2, k, M соответствуют обозначениям задачи 4. Для режима холостого хода (/? = оо, 7,2 = 0, k=l) имеем u2{t) = -Mpll(t), (3) откуда видно, что в режиме холостого хода трансформатор представляет собой идеальное дифференцирующее звено, если входной величиной "является ток, а выходной — напряжение.
m § l.I. УРАВНЕНИЯ И ПЕРЕДАТОЧНЫЕ ФУНКЦИИ ЗВЕНЬЕВ 19 6. Найти дифференциальное уравнение и передаточную функцию относительно напряжений н, и щ пассивной электрической цепи RC в виде моста (рис. 6). Решение. Токи плеч моста (см. решение задачи 3) h = и\Сф 4 = Т\ = RyCy, T2 = R2^2t Р Тогда u^t) = -^pkit)-Rxh{t) 1 - TJzp* "■lit), Рис. 6. Мостовая схема к задаче 6. (TiP+l)(TiP+l) откуда искомое дифференциальное уравнение имеет вид {TlP.+ 1) iTip + i) щit) = (Г- т?/?2) щ it) (1) и передаточная функция равна 1 - TJrf2 WW-(TiP+1)(tTp~+TT {TlP + l)(T2p+\) (2) где rf = ТХТа. 7. Найти передаточную функцию электрического моста (рис. 6), если сопротивление резисторов /?, = R2 и емкость конденсаторов С{ — С2. Решение. При равенстве сопротивлений и емкостей противоположных плеч моста (рис. 6) постоянная времени Т\ = Т2—Т и передаточная функция (2) задачи 6 принимает вид w(p)= »-?V =Lzl£ ■кр> (i+гр)2 \ + тР-- 8. Найти передаточную функцию гидравлического демпфера (рис. 7), если пренебречь влиянием массы подвижных частей и принять за входную величину силу F, а за выходную — перемещение поршня х. Решение. Приложенной силе F будет противостоять демпфирующая сила Рл = ctx, где с, — коэф фициент
20.. .. ГЛ. 1. . УРАВНЕНИЯ И ПЕРЕДАТОЧНЫЕ ФУНКЦИИ ; [8 демпфирования, пропорциональный вязкости жидкости и площади поршня и обратно пропорциональный площади пропускного отверстия. aj Тогда имеем рх = kF, где с L F(P) Г ■ х к »- Рис. 7. Поршень с цилиндром (демпфер). и, 0- i. Схема и график к задаче 10. 9. Найти передаточную функцию по условиям предыдущей задачи, если учесть массу подвижных частей. Ответ. W(p) = Х(р) __ F(p) pitp+i) г=- С\ m — масса подвижных частей. 10. Найти передаточную функцию, электрической цепи (рис. 8, а) ■■ по огибающей модулированного сигнала с несущей частотой сос = 2я/с, где /с — частота сети. Решение. На основании формулы (4) задачи. 3- передаточная функция электрической цепи (см. рис. 8, а) W(p) и* (р) Тр. и Ар) ту+тр + i T = RC, Tl = LC. (1) Амплитудная частотная характеристика (а. ч. х.) имеет ВИД Та> . . Л(сй) = |1Г(/сйД= ■ У0^т^2)2 + г •2^2' (2)
101 § I.I: УРАВНЕНИЯ Н- ПЕРЕДАТОЧНЫЕ ФУНКЦИИ; ЗВЕНЬЕВ 21 Анализ зависимости (2) показывает, что график а. ч. х. электрической цепи рис. 8, а имеет вид, изображенный на рис. 8, о, причем при резонансной частоте са==(й0=-^-, а. ч. х. принимает, максимальное значение Л(са0)=1, а при 0 ^а<щ и щ<(а^.оо А(а>)<1. •Амплитудная частотная характеристика на рис. 8, б напоминает а. ч. х. апериодического звена первого порядка-с коэффициентом передачи k—l и са0 = 0. Найдем условие, при котором а. ч. х. с достаточным приближением является симметричной относительно резонансной частоты са0, т. е. может рассматриваться как а. ч. х. апериодического звена первого порядка относительно резонансной частоты са0. Для этого найдем частоты toj и са2 из условия идентичности подавления боковых частот апериодическим звеном первого порядка и электрической цепью (см. рис. 8, б): А(а>) = ~ = -^. (3) Подставив (2) в (3), получим уравнение Гю ' (4) решив которое, найдем выражения для боковых частот: «.- Щ . Щ = —~-щ • (5) Чтобы а. ч. х., изображенная на рис. 8, б, была, симметричной относительно резонансной частоты а0—То необходимо, чтобы выполнялось условие со, + ©2 / 1 . Г2 |/V;£—■ (6) Условие (6) вы полняется при ~i<^, т.е. Т<2Т0, или #<2l/-£-. (7) Следовательно, электрическую цепь (рис. 8, а) можно пред ставить в виде апериодического звена первого
22 ГЛ. U УРАВНЕНИЯ И ПЕРЕДАТОЧНЫЕ ФУНКЦИИ ГП порядка по огибающей модулированного сигнала, если выполняется условие (7) и если несущая частота или частота сети ©с = а)0. Для определения эквивалентной постоянной времени апериодического звена первого порядка по огибающей необходимо найти полосу пропускания рассматриваемой электрической цепи Т R Доп = (й2-со1 =-2- = —. (8)- /0 L, Эквивалентная постоянная времени Тэ^-^- = 2-^. (9) Тогда при выполнении условия (7) и при подборе параметров L, С так, чтобы са0 = (йс, можно записать выражение для передаточной функции электрической цепи на рис. .8, а по огибающей модулированного сигнала в виде 11. Найти передаточную функцию электрической цепи (рис.>8, а) по огибающей модулированного сигнала при /?=1000 ом, С = 0,2 мкф,- L = 0,8 гн и несущей частоте входного сигнала fc=400 гц. Решение. Воспользуемся формулами предыдущей задачи. Постоянные времени Т0 = VTc = /o,8 • 0,2 ■ 10" ■6 = 0,4 • 10"3 сек, T = RC= 1000 • 0,2 . 10~6 = 0,2 • 10"3 сек. Условие (7) выполняется. Резонансная частота са0=^- = •"о = 5-=2500 сект1. 'Частота входного сигнала ©с= 0,4-Ю-3 = 2тс/с = 6,28 • 400 = 2512 сек'1, т. е. условие. оо0 = (йс практически выполняется. Условие (7) можно уточнить по формуле (6)
M] § 1.1. УРАВНЕНИЯ И ПЕРЕДАТОЧНЫЕ ФУНКЦИИ ЗВЕНЬЕВ 23 откуда следует, что а. ч. х. симметрична относительно COi+Gb резонансной частоты, так как ^— «* со0. Эквивалентная постоянная времени Тэ = 2 ^ = 2 щ^ = = 1,6 • Ю-3 сек. Передаточная функция по огибающей модулированного сигнала W(P): I 1,6.1(Г)р + 1 ила <ш 12* Найти передаточную функцию пружины и демпфера (рис. 9), если пренебречь влиянием массы подвижных частей и принять за входную величину силу F, а за выходную — перемещение точки А (поршня) х. -Решение. Составляем уравнение равновесия сил F = Рл + Fп = схх + с2х, где Су — коэффициент демпфирования, а с2 — коэффициент упругости пружины. Тогда имеем (Tip + 1) х = kF, где Г] = — , k = cl , откуда передаточ- ная функция W(p) = к к TiP+l Щй Рис. 9. Поршень с цилиндром и пружиной. 13. Найти передаточную функцию по условиям предыдущей задачи, если учесть приведенную к точке А (см. рис. 9) массу подвижных частей. Ответ. Искомая передаточная функция W(p)- ry + TlP+i^ /in й- ш — масса подвижных частей. 14. Изменится ли тип динамического звена, к которому относится .демпфер, рассмотренный в задачах 8 и 9, если входную и выходную величины поменять местами? Найти передаточные функции.
24 ГЛ. 1. УРАВНЕНИЯ И ПЕРЕДАТОЧНЫЕ .ФУНКЦИИ "' [IS Ответ. Да, ная функция изменится. Без учета массы передаточ- W{p). F-{P) kp, Х(Р) где k=>'Ci. С учетом массы — НТр+\)Р, определены в зада- г — HL *Ы = Т& Коэффициенты m и Cj чах 8 и 9. 15. Составить дифференциальное уравнение движения и передаточную функцию двигателя с независимым возбуждением • (рис. 10, а) относительно угловой скорости Q при моменте нагрузки М„=*0. • Ответ. Дифференциальное, уравнение движения (Г„7>2 + 7> + 1)0(0 = La^^J т = * я L* + LB /швх \t). — электромаг- #Я + #В нитная постоянная времени цепи якоря; La, Яя — индуктивность и активное сопротивление якоря; LB, RB — индуктивность и внутреннее сопротивление оконечного каскада усилителя, питающего двигатель. Рис. 10. Схема и механические характеристики к задаче 15. Т = J 1 м J Ая м„ ■ . электромеханическая постоянная времени двигателя; / — приведенный к валу двигателя момент инерции вращающихся частей; Мп — пусковой момент двигателя при Q = 0; 'Qxx—угловая скорость холостого хода при моменте двигателя М = 0;
18] $ 1.1. УРАВНЕНИЯ И ПЕРЕДАТОЧНЫЕ ФУНКЦИЙ ЗВЕНЬЕВ 25 ток короткого замыкания цепи якоря двигателя при q = О, В = -гтт = -пг- — коэффициент, наклона механиче- о?. 1 коэффи- dM | Мп ских характеристик двигателя, &=■- циент передачи. Для двигателей постоянного тока с независимым возбуждением В = const при ывх = var. Передаточная функция двигателя k W0(p) = УяУмР2 + 7"мР + 1 ' 16. Найти дифференциальное уравнение движения и передаточную функцию двигателя с. независимым возбуждением (см. рис. 10, а) относительно угла поворота а. Ответ: (ТиТяр2+Тмр+\) pa(t) = kuBX(t), '" иир) = р(7яГм„2+ГмР+1). 17; Найти передаточные функции двигателя постоянного тока с независимым возбуждением, пренебрегая влиянием электромагнитных переходных процессов в цепи якоря (см. задачи 15 н-16). Ответ, . Wa(p)- к Wa(p) = ТыР+1 ' k Рис. 11. Схема и механические характеристики к задаче 18. 18. Найтл передаточные функции двухфазного асинхронного двигателя (рис. 11, а) при моменте нагрузки М„ — 0. Механические характеристики имеют вид рис. 11, б, а электромагнитными переходными процессами в статоре и роторе можно пренебречь. Решение. Аналогично предыдущей задаче, передаточные ' функции асинхронного двигателя по угловой
26 ГЛ. I. .^РАВНЕНИЯ И ИЕРЕДАЯОЧНИШ ФУНКЦИИ ИВ скорости й по углу Электромеханическая постоянная времени Тм пропорциональна коэффициенту наклона -механической характеристики р (см. задачу 15): Гм = /р0 = / Q0 X. X где / — приведенный к валу двигателя момент инерции вращающихся частей; Q°. x, mB, Ро — соответственно угловая скорость холостого хода, пусковой момент и коэффициент наклона аппроксимированной прямой линией механической характеристики, соответствующей наиболее часто принимаемым значениям управляющего напряжения иу = U° в автоматической системе (см. рис. 11, 6),k- передачи ч> в) 1 Т,Р+1 "а к hp+l Si 1 Р IX. коэффициент двигателя. 19. Для компенсации индуктивного сопротивления обмотки управления двухфазного асинхронного двигателя в ее цепь включают конденсатор емкостью С (рис. 12, а). Требуется найти передаточную функцию двигателя с учетом динамических свойств образовавшегося контура LCR в цеци обмотки управления. Решение. Динамические свойства, выражающиеся инерционностью электромеханических процессов 'двига- Рис. 12. Электрические и структурная схемы к задаче 19.
ffl] § I.I. УРАВНЕНИЯ И ПЕРЕДАТОЧНЫЕ ФУНКЦИИ ЗВЕНЬЕВ 27 теля, полностью определяются, передаточными ф.ункг цйями Wq(p) и Wa(p) (см. задачу 18). Для определения передаточной функции контура LCR обмотки управления составим эквивалентную схему цепи обмотки управления рис. 12* б, где /- — индуктивность, R = —~ — приведенное активное сопротивление обмотки у управления, /у — номинальный ток, Ру — номинальная активная мощность обмотки управления, С — емкость конденсатора, включаемого в цепь управления. Влиянием внутреннего сопротивления источника, питающего обмотку управления, пренебрегаем. Контур LCR подробно рассмотрен в задаче 10. Его передаточная функция по огибающей модулированного сигнала с несущей частотой, равной частоте сети fc или круговой частоте сети сас = 2зх/с, Передаточная функция (1) справедлива при выполнении условий /?<2 т/ — , , ■ ^ 2зх/с. При выполнении обоих условий передаточные функции двухфазного асинхронного двигателя п/п(„\ ^ й (р) * WQW> Uy(p) (ТэР+1)(тиР+1) • WayfJ> Uy(p) р(ТэР+1)^+1) • Структурная схема двигателя принимает вид, изображенный на рис. 12, в. После незначительных преобразований можно получить новые выражения для определения эквивалентной постоянной времени 2 х. 2 2 ]/■[- COS2 ф r8 = 2-K- = ^=— tg<p= -, (2) 8 R coc R шс 6 т сос cos ф ' w где (ос » щ = ■:..— , xL = cacL—индуктивное сопротивление обмотки управления, cos ф —коэффициент мощности
ГЛ. 1." УРАВНЕНИЯ И ПЕРЕДАТОЧНЫЕ ФУНКЦИИ" ' 1 [20 обмотки управления при работе без "конденсатора (в номинальном режиме). 20. Индуктивность обмотки управления двухфазного' асинхронного двигателя L = 0,05 гн, а приведенное активное сопротивление /?■= 150 ом. Какой должна быть емкость конденсатора, включаемого в цепь обмотки управления, если частота сети /с = 400 гц7 Можно ли пользоваться передаточной функцией (1) из предыдущей задачи? Ответ. 1) С = 3,2 мкф. 2) Можно, так' как R'= 150 ом< < 2 "J/ ~ = 250 ом. . 21. Найти передаточную функцию пассивной электрической цепи LC в виде моста, изображенного на рис. 13 (см. задачи 6 и 7). - Ответ. W(p) = ^~^, т = уТс. -22. Найти дифференциальное уравнение движения поршня х,\\ —яг; Рис. 14. Поршень с цилиндром и пружиной. Рис. 13. Мостовая схема к задаче 21. относительно корпуса хх под действием силы F (рис. 14) без учета массы подвижных частей. Решение. Составим уравнение равновесия сил F = Гд + Fn = с{х3 + с2лг2, где хъ = хх — х2 — перемещение поршня относительно цилиндра, х2—перемещение точки А. Подставив в уравнение сил вместо х3 его значение, получим рхх (/) = kxF (t) + k2(x{p - 1)x2(t), где kx = c~x, fej^CjjC,1,. т, = 0,0^' (см. задачи 8 и 12)..
Ml § LI. УРАВНЕНИЯ И ПЕРЕДАТОЧНЫЕ ФУНКЦИИ ЗВЬИЬКЬ 2J 23. Найти дифференциальное уравнение движения по условиям предыдущей задачи, учитывая массу подвижных частей. Ответ. ■ ' (7> + 1) рхл (0 = kxF (t) + k2 {x%f + xlP - 1) хг (t), „ m, -, / in, — m2 где 7i~-—-. ^2— V ;;—~> mi— масса поршня с ры- i С ■у г ■ С 2 чагом, т2 —приведенная масса пружины с цилиндром (см. задачи 9 и 13). 24. Найти дифференциальное уравнение и передаточную функцию центробежного измерителя угловой Рис. 15. Центробежный измеритель скорости и график к задаче 24. скорости (ЦИС) на'рис. 15, а, если принять за выходную величину перемещение муфты х, а за входную — приращение угловой- скорости AQ и считать известными приведенную к точке М массу всех шаров т; длины 'рычагов lu k, 1г\ приведенные в точке В муфты а) силу пружины Fn, б) силу вязкого трения и демпфера FR, в) инерционные силы приведенных масс Fn и г) приведенные силы от веса всех подвижных частей Fa. •Влиянием сил сухого трения пренебречь. - Решение. Выберем- прямоугольную систему координат 2, х. Ось х совмещена с осью вращения ЦИС, а ось z — с положением точки 'В при Q —. О,, когда муфта под действием пружины находится в положении х = О, где выхрдная величина : х есть координата точки- В.
30 ГЛ. 1. УРАВНЕНИЯ И ПЕРЕДАТОЧНЫЕ ФУНКЦИИ [24= Движущей является центробежная сила шаров Fn = mrU\ (1) где г — гы — расстояние точки М от оси х\ На муфту действуют приведенные силы сопротивления Р и приведенная движущая сила F (см. рис. 15, а). Приведем к точке В силу Fu на основании равенства мощностей FxB = FaZM, F = Fn^, (2) где хв, гы — составляющие скорости перемещения точек В и М по соответствующим координатным осям. Определим zM: £M = l/Mcosa^l/A-icosa = xB-i- tga|tgp , - (3) где / = /j +12, Vh> VM — линейные скорости точек А и М при их вращательном движении относительно общего центра с координатами (Ь, а), а, р —углы, показанные на рис. 15, б. Подставив (3) в (2) с учетом (1), получим F=mii^b^Q2=k>h{r'a'®Q2' (4) где £,.= «{,/,(г, a, р) = tga;tgp. Из рис. 15, б находим г = b +1 sin a, х = а — 1х cos a —13 cos p, . (5) гдео = ^ + /3, Ь — радиус муфты и фланца, к которому крепятся рычаги-держатели шаров. Из соотношения (5) видно, что переменные г, х, а н р связаны между собой нелинейной функциональной зависимостью. Следовательно, можно найти М', «, Р) = /<*)• ■ (6) Например, при l3=li (a = p, a = 2/j) , /i (r, a) = f(x) = (2/, - ж) Г b + -L] . (6, a)
421 § 1.1. УРАВНЕНИЯ И ПЕРЕДАТОЧНЫЕ ФУНКЦИИ ЗВЕНЬЕВ 31 Подставив (6) в (ft), получим F = kxf{x)Q\ (7) Линеаризуем выражение (7) в окрестности малых отклонений переменных х и Q относительно выбранного установившегося режима Q = Q0, х = х0: AF = {ж)°Ах+(ж)°ш=*lQ°D Ajc+2kiQoE AQ- <8) где £) = AIM. I £ = f ( *Л I dx L=*0' /W|x=*-- . В установившемся режиме приведенная сила сопротивления Р = Fp + Рв. При этом приведенная сила от веса подвижных частей (в основном от веса шаров) Рв также зависит от перемещения муфты х\ эта зависимость также является нелинейной. Приближенно примем FB = const. Тогда в динамическом режиме для малых отклонений уравнение равновесия' сил примет вид ДР„ + ДР„ + ДРП = АР = ДР, или тп Ах -£ с, Ах + с2 Ах = klQ20D Ах + 2kfil0E AQ, (9) где тп — приведенная к точке В масса подвижных частей, х, х — скорость и ускорение муфты, сх — коэффициент демпфирования, с2 — коэффициент упругости пружины. Преобразуем уравнение (9)" к виду {Tip2 + TiP+i)Ax{t) = kAQ(t), (10) где 2k,&0E Т = Л / тп у _ с1 и . ■ k,Q%D с2 - ^ЩЛ. I Для всех практически осуществимых ЦИС по схеме рис. 15, а функция f(x) имеет падающий характер (рис. 15, в), а коэффициент D= d имеет отрица- тельный знак, который необходимо учитывать при
82 ' ГЛ. I. УРАВНЕНИЯ И ПЕРЕДАТОЧНЫЕ ФУНКЦИИ |£5 вычислении параметров к, Тъ Т2 и при записи уравнений (9) и (10). Передаточная функция ЦИС WW^-zjr1- ту + т1Р+1 ■ § 1.2. Типовые динамические звенья 25. Какое динамическое звено имеет функцию веса w (t) = 50 {е~ы — е~ш) • !,(/)? Найти параметры этого звена и записать, выражение передаточной функции. Решение. Способ 1. Приведенная функция веса состоит из двух экспонент. Следовательно, это апериодическое звено второго порядка с весовой функцией вида w (t) = -=нЦг- [е~к - е~Ц • 1 (0, откуда находим Г3 = 0.2 сек, Тл — 0,1 сек и k — (0,2 —0,1)Х ХБ0 = 5, WlP)= (0,2р+1)(0,1р+1) " Способ 2. '(Р)={.^(0 W(p)= т<Л\е-г>Ч1- (1 + 0,2р)(1+0,1р) ' откуда 7'3 = 0,2 сек, Г4 = 0,1 сек, k = 5. 26. Найти переходную функцию неустойчивого звена с передаточной функцией W (р) = р .' _ ■.- . Ответ, h (t) = 5 ( - 1 + еш) • 1 (t). 27. Найти параметры передаточной функции колебательного звена, если его переходная функция имеет вид, изображенный на рис. 16. «• Решение. Способ 1. Переходная характеристика колебательного звена записывается в виде . h (/).===' k\\ - е-* (cosM + -^ siii Afj] • \{t).
271 § 1.2. ТИПОВЫЕ ДИНАМИЧЕСКИЕ ЗВЕНЬЯ 33 Затухание колебаний происходит по экспоненте с постоян- 0,5 сек, откуда у = 2 сек'1. Период пой времени Т. h(t) колебаний 7Y = -г- = 0,628 сек, откуда Я, = 10 сек~1. А,40 Аг=5,3 ■ 0,5 '•''■ /-'■-' Рис. 16. Переходная функция. t.cen Составляем систему уравнений Я т = ю, ; (О решив которую, находим Г = 0,1 сек, £ = 0,2. Из графика рис. 16 определяем k = 20. Способ 2. Определив амплитуды Д и А2 (см. рис.. 16), можно найти коэффициент затухания переходного процесса y по формуле п А2 10 . 10 0 л 5,3 Подставляя значения частоты затухающих колебаний Я. и коэффициента y в систему уравнений (1), находим постоянную времени Т и параметр затухания |.
34 '"' ГЛ. I. УРАВНЕНИЯ И ПЕРЕДАТОЧНЫЕ ФУНКЦИИ "' 128 Передаточная функция Wi \ 'к 20 КГ>) ~ ТУ + 2\Тр + 1 0,01р2 + 0,04р + 1 ' .. 28. Устройство работает на переменном токе. Каким типовым звеном определяются его динамические свойства по огибающей, если переходная характеристика имеет вид, изображенный на рис. 17? Колебания с частотой Рис. 17. Переходная функция звена, работающего на переменном токе. сети на графике показаны без соблюдения масштаба времени. Определить параметры передаточной функции звена. Ответ. Колебательное звено с передаточной функцией W = 0,0042р2 + 0,028р+1' 29. Известны следующие параметры центробежного измерителя скорости (см. рис. 15): приведенная к точке М масса шаров т = 0,02 кг; /=-6 см, /( = 3 см; стабилизируемая угловая скорость Q0= 150 сек~1\ коэф-
3D § T.2. ТИПОВЫЕ ДИНАМИЧЕСКИЕ ЗВЕНЬЯ 35 фициент D = — 0,11 • Ю~; приведенная к точке В масса подвижных частей тп = 0,09 кг; коэффициент упругости пружины с2 = 0,7 н ■ м~1. Каким должен быть коэффициент демпфирования clt чтобы центробежный измеритель скорости был апериодическим звеном второго порядка? Ответ. с1 ^ 0,54 н • сек • м~1 Для решения этой задачи следует воспользоваться данными задачи 24. 30. По переходной функции, изображенной на рис. 18, определить тип и передаточную функцию звена. Переходная функция представляет собой сумму линейного и экспоненциального членов. I, сел 0,4 0$ V 1,6 Рис. 18. Переходная функция. 0,1 0,2 0.3 Рис. 19. Весовая функция. Ответ. Это — интегрирующее звено с замедлением. Его переходная функция h{t) = k[t-T\\-e~)\- l{t). Уравнение асимптоты переходной функции Лд(0 = k (t — T) позволяет определить параметры передаточной функции ь_ 9,6-4,8 ,п __.._, „ пл __.. IW,_4 12 1,2 - 0,8 12 сек'1, Т = 0,4 сек, W(p)- Р(0,4р+1) 31. Функция! веса апериодического звена первого порядка изображена на рис. 19. Определить параметры пеРедаточной функции.
36 ГЛ. Т. УРАВНЕНИЯ И ПЕРЕДАТОЧНЫЕ ФУНКЦИИ (32 ^Hg> 7\х, g - . *, тгу*г,р ч *В ■г. Рис. схема Ответ. Коэффициент передачи k = 2 и постоянная времени Т = 0,2 сек. 32. Как изменятся постоянные времени, коэффициент передачи, время и форма переходного процесса апериодического второго порядка или колебательного звена при охвате его жесткой отрицательной обратной связью с коэффициентом передачи /г,} (рис. 20)? Ответ. Время переходного процесса уменьшится, так как уменьшатся обе постоянные времени Т2 и 1\; форма переходного процесса изменится (например, вместо апериодической может стать колебательной), так как постоянная времени Т{ уменьшится в большей степени (в 1-Ь/г^о раз), чем Т2 (в ]Л + kxku раз). Коэффициент передачи уменьшится в I + kxku раз. 33. Как изменятся постоянная времени Tt и коэффициент передачи /г, апериодического звена первого порядка, если его охватить идеальной гибкой отрицательной обратной связью с передаточной функцией цепи обратной связи Woc (р) = fe0p? Ответ. Постоянная времени возрастает (Т = Т1 + /г,йс), а коэффициент передачи останется тем же (k = /е(). 20. Структурная к задаче 32. § 1.3. Дифференциальные уравнения и передаточные , функции автоматических систем 34. На рис. 21, а представлена принципиальная схема автоматической системы регулирования (стабилизации) скорости теплового двигателя. Чувствительным элементом (ЧЭ) является центробежный измеритель скорости (ЦИС). Исполнительным устройством {ИУ) является гидравлический двигатель, состоящий из золотника 2, связанного с муфтой ЦИС 3, и силового поршня /, связанного с заслонкой, или регулирующим органом {РО). Составить структурную схему, найти передаточные функции разомкнутой системы W(p), замкнутой системы относительно регулируемой величины Ф (р), относительно
.■hi § |ч урлши-.ппя и пкргдлючиыг фупкнпи г.ппрм ошибки Ф,(/>) и по возмущению Ф, (/»). если .iiinc.ipi! ui- н.лтые уравнения отдельных звеньев имеют такой iv u 1) Овигатель (офт,ект) * (Г0/?+1)и=*пУ-*1Л1 , * ■# г i,e О —угловая скорость, (/ — перемещение заслонки, ;W„ — момент нагрузки; /Уи Я двигатель ") б) -К Г/р'ЦрЧ <L TqP" S2' S2 Рис. 21. Принципиальная (а) и структурная (б) схемы к задаче 31. 2) центробежный измеритель скорости (см. задачу '24) (T'2ps + Ttp+ l)jc = fe2AQ, где х — перемещение муфты и золотника, Т2, 7"i — постоянные времени ЦИС; 3) гидравлический двигатель РУ = V. /?о> feh fe2 и k3 — коэффициенты передачи.
38 ГЛ. Т. УРАВНЕНИЯ И ПЕРЕДАТОЧНЫЕ ФУНКЦИИ '185 Решение. Составим структурную схему (рис. 21,6), где обозначены: QHC — приведенная или эквивалентная угловая скорость настройки, задаваемая поджатием пружины^ЦИС (см. рис. 21, a); Q' — составляющая угловой скорости от перемещения заслонки у, a Q," — от момента нагрузки М„, причем Q = Q' + Q", и- ошибка или отклонение AQ = QHC — Q. Тогда передаточная функция разомкнутой чсистемы по задающему воздействию rw"3i=*rwwo' «-«*[--]. и по возмущению (по нагрузке) Передаточная функция замкнутой системы относительно регулируемой величины ф{р)= Q(p) _ У(р) , К fiHC(p) l + W(p) р(Г0р-+1)(7|р8+Г1р+1) + /С" Передаточная функция замкнутой системы относительно ошибки ф ( ч_ №(р) _ 1 __ p(7-0p-M)(r|P+rlP+l) !М I Ф/(Р) = " и по возмущению о(р) _ yf(p) _ кхР {ту +. г,р +1) Л*н(р) 1 + Г(р) р(Г0р+1)(Г|р2+7'1р+1) + /С' 35. Найти передаточные функции системы стабилизации угловой скорости (см. предыдущую задачу), если гидравлический двигатель охватить гибкой отрицательной „ обратной связью в виде демпфера и пружины (см. рис. 2). Ответ. п//^\_ К(ТрсР+1) . ^' p(тy+тlp+i)(тp + l)(т0p+l),
371 § 1-3. УРАВНЕНИЯ И ПЕРЕДАТОЧНЫЕ ФУНКЦИИ СИСТЕМ 39 ур) р (г|Р2 + г,р +1) с^Р + D (гоР + 1) + ^с (УосР +1) * ■ ф p(7fP2+:r,p+i)(7>+i)(V-H) '* p(laaP2 + Tlp+i)(TP+i)(Tip+l)+-K(Tocp + \) ' ' КР) р(Г^р2 + Г1Р + 1) (Гр + 1) (Г0р + 1) + К (focP + I) ' где коэффициенты Гос, Т uk'3 = k определены в задаче 2, а остальные коэффициенты — в задаче 34. 36. Найти дифференциальные уравнения движения автоматической системы стабилизации угловой скорости (рис. 21) относительно регулируемой величины (Q) при задающем воздействии (QHC) и при возмущении (Мн). Дифференциальные уравнения отдельных звеньев системы рис. 21 приведены в задаче 34. Ответ. а) [р (ГоР + 1) (Т2Р2 +TlP+l) + K]Q (0 = *Q« С) или (я0р4 + я,р3-+ а2р2 + а3р + а4) Q (t) = b0Qm (t), где а0 = Т0Ц, ах = Г2 + T0TV а2 = Т0 + Г,гя3 = 1, а4 = b0=Ki б) [р (ТоР + 1) (Г1р2 + Тхр + 1) + /С] Q (0 = = -feip(r22p2+r,p + l)MH(0 или (й0р4 + с, р3 + с2р2 + а3р + g4) Q (0 = = - {d0p3 + dlP2 + d2p) MH (0, где do — kj-z, d\ = k\Ti, d2 = ki. 37. Найти дифференциальные уравнения движения системы стабилизации угловой скорости (см. рис. 21) относительно ошибки (ДО) до задающему воздействию (2нс) й по возмущению (Мн). Дифференциальные уравнения отдельных звеньев системы рис. 21 определены в задаче 34. i
40 ГЛ. I. УРАВНЕНИЯ И' ПЕРЕДАТОЧНЫЕ -ФУНКЦИИ 138' Ответ. а) [р(Тор.+ \){tV, + TiP+ 1). + /С].АЙ(0 = = р {т0р + 1) (fip2 + г,р +1) Q„t. (0; б) [р {Тор + 1)(rip2 +Ttp+l) + K] ДО'ОО = - = feip (rip2 + Tip + l) Af „ (0- 38. Составить структурную схему и найти передаточные функции разомкнутой W(p),- Wf(p) незамкнутой /v<- p(hp -") ~м« -^н^ v+/ p(hh>) OCi, ее т~ Рис. 22. Принципиальная (а) и структурная (б) схемы следящей системы. Ф(р). Фх(р). (lJf(p) (см. задачу 34) следящей системы (рис. 22, а), если звенья системы описываются следующими уравнениями: 1) элемент сравнения ■& = fj'i — fJ-2, 2) потенциометрический датчик и — kfi; 3) усилитель (Тур + 1)1^ = k2u, Ty— постоянная времени усилителя; 4) двигатель(Тыр + \)ра = k3Ui —1гЛМю Тн— постоянная Еременн двигателя-
§ !.S. УРАВНЕНИЯ И ПЕРЕДАТОЧНЫЕ ФУНКЦИИ СИСТЕМ 41 5) редуктор •&2 = ^5а' ^i» *2. k3,'ki, k5~коэффициенты передачи. Ответ. Структурная схема представлена на рис. 22, б; ktks W,{p) = Ф(р) = p(T»p+l) ' К PiTyp+DiTvp+D + K» ktkslTyp+l) u fw ~ р(тур + 1)(тир+1)-Л<' ЛР) .plTyp+l)(Tup+l) + K-' 39. Составить структурную схему и найти передаточные функции следящей системы (см. рис. 22, а), если V Wl7^ ■4-® "4 ^ <% v* £ XW; -/*, «J„ cc. ■€>■ V a; 4 **(У+0 ^2^3 0f K>* h^fll-4b< 0 r< r "ir ^ y+/ ^ V *s p(t*p+i) ОС *s Рис. 23. -Структурные схемы следящей системы. непосредственно с валом двигателя соединить тахогенера- тор,- а его напряжение подать на вход усилителя в проти- вофдзе с выходным напряжением потенциометрического
42 _ ГЛ. Г. УРАВНЕНИЯ И' ПЕРЕДАТОЧНЫЕ ФУНКЦИИ' "" [40 <Мр) = датчика угла рассогласования. Дифференциальное уравнение тахогенератора uTT = k6pa. Уравнения остальных звеньев следящей системы приведены в условии предыдущей задачи. Ответ. Структурная схема представлена на рис. 23, а. Для вывода передаточных функций структурную схему* рис. 23, а необходимо преобразовать: перенести сумматор 2 на вход сумматора 1 (рис. 23, б). Тогда WiP)= P[(7'yp+1)(7>+1) + W6]' ^ = М^' W ( у_ feA(V-bl) iKP) p[(Typ+l)(TKp+l) + k2kskB]' К . ф (р)=' р [(Тур + 1) (ТмР + i) + k2k3kB] + к ' р[(У+1)(7>+1) + Ы^б] Р 1(ТуР + 1) (Гмр + 1) + k2k3k6] + К ' , . k4kS{TyP+l) f {Р) ~ Р [(ТуР + 0 (ТмР + 1) + k2k3k,] + К " 40. Найти дифференциальные уравнения движения следящей системы (см. рис: 22) относительно ошибки {&) по задающему воздействию {&i) и по возмущению (Ми). Дифференциальные уравнения отдельных звеньев приведены в условии задачи 38. Ответ. а) [p(TyP+l)(TMp + l) + K}&{t) = = р(Тур + \)(Тыр+1)«,(/), или (а0р3 f alP2 + a2p + а3)Ъ(/) = (й0р3 + blP2 + b2p)u, (0, *■ а0 = Ь0 = ТуТы, а, = Ьх = Ту + Гм, a2 = b2=l, б) [р (Тур + 1),(7> + 1) + /С];»(0■= W^P + О М„ (/) или (с0р3 + GiP2 + а2р + а3) й (*) .= (d0p + d,) MH (/),
§ 1.3. УРАВНЕНИЯ И ПЕРЕДАТОЧНЫЕ ФУНКЦИИ СИСТЕМ 43 41. На рис. '24, а язображена принципиальная схема дистанционной следящей системы с синусно-косинусными. вращающимися трансформаторами (СКВТ), на которой обозначены: #If ■&2 — углы поворота командной и исполнительной осей, •&='в'1—•0,2 — ошибка, РМ — рабочий механизм (объект), Р — редуктор,.Д— двигатель, ТГ—тахогене- ратор. Параметры элементов следующие: k^e-pad'1] — коэффициент передачи чувствительного элемента (СКВТ) а) СКВТ-2 *s$Gj^ Рис. 24. Следящая система. в линейной части характеристики, k2 и k3 — коэффициенты усиления усилителей по напряжению, k4 [pad • е-1 • сек'1] — коэффициент передачи исполнительного двигателя, fes — = п'1 — коэффициент передачи редуктора, п — передаточное отношение, k6[в-сек-рад-1] — коэффициент передачи тахогенератора, k7 [pad-н'1-см'1 -сек'1] — коэффициент наклона механической характеристики двигателя, Т\ и Т2—постоянные времени усилителя и двигателя, Т = непостоянная времени дифференцирующей цепи. Требуется составить структурную схему и определить передаточную функцию разомкнутой системы, передаточные функции замкнутой системы: а) относительно Управляемой величины по. задающему воздействию, б) относительно ошибки по задающему воздействию.
44 ГЛ. 1. УРАВНЕНИЯ И ПЕРЕДАТОЧНЫЕ ФУНКЦИИ [4Й в) относительно ошибки по возмущающему воздействию и добротность следящей системы по моменту нагрузки Ма. Ответ. Структурная схема изображена на рис. 24, б. Передаточная функция разомкнутой системы w( ч /С (1 + 7» W. КР) р [(1 + 7» (1 + 7» (1 + Тр) + кък^Тр\ ' I где добротность по скорости (отношение постоянной скорости слежения к установившейся ошибке) /С = ktk2k3k4k5 [сек~1]. Передаточная функция замкнутой системы относительно управляемой величины по задающему воздействию ■KfJ> р [(1 + 7» (1 + 7» (1 + Тр) + k3k<keTp] + K(l + Тр) ' Передаточная функция замкнутой системы относительно ошибки по задающему воздействию Л) („\ -. р[{1 + Г,р) (' +"Гар) (1 + Тр) + *»*^гр1 "*КР' Р [О + 7» (1 + 7» (1 + Тр) + Ыц1гвТр]+К[1 + Тр) • Передаточная функция замкнутой системы относительно ошибки по возмущающему воздействию (моменту нагрузки Ми) ф , v k7kj(l + TlP)(l + Tp) Wm КР> р [(1 + Т,р) (1 + 7» (1 + Тр) + k3ktkeTp) + K(l + Тр) * Добротность по моменту (отношение момента нагрузки М» на исполнительной оси системы к рассогласованию в установившемся режиме) Am 2 I i * 42. Для предыдущей задачи определить численные значения коэффициентов, входящих в передаточную функцию разомкнутой системы, при'- следующих исходных данных: крутизна чувствительного элемента kx = -= 1 в/град = 57,3 в/рад, коэффициенты усиления усилителя k2 = 2,5 и &з = 80, номинальное значение напряжения двигателя [/„=110 в, скорость холостого хода "хх = 9000 об/мин и пусковой момент Мп = 55 Г-см =
42] .§ 1.3. УРАВНЕНИЯ И ПЕРЕДАТОЧНЫЕ- ФУНКЦИИ СИСТЕМ 45 = 0,54 н • см, момент инерции двигателя с объектом / = 0,098 г • см2=0,01 Г-см- сек2, передаточное отношение редуктора п = 1000, коэффициент передачи тахогенера- тора /fe6 = 0,001 в • мин/об = 9,6 • Ю-3 е ■ сек/рад, постоянная времени усилителя 7,1 = 0,01 сек, постоянная времени дифференцирующей цепи Т = 0,14 сек. Решение. Коэффициент передачи двигателя , йх х лих.х 3,14-9000 0„ т -1 • -1 fe4—Ёг^—-зоёл;— 30-1Ю =8-6 рад-в1-сек К Коэффициент наклона механической характеристики -, Qx у пихх 3,14-9003 .„ „ т „_, _, _, к^-М7 = Ш^^\ 30-55 =иЛрад-Г*-см*-секК Постоянная времени двигателя Т2 = J Ьр. = Jk7 = 0,01 - 17,2 = 0,172 сек. Добротность системы, по скорости is i. и и и и 57,3-2,5,80-2,6 1П_ _., /С = М2*3*4К5= "^ [000^1 ~ 100 СеК • Передаточная функция разомкнутой системы 100(1 +.0,14р) W(p)- р (1 + 1,18р + 0,027р2 + 0,00024р3) Раскладывая знаменатель последнего выражения на множители, передаточную функцию разомкнутой системы можно представить в следующем виде: W tp) = /С (1 + 7» p(l + T,p)(l+2lTiP+Ty) ' где Т3= 1,16 сек, Г4 = 0,0145 сек и £ = 0,8. Добротность по моменту к Кп* 100-1 OOP2 r- Q 1П6 г д-1 Ам = —у = yj~2— ~ 5,8 • 10ь Г • см • рад = = 1700 Г • см {угл. мин)~ •
46 ГЛ. \. УРАВНЕНИЯ И ПЕРЕДАТОЧНЫЕ ФУНКЦИИ [43 § 1.4. Структурные схемгы и их преобразование 43. Преобразовать динамическое звено, описываемое дифференциальным уравнением (Tlp4-TlP+l)x2 = kxr (1) во встречно-параллельное (с обратной связью) соединение консервативного и идеального дифференцирующего звеньев. Решение. Преобразуем дифференциальное уравнение (1) к виду _ * 7"iP #2 ■4р2+1 тУ+\ %2- (2) а) По уравнению (2) составим структурную схему (рис. 25, а), которая переносом сумматора,.или элемента сравнения, и объединением двух последовательно соединенных звеньев преобразуется в искомую схему на рис. 25, б. 44. Найти передаточную функцию замкнутой системы Ф(р) автоматической системы, структурная схема которой изображена на рис. 26, а. Решение. Освободимся от перекрестных связей в^структурной схеме на рис. 26, а, для чего . перенесем узел / через звено W3 по направлению действия сигнала (рис. 26, б). По полученной структурной схеме, находим искомую передаточную функцию ■ 45.\Найти дифференциальное уравнение автоматической системы, структурная схема которой изображена на рис. 26, а, относительно управляемой величины y(t) X, А /"//*/ К > >УЛ/ х> _ /0\ - 1 Г б) \ к &*' Рис. 25. Структ ' . да Т> о кр У! че эньн 43 Ър Th2*' ■Zz .хг з схемы к за-
461 § T.4. СТРУКТУРНЫЕ' СХЕМЫ И ИХ ПРЕОБРАЗОВАНИЕ 47 по задающему воздействию g {t), если *■''- wApY=%-. w3(p) = k3. Решение. Воспользовавшись решением предыдущей задачи, найдем а» (п\ - *Ш. = ь° где-F(p), G (р) — изображения управляемой величины «/ ^ / -•-»■ ^ / г ... ъ(9> - ^ г^Г У - щ 3 . В) »1 < »3 t ? ' ? У Рис. 26. Структурные схемы к задаче 44. и задающего воздействия, р = с + /to — комплексная переменная, bc= kik3, ao^Tikl1, ay = k2 +k3Ti,.a2 — k\ + k3+ + k[k3. Тогда искомое дифференциальное уравнение (а0р2 + ахр Л- а2) у (t) = £>0g (*), где P — -jf~ символ дифференцирования. 46. Найти дифференциальное уравнение автоматической системы, структурная схема которой изображена на рис. 27, а, относительно управляемой величины y{t) по возмущению f(t), если WMp) = fct, W2(p) = rp, Ws(p)-k3, Wifr)- k5 TiP+l ' W5(p) = ■ Ttf+Tgp + l
48 ГЛ. 1. УРАВНЕНИЯ И ПЕРЕДАТОЧНЫЕ ФУНКЦИИ МП Решение. Вначале получим передаточную функцию автоматической системы по возмущению CDf(p), для чего преобразуем структурную схему рис. 27, а. Перенесем сумматор 2 через звено W5 и заменим W\, W2 одним о) т 1—1 f 1—1 А > т^ т^ f PI 1 1 J 1 1 2 1 > й/ 4 ■» "j # J '** Щ+Щ, —* % -—©-*■ И * '*& 1 1 ^ У у Рис. 27. Структурные схемы к задаче 46. звеном (рис. 27, б). Найдем передаточную функцию разомкнутой системы по задающему воздействию w^p)=aVu=[{Wi+W2) Wa+^з] w* и по возмущению Wf(p) _ У (р) Тогда l + W(p) nop* + alP2 + a2p + a3 ' где Y (p), F(p) — изображения управляемой величины y(t) w возмущения /(t), p = c + ja — комплексная переменная, d0 = kjr ^ = /г5, a0=»7'17,|, a, = 7| + 7;17\J. «2 = Ti + T3 + /г4&5Т + Л3^5ГЬ G3 = I + k3k5 + kikjt5.:
«] § 1.4. СТРУКТУРНЫЕ СХЕМЫ И ИХ ПРЕ0ПРА30ВАНИЕ 49 Отсюда искомое дифференциальное уравнение имеет вид d (a0pl + ахр2 + а2р + а3) у (f) = (d0p + d,) / (/), р = м . 47. Найти следующие передаточные функции автоматической системы, структурная схема которой ' о) ' 1 2 г^ »"/ i—Г^1-<±>- <j> i <±>- ~йГк<±> /o> / 3 2 # i» ад ФЬ ад "-4>*R" г<? /32 H H—f •-»- X ЩЩ B) m л W,W2W3 ■~<&Wb 2 У \ > > X f г) ~~ w,^*(/*wlw2w3)w4*w5 %w, \ 1 >r '\L Р.ис; 28. Структурные схемы к задаче 47. изображена на рис. 28, а: разомкнутой системы по задающему воздействию W(p) и по возмущению 'Wf(p};.
50 " ГЛ. t. УРАВНЕНИЯ И ПЕРЕДАТОЧНЫЕ ФУНКЦИИ [48 основную Ф(р), по возмущению Of(p) и относительно ошибки по задающему воздействию Фх(р). Решение. Преобразуем структурную схему рис.28, а (см. рис. 28, б, в и г). По структурной схеме рис. 28, г найдем W (р) = Wx W2 + (1 + W, W2W3) Wi + Ws, Wf (p) = WbWA, ~ /nx _ W&i + il + WiWiWJWt+Wt WKP) i + WlWi + (l + W1WaWs)Wt+Ws ' ^flP/ l + W.^ + d + ^.^jTrgJ^ + U^B • Ф,(Р) ' 1 + W, Ws + (1 + W, №2Г3) №4 + Ws 48. Найти дифференциальные уравнения автоматической системы, структурная схема которой изображена на рис. 28, а, относительно управляемой величины y(t) по задающему воздействию g(t) и по возмущению f{t), а также относительно ошибки x(t) по задающему воздействию g{t) и по возмущению f(t), если Wi(p) = klt Ответ. ' D{p)y{t) = {buP* + bxp + b2)g{t), D(p)y(t) = (d0p* + dlP + d2)f(t)., D(p)x(t) = (coP3 + clP2 + c2P)g{t), ■ D{p)x(t)=-(d0p2 + dlP + d2)f{t), где характеристический полином системы D (Р) = аоР3 + сцр2 + а2р + а3; а0 = с0, ах = ЬаЛ-съ a2 = bi-+c2, a3=b2; b0 = kik2k^T2 + TiT2, bi = kifak? + Tx + T2 + kik2k3T2 + ksfHlTu b2 = l + /jifo&3 + fefe? ; fo=k?TiT2, Cy = kil{Ti + T2), c2 = k~i\ d0 - k3TiT2, dt = /г3 (Г, + T2), d2 = k3; p = -^.
ГЛАВА2 ЧАСТОТНЫЕ ХАРАКТЕРИСТИКИ ДИНАМИЧЕСКИХ ЗВЕНЬЕВ И СИСТЕМ АВТОМАТИЧЕСКОГО РЕГУЛИРОВАНИЯ § 2.1. Характеристики динамических звеньев 49. Построить амплитудно-фазовую характеристику звена с передаточной функцией >(p)=-f • Ответ. Амплитудно-фазовая характеристика совпадает с отрицательной полуосью мнимых чисел (рис. 29, а). 50. Построить амплитудно-фазовую характеристику звена с передаточной функцией W(p) = jr- Ответ. Амплитудно-фазовая характеристика совпадает с отрицательной полуосью вещественных чисел (рис. 29, б). 51. Построить амплитудно-фазовую характеристику цепи RC, представленной на рис. 30, a; R = 1 ком, С= 10 мкф. Решение. Частотная передаточная функция цепи равна где Г = /?С=103-10"5=10~2 сек. Преобразуем выражение (1) таким образом, что* бы оно представляло собой комплексное число в
52 ГЛ. 2. ЧАСТОТНЫЕ ХАРАКТЕРИСТИКИ ЗВЕНЬЕВ И СИСТЕМ [61 алгебраической форме: io-V 1 + io-V 1 10_2сй 1 + io-V (2) Задаваясь отдельными значениями со,, можно по формуле (2) вычислить ряд пар значений 0(ы) и V(w) j'Vfw) U«J) О woo -0,5 «-w2K -1.0 "W'K -15- ■ -2,0- *ОЦ5К л а <5f -f- K tJ II O^-w -3.0 . -2,0 -1,0 6) jV(w) 0 Ufwt Рис. 29. Амплитудно-фазовые характеристики интегрирующих звеньев первого порядка (а) и второго порядка (б). и построить по мим амплитудно-фазовую характеристику цепи. Однако анализ выражения (2) показывает, что эта характеристика определяется уравнением 1/2 + (£/-0,5)2 = 0,52 и для положительных частот является полуокруж-. ностыо, расположенной в верхней полуплоскости с центром в точке (0,5; /0) и радиусом 0,5 (рис. 30,6). Из выражения (2) видно, что при со =.0 Щ/со) = = 0-f-/0, а при (о — оо U^O'ico) ==.l + /0. Точки, соответ-
53]" § 2.1. ХАРАКТЕРИСТИКИ ДИНАМИЧЕСКИХ ЗВЕНЬЕВ 53 ' ствующие этим, а также некоторым промежуточным частотам, указаны на рис. 30, б; значения частоты на этом и всех- последующих рисунках даны в рад/сек. С 0— я\ ивь JV(W) 0.50 O.ZS а) Рис. 30. Амплитудно-фазовая характеристика дифферен- \ пирующего звена (случай 1). Частоты, соответствующие промежуточным точкам кривой, могут быть найдены следующим образом. Аргумент комплексного числа (2) равен 1 ,100 ■ф = arg W (/©) == arctg ыТ arctg |3) ем поэтому луч, проведенный из начала, координат код углом 1|з к оси абсцисс, пересекает амплитудно-фазовую характеристику в точке, в которой величина со определяется через 1|з согласно (3). Один такой луч показан на рисунке. 52, Построить амплитудно-фазовую характеристику апериодического звена с передаточной функцией W (р) = k 5 (О'Юсвп' Рис. 31. Амплитудно-фазовая х^* рактеристика апериодического звена первого порядка. 1 + Тр 1+0,1/э ' ■ Ответ. См. рис. 31 (а. ф. х. представляет собой полу» окружность). 53. Найти уравнение кривой, представляющей собой амплитудно-фазовую. характеристику дифференцирую-.
54 ГЛ. 2. ЧАСТОТНЫЕ ХАРАКТЕРИСТИКИ ЗВЕНЬЕВ И СИСТЕМ |63 щего звена, изображенного на рис. 32, а. Построить амплитудно-фазовую характеристику звена для случая #, = 40 ком, R2 = 10 ком, С — 2,5 мкф. С ..»-*—CZZ1 \-0 &.\ —0 а) и(ш) б) Рис. 32. Амплитудно-фазовая характеристика дифференцирующего ' звена (случай 2). jV(a» 0 'oo£f *®<аяг\ -и 0=ОО --Е 20ш 2 1 4 1 ъ /uce/f е ■ ■ в I/a \<а=0 Iеceit /rJ- uce/t - Рис. 33. Амплитудно-фазовая характеристика апериодического звена второго порядка. Ответ. Уравнение кривой' имеет вид (1) где Я» Согласно (1) амплитудно-фазовая характеристика является для положительных частот полуокружностью, расположенной в верхней полуплоскости, с центром
BSI § 2.Г. ХАРАКТЕРИСТИКИ ДИНАМИЧЕСКИХ ЗВЕНЬЕВ 55 5 точке (р 2 , /О) и радиусом —?р . Эта характеристика построена для указанных данных на рис. 32, б. 54. Построить амплитудно-фазовую характеристику апериодического звена второго порядка с передаточной функцией ^(p)=YT+r,p)(l + r2P)' если ^ = 8' Г, = 80 и^се/с, Т2 = 12 .мсек. Ответ. См. рис. 33. оисек Рис. 34. Амплитудно-фазовая характеристика колебательного звена. 55. Построить амплитудно-фазовую характеристику колебательного звена с передаточной функцией К при k=l; g = 0,15, T = 0,02 сек. ■ Ответ. См. рис. 34.
56 ГЯг 2. ЧАСТОТНЫЕ ХАРАКТЕРИСТИКИ ЗВЕНЬЕВ. И СИСТЕМ [56 ^6. Построить амплитудно-фазовую характеристику зв-'на с передаточной функцией k W(p)- 'pV + TpV k = 10 сект1, Т = 0,25 сек. ртвет. См. рис. 35; штриховой линией показана асимптота, к которой стремится, а. ф, х. при со—>0. 57. Построить логарифмические амплитудную L(to)= = 201g| W(j&)\ и фазовую %\>(в>) характеристики апериодического звена с передаточной функцией jV(v) Ш Рис, 35. Ам ыитудно- фазоаая характеристика последовательного соединения интегрирующего звена и апериодического звеиа первого порядка. 1 + Тр для двух случаев: а) в виде, пригодном для любых k и Т; б) для k = 100, Т = 50 мсек. Решение. Логарифмическая амплитудная характеристика, со- ■ ответствующая выражению (1), равна Mco) = 201g(W4/co)N = 201g-^=i=. (2) S У 1 + (<оГ)2 Асимптотическая логарифмическая амплитудная характеристика, соответствующая (2), построена на рис. 36, а; по оси абсцисс отложена величина «Г в логарифмическом масштабе, по оси ординат — L (со) в децибелах. л. а. х. имеет согласно (2) излом Слева от излома она горизонтальна Асимптотическая в точке, где соГ = 1. и расположена на высоте 20 lg&, справа от излсГйа она имеет наклон — 20 дб/дек. Точка пересечения характеристики с осью частот, т. е. частота сос среза, определяется из условия fe_ г. k (осТ L (сос) ~ 20 lg —~ — 0 или »с~ ~f
■§" 2.T'. ХАРАКТЕРИСТИКИ ДИНАМИЧЕСКИХ ЗВЕНЬЕВ 57 Наибольшее '' отклонение асимптотической характеристики от точной имеет место при соТ'=' 1 и составляет, как можно найти из выражения (2), 3 дб. При соГ = 0,5 ЦшТ) Ltd». . ф«а>. Об''-— ' •—— ' »■'■'■ ' --— 'г ' 40 го -760 -160 -Ев ^N; ( —^-740 ''"' 7ZO -fO MM 1 1 iH-v--j...._ Mil 1 1—Hi 1 - 700 80 -60 \\ I j 1 sT\ ■ I I 1 1—НЧ 1 .1 i I I 1 -80 -40 -801 JHi i—H-H—I—1 ■■[ II—I—I—Hi—I -го 0 Рис. 36. Логарифмические характеристики устой швлго и неустойчивого апериодических звеиьео к задачам 57 и 58. *• и ■8 0 *х ю г 3 0 41 N? ф* 761 (и» Ш, 780 ""' ■ 700 го j -.. -- Ь(ш) "•n,. 0 401 ^Щ 766 7 71 &^ mm mo. т<\ м <ю - - г".<4 - - - и оГ = 2 отклонение асимптотической характеристики от точной равно примерно 1 дб, а за пределами участка иТ = 1 ± 1 октава это .отклонение ничтожно мало. Фазовая характеристика звена определяется согласно (1) выражением ■ф (©)' = a-rg W (/со) = — arctg «Г. (3)
58 ГЛ. 2. ЧАСТОТНЫЕ ХАРАКТЕРИСТИКИ ЗВЕНЬЕВ И СИСТЕМ [58 В области низких частот -ф (<») —*- 0, в области высоких частот 1|з (со) -> — 90°; при аТ = 1 -ф (о) = — 45°. Из выражения (3) следует также, что фазовая характеристика симметрична относительно точки <оГ = 1, -ф == — 45°. Фазовая характеристика апериодического звена с передаточной функцией (1) построена согласно (2) в виде 1|)(<йГ) в приложении 3. При построении использована следующая таблица: е>Т град 0 0 0,05 —2=50' 0,1 —5-40/ 0,2 — П-20' 0,5 —26°30' 1 —45 2 —63°30' 5 —78-40' 10 —84-20' 20 ч —87-10' оо —so- Логарифмические амплитудная и фазовая характеристики звена с передаточной функцией W(p) = 100 1 + 0,05р (4) построены на рис.- 36, б; штриховой линией показана точная амплитудная характеристика в части, где она не совпадает с асимптотической, построенной согласно формуле (2). По оси абсцисс отложена частота о в логарифмическом масштабе, по оси ординат — децибелы и градусы. 58. Построить логарифмические амплитудную и фазовую характеристики неустойчивого апериодического звена с передаточной функцией W(p) = - 100 1 + 0,05/э ' Ответ. Амплитудная характеристика /,(©) та же, что и для устойчивого звена с передаточной функцией (4) в предыдущей задаче (см. рис. 36, б). Фазовая характеристика ^>{ч>) дана на рис. 36s б пунктирной кривой. 59. Передаточная функция динамического звена равна
6Ц § 2.1.. ХАРАКТЕРИСТИКИ ДИНАМИЧЕСКИХ ЗВЕНЬЕВ 59 Построить логарифмические амплитудную L{&) и фазовую -ф (со) характеристики звена при Д" = 400 сек-1 для- трех случаев: 1) Г = 25 мсек, 2)Т=5 мсек, 3) Г = 2,5 мсек. Указание. При построении фазовой характеристики целесообразно использовать приложение 3. Ответ. См. рис. 37; индекс при L (о) И1|з(о) означает номер случая. Для первого случая (Т = 25 мсек) пунктиром показана точная амплитудная характеристика. 4 в ю го «о Рис. 37. Логарифмические характеристики к задаче 59. 60. Построить логарифмические амплитудную и фазовую характеристики системы с передаточной функцией ^ w( \ — к == 300 WKP) (\ + ТрУ (Ц-0,025р)2 ' Ответ. См. рис. 38. Из рисунка видно, что при вычерчивании логарифмических характеристик не обязательно строить логарифмическую сетку частот, достаточно лишь сделать соответствующие отметки на оси ч'астот. - * Для нанесения этих отметок обычно используется шкала логарифмической линейки; удобный масштаб имеет шкала кубов малой логарифмической линейки. 61. Построить логарифмические амплитудную и фазовую характеристики колебательного звена с переда-
60' ГЛ. 2 ПЛСТОТНЫЕ ХАРАКТЕРИСТИКИ ЗВЕНЬЕВ И СИСТЕМ [61 точной функцией : ' ft W(p)= 1+'2£Гр+ГУ • (1) • Рассмотреть случаи: 1) характеристики L (аТ) и ty(aT) при k=\ и |=0,ш; 0,10; ...; 0,8; 1,0; 2) характеристики L(a) и -ф(со) при k = 30, | = 0,2, Т = 50 мсек. <p(W. Рис. 38. Логарифмические характеристики к задаче 60. Решение. 1) Частотная передаточная функция, соответствующая (I) при £==!., равна W {jet)-- 1 (1-№) + йГш (2) Из (2) находим логарифмическую амплитудную характеристику
6«S. § 2.1. ХАРАКТЕРИСТИКИ ДИНАМИЧЕСКИХ ЗВЕНЬЕВ 61 и логарифмическую фазовую характеристику: г]) (со 7') = — arctg - Ч<*Т (4) -1 - (со Г)2 - ■По формулам (3) и (4) строим амплитудную- и фазовую характеристики, задаваясь различными значениями £ Lim, as «О 30 го ю о чо -го -зо -10 -so -GO а) - ■ V ■ - ■ О f £т^ 10 ш *ч V ч* 4s \фт> \JDO Рис. 39. Логарифмические характеристики колебательного звена. от 0,05 до 1,0. Эти характеристики приведены в приложении 4. Амплитудная характеристика (3) имеет две асимптоты: L'(co7') = 20Igl=0 при ©7,<1 L" (©Г) = - 20 lgjcof)2 при ©Г > !.} (5)
62 ГЛ. 2. ЧАСТОТНЫЕ ХАРАКТЕРИСТИКИ ЗВЕНЬЕВ И СИСТЕМ [62 Асимптотическая амплитудная характеристика, определяемая выражениями (5), построена на рис. 39, а. Для колебательного звена асимптотическая ампли*. тудная характеристика может очень отличаться от точной, что следует из сравнения рис. 39, а с рисунком, приведенным в приложении. 5. Поэтому для колебательного звена обычно строится точная амплитудная характеристика. Это построение удобно проводить, суммируя Цы),дб Рис. 40. Логарифмические амплитудные характеристики к задаче 63. ординаты асимптотической характеристики с ординатами кривой отклонения At(<o) асимптотической характеристики от точной; такая кривая дана в приложении 5. 2) Характеристики L (со) и яр (со) для звена с передаточной функцией ™, ч 30 30 ,™ w (Р) - j + 2 • 0,2 • 0,05р + 0,0025р2 1 + 0,02р + 0,0025р2 ' V ; построенные с использованием приложений 4. и 5, приведены на рис..39, б. 62. Построить логарифмические амплитудную и фазовую характеристики неустойчивого колебательного звена с передаточной функцией W M = 1-2|7> + Т2р2 при k = 30, Т = 50 мсек, £ = 0,2.
84] $ 2.2. АМПЛИТУДНО-ФАЗОВЫЕ ХАРАКТЕРИСТИКИ 63 I " Ответ. Амплитудная характеристика совпадает с L (со) устойчивого колебательного звена в предыдущей задаче, имеющего передаточную функцию (6) (см. рис. 39, б). Фазовая характеристика отличается от -ф (со) для звена с передаточной функцией (6) только знаком. 63. На рис. 40 изображены асимптотические логарифмические амплитудные характеристики минимально- фазовых звеньев. Найти передаточные функции этих звеньев. Ответ. Wi(P)= (1 + т,рИ1 + г2р) » ft= ЮО,r,=J00леек,Га=8лссе/с; Wo kp '1 + Гр' & = 0,025 сек, Т = 0,15 сек. § 2.2. Амплитудно-фазовые характеристики разомкнутых систем автоматического регулирования 64. Система автоматического регулирования имеет структурную схему, показанную на рис. 41; ЧЭ — чувствительный элемент, Д—двигатель, Р—редуктор. Передаточная функция разомкнутой системы равна W(p)- К p(l + TlPy(l + T2p) Построить амплитудно-фазовую характеристику системы при К = 400 сек'1, Гг= 80 мсек, 7,2=12 мсек. Ответ. Амплитудно-фазовая характеристика может быть построена по приведенным в таблице значениям модуля Л (со) и аргумента -ф (со) частотной передаточной функции W (/со) = А (со) е'Ф «°>. е>,сек А (ну) град 0 оо -90 2 196 -100 5 74 -115 10 31 -135 20 10,3 -162 50 1,66 -197 100 0,319 -223 300 0,015 -252 ОО 0 -270
64 ГЛ. 2. ЧАСТОТНЫЕ ХАРАКТЕРИСТИКИ ЗВЕНЬЕВ И СИСТЕМ 165 65. Система автоматического регулирования имеет структурную схему, показанную на рис. 41. Передаточная функция разомкнутой системы имеет, вид т = Ч (\\ Найти такой способ изображения амплитудно-фазовых в, **= W ■ Д Рис. 41. Структурная схема к задачам 64 и 65. характеристик, который позволил бы охватить случаи различных сочетаний параметров К, Ти Т2' системы. Решение. Представим выражение (1) в виде У(Р)?^,^Л',.,^-Г^> (2) Т1р-Ц+Г1р){1+-аТ1р).' где а — Частотная передаточная функция, соответствующая выражению (2), имеет вид . = KTlU0(Tl<i>) + jKTlV0pl<a). #) Задавшись последовательностью близких друг к другу значений a-=TjTi от а = 0 до й=1, можно построить семейство амплитудно-фазовых характеристик, практически охватывающих все возможные варианты систем с передаточной функцией (1). , На рис. 42 построено семейство подобных амплитудно-фазовых характеристик для а = 0; 0,2; 0,4; 0,6; 0,8. Построение выполнено на основе выражения (3) по его модулю и аргументу для различных значений частоты;
§ 2.2. АМПЛИТУДКО-ФАЗОВЫЕ ХАРАКТЕРИСТИКИ 65 по осям координат отложены величины U0 (ш) = (/СГ,)"'1 U («О- ^ и У0 (со) = (KTf1 V (©). ■ Переход к характеристике, соответствующей определенному значению /(Г,, осуществляется умножением чисел, отложенных по осям координат, на величину КТ\. Jw,vm Um wT,~o.2^ wT,=D.1 - wT^O.OS T -10 ■is -го Рис. 42. Универсальные амплитудно-ф'азовые характеристики к задаче 65. Интерполирование позволяет легко получить амплитудно-фазовые характеристики систем, для которых величины а = TJTi отличаются от приведенных на рис. 42. 66. Построить амплитудно-фазовые характеристики двух' систем, . имеющих . в .разомкнутом состоянии
66 ГЛ- 2. ЧАСТОТНЫЕ ХАРАКТЕРИСТИКИ ЗВЕНЬЕВ И СИСТЕМ 467 передаточные функции: a) Wl{p) = 20 б) №2(р) = Р (1 + 0,1р) * 200 р(1+0,05р)(1+0,02р) Указание. Возможно использование готовых кривых из предыдущей задачи. Ответ. См. рис. 43. Ufa» -1.5 -1.0 jVtw) -0.5 /ZUcek tin— I шсек f 'сек I г 1 f<o-oce/( 0 Urn w=oo -0.5 -1.0 -1,5 -2.0 -Z.S --3.0 a) 6) Рис. 43. Амплитудно-фазовые характеристтш к задаче 66: а) кривая для первой системы, б) для второй системы. 67. Построить амплитудно-фазовую характеристику системы, блок-схема которой дана на рис. 44; СЛ9 —чувствительный элемент, Д — двигатель, Р — редуктор. Передаточная функция разомкнутой1 системы имеет вид W(p) = Kd + TiP) 500(1-Ю.ОЗр) р (I + Г,р) (1 + Tsp) р.(1 +0,1р) (1 +0,006р)
ев] § 2.2. АМПЛИТУДНО-ФАЗОВЫЕ ХАРАКТЕРИСТИКИ Ответ. См. рис. 45. А 68. Найти уравнение кривой, представляющей собой амплитудно-фазовую характеристику системы, имеющей в, \Р чэ •^т}— Д Рис. 44. Блок-схема к задаче 67. Jl/m Ol/(uii jVm iw -го *Щ / ' 1'исек / 1 ю'Зсёй п(!а -го -40 -Б0 -во -100 р вг Рис. 45. Амплитудно-фазовая характеристика к задаче 67. следующую передаточную функцию: Построить амплитудно-фазовую характеристику для случая /С= ЮО сект2 и Т =«0,2 сек. Решение. Частотная передаточная функция равна K(l+jTe>) №(/*>) = £/(©) +/У (©),
68 ГЛ. 2. ЧАСТОТНЫЕ ХАРАКТЕРИСТИКИ ЗВЕНЬЕВ И СИСТЕМ [69 где Из (1) находим кт (О* (1) (2) Согласно (1) н (2) амплитудно-фазовая характеристика представляет собой для положительных частот ветвь параболы, лежащую в третьем квадранте комплексной плоскости. jvm -ff-f-rrf-f-ft-f-f-f-f Ol/ra» Рис. 46. А. ф. х. в виде параболы к задаче 68. Точка амплитудно-фазовой характеристики, соответствующая какому-либо значению частоты &, легко определяется как точка пересечения параболы с лучом, проведенным из начала координат и составляющим с осью вещественных чисел угол - ^^arctg-gJ^l.^ — зт + arctg со7\ Амплитудно-фазовая характеристика для заданных параметров построена на рис. 46. 69. Передаточная функция разомкнутой системы равна W(D) _ -*(1+0.15р) Построить ее амплитудно-фазовую характеристику при К = 50 сект2 и К = 200 сект2. Ответ. См. рис. 47. ..
•69] § 2:2. АМПЛИТУДНО-ФДЗОВЫЕ ХАРАКТЕРИСТИКИ 69 ^ * \дит Рис. 47. Амплитудно-фазовые характеристики к задаче 69. Шкала /—для К = 50, шкала //—для Я=200. jV(a» jV(o» $ V ,s i -15 А -ЮО 6 -SO ^^•<2 -to1^ -so -zs It- \\p Um» \bj-00 -/ -as S3 f A w- 0.5- 1 1^ -O.S- Б 0.5 0,25 0 l/fitf) w=co -0,25 Рис. 48. Амплитудио-фазовые характеристики к задаче 70. Шкала А - для К = 200 сек~3, шкала Б - для К = 100 се/Г3,
70 ГЛ. 2. ЧАСТОТНЫЕ. ХАРАКТЕРИСТИКИ ЗВЕНЬЕВ И СИСТЕМ [70' 70. Построить амплитудно-фазовую характеристику системы с передаточной функцией в разомкнутом состоянии W \Р) рз(1+0,04р) при К = 200 сек"3 и К=* ЮО сек~\ Ответ. См, рис. 48. 71. На рис. 49 приведена цепь тахометрической обратной, связи с пассивными корректирующими контурами; ¥* тг J—Z Рис; 49. Схема к задаче 71. jVfai) 8 . 'сек 1 /6 '' 4 \ Z 1 I Г^ '3 -2 -10 -г -4 Г ьсек - - w~0 г СО'ОО - Ws? # в б 1 1 \в-!- \°сен f Щит h-L- J' ce/t /jo-J- Л JO* Рис. 50. А. ф. х. цепи тахометрической обратной связи к задаче, 71. ТГ — тахогенератор. Построить амплитудно-фазовую характеристику этой цепи, если ее передаточная
74j <§ .2.2. АМПЛИТУДНО-ФАЗОВЫЕ ХАРАКТЕРИСТИКИ функция равна Кр2 71 №(p)- (1 + Г1Р)г(1 + Г2р) ' Я = 4 в-сек2/град, Г, =0,5 сек, Т2 = 0,1 сек. Ответ. См. рис. 50, где числа, отложенные вдоль осей, имеют размерность в/град. 72. Построить амплитудно-фазовую характеристику цепи с передаточной функцией W(p)- р(-1 + Тр) 100 р(-1+0,1р) Ответ. См. рис. 51. 73. Построить амплитудно-фазовые характеристики систем с передаточными функциями (А) Щр) = ( - 1 + 2Г,р + Г2р2) (l + Г2р) ' (Б) W(p)~ К (1 + Тф) ( - 1 + 2Г ,р + Т2р2) (l + Т2р) (1 + Т,р) -в -7 -6 -5 -и -з -г -1 о um Рис. 51. А. ф. х. системы с неустойчивым звеном. для случая К — 5, 7\ = 0,1 сек,. Т2 = 0,05 сек, Г3 == 0,03 сек, Т4 = 0,006 сек. Ответ. См. рис. 52. 74. Блок-схема системы гироскопической стабилизации, разомкнутой на входе датчика угла прецессии, может быть представлена [4, 5] в виде, показанном на рис. 53; ДУП — датчик угла прецессии, Д — двигатель, Р — редуктор, Г —гироскоп. Передаточная функция разомкнутой системы может быть . при безынерционном
ГЛ. 2. ЧАСТОТНЫЕ ХАРАКТЕРИСТИКИ ЗВЕНЬЕВ И СИСТЕМ [74 ОUiiui Рис. 52. Амплитудно-фазовые характеристики к задаче 73. $* ДУП д ось стабилизации Ф Ось прецессии Рис. 53. Блок-схема системы гироскопической стабилизации к задачам 74 и 75. ■ Ч ''? ~S. 'I Jiff l/rwl а I -Й5 -10 -1.5 Рис. 54. Амплитудно-фазовые характеристики гироскопических систем.
76j § 2.3. ВЕЩЕСТВЕННЫЕ ЧАСТОТНЫЕ ХАРАКТЕРИСТИКИ 73 » ч усилителе записана в виде К WM = р(1+2%Тр+Т2р2) • Построить амплитудно-фазовую характеристику этой системы при К = 20 сек'1; | = 0,15, Т2 = 0,02 сек. Ответ. См. кривую Л на рис. 54. 75. Система гироскопической стабилизации, блок- схема которой дана на рис. 53 (см. также задачу 74), имеет при инерционном усилителе следующую передаточную функцию в разомкнутом состоянии: W(P) ' К р(1 + Т1Р)(1+21Т2р+тУ)' Построить амплитудно-фазовую -характеристику этой системы при К = 20 сек"1, 7Л = 0,2 сек, Г2 = 0,02 сек, £ = 0,15. Ответ. См. кривую Б на рис. 54. § 2.3. Вещественные частотные характеристики замкнутых систем автоматического регулирования 76. Построить вещественную частотную характеристику Р(а) замкнутой системы автоматического регулирования. Передаточная функция разомкнутой системы ^)-7aW' (I) К = 20 сек~\ Г =0,1 сек. Решение. Вещественная частотная характеристика строится по точкам. Эти точки могут быть найдены различными способами. а) Вещественная частотная характеристика Р(а>) может быть построена по ее аналитическому выражению Р (<d> = Re [Ф (/©)], (2) где Ф(/со) —частотная передаточная функция замкнутой системы, равная
74 ГЛ. 2. ЧАСТОТНЫЕ ХАРАКТЕРИСТИКИ ЗВЕНЬЕВ И СИСТЕМ [78 Согласно (3) и (1) получаем Ц®' ~ (К - Гю2)2 + о2 1 {К - Гю2)2 + ю2 ' Из (4) и (2) находим /((/С-Гсо2) Р(©) = 20 (20-0,1 со2) (iC - Гю2)2 + со2 (20 - 0,1е>2)2 + и2 * (4) (5) Подставляя в (5) различные значения to, получаем табл. 1 для построения Р (to): Таблица 1 сек~ />(<й) 0 1,00 5 1,06 7 1,08 10 1,00 15 —0,23 18 —0,25 20 —0,50 25 —0.35 30 —0,24 40 —0,15 50 —0,08 60 —0,06 оо 0 По данным табл. 1 на рис. 55, а построена вещественная частотная характеристика. б) Если для ряда значений частоты со имеются координаты U (to) и V («>) точек амплитудно-фазовой Таблица 2 и, сек ' V(«) 0 — оо — оо 5 -1,60 -3,18 7 -1,34 -1,93 10 -1,00 -1,00. 20 -0,41 -0,21 40 -0,13 -0,02 ОО О' 0 характеристики разомкнутой системы (табл. 2), то соответствующие значения Р(<о) можно найти по, формуле ГЛ{а> [1 + <7(<о)]2+К2(<о) (6) В случае, когда координаты точек амплитудно-фазовой характеристики заданы в виде модуля Л(<о) и аргумента "ф(«>) (табл. 3) частотной передаточной функции W (/to), характеристика Р (о) может быть построена по формуле Р ((-Л = А* (<°) + А Ф) cos ^ (ta> (7\ к ' А*((а) + 2А (со) cos iM«) + ! ' . - кч
7eJ § 2.3.- ВЕЩЕСТВЕННЫЕ ЧАСТОТНЫЕ ХАРАКТЕРИСТИКИ 75 Таблица 3 —I и, сек- А (а) •ф(со), град 0 оо -90 5 3,56 -116 7 . 2,34 -125 10 1.41 -135 20 0,448 -154 30 0,211 —162 40 0,121 -166 50 0,078 -169 60 0,054 -170 ОО 0 —180 получаемой из формулы (6) при подстановке £/8(ю) + У2(©) = Л2(©) и £/(«>) = Л (со) cos ij)(fi>)- в) Если имеется амплитудно-фазовая характеристика разомкнутой системы, то для построения Р(а) удобно использовать формулу п([л— А* (<*>) +U {а) (т получающуюся из (6) при подстановке Л2(ю)= f/2(co)-}-' И- V2(«>) и В2((о) = [1 + С/(со)]2+ УЦа). Величины Л(ю) jV<co) О U<co) 5 70 \75 га 25 30 35 4/0 45 50 55 Щщ} -0.6- а) Рис. 55. Вещественная частотная характеристика к задаче 76. и В (ю) для каждой заданной частоты ю легко получить из амплитудно-фазовой характеристики, так как Л(о>)
76 ГЛ. 2. ЧАСТОТНЫЕ ХАРАКТЕРИСТИКИ ЗВЕНЬЕВ И СИСТЕМ [77 представляет собой модуль вектора W(j(a), т. е. расстояние от начала координат до заданной точки k характеристики, а В ((а) — расстояние от точки (— 1, /0) до точки k (рис. 55). Числа, входящие в формулу (8) и необходимые для построения вещественной частотной характеристики, приведенной на рис. 55, а, можно получить из амплитудно-фазовой характеристики системы с передаточной функцией (1), изображенной на рис. 43, а. . г)- Для построения вещественной частотной характеристики системы по имеющейся амплитудно-фазовой характеристике (рис. 43, а) можно воспользоваться номограммой, называемой вещественной круговой диаграммой. Такая номограмма дана в приложении 11. < сен 50 Ю0/50\ 200 250 300 350 Ш7 450 500 Рис. 56. Вещественная частотная характеристика к задаче 77. 77. Построить вещественную частотную характеристику Р(о) замкнутой системы автоматического регулирования, если передаточная функция разомкнутой си- XV7I . 500(1+0,03р) стемы ^(/>)=p(1+o,ip)(,+o,o06p)- 'ч at сек" А (о) ■ф(со), град 0 оо -90 10 36,8 -122 20 13,0 -130 30 6,85 -130 40 4,6 -130 50 3,38 -130 100 1,35 -133 Та 200 0,488 -136 б л и 500 0,095 -165 ц а 4 оо 0 -180
781 § 2.3. ВЕЩЕСТВЕННЫЕ ЧАСТОТНЫЕ ХАРАКТЕРИСТИКИ 77 - При построении Р (со) может быть использована амплитудно-фазовая характеристика системы, приведенная на рис. 45, либо табл. 4 величин модуля Л (со) и аргумента ф(<о) частотной передаточной функции системы. Ответ. См. рис. 56 и задачу 76. 78. Построить вещественную частотную характеристику Я (to) замкнутой системы. Амплитудно-фазовая ■Q54 ** W Ufa» Рис. 57. Амплитудно-фазовая,- и вещественная частотная характеристики .к задаче 78. характеристика разомкнутой системы-дана на рис. 57, а. При построении могут быть использованы данные табл. 5. Ответ. См. рис. 57, б и задачу 76. Таблица 5 ю. сек М<о) ty[(o), град 0 оо —180 2 10,32 -175 4 2.80 —172 7 1.05 —172 10 0.58 —177 15 0,28 —188 20 0,16 -199 ОО 0 -180
78 ГЛ. 2. ЧАСТОТНЫЕ ХАРАКТЕРИСТИКИ ЗВЕНЬЕВ И СИСТЕМ 179 79. Построить вещественную частотную характеристику замкнутой статической системы. Амплитудно- фазовая характеристика разомкнутой в виде кривой Б на рис. 52. Ответ, См. рис. 58 и задачу 76. системы дана Рис. 58. Вещественная частотная характеристика к задаче 79. WOwl7fa, Рис. 59. Вещественная частотная характеристика к задаче 80. 80. Построить вещественную частотную характеристику замкнутой статической системы. Амплитудно- фазовая характеристика разомкнутой системы совпадает с' приведенной на рис. 33. Ответ, См. рис. 59 и задачу 76.
82] § 2.4. ЛОГАРИФМИЧЕСКИЕ ХАРАКТЕРИСТИКИ 79 81. Построить вещественную частотную характеристику замкнутой системы с астатизмом третьего порядка. Рис. 60. Вещественная частотная характеристика к задаче 81. Амплитудно-фазовая характеристика разомкнутой системы дана на рис. 48 (шкалы с буквой А). Ответ. См. рис. 60 и задачу 76. § 2.4. Логарифмические характеристики систем автоматического регулирования . 82. Построить логарифмические амплитудную и фазовую характеристики системы с передаточной функцией 40 ^' I+0,12р + 0,002р2 (1) Решение. Для построения логарифмических характеристик следут разложить знаменатель (I) на два сомножителя.
80 ГЛ. 2. ЧАСТОТНЫЕ ХАРАКТЕРИСТИКИ ЗВЕНЬЕВ И СИСТЕМ [82 Для этого определяем корни знаменателя, которые ■50 сек-1, и пред- 40 оказываются равными —10 сек * и ставляем (1) в виде W{p) = () + Тф) (J + вд = {1 +01р) (1 +0,02р) • <2) Отсюда находим логарифмическую амплитудную характеристику системы 40 L(co) = 201g|(1+Alg()(1 + .0>02g() = 20lg- 40 /Ц + (0,1с))2][1 + (0,02ю)2] " * ' Из выражений (2) или (3) следует, что асимптотическая л. а. х. имеет два излома; в точках щ — 1/F, = Ышдб ' - ф(а>), чо j | | 110 | ТТЛ j гт~п I ТТЛ Iград 30 го ю о -JO -го -зо -ио -so о - -^^^ f% > 4> $? г %? # ч Y Хз ■ -ISO -wo -mo -120- -wo -во -во -40 -го Рис. 61. Логарифмические характеристики к задаче 82. о,2 том i г 4>в iff го 40\Ш т zoo таютощ^ — 10 сек'1 и со2= 1/Г2 = 50 секГ1 и состоит из трех участков: горизонтального, проведенного на высоте 201g40 = — 32 дб; участка с наклоном —20 дб]дек; участка с на-
щ § -2.4. ЛОГАРИФМИЧЕСКИЕ ХАРАКТЕРИСТИКИ 81 клоном —40 дб/дек. Эта асимптотическая характеристика представлена на рис. 61. Так как отношение Т1/Т2 = 5, т. е. превышает две октавы, то из решения задачи 57 следует, что отклонение асимптотической амплитудной характеристики от точной в районе каждого излома имеет такой же вид, как и для апериодического звена и не превышает 3 дб. Фазовая характеристика имеет вид ■ф (са) = — arctg 0,1 со — arctg 0,02(а. (4) Последнее выражение позволяет построить -ф(со) по точкам. Однако проще построить гр(са) как сумму ординат фазовых характеристик ^(ю) и ^(сэ) двух апериодических звеньев с постоянными времени Ту = 1 сек и Г2 = 0,2 сек, так как каждая из этих характеристик легко строится при помощи графиков приложения 3. Фазовая характеристика ф(<о) системы дана на рис. 61. 83. Построить логарифмические амплитудную и фазовую характеристики системы с передаточной функцией W{p)= 12*5 р(1+0,004р + O.000V) Указание. Передаточную функцию следует привести к виду, удобному для построения логарифмических характеристик, т. е. найти, соответствует ли полином второй степени в знаменателе двум апериодическим звеньям или он соответствует колебательному звену, и определить необходимые параметры этих звеньев. Ответ. См. рис. 62. " 84. Система автоматического регулирования, блок- схема которой построена по образцу, представленному на рис. 44, имеет в разомкнутом состоянии передаточную функцию 1 W(P): К(1 + Т2р) р (1 + Г,р> (1 -Ь ГэР) (1 + Г4р) К(1+0,017р) ^ч р (1 + 0,05р) (1 + 0,0025р) (1 + 0,001р)
82 ГЛ. 2. ЧАСТОТНЫЕ ХАРАКТЕРИСТИКИ ЗВЕНЬЕВ И СИСТЕМ (в« Построить логарифмические асимптотическую амплитудную и фазовую характеристики системы для двух зна^ чений коэффициента усиления: К = 500 сек~1 и К = = 2000 ceir1. Решение. Частотная передаточная функция, соответствующая (1), имеет вид К (I + j0,0l7(o) И?(/<Ф (2) /о (1 + /0,05©) (1 + /0,0025(й) (1 + /0,001ю) ' Из выражения (2) или из выражения (1) видно, что асимптотическая амплитудная характеристика имеет вид ог wots t г и в т го 4В6В wo гоо meoowJL Con Рис. 62. Логарифмические характеристики к задаче 83. ломаной прямой с участками, имеющими отрицательный наклон 20—40 — 20—40—60 дб/дек, и с изломами в точках <о, = 1/Г,. = 20 сек'-1, со2 = \/Т2 = 59 сект1, са3 = 1/Г, = = 400 сек"1, са4 = 1/7"4 = 1000 сек"1; первый участок характеристики является частью прямой с наклоном — 20 дб/дек, пересекающей ось частот в точке и> = К- . Асимптотические амплитудные характеристики L^ca) для случая /С=500 сек'1 и L2(v>) для случая /С =^2000 сек"1 изображены на рис. 63. Фазовая характеристика для обоих случаев совпадает и, согласно. (1) или (2), может быть найдена как
g8j § £.4. ЛОГАРИФМИЧЕСКИЕ ХАРАКТЕРИСТИКИ 83 сумма ординат фазовой характеристики -ф0(сй) идеального интегрирующего звена, фазовых характеристик ф, (со), "фз(ю) и яр^(сй) апериодических звеньев с постоянными времени Ть Т3 и Г4. и ^(ш) -дифференцирующего звена с постоянной времени Т2- 2 U 6 10 20 4060100 200400600/000 W00 10000 аг._4 Рис. 63. Логарифмические характеристики к задаче 81. - Указанные фазовые характеристики звеньев и результирующая фазовая характеристика -ф(со) всей системы построены на рис. 63. 85. Построить логарифмическую асимптотическую амплитудную характеристику L(v>) и логарифмическую фазовую характеристику г^(со) системы с передаточной
84 .. ГЛ. 2. ЧАСТОТНЫЕ ХАРАКТЕРИСТИКИ ЗВЕНЬЕВ И СИСТЕМ [86 функцией W(p) = ' K(l + TlP) при К — 75 сек~2, Tt = 200 мсек, Т2 — 25 мсек, Т3 — 6 мсек. Ответ. См. рис. 64. Urn, Ш w го о (-го ;~40 '-60- \ 1 \ ^ >s N S Ч"^[ ч ч< V ч t. • ■ - - - град -280 -2Б0 -240 -гго -zoo -wo -wo -HO -120 -wo -80 -BO аг mas 1 г и в ю го шзвоюо гоошвоош,^. ■Рис. 64. Логарифмические характеристики к задаче 85. 86. Построить логарифмическую амплитудную и фазовую характеристики системы с передаточной функцией /С(1 + Г,р)2 Л-(1+0,25р)2 W{p) = Ръ (I + Т2р) (I + Т3р) р3 (1 + О.ОЗр) (1 + 0,008р) для трех случаев: 1) /С = 250 секГъ\ 2) /С = 75 сек~3; 3) К. = 1000 сек~г. Ответ. На рис. 65 показаны асимптотические, амплитудные характеристики L, (се), L2(a>) и 13(са), причем индекс соответствует номеру случая; для случая 1 пунктиром показана точная амплитудная характеристика. Фазовая характеристика if (©) для всех случаев одна и та же. 87. Построить логарифмические амплитудную й фазовую характеристики системы со следующей передаточной
>■ 40 го .0 -го -40 -60 \ Ч\ - 1 V4, ** л. $ с ■ \ • - - . ■ gpaff -260 -2W -220 -200 -160 -160 -U0 -120 -100 -во -во од 0,406 1 г 4 6/0 20 4060 ЮО гОО 4006001000 w,'^ Рис. 65. Логарифмические характеристики к задаче 86. Lfai), Об во 60 40 20 О -20 -40 -ВО Й| «Р % ((А, 4 - л й 1 \ V\ V с* с \ СУД |"Л ■ - - - ■ ■ • ерав -340 -320 -300 -2в0 -260 -240 -220 -гоо -то -160 -140 -120 -то -во -60 Ц2 0.40,6 1 2 4 6 10 20 4060100 200 Рис. 66. Логарифмические характеристики к задаче 87. сея
86 ГЛ. 2. ЧАСТОТНЫЕ ХАРАКТЕРИСТИКИ ЗВЕНЬЕВ И СИСТЕМ 188 функцией: W(p) 20 р (1 + 0.104Р + 0,0008р2 + 0,0004р3) ' Указание. Знаменатель передаточной функции следует разложить на множители, чтобы привести W{p) к виду, удобному для построения логарифмических характеристик. Ответ. Амплитудная L(ca) и фазовая -ф(со) характе~ ристики построены на рис. 66. Ш»,дб го -20 -40 -60 -80 -100 -120 UJ ■b," *t\ 0 *,* I i^^T-^ С r ** -'Л \ '/ / * \ N\ V n V - - - - ф(Щ -160 -wo -no -120 -100 -80 -60 -40. -20 20 40 60 80 0,2 0,10,6 1 2 t6 10 20 40 60100 200 4006001О00о>,^ Рис. 67. Логарифмические характеристики к задаче 88. 88. Построить логарифмические амплитудную и фазовую характеристики системы с передаточной функцией Г(р) = КР (1 + 7»(1+2|Г2р+7|р2) при К = 0,0645 сек; Г, = 30 мсек; Тг = 7 мсек; I = 0,2. Ответ. См. рис. 67.
§ 2.4. ЛОГАРИФМИЧЕСКИЕ ХАРАКТЕРИСТИКИ 87 89. Построить логарифмические амплитудно-фазовые характеристики системы с передаточной функцией _-, , К(\ + Т2р) W W _ р (1 + Тф) (1 + Т3р) (1 + 7>) ~ р(1 + 0,05р) (1 + 0,0025р) (t + O.OOlp) для двух случаев: 1) К = К\ = 500 се/Г1; 2)^ = ^2 = = 2000 сек-1. цШ.град , ^# Л #7 #7 Д? 1QO 120 740 160 180 Uw), дб 40 го о -го -40 -во -80 -tea-wo -no'120-m -во -во -4о -го о -гоо -гго-яю-ш-гво ф(и», град Рис. 68. Логарифмические амплитудно- ■ фазовые характеристики к задаче 89. Решение. Для построения логарифмической амплитудно-фазовой характеристики 201g| W(j(i>)\ = f[ty(ja>)] предварительно строятся логарифмические амплитудная и фазовая характеристики системы. Используя эти 400 го, to/ 1 / J 4 10/ {' /10 *20 m\W гоо/! ' У\]Ю <600l2od Щ£ф* si v юоо\ i \ K' Ч1 •N ? \4000 \4 4000\ * \\ lOOOO l\', 70000\\
83 ГЛ. 2. ЧАСТОТНЫЕ ХАРАКТЕРИСТИКИ ЗВЕНЬЕВ И СИСТЕМ [»0 характеристики £|(со) и -ф(со), изображенные на рис. 63 (см. задачу 84), строим по точкам логарифмическую амплитудно-фазовую характеристику для случая K = Ki = 500 секгК Эта характеристика представ- 0 7](ш),гра9 Z0 «О ВО 80 100 120140 160180 60 Ш), дб «0 го о -20 -40 -60 iW 1 11 ^ 40 ОТ V 0,6 г V \ Г40 А ч 200 S® \Ю .200 \ \ \ N 1^ 2 \ \В00 600^{ о -180-160-МО-120-100-80 -60 -40 -20 -200-220-240-260-280 ф«1», град Рис. 69. Логарифмические ампли- тудно-фазоиые характеристики к задаче 90. лена на рис. 68 (кривая /). Числа возле отметок на кривой указывают соответствующие значения частоты са в сек"1. Высокочастотная часть кривой, для которой ■ф(са) < — 180°, заменена ее зеркальным отображением в оси ординат. Для этой части кривой, показанной на рисунке пунктиром, на оси абсцисс предусмотрена дополнительная шкала углов от —180 до —280°. На рисунке имеется также шкала запаса по фазе, равного т}(со) = = ф(со)+180°. Для случая К = Ко = = 2000 сек~х аналогичная кривая может быть построена переносом всех точек кривой / на 12 дб BBepx(201g/C2//C,= 12a6), см. рис. 68, кривая 2. ■ 90. Построить логарифмические амплитудно-фазовые характеристики системы с передаточной функцией K(l + TlP) . АГ(1+0,2р) W{p) P2{i + T2p)(l + Т3р) р2 (1 + 0,025р) (1 + О.ООбр) для двух случаев: 1) /С = 75 сек~2; 2) К = 400 сек~2. Указание. Возможно использование решения задачи 85. Ответ. См. рис. 69, где кривая / относится к первому случаю, а кривая 2 — ко второму.
ГЛАВА 3■ УСТОЙЧИВОСТЬ ЛИНЕЙНЫХ СИСТЕМ § 3.1. Алгебраические критерии устойчивости 91. Характеристическое уравнение системы имеет вид />3 + р2 + 2р+1=0. Определить устойчивость системы. Ответ. Система устойчива. 92. Характеристическое уравнение системы имеет вид 5р3+2р2-Зр+1 = 0. Определить устойчивость системы. Решение. Система неустойчива, так как не выполняется необходимое условие устойчивости. 93. Передаточная функция разомкнутой системы ун' р (1 + Тр) Определить условия устойчивости замкнутой системы. Ответ. К>0, Т>0. 94. Передаточная функция разомкнутой системы где К= 100 сект2. Определить устойчивость замкнутой системы. Ответ. Замкнутая система находится на границе устойчивости. 95. Определить устойчивость замкнутой системы, если передаточная функция разомкнутой системы имеет вид tF(p)=p2(1 + rp).
90 ГЛ. 3. УСТОЙЧИВОСТЬ ЛИНЕЙНЫХ СИСТЕМ 196 где К = 20 сек'2 — добротность системы по ускорению, Т = 0,01 сек — постоянная времени. Ответ. Замкнутая система структурно неустойчива, т. е. неустойчива при любых значениях К и Т ф 0. 96. Структурная схема системы приведена на рис. 70. Коэффициент усиления разомкнутой системы К>0, постоянная времени Г>0. Определить устойчивость ■*-у разомкнутой системы и условие устойчивости замкнутой системы. ^-HgH_/ IP. ь ~, п „ „ Ответ. Разомкнутая си- Рис. 70. Структурная схема „ J„ системы к задаче 96. стема неустойчива. Замкнутая система устойчива при К>1. 97. Характеристическое уравнение системы имеет вид (fe, г k2) р3 + ахр2 + а2р + а3 = 0, где k\ = 25 сек3, k2 = 25 сек3, ах — 10 сек2, а2 = 5 сек, а3 = 25. Определить устойчивость системы. Решение. Коэффициент при старшем члене характеристического полинома a0~kl — k2. При kx — k2<0 система неустойчива, так как не выполняется необходимое условие устойчивости. При aQ = k{ — k2 > 0 и при выполнении условия а^а? — а0а3>0 (см. приложение 6) система устойчива. В данной задаче a0 = k{ — k2 = 2b — — 25 = 0. Система находится на границе устойчивости. 98. Характеристическое уравнение системы имеет вид а0р4 + atp3 + а2р2 + а3р = 0, где й0=10 сек4, а, = 5 сек3, а2 = 2 сек2, а3=10 сек. Определить устойчивость системы. Решение. Характеристическое уравнение системы запишем в следующем виде: (а0р3 + ахр2 + а2р + ай) р = 0. (1) Из (1) видно, что один из корней характеристического уравнения равен нулю. Система будет находиться на границе устойчивости, если все остальные корни характеристического уравнения лежат в левой половине
Ю21 § 3.1. АЛГЕБРАИЧЕСКИЕ КРИТЕРИИ УСТОЙЧИВОСТИ 9] плоскости корней. Для этого должны выполняться условия устойчивости для полинома а0р3 + а,р2 + а2р + а3, которые имеют вид а0>0, щ>0, а2>0, а3>0, а1а2>а0а3- Для значений коэффициентов а0, ..., а3, принятых в задаче, последнее неравенство не выполняется. Поэтому система неустойчива. 99. Решить предыдущую задачу для следующих значений коэффициентов: а) а0=10 сек4, «| = 5 сек3, а2 = 2 сек2, а3 = i сек; б) а0=10 сек4, а(=5 сек3, а2 = 2 сек2, аз = 0,5 сек. Ответ, а) Система находится на границе устойчивости; б) система находится на апериодической границе устойчивости. 100. Передаточная функция разомкнутой системы имеет вид ^^"Vu + r.pMi + TVp) ' где Д" = 50 сек~2 — общий коэффициент усиления разомкнутой системы; Т{ = 1 сек, Т2 — 0,05 сек — постоянные времени. Определить устойчивость замкнутой системы. Ответ. Система структурно неустойчива, т. е. неустойчива при любых значениях общего коэффициента усиления разомкнутой системы К и постоянных времени Г| Ф 0 и Т2 Ф 0. 101. Передаточная функция замкнутой системы автоматического управления имеет вид где К = 50 сект1, Тг = 0,2 сек, Т2 = 0,2 сек. Определить устойчивость системы. Ответ. Система неустойчива. 102. Решить задачу 101, если К = 50 сек*1, Т{ = 0,1 сек » 7-2 = 0,02 сек. Ответ. Система устойчива.
ГЛ. 3. УСТОЙЧИВОСТЬ ЛИНЕЙНЫХ СИСТЕМ. [103 103. Движение автоматической системы описывается следующей системой дифференциальных уравнений: ф - Q (vo + v) = c2Yo + At Yo + Y + H> = -CiYo + *2. Y + йя|з = -ky (Yi - Y«) + 63. (1) где Y« ~~ задающее воздействие; 61, At 63— возмущающие воздействия; Yit Yot Ф ~~ координаты системы; Q = 1,16 • Ю-3 сек'1, kt = 10 cetf1, clt c2— коэффициенты. Определить условия устойчивости системы. Решение. "Заданием систему дифференциальных уравнений (1) в символической форме: pty-Q (yo + y) = C2Y0 + *i. />Yo + PV + Qty = - C1Y0 + A. py + Qy = — fei(Y-Y4) + A» (2) где р — символ дифференцирования. Характеристический полином автоматической системы равен определителю системы уравнений (2): £>(/>) = Д(/Ф -Q —(Q + c2) p р p + ci Q р + 6, ~0 Q Характеристическое уравнение системы: р3 + (с, + ftj р2 + (Q2 + kiCi) р + Q2(fc, + с,) + (ОД = = аор3 + о,р2 + а2р,+ а3 = 0, а2 = £P + ktcu а3 где а0 =1, щ = Ci + ki, = Q2(fe, + c,) + Qc2A;1. Условие устойчивости получим, используя критерий устойчивости Гурвица (приложение 6). В данной задаче система будет устойчивой при выполнении следующих неравенств: а0>0, fli>0, а2>0, а3>0, а^ — аоаъ>0. (З)
J071 § 3.T. АЛГЕБРАИЧЕСКИЕ КРИТЕРИИ УСТОЙЧИВОСТИ 93 Подставляя в неравенства (3) значения коэффициентов системы, получим условие устойчивости с2< ' Q' '■ = 863с,2 + 8630с,. 104. Определить устойчивость автоматической системы управления, если ее движение описывается следующей системой дифференциальных" уравнений: ■ф - Q (vo + Y) = c3Yo. Yo + V + Q* = — CiYo. Y + Qij5 = — kl(y0-yK) + J" MYo-Y.i)^, где Yh —задающее воздействие; у, у0, if —координаты системы; Q = 1,16 ■ Ю-3 сект1, k2- 3 • 10~S сек'2, kx = 1X X Ю-2 сек'1, cx = Ю-1 сект1, c2 = 2,5 • Ю-2 сек'1 — коэффициенты. Определить устойчивость системы. Ответ. Система устойчива. 105. Передаточная функция разомкнутой системы имеет вид Определить условие устойчивости замкнутой системы. Ответ. ТХ>Т2. 108. Передаточная функция разомкнутой системы имеет вид где К = 5 — общий коэффициент усиления разомкнутой системы, Т = 0,5 сек — постоянная времени. Определить устойчивость замкнутой системы. Ответ. Замкнутая система устойчива. 107. Передаточная функция разомкнутой системы одноосной гироскопической стабилизации имеет вид W(p) = ^ 5—. Общий коэффициент усиления разомкнутой системы /С = 50 сек~1, постоянная времени 77 = 0,01 сек. Опреде-
94 ГЛ. 3. УСТОЙЧИВОСТЬ ЛИНЕЙНЫХ СИСТЕМ Ц08 лить: а) при каких значениях коэффициента демпфирования £ гироскопический стабилизатор устойчив; б) условие устойчивости. Ответ, а) Гироскопический стабилизатор устойчив при коэффициенте демпфирования £>0,25; б) К<-^-, 108. В одноосном гироскопическом стабилизаторе, рассмотренном в задаче 107, для увеличения области устойчивости введен сигнал, пропорциональный производной от угла прецессии. При этом передаточная функция разомкнутой системы будет следующей: у(р), *(1 + тр> . р(1+2|7>+7У) Определить: а) условие устойчивости гиростабилизатора; б) устойчивость гиростабилизатора при общем коэффициенте усиления разомкнутой системы /С = 50 сек'1; при постоянных времени 7"г = 0,01 сек, т = 0,01 сек; коэффициенте демпфирования | = 0,1. 2Ё Ответ, а) К<-*—%г~; б) гиростабилизатор устойчив. 109. Передаточная функция одноосного гиростабилизатора имеет вид где k{ = 25 секГ1 — коэффициент усиления разомкнутой системы; 7"г = 0,01 сек — постоянная времени. Для демпфирования системы последовательно в канал управления введено звено с бесконечной полосой пропускания (рис. 71) и коэффициентом усиления 62= 1- Выбрать постоянную времени корректирующего звена Т из условия устойчивости. Решение. Передаточная функция разомкнутой системы W(p) Р(1 + ту)(1 + гР)' где К — k{k2 — общий коэффициент усиления разомкнутой системы.
ИИ § ЗХ АЛГЕБРАИЧЕСКИЕ КРИТЕРИИ УСТОЙЧИВОСТИ 95 Характеристический полином замкнутой системы равен сумме полиномов знаменателя и числителя передаточной функции разомкнутой системы: D (p) = р (1 + ЗГУ)0 + ТР) + К (1 - Тр) = = T2TTpi + Тгр3 + Тр2 + (1-КТ)р + К. Используя критерий устойчивости Гурвица для автоматических систем, имеющих характеристический полином четвертого порядка (приложение 6), получаем условие устойчивости гиростабилизатора при Т- Гиро =0,015 - стабилизатор сек. I Tl А "С. „ „з . устойчив, например, 1-Тр /+Тр А, * Р(>+Т*р2) ■-' Рис. 71. Структурная схема гиростабилизатора к задаче 109. ^-*0» зз~<±>- (ЩЩр *з т^ё -у Рис. 72. Структурная схема к задаче ПО. 110. Структурная схема автоматической системы приведена на рис. 72. Постоянные времени звеньев Г( = 0,01 сек, Т2 = 0,5 сек, Г3 = 0,05 сек. Определить критическое значение общего коэффициента усиления разомкнутой системы K="kik2k3k4, при котором автоматическая система находится на границе устойчивости. Ответ. Критическое значение общего коэффициента усиления разомкнутой системы Д"к=16,8 сек'1. 111. Передаточная функция разомкнутой системы имеет вид WW p(l + T.lP)(l+T3p)>
96 ГЛ. 3. УСТОЙЧИВОСТЬ ЛИНЕЙНЫХ СИСТЕМ ..,.,- Ц12 где К — добротность системы по скорости; Ti = 0,2,сек, Г3 = 0,02 сек — постоянные времени объекта управления и усилителя; 7"2 — постоянная времени корректирующего устройства. Определить значения постоянной времени корректирующего устройства Т2, при которых замкнутая- система устойчива при любых положительных значениях добротности системы по скорости. Ответ. Т2 > TTlJl = 0,018 сек. 112. Структурная схема электромеханической следящей системы приведена на рис. 73. Коэффициент передачи измерительного элемента &и, 8= 1 в/град — 57,3 в/рад, #, —*0-*^, ^^ЖЧ *д р('*ткр) ри/Тр-- ■4 Рис. 73. Структурная схема электромеханической следящей системы. коэффициент усиления усилителя ky = 1000, коэффициент передачи двигателя &д = 50 рад/в сек коэффициент передачи редуктора fep= 10-3, постоянная времени двигателя Ти = 0,05 сек, постоянная времени усилителя 7"у = = 0,005 сек. Определить: а) устойчивость электромеханической следящей системы при отсутствии тахометриче- ской обратной связи; б) значения коэффициента передачи тахогенератора kTT, при которых следящая система устойчива. Ответ, а) Следящая система неустойчива; б) krt> -1 ^м 1 v (kyk„) * = 2,4 • 10 в • сек/рад. 113. Схема электромеханической следящей системы приведена на рис. 74, а. Коэффициент передачи измерительного элемента, выполненного на СКВТ, &и.8 = 1 в/град; коэффициент усиления первого усилителя — kt; коэффициент усиления второго усилителя — fe2"» постоянная времени второго усилителя —7" = 0,0у05 сек; коэффи-
1131 § 3.f. АЛГЕБРАИЧЕСКИЕ КРИТЕРИИ УСТОЙЧИВОСТИ 97 циент передачи двигателя &д = 50 рад/в • сек; постоянная времени двигателя Г», = 0,05 сек; коэффициент передачи редуктора kp— 1 : 1000= Ю-3. Для демпфирования системы первый усилитель охвачен отрицательной обратной связью. Общий коэффициент усиления разомкнутой системы из/условия обеспечения точности работы следящей системы должен быть не менее 300 сект1. О, —ЧЯ>-~ *aj -Н£К к, ЧЖХ Ю ч Рис. 74. Схемы электромеханической следящей системы к задаче 113- Определить коэффициенты усиления усилителей и параметры корректирующего уконтура из условий обеспечения устойчивости системы и заданного значения общего коэффициента передачи разомкнутой системы. Решение. Передаточная функция корректирующего контура . ' где £к=1, T = RC. Структурная схема электромеханической следящей системы, начерченная в соответствии со схемой 74, а, приведена на рис. 74, 6.
98 ГЛ. 3. УСТОЙЧИВОСТЬ ЛИНЕЙНЫХ СИСТЕМ H1S При отсутствии корректирующего контура, передаточная функция разомкнутой системы равна К' №г(р) = р(1 + туР)(1 + Тыр) ' где /С = ka. ^kik2kakp — общий коэффициент усиления разомкнутой системы. "Условие устойчивости замкнутой системы имеет вид /С'<4- + 4- = 220се/с-1. По условию задачи общий коэффициент усиления разомкнутой системы должен быть больше 300 сек~х* Для обеспечения устойчивости введем в систему корректирующее звено (пунктир на рис. 74). Передаточная функция разомкнутой системы при введении в систему корректирующего звена имеет вид *(„)- *<'•"» . , р[1+ -т-^-р) (1 + тур) (1 + тыР) где /( = ——г—г- общий коэффициент усиления разомкнутой системы. Постоянную времени корректирующего звена Т выберем равной постоянной времени двигателя Ты. Это всегда можно сделать выбором параметров /? и С. Положим Я = 0,1 мом. Тогда С = -#- = -g-r- = 0,5 лш/>. При 7" = 7"„ передаточная функция разомкнутой системы равна w(P) = - ^-^Ц , "(1+-rf5r')<1 + r'rt а характеристическое уравнение замкнутой системы имеет вид 1^т>р3+{тть+т*)р2+р+к=0-
им § 3.1. АЛГЕБРАИЧЕСКИЕ КРИТЕРИИ УСТОЙЧИВОСТИ 99 Условие устойчивости записывается в виде /\ "V. гр "Г гр , Из последнего неравенства получаем выражение для определения kt: Выбираем kt =■= 9. Величину коэффициента усиления k2 выбираем из условия обеспечения заданного значения общего коэффициента усиления разомкнутой системы: , K(l + ki) _ 300(1+9) 57,3 • 9 ■ 50 • 10~3 174. 114. Передаточная функция разомкнутой системы К W(p) р (1 + Г,Р) (1 + TiP) * Общий коэффициент усиления разомкнутой системы /( = 500 сект1, постоянная времени ^ = 0,02 сек. Определить значение постоянной времени 7"2, при котором замкнутая система оказывается на границе устойчивости. Ответ. 7"2 = 2,22- 10~3 сек. 115. Структурная схема системы управления статически устойчивого летательного аппарата приведена на %—-®-1—-е- т М,р у'гР' Рис. 75. Структурная схема системы управления статически устойчивого летательного аппарата по углу тангажа, рис. 75. Коэффициенты передачи kx = 1, А2 = 5; постоянные времени Г, = 0,5 сек и Г?. = 2 сек. Определить: а) устойчивость системы без корректирующего звена
100 'гл. з. устойчивость линейных систем ("в (WK — xp на рис. 75); б) величину постоянной времени корректирующего звена т из условия устойчивости. Ответ: а) система неустойчива; б) т > Т, = 0,5-сек. 116. На рис. 76 приведена структурная схема системы управления жесткого статически неустойчивого летательного аппарата. Постоянная времени рулевого привода Г, = 0,5 сек; постоянная времени объекта Г2=2 сек. "пр ф- •\*Р h Ч±Ь- Т" 1*Т,Р ~dh Рис. 76. Структурная схема системы управления статически неустойчивого летательного аппарата. Определить значения постоянной времени корректирующего звена т и общего коэффициента усиления разомкнутой системы К — k\k% из условия обеспечения устойчивости системы. Ответ. Условия устойчивости системы имеют следующий вид: т>Г„ K>U К>^-. Система устойчива, например, при К = 5, т = 0,7 сек. 117. Структурная схема автоматической системы приведена на рис. 77. Коэффициенты передачи звеньев ff Ни-*- W*tp) 1*4у ~?7,г •Ф к-: юи,р+т?рг -у Рис!. 77. Структурная схема системы к задаче 117. fej = 2 • 103, k2 =="6, kz — 0,25« 10~3; постоянные времени т = 0,7-10-3 сек, Г2= 1,42 сек, Т3 = 2,2-10-2 сек. коэф.
1211 § .3.2. КРИТЕРИИ УСТОЙЧИВОСТИ МИХАЙЛОВА 101 фициент демпфирования 1 = 0,68 • 10~". Определить устойчивость системы. / Ответ. Система устойчива. 118. Определить устойчивость автоматической системы, характеристическое уравнение которой имеет вид " а0р5 + аур4 + а2р3 + а3р^ + а4р + аь = 0 при следующих значениях коэффициентов: а) fi0 = 0,005 сек5, Я|=0,15 сек4, о?—1,25 сек3, а3 = 5 сек2, а4 ~ 50 сек, а5 = 300; б) йо = 0,005 сек5, а\ = 0,1 сек4, а2 = 2,5 сек3, а3 = 20 сек2, а4 = 50 сек, 200. Ответ, а) Система неустойчива; б) система устойчива. . 119. Характеристическое уравнение замкнутой автоматической системы имеет вид «оР6 + atp5 + а2р4 + а3р3 + а4р2 + аьр + ас = 0, а2 ='3 сек4, а3 — 4 сек3, где йо — 1 сек%, а, = 2 сек! % —5 сек2, а5 — 6 сек, а6= = 100. Определить устойчивость системы. Ответ. Система неустойчива. § 3.2. Критерий устойчивости Михайлова 120. Система автоматического управления имеет характеристическое уравнение четвертого порядка. Кривая Михайлова системы приведена на рис. 78. Определить устойчивость автоматической системы. Ответ. Система устойчива. 121. Используя критерий устойчивости Михайлова, определить устойчивость электромеханической следящей системы, передаточная функция которой в разомкнутом Рис. 78. Кривая Михайлова к задаче 120.
102 ГЛ. 3: УСТОЙЧИВОСТЬ ЛИНЕЙНЫХ СИСТЕМ - 1181 состоянии равна к W(P)~ р(1+Тур)(1+Тыр) ' где К = 58 се/с-1 — общий коэффициент усиления разомкнутой системы, Тм = &,57 сек — постоянная времени двигателя, Tv = 0,01 сек — постоянная времени усилителя. ш=Юш' <о=5сек~ ч -8В-70-Б!Г-5Н1]-ЗНд -Iff Рис. 79. Кривая Михайлова к задаче 121. Решение. Характеристический полином замкнутой системы имеет вид D(p) = p(l + Typ)(l + TMp) + K~ ~ТуТмрЗ + (Ту + Ты)р* + р + К. Для построения кривой Михайлова определим вещественную и мнимую части функции D (/со): X (со) = Re D (/со) = К - (Ту + Тм) со2 = 58 - 0,58со2, Y (со) = Im D (/со) = со - Гу7>3 = со - 5,7 • 10"V- Вычислим А'(со) и К (со) для ряда значений частоты со. Результаты вычислений сведем в таблицу: ю, сек К (и) У (to) 0 58 0 5 44 4 10 0 4,5 13 -40 0 15 -70 -5 СО — оо — ОО'
лаз) § -3.2, КРИТЕРИЙ УСТОЙЧИВОСТИ МИХАЙЛОВА ЮЗ По данным таблицы построим кривую .Михайлова (рис. 79). Кривая Михайлова последовательно проходит через три квадранта. Следовательно, система устойчива. ' 122. Передаточная функция разомкнутой системы имеет вид где К — общий коэффициент усиления разомкнутой системы, Т > 0 — постоянная времени. Используя критерий устойчивости Михайлова, получить условие устойчивости замкнутой системы. Решение. Характеристический полином замкнутой системы равен сумме полиномов числителя и знаменателя передаточной функции А ы^ /с</- «-•*= ,, ^JJlJCO) Mj(o)/ -I w=Z7 - 1-K ■ -/<-н со -»<>» /r>/ 0)'D - X Рис. 80. Годограф вектора D 0'ю) к задаче 122. разомкнутой системы: D(p) = Tp + K-l. ■ Вектор D(/co) получаем, заменяя в характеристическом полиноме р на /со, D (/со) = /соГ + К ~ 1 = = Х(со) + /Т(со), где>>Цсо) = К-1, К(сй) = со7'. Для устойчивости системы необходимо и достаточно, чтобы вектор Л (/со) при изменении частоты со от нуля до то повернулся на угол ф = -у (рис. 80) .При К< 1 кривая Михайлова расположена во втором квадранте ^ угол поворота вектора D(/co) при изменении частоты со от нуля до оо равен ф=—-^i a ПРИ К>1 равен Ф = -тг. Итак, замкнутая система устойчива при К>1. 123. Система автоматического управления имеет характеристический полином шестого порядка. На рис. 81 приведены кривые Михайлова для различных значений параметров системы. Определить устойчивость системы.
104 ГЛ. 3. УСТОЙЧИВОСТЬ ЛИНЕЙНЫХ СИСТЕМ [124 Ответ. 1 — система находится на границе устойчивости; 2 —система устойчива; 3 —система неустойчива. Рнс. 81. Кривые Михайлова к задаче 123. 124. Система автоматического управления имеет характеристическое уравнение пятого порядка. На рис. 82 приведена кривая Михайлова системы. Определить число корней характеристического уравнения с отрицательной вещественной частью и число корней характеристического уравнения с ', положительной вещественной частью. Решение. Угол поворота вектора D (/со) при изменении частоты со от О до оо равен <Р; : п "5— » (О Рнс. 82. Кривая Михайлова к задаче 124. где я—порядок характеристического уравнения; /—число корней характеристического уравнения с положительной вещественной частью. Из рис. 82 видно, что угол поворота вектора D(jv>) при изменении частоты со от 0 до о° равен я После подстановки в (1) значения угла ф = -2~ и я = 5
§ 3.2. КРИТЕРИЙ УСТОЙЧИВОСТИ МИХАЙЛОВА ДО5 получаем, что число корней характеристического уравнения с положительной вещественной частью п л 5~™ / = = 2. Кривая Михайлова через начало координат не проходит, поэтому число корней с отрицательной вещественной частью равно п —1 — 5 — 2 = 3. Imp У-Рз Pt Pi -Rep Рис. 83. Кривая Михайлова к задаче 125. Рис. 84. Расположение корней характеристического уравнения к задаче 125. 125. На рис. 83 приведена кривая Михайлова автоматической системы, имеющей характеристическое уравнение пятого порядка. Нарисовать качественную картину расположения корней характеристического уравнения на плоскости корней. Ответ. Картина расположения корней приведена на рис. 84. 126* Автоматическая система имеет характеристическое уравнение четвертого порядка. Кривая Михайлова системы приведена на рис. 85. Определить число корней характеристического уравнения с отрицательной вещественной частью. Ответ. Характеристическое уравнение системы имеет Два корня с отрицательной вещественной частью. <W—°о Рис. 85. Кривая Михайлова к задаче 126.
106 ГЛ. 8. УСТОЙЧИВОСТЬ ЛИНЕЙНЫХ СИСТЕМ №7 127. Передаточная функция замкнутой автоматической системы имеет вид Ф № = а0р4 + о,р3 + а2р* + а3р + К' где К =100 сект1, а3—1, а2=1 сек, «[ = 0,02 сек2, Оо — 0,001 сек3. Определить устойчивость системы с помощью критерия устойчивости Михайлова. Ответ. Система неустойчива. 128. Передаточная функция разомкнутой системы имеет вид W (р) = р (1 + Г,р) (1 + Т2р) (1 + г8р) • где К — общий коэффициент усиления разомкнутой системы; Тх = 0,5 сек, Т2 = 0,1 сек, Т3 = 0,02 сек — постоянные времени. С помощью критерия устойчивости Михайлова определить значение общего коэффициента усиления разомкнутой системы /Ск, при котором система оказывается на границе устойчивости. Решение. Характеристический многочлен замкнутой системы равен 2)(р) = р(1 + Г1р)(1+Гар)(1 + Г3р) + /С = " = Т{Г2Т^ + (Г,Т2 + Т{Г3 + Т2Т3) р3 + + (Т1 + Т2 + Т3)р2 + р + К. После подстановки вместо Т^, Т2, Т3 их числовых значений получаем D (р) = Ю"У + 62 • 10"V + 610 • 10"3р2 + Р + К. £)(/©) получаем, заменяя в характеристическом полиноме р на /со, Д(/со) = Х(со) + /К(со), где X (со) = К - 6 Ю • 10"V + 10"V, У (со) = со-62- 10"V. ■ При нахождении системы на колебательной границе устойчивости кривая Михайлова проходит через начало
13(1) § 3.2. КРИТЕРИЙ УСТОЙЧИВОСТИ МИХАЙЛОВА Ю7 координат при частоте со^О. Поэтому при К = КК. X(v>) = Кк - 610 • 10"V + 10"V = 0, (1) У (0) = со - 62 • 10"V = 0. (2) Из второго ураЪнения находим значение квадрата частоты, при котором кривая Михайлова проходит через начало координат: со2 = (б2- Ю-3)"' сек'2. (3) Подставляя (3) в (1), после несложных преобразований получаем 610 -КГ3 Ю~3 62-Ю-3 ■ 622 ■ 10~ 129. Передаточная функция разомкнутой системы автоматического управления имеет вид 1С — !_- Q fi W(p) = - (1 +25Г.Р-+ rfp3).(l +2-#)(l + Т3р) ' где д" —общий коэффициент усиления разомкнутой системы; 7*i = 0,05 сек, Г2 = 0,2 сек, Г3 = 0,1 сек — постоянные времени: g = 0,5 — коэффициент демпфирования. С помощью критерия устойчивости Михайлова определить значение общего коэффициента усиления разомкнутой системы Кк, при котором замкнутая система находится на границе устойчивости. Ответ. /(« = 0,46. 130. Структурная схема автоматической системы приведена на рис. 86. f,(htp) ILL -У Рис. 86. Структурная схема системы к задаче 130; i Общий коэффициент усиления разомкнутой системы K==k1k2k3ki=l0; постоянные времени Г = 0,2 сек, Т0 = = 0,8 сек. Используя критерий устойчивости Михайлова, определить величину постоянной времени корректирую-
108 ГЛ. 3. УСТОЙЧИВОСТЬ ЛИНЕЙНЫХ СИСТЕМ;; [J31 щего устройства т = тк, при которой система находится на границе устойчивости. Ответ. гк = 0,2 сек. 131. Структурная схема системы автоматической стабилизации статически неустойчивого объекта приведена на рис. 87. —Hgb* \ к, '/^ —** k (l*T,p)fl*Txp) »- "в _Рис. 87. Структурная схема системы автомати- • ческой стабилизации статически неустойчивого объекта. Постоянные времени привода Tt*=0,5 сек, Т2=0,1 сек. Постоянная времени объекта Г0 = 2 сек. Коэффициент передачи объекта &0= 1- Коэффициент передачи привода &п = 0,5 град • е-1; Коэффициент передачи корректирующего устройства &2 = 20 град • сек"1 в"1. Используя критерий устойчивости Михайлова, определить значения коэффициента передачи klt при котором система находится на границе устойчивости. Ответ. При'kx = 2 в/град система находится на апериодической границе устойчивости. При k{ = 27 в/град система находится на колебательной границе устойчивости. 132. Характеристическое уравнение автоматической системы имеет вид * а0р5 + atp4 + а2р3 + azp2 + а4р + а5 — 0, где о0 = 3 10~4 сек, а, = 5 10~3 сек, а3 = 0,5 сек2, ак = 0,9 сек, Определить устойчивость системы. Решение. Я 0'юН *(«>) + №), а2 = 0,1 сек , ак= 1. где X (со) = а5 — а3е>2 + аха>\ Y (и) = а4а> — а2&3 + а0&5.
1381 § 3-3. КРИТЕРИЙ УСТОЙЧИВОСТИ НАЙКВИСТА 109 После' подстановки в выражения для Х(со) и Y (со) числовых значений а0, . ..,,а5, получаем X (со) = I - 0,5©2 + 5 • КГ8©4, Y (to) = 0,9со - О, I©3 + 3 • КГ4©5. Неотрицательные корни уравнения У(со) = 0: ©i ==0, ©2 = 3,2 сект1, ©3—18 сек'1. Положительные корни уравнения Jf(co) = 0: ©4 = 1,41 сек'1, <о5 = 9,9 сект1. Неотрицательные корни уравнений X (со) = 0 и У(©)=0 перемежаются. Это говорит о том, что кривая Михайлова последовательно проходит пять квадрантов. Следовательно, система устойчива. 133. Используя критерий устойчивости Михайлова, определить устойчивость автоматической системы, если ее характеристическое уравнение имеет вид а0р5 + «!/?" + а2р3 + «зР2 + а4р + а5 = 0, где а0 = 0,15 • 10~2 сек5, а, = 5 - 1(Г2 се/с4, я2 = 0,6 сек3, ■ Оз = 4 сек2, а4 = 20 сек, аъ — 500. Ответ. Система неустойчива. 134. Характеристический многочлен автоматической системы равен D (р) = 2 • 10~V6 + 80 • 10~У + 3 . 10"У + + 1,24р3 + Юр2 + 40р + 34. Определить устойчивость системы, Ответ. Система устойчива. § 3.3. Критерий устойчивости Найквиста 135. Амплитудно-фазовые характеристики устойчивых в разомкнутом состоянии систем приведены.на рис. 88. Определить устойчивость замкнутых систем.
МО ГЛ. 3. УСТОЙЧИВОСТЬ ЛИНЕЙНЫХ СИСТЕМ f/se Ответ. 1 —замкнутая система устойчива; 2 — замкну» тая система неустойчива; 3 — замкнутая система устойчива. Рис. 88. А. ф. х. к задаче 135. 136. Передаточная функция электромеханической следящей системы в разомкнутом состоянии имеет вид К W(P) = Р О + Тмр) (1 + Тур) ' где К = 100 сект1 — добротность следящей системы по Скорости; 7"м = 0,1 сек — постоянная времени двигателя; 7^ = 0,02 сек — постоянная времени усилителя. Определить, устойчивость электромеханической следящей системы," используя критерий устойчивости Найк- виста. Решение. Для построения а. ф. х. разомкнутой системы определим амплитудную частотную характеристику Л (со) и фазовую частотную характеристику if (со): К Л (со) = №(/со)| = К' /cu(l+/fi.rM),(l+/cuiry) 100 со V\ + (соГм)2 V\ + (соГу)2 со V\ + (со • 0,1)2 Vl + (со • 0,02)^' ^((o)=arg^(y^)^arg/coU + .to^)(1 + .(o7>r-90°+^+fe где % (со) = — arctg соГм = — arctgO, lto, fit®) — — arctg cofj, = —arctgO,02co.
ISTI - § S3, КРИТЕРИИ УСТОЙЧИВОСТИ НАЙКВИСТА - Hi -Вычислим Л (со), %((»), -^{(й), Ф(ю) для ряда значений «в. Результаты вычислений сведем в таблицу: (0, сек~ А i|)i, град ^2. град ч]>, град 0 •оо 0 0 -90 5 18 -26 -6 -122 ш 6,9 -45 -11 -144 15 3,56 -56 -17 -153 25 1,32 -68 -26 -184 50 0,28 -79 -45 -214 100 0,045 -84 -64 -238 По данным таблицы построена а. ф. х. разомкнутой системы (рис. 89). Знаменатель передаточной функции разомкнутой системы имеет один нулевой корень. Поэтому ветвь а. ф. х., соответствующую частотам ы,~>0, дополним дугой окружности бесконечно большого радиуса так, чтобы вектор V ',Яя* ч г 1 / ? ■* -. ■5 ,6 -7 _,,'" Рис, 89. А. ф. х. разомкнутой системы. W(ja>) повернулся по часовой стрелке на угол, равный 90° (рис. 89). Из рис. 89 видно, что а. ф. х. разомкнутой системы охватывает точку (-1, 0). Следовательно, замкнутая система неустойчива. 137. Используя критерий устойчивости Найквистау определить устойчивость электромеханической следяще, системы, рассмотренной в задаче 136 при следующий параметрах системы: а) /( = 50 сек'1-, 2"м = 0,1 сек, Г=х
112 .. гл. а. устойчивость линейных систем Язе ■= 0,025 сек; б) К = 200 сек~\ Тм = 0,02 сек, Ту = 0,002 сек; в) К ==50 се/г1, Гн = 0,1 сек, Гу = 0,005 сек. Ответ, а) Система находится на колебательной границе устойчивости; б) система устойчива; в) система устойчива. 138. Передаточная функция электромеханической следящей системы в разомкнутом состоянии имеет вид ^•.(РН р{1+ткР)(1 + Тур) ■ На рис. 89 приведена а. ф. х. разомкнутой системы, построенная для добротности системы по скорости К.- ЮО сек4. * Определить, при каких значениях К. замкнутая система устойчива. Ответ. Замкнутая система устойчива при /С<57 ее/С1. 139. Передаточная функция одноосного гироскопического стабилизатора в разомкнутом состоянии.имеет у вид Sffc&i W(p) = К р(1+21ГгР+Г2У)' Тг = 0,02 сек, где К = 40 сек' 1-0,15. Используя критерий устойчивости Найквиста, определить устойчивость гиростабилиза- тора в замкнутом состоянии. Ответ. А. ф. х. разомкнутой системы приведена на рис. 90. Гиростабилизатор неустойчив. 140. Передаточная функция системы управления статически устойчивым объектом в разомкнутом состоянии имеет вид W{p) -{i+tMi + W где К = 1 ~ общий коэффициент усиления разомкнутой системы; т = 0,1 с£к — постоянная времени .корректирующее о устройства; Тх = 0,2 сек — постоянная времени Рис. 90. А. ф. х. разомкнутой системы к задаче 139.
14C] § 3.3. КРИТЕРИЙ УСТОЙЧИВОСТИ НАЙКВИСТА * 1-13- исполнительного устройства; 70 = 0,5 сек — постоянная времени объекта. Используя критерий устойчивости Найквиста, определить устойчивость замкнутой системы. Решение. Амплитудная частотная характеристика разомкнутой системы • • ._ кУ~1+(вп)г '_ 1Л+(о,1ю)г (0)) VTTWtf-11 - (<аГ0)2 | VT+ (0,2ш)2 • 11 - (0,5ш)2 | ' Фазовая частотная характеристика arctg ют — arctg юГ, = arctg 0,1© — arctg 0,2(о -ф (оэ) = при w<-~- = 2 сек !, arctg ют — arctg &ТХ — 180е = v = arctg 0,1©.- arctg 0,2<в- 180° при о) >-jr- — 2 сек~1. 'о Вычислим А (а) и ip(<o) для ряда значений частоты ©. Результаты вычислений сведем в таблицу: <а, сек А (оо) ■ф (га), град 0 1 0 1 1,33 -6 1,5 2,2 -9 0>->2—0 со -л ю->2+0 со -191 2|4 2,1 -192 3 0,7 -204 5 1,15 -198 'ой 0 -180 - По данным таблицы построена а. ф. х. разомкнутой системы (рис. 91). При частоте о) = 2 сек ' а. ф. х. имеет разрыв. Ветви а. ф. х., соответствующие частотам &->-=- — 0 и 'о 1 © То + 0, дополним полуокружностью бесконечно большого радиуса. Полуокружность проводим по часовой стрелке от ветви а. ф. х., соответствующей <а~>у—0, к ветви, соответствующей ю—>-~- + 0 (рис. 91).
1-14 , ГЛ- 3. УСТОЙЧИВОСТЬ ЛИНЕЙНЫХ СИСТЕМ [И1 Из рис. 91 видно, что а. ф. х. разомкнутой системы охватывает точку (—1, 0). Следовательно, замкнутая система неустойчива. Определить устойчивость этой системы можно и более простым способом. Из выражения для фазовой характеристики следует* что при T>rt для всех частот ■$(©)>—180°. Поэтому Рис. 91. А. ф. х. разомкнутой сисге- 'мы к задаче 1401 а. ф. х. при х>Т] не заходит в третий квадрант и система устойчива при любых значениях /^>0 и Т0. При T<rii|)(oi>)< —180° для всех частот ©>-=-. По- этому часть а. ф. х., соответствующих частотам ю>^г-, лежит в третьем квадранте, причем ветвь а. ф. х., соответствующая (й-н>--^ + 0, уходит в бесконечность. 'о Поэтому при т<Г, система неустойчива при любых К и Т0- В данной задаче x<Ti. Поэтому система неустойчива. 141. Передаточная функция разомкнутой системы равна {l + T^^+T^l + Tlp2)'
И31 6 3.3. КРИТЕРИИ УС.ТОЙЧИВОСТ.И*НАЙКВИСТА 115 где /( = 1, т=0,4 сек, Tj =0,2 сек, Г2 = 0,1 сек, Т0 = 0,5 сек. Используя критерий устойчивости Найквиста, опреде* лить устойчивость замкнутой системы. Ответ. Замкнутая система устойчива. 142. Передаточная функция разомкнутой системы имеет вид где В (р) — многочлен, все корни которого имеют отрицательные вещественные части. На рис. 92 приведена а. ф. х. разомкнутой системы. Определить устойчивость замкнутой системы. 1^,, Ответ* Замкнутая система устойчива. Рис. 92. А. ф. х. разомкнутой Рис. 93. А. ф. х. .к задаче 143. системы к задаче 142. ■143. Передаточная функция разомкнутой системы равна К W(P): {i+Tp)n > где К>0, Т>0, п>% Определить условие устойчивости замкнутой системы. Решение. Вид а. ф. х.'разомкнутой системы показан на рис. 93.
J16 ГЛ. 3. УСТОЙЧИВОСТЬ ЛИНЕЙНЫХ СИСТЕМ 1144 Фазовая частотная характеристика системы равна ■ф (е>) = — п ar ctg <oT. -Определим значение частоты о = <»_„, при котором ■ф(е>) = — п arctg аТ = — п. (I) Из (1) находим, что Для устойчивости данной системы необходимо и достаточно, чтобы К Из (2) находим условие устойчивости K<(/lWf)"—^ <1. (2) и-и 1_ COS" п Следует отметить, что устойчивость этой системы не зависит от величины постоянной времени Т. 144. Определить устойчивость системы, передаточная функция которой в разомкнутом состоянии имеет вид 1Нр)= к р(1 + 7>)" » где К>0, Г>а Ответ. При п=\ система устойчива при любых значениях /С>0 и Т>0. При п^2 система устойчива при к .»■&(/■+*£)' Г 145. Используя критерий устойчивости Найквиста, определить устойчивость системы автоматической стабилизации летательного аппарата, передаточная функция
»461 § 3.3. КРИТЕРИИ УСТОЙЧИВОСТИ НАЙКВИСТА 117 которой в разомкнутом состоянии.имеет вид W(p)- К(\+.хр) Р(- 1 + Тр) ' где К = 4 сек'1, Т — 1 сек, х = 0,5 сек. Решение. Амплитудная частотная характеристика разомкнутой системы имеет вид КУ\ + (Ш)2 =_2.-i/ l+(0.5^F (О I - И (в))- Фазовая частотная характеристика равна ф (со) == a rctg сот - 90° - (180° - arctgcor) = = — 270° + arctg 0,5ю + arctg о. ' На рис. 94 приведена а. ф. х. разомкнутой системы. Знаменатель передаточной функции разомкнутой системы имеет один нулевой корень. Поэтому ветвь а. ф. х., соответствующую ча- ■ ... стотам со -> 0, дополним дугой окружности бесконечно большого радиуса (см. рис. 94). Многочлен знаменателя передаточной функции разомкнутой системы содержит один положительный кЪрень. Угол поворота вектора, начало которого находится в точке (— 1,0), а конец на а. ф. х., при изменении частоты о от + 0 до оо равен 180°. Следовательно, замкнутая система устойчива. 146. Структурная схема автоматической системы приведена на рис. 87. Постоянные времени г = 0,1 сек, Г, = 0,05 сек, 7*2 = 0,01 сек, Г0 = 2 сек. Коэффициенты передачи ky = § в'град"1, £„ = 0,5 град ■ в'1, &о=1, *а = 0,2 в • сек - град'1. ' Определить устойчивость системы, используя критерий устойчивости Найквиста. ■' Ответ. Система неустойчива. Рис.. 94. Л. ф. х. к задаче 145.
118 ГЛ. 3. УСТОЙЧИВОСТЬ ЛИНЕЙНЫХ СИСТЕМ - (147 147. На рис. 95 изображена структурная схема двух- канальной следящей системы с антисимметричными перекрестными связями. Коэффициенты передачи звеньев fc| = l, k2 — 2, k3 = ■=5 се/с"1. Постоянная времени Т—1 сек. Коэффициент перекрестной связи а = 2. Определить устойчивость системы. Рис. 95. Структурная схема двухканадьной следящей системы с антисимметричными связями^ Решение. Двухканальные автоматические системы с идентичными каналами и антисимметричными связями удобно рассчитывать путем введения комплексных координат. Согласно структурной схеме рис. 95 напишем уравнения движения системы: - k 1 + Тр k2 —- (г, - ах2), (г2 + ах{), х{ *'~ l + Tp z\ = h {gi ~ yd, Z2 = ki(g2~ys), 0) (2) (3) (4) (5) (6)
1471 5 3.3. КРИТЕРИЙ УСТОЙЧИВОСТИ НАЙКВИСТА 119 Умножая уравнения (2), (4) и (6) на ] и складывая их соответственно с уравнениями (1), (3), (5), после несложных преобразований получаем х== Tp + i-jakt2' . ^ . z-fe,(g-jl), (8) *~£*. ' (9) где х = л;, + jx2, S = zt + \гъ g = g, + /g2, «/ = »/i + jy2- v \2 -//aw"7 -UScetf' <&*. ^ Рис. 96. А. ф. x. к задаче 147. В результате решения системы уравнений (7) —(9) имеем W (.р) _ у~ l + W(p) £' где 117 („\ kik2ks _ К_ wyp) p(Tp+l-jak2) p(Tp+\-iak2) — передаточная функция разомкнутой системы. Частотная характеристика разомкнутой системы равна W (/СО) = ■ -. *'. г-у-г = U (СО) + V (СО), где K(aT~ak2) _ -10 (со - 4) £/(со) = У(со) = со [ 1 + («Г - aft2)2] to [ 1 + (со - 4)2] ' К 10 co[l + (cor-afe2)2] co[l+(to-4)2]
120 ГЛ. 3. УСТОЙЧИВОСТЬ ЛИНЕЙНЫХ СИСТЕМ 1148 Передаточная функция разомкнутой системы имеет комплексные коэффициенты. Поэтому для определения устойчивости необходимо построить а. ф. х. в диапазоне частот — оо ч- + оо. А. ф. х. системы приведена на рис. 96. Из рис. 96 видно, что а. ф. х. разомкнутой системы не охватывает точку (— 1, 0). Следовательно, двухканальная следящая система устойчива. , 148. Структурная схема системы приведена на рис. 97. Параметры системы равны: К = 20 сек'1, Т — 1 сек, а = 2. Определить устойчивость системы. ~Уг Рис. 97. Структурная схема двух- канальной следящей системы к задаче 148. Ответ. Система неустойчива. Следует отметить, что при отсутствии перекрестных связей (при а — О) система устойчива при любых,/С>0 н Г>0. § 3.4. Определение устойчивости по логарифмическим частотным характеристикам разомкнутой системы 149. Передаточная функция электромеханической следящей системы в разомкнутом состоянии имеет вид Г(р)== p(i + r„p)(i+r,p) ' где К = 75 сек~\ Гм = 0,02 сек, 7*у = 0,005 сек. Определить устойчивость замкнутой системы по логарифмическим частотным характеристикам разомкнутой системы. -
МЙ § 3.4. ЛОГАРИФМИЧЕСКИЕ ЧАСТОТНЫЕ ХАРАКТЕРА 'Ч<11 J 23 Решение. Устойчивость системы будем определять по асимптотической л: а. х. и л. ф. х. Частоты излома асимптотической л. а. х. равны щ = = 50 сек'1, щ — у- = щ^ = 200 сек'1. тх 0,02 \ Низкочастотная асимптота л. а. х. пересекает ось ч^1£тот при частоте \ <о = /<Г = 75 сек'1. По этим*1' данным строим асимптотическую л. а. х. (рис. 98). сказовая частотная характеристика равна * 1]> {&) = +1 («>) + Ь (03) + фз (0)), где i|>, (ю) = — 90°, ' % («>) = — arctg <йГм == — arctg 0,02<й, •фз<<о) = — arctg Гу<о = --arctg 0,005ю. *•* -225z -J80" -135°- L, дб -40 -30 •^гОдб/ден j j /0 20 ЗОЬОър* *^-J 1 1 1 1 1111 \\?60д№еп Рис. 98. Асимптотические л. а. х. и л. ф, х. к задачам 149 и 152. Графики функции i])2(<o) и г])3(<о) строим с помощью шаблона. Л. ф. х. получаем графическим сложением функций Ы«>), 1М«>), ^з(«>) (см. рис. 98).
120 "гл. з. устойчивость линейных систем iko Л. ф. х. пересекает линию -ф = — 180° при отрицательных значениях асимптотической л. а. х. Следовательно, замкнутая система устойчива. В данной задаче л. ф. х. является монотонной функцией от частоты со, поэтому задачу можно решить без построения л. ф. х. После построения асимптотической л. а. х. опреде-; ляем частоту среза разомкнутой системы со = сос = 60 ce%-i (см. рис. 98). Значение фазы при частоте среза ф (сос) = — 90° - arctg (0,02 • 60) - arctg (0,005 • Щ = = — "157° >— 180°. Следовательно, замкнутая система устойчива. .150. Определить устойчивость системы, рассмотренной в задаче 149 при Гу = 0,00-5-" сек, Гм = 0,02 сек, /<Г = 300 сек'1. .,--" Ответ. Система неустойчива. 151. Передаточная функция разомкнутой системы имеет в.ид w(P)r- ки±ж1 р(1+7»(1+Тур) ' где /С = 300 сек'1, Гм = 0,02 сек, Гу = 0,005 сек, т = 0,0045 сек. Определить устойчивость системы. Ответ. Система устойчива-. 152. Для электромеханической следящей системы, рассмотренной в задаче 149, определить значение добротности системы, при котором система находится на колебательной границе устойчивости. Решение. Логарифмическая фазовая характеристика определяется выражением •ф (со) = -ф, (со) + -ф2 (со) + ^фз (со), где -ф, (со) = — 90°, % (со) = — arctg соГм, ij>3 = — arctg соГу. Графики функций ^(со) и ^3(co) строим с помощью шаблона. Л. ф. х. получаем графическим сложением характеристик ф, (со), г|)2(со) и ф3(со) (см. рис. 98). Система находится на колебательной границе устойчивости, если л. а. х. пересекает ось частот при частоте пересечения л. ф. х. линии ■ф = '—180°, со ==100 сек'1.
1S8I § 3.4. ЛОГАРИФМИЧЕСКИЕ Ч*\СТОТН,ЫЕ ХАРАКТЕРИСТИКИ 123 Асимптотическая л. а. х. системы в диапазоне частот О -^ -=г- имеет наклон —20 дб/д^ек, в диапазоне частот J- -н -= —40 дб/дек, в диапазоне частот -~- -=- оо— Ты 'у * у — 60 дб(дек. Зная наклоны асимптотической л. а. х., легко начертить асимптотическую л. а. х., пересекающую ось частот на частоте пересечения л. ф. х. линии •ф ——180° (см. рис. 98). Добротность системы по скорости К — Кк определяем по точке пересечения низкочастотной асимптоты л. а.х. с осью частот Кк = 200 сек~х. Точное значение Кк~250 сек'1. Ошибка в определении Кк объясняется отличием асимптотической л. а. х. от действительной. 153. Определить устойчивость замкнутой автоматической системы, если ее передаточная функция в разомкнутом состоянии имеет вид ■ ~ к WW= р (1+2» (1 + 2» (1 + 7» ' где К = 300 сек~\ Г, = 0,2 сек, Т2 = 0,05 сек, Т3 = 0,02 сек. Ответ. Система неустойчива. 154. Для системы, рассмотренной в задаче 153, определить значение общего коэффициента усиления разомкнутой системы К«, при котором система находится на .колебательной границе устойчивости. Отёет. /<"к = 20 сек~х. 155. Структурная схема автоматической системы приведена на рис. 76. Определить устойчивость системы, если К = kik2= 10, Г2 = 2 сек, 7^ = 0,05 сек, т = 0,1 сек. Ответ. Система устойчива. 156. Передаточная функция одноосного гиростабили- затора при коэффициенте демпфирования | = 0 в разомкнутом состоянии имеет вид где К = 40 сект1, Гг = 6,5. 10~3 сек. Для демпфирования системы последовательно в цепь управления введено . корректирующее звено с
124 ГЛ. 3. УСТОЙЧИВОСТЬ ЛИНЕЙНЫХ СИСТЕМ [157 передаточной функцией Определить величину постоянной времени Т, при которой гиростабилизатор будет устойчив. Решение. На рис. 99 приведены асимптотическая л. а. х. и л. ф. х. нескорректированной системы (сплошная линия). Для устойчивости системы необходимо, о), сен Рис. 99. Асимптотическая л. а. х. и л. ф. х. к задаче i56. чтобы л. ф. х. пересекала линию -ф = —180° в диапазоне частот 1( -*- -у-. Поэтому постоянную времени корректирующего звена необходимо выбирать из условия Система устойчива, например, при Г = 0,01 сек. 157. Определить устойчивость одноосного гиростаби- лизатора, передаточная функция которого в разомкнутом состоянии имеет вид W{p) = ~r- ^ g-jgr, -р(1+2£7"гр + тУ) где К = 40 сек'1. Гг~6,5'. 10~3 сек, 6-0,2. Ответ. Гиростабилизатор устойчив.
ISO] § 3.5. ПОСТРОЕНИЕ ОБЛАСТЕЙ УСТОЙЧИВОСТИ 125 158. Определить устойчивость системы, структурная ехема которой приведена на рис. 87, если k\ = 20 в/град, k2 = 40 в • сек • градт*, 7\ = 0,5 сек, Т2 = 0,1 сек, Т0 = ж= 2 сек, ftn = 0,5 град • е-1, k0 = 1. Ответ. Асимптотическая и точная л. а. х. системы приведены на рис. 100. Система устойчива. Рис. 100. Асимптотическая л. а. к. и л. ф. х. к задаче 158. 159. Решить предыдущую задачу при ft, ==20 в]град, ft2=100 в-сек-град, Т-, = 0,5 сек, Т2 — 0,1 сек, Г0 = = 2 сек, ftn === 0,5 град/в, k0 = 1. Ответ. Система неустойчива. § 3.5. Построение областей устойчивости 160. Передаточная функция статической системы автоматического управления в разомкнутом состоянии имеет вид К W (/,)== (1+Г,р)(1+Г,р)(1+7'1р) ' где 7*2 = 0,2 сек, Т3 = 0,1 се/с. Построить область устойчивости системы в плоскости параметров Тх а К.
126 ГЛ. 3. УСТОЙЧИВОСТЬ ЛИНЕЙНЫХ СИСТЕМ 1160 Решение. Характеристический многочлен замкнутой системы имеет вид D (р) = (1 + TlP) (1 + Т2р) (1 + Тър) + К = ~Т1Т2Тър* + {ТхТ2+ Г,Г3 + Г2Г3)Р2 + (Г1 + Г2+ Г3)Р + /С= - 0,02Г,/73 + (0,02 + 0,37,) /72 + (0,3 + Г,) р + К + 1. Для построения областей устойчивости найдем выражение для границ области устойчивости. Для получения уравнений границы области устойчивости, соответствующих наличию в характеристическом*- многочлене системы бесконечного и нулевого корня, приравняем нулю коэффициент при старшей степени характеристического многочлена и свободный член характеристического многочлена. В результате получим следующие уравнения границ области устойчивости: 7*1-0, (1) Т'(Тт^т^ Т»ш К*=-1- (2) Уравнение для грани- 101. Область устойчивости КЫ области устойчивости, к задаче' 159. соответствующей нахождению системы на колебательной границе устойчивости, найдем, приравнивая нулю предпоследний определитель Гурвица Д„-1 = 0. В дайной задаче это условие принимает вид (0,02 + 0,377) (0,3 + 7",) = 0.02Г, (1 + К). Отсюда получаем ' к__ (1 + 15Г.) (15 + 50Г.) ^ (3) В соответствии с уравнениями (1), (2), (3), на рис. 101 построены границы области устойчивости. Линия, соответствующая уравнению К— — 1» практически сливается с осью абсцисс.
I03| § 3.5, ПОСТРОЕНИЕ ОБЛАСТЕЙ УСТОЙЧИВОСТИ 127 Областью устойчивости . является область А (см. рис. 101), так 'как для любой из точек внутри >той области выполняется условие устойчивости. 161. Передаточная функция разомкнутой системы равна W( П) = К _ W УН> (1 + TlP) (1 + 2|7'р + ГУ) ' • где Ту ~ 0,2 сек, Т = 1 сек — постоянные времени исполнительного устройства и объекта; К — общий коэффициент усиления разомкнутой системы; | — коэффициент демпфирования. .Построить область устойчивости замкнутой системы в плоскости параметров К, £• Ответ. Область устойчивости системы приведена на рис. 102. 162. Передаточная функция ОДНООСНОГО ГИрОСТабИЛИ- №щ/!1/гг(>ггн</«м<«му«а £ затора в разомкнутом со- ^ ^ ^ °^ № стоянии имеет вид _ 1ПО ' " Рис. 102. Область устойчивости Ш(п\ — t± к задаче 160. WKP) р(1+2|Ггр + гУ)' где /С —общий коэффициент усиления разомкнутой системы, |—коэффициент демпфирования, Тт — постоянная времени. Построить область устойчивости одноосного гироста- билизатора на плоскости параметров /С, Тг для значений коэффициента демпфирования g = 0,1, | = 0,2, g = 0,3. Ответ. Область устойчивости системы приведена на рис. 103. 163. На рис. 87 приведена структурная схема системы автоматической стабилизации летательного аппарата. Постоянные времени привода исполнительного устройства Тi = 0,5 сек, Т2 — 0,1 сек. Постоянная времени объекта Т0 = 2 сек. Коэффициенты передачи привода исполнительного устройства и объекта йп = 0,5 град/в, Ао=1.
128 ГЛ. 3. УСТОЙЧИВОСТЬ ЛИНЕЙНЫХ СИСТЕМ [163 Построить область устойчивости системы на плоскости К\, К* Решение. Характеристическое уравнение замкнутой системы (1 + Г,р) (1 + Т2р) (- 1 + Tip2) + kako {ki + k2p) = = Т\ТД%р* + (Г, + T2) Top3 + {To - TiT2) p2 + + {k0kDk2 —Ti — T2)p + k0knkl — 1. Найдем уравнения границ области устойчивости. Уравнение апериодической границы устойчивости Т""1 'I Iг'г ■ Ч" 0,0/0 0,015 ЩШ ,сен Рис. 103. Область устойчивости одноосного гидростабилизатора к задаче 161. найдем, приравняв нулю свободный член характеристического уравнения. Тогда k^l^ = 2e-ePad~l- M Колебательной границе устойчивости соответствует равенство нулю характеристического комплекса или D(ja)='X(a>) + iY(a>) = 0, X (©) = Minft, - 1 - (Го - TiT2) ©2 + Т<T2TW = 0, (2) У(со) = (МпА:2-Т1-Г2)со-(Г, + Г2)7,осо3 = 0. . (3)
183) § 3.5. ПОСТРОЕНИЕ ОБЛАСТЕЙ УСТОЙЧИВОСТИ 129 Из уравнения (2) получаем 1 «о&п (4) В дальнейшем можно из (3) найти выражение для (б) k2 = L+l3- + li+Is.a? Уравнения (4) и (5) — уравнение границы устойчивости, записанное в параметрической форме. В данной задаче проще поступить следующим об» разом. 20 30 W 50 60 70 80 90 Рис. 104. Область устойчивости к задаче 162. Seen град Из (3) найдем выражение для со2 и подставим его в (2). В результате получим уравнение параболы , ._ J , (7o-VV)(W2-riTrg) *0*п ЬЬ(*оМ«-г,-т2у _^(„ J)73 .10-% + 1 >7). (6) WATi + T2?rl По уравнениям (1) и (6) на рис. 104 построена граница области устойчивости. Областью устойчивости является область А. Это можно проверить, применив любой критерий устойчивости к одной из точек,- принадлежащих этой области.
ГЛАВА 4 ПОСТРОЕНИЕ ПЕРЕХОДНЫХ ПРОЦЕССОВ В СИСТЕМАХ АВТОМАТИЧЕСКОГО РЕГУЛИРОВАНИЯ § 4.1. Классический метод решения дифференциальных уравнений 164. Найти выходную величину y(t) системы, описываемой уравнением T-§iy{i) + y<fi*=g(i)- для двух случаев. 1. На вход системы подается задающее воздействие, изменяющееся по гармоническому закону g(t) = Gu.sinQt; начальное условие у{0) — уо. 2. При установившемся режиме, соответствующем задающему воздействию g(t) = GMsin Qt, происходит скачкообразный сдвиг фазы управляющего воздействия на +90°; сдвиг происходит в момент, когда Ш — 2пп, где п — целое число. Ответ. t_ 1. y(t) = Yusm(Qt-ty) + (yo+Yusin$)e~T, м V t + (йГ)« s t_ 2. y{t) — Yucos{Qt — ф) — KM (sin ф + cos if) e T.
§ 4.1. КЛАССИЧЕСКИЙ МЕТОД РЕШЕНИЯ УРАВНЕНИИ 13t 165. Дана следящая система, представленная на рис. 105. На-вход5силителя / подается разность между задающим воздействием g и выходной величиной у. "' . V х-х 5—*М~* Т 7 2 У Рис. 105. Блок-схема следящей системы к задаче 165. Кроме того, на усилитель подается первая производная g1 задающего воздействия. 2 — двигатель, редуктор и исполнительный механизм. Система описывается уравнением (Тр? + р + Юу V) = (Ктр + K)g V). (1) Постоянная времени 7 = 5 мсек, коэффициент усиления- по задающему воздействию К = 40 сект1, коэффициент усиления по производной задающего воздействия Кх =*- = 0,8. Найти закон изменения выходной величины g для двух следующих случаев. 1. При отработке системой рассогласования у0 при отсутствии задающего воздействия и нулевой' начальной скорости. 2. При задающем воздействии в виде единичной- ступенчатой функции I (0 и нулевых начальных условиях ^_o==t/'_o==0. Решение. 1. Дифференциальное уравнение системы для первого случая имеет вид (ТУ + Р + Юг/Ф^О или (0,005р2 + р + 40)у(0 = 0. (2) Характеристическое уравнение 0Д)05р2 + /} + 40 = 0 (3) имеет два вещественных рц — —144,7 сект1. корня —ps — —55,3 сек*1,
132 ГЛ. 4.. ПОСТРОЕНИЕ ПЕРЕХОДНЫХ ПРОЦЕССОВ 116» Для случая вещественных корней решение уравнения (2) имеет вид y{t)^Ale'a't + A2e'a'i, (4) где а, и а2 — абсолютные значения корней характеристического уравнения. Начальные условия: при t = О У = Уо> У'-У'о Из (4) и (5) получаем А1 + А2 = у0, | — а1Ах — а2А2 = 0. J Из (6) находим Л1==-^-, Л2=-^-. (7) Решение задачи для первого случая имеет вид согласно (4) и (7): U} г/о или yW^X—i^e-^-aie^Kt) y{t) = %(1.619е-55-м - 0,619е-14".70 1 (0- (8) Выражение (8) можно также получить непосредственно по данным задачи, если воспользоваться приложением 10, где приведены решения однородных уравнений первой, второй и третьей степеней как при вещественных, так и при комплексных корнях. 2. Дифференциальное уравнение системы для второго случая может быть записано, согласно (1), в виде {а0р2 + щр + а2) у (t) = (b0p + Ь{) g {t), (9) где а0 — Т = 0,005 сек, at = l, a2 = К = 40 сект1, b0 = Kt = = 0,8, &,-=ЛС='40 сек~1. Прежде всего найдем начальные условия, имеющие место непосредственно после воздействия на систему единичной ступенчатой функции. ■ Для этого удобно воспользоваться приложением 9, В соответствий с указанным приложением найдем из' (9)'
,s5j " § 4.1. КЛАССИЧЕСКИЙ МЕТвД РЕШЕНИЯ УРАВНЕНИЙ 133 п == 2, т = 1 и получим \ У;+^ У-* + ■% НО = о + да l W = 16°-l W '«-'• <10> Решение уравнения (9) удобно свести к решению однородного уравнения с теми же коэффициентами, перейдя к новой переменной z(t) = y(t) — yyCT, где Уус ^-1(0 = -^1(0=К0 (И) (12) — частное решение уравнения (9), т. е. установившееся значение выходной величины у. Таким образом, вместо ((9) получаем уравнение (a0f? + щр + а2) z (0 = 0 при начальных условиях Z+0. У+0 Уус-с' Z+0 У+(Г Эти соотношения получены из уравнения (11). Решение (13) имеет вид • z (0 = л,е-а'' + A2e~a!t, (13) (14) (15) где, согласно первому случаю, ai = 55,3 сект1, се2 = '=*= 144,7 сект1. Для определения постоянных интегрирования А\ и Л2 получаем из (15), согласно (10), (12) и (14), уравнения Al + A2 = z+0 = y+0-yyc. ИЛИ 13ЛИ А + л2 = -1(0, а. А. — а, А, ■z', Ч'Ч «2**2 ~+0 #+0 — <Х|Л, — а2л2 =160-1 (0- (16)
134 ГЛ. 4. ПОСТРОЕНИЕ ПЕРЕХОДНЫХ ПРОЦЕССОВ [JS5 Из (16). получаем л-~Г.2-Г°но=о,171-на А2 = a2 — tti -.а, + 1-60 ai — a2 1(0=- 1,171-1(0. (17) Заметим, что решение уравнения (13) можно получить, используя "приложение 10. , у_ У, W 0.8 \ 0,6 ■0,4 '№ О ^^/,те-!т-дежт'п)1Ш Ш т 0,03 DJ04 № ffMS D,07tfiBH ■ff(t)=(^Gme-f3-3t-/lJ7/ei,4m)rm О 0{I1 0,02 003 0/J4 005 OflB Oj07t,csH Рис. 106. Кривые переходных процессов к задаче 165; I — ненулевые начальные условия, 2 — реакция системы • =на- ступенчатое воздействие. Из (15) получим, согласно (11), (12) и (17), y(t) = z(t) + i/уст = (0,171в-кл*— 1,171<г"*.«) 1 (0 + 1 (/). Таким образом, при воздействии на систему единичной ступенчатой функции 1 (t) выходная величина изменяется по закону y(t) = [l +0,171е-55.з'- 1,171е-1<4.7<] Щ. (18)
tm § 4.Ц КЛАССИЧЕСКИЙ* МЕТОД' РЕШЕНИЯ УРАВНЕНИИ 135* Па- уравнению (8) на рис. 106. построена.кривая /, а 1 №•• уравнению (18) г- кривая 2, 166. Решить задачу 165 при следующих' данных: 7V= 0,005 сек, # = 200 сек'1, #т = 0^8.. Ответ. 1. При согласовании- системы ее закон движения yit) = U55z/be~100' sin(173/ + 60°) (кривая/, на рис 107). ■fits О 0.005 0,010 ОМ 0,020 0,025 t.ceii Рис. 107. Кривые переходных процессов к за- - дане 166; 1 — ненулевые начальные условия, 2 — реакция системы на ступенчатое воздействие. 2. При воздействии на систему единичной ■■ ступенчатой функции. У{*) = [1 + 1,059е-"М sin (173* - 70°50')] 1 V). (кривая 2 на рис. 107).
J36 IVIv 4, ПОСТРОЕНИЕ ПЕРЕХОДНЫХ ПРОЦЕССОВ -: [167 167. Найти функцию -веса w(t): 1) для-системы, представденной в задаче 165; 2) для системы, представленной в задаче 166. Указание. Возможно, использование переходных функций этих систем, полученных в задачах 165 и 166. Ответ. ■ 1) w if) = (169,2е-'«-»- 9,45е-55-30 1 (t); 2) w (0 = 2\2e-™cos{mt - 40°50') 1 (t). 168. Найти переходную функцию h(t) и функцию веса w(t) системы, описываемой уравнением Цр2 + щр + а2) у (t) = b0g(t). Все коэффициенты уравнения положительны; Ь0 = а-2, а\>Аа(р.2. Ответ. (^ = gi°» (e-erf _ ечЦ) l (0, 4 ' (q2 — a,) v / \ /' где а, и а2 — абсолютные значения корней характеристического уравнения системы. ,,169. Дана статическая система автоматического регулирования, описываемая уравнением (а0р2 + щр + а2) у (0 = b0g (О, где Оо = 0,002 сек2, aj = 0,12 сек, а2 = 5, Ь0 = 4. Найти реакцию- системы на ступенчатое воздействие g(t) = gn-l(t). Ответ. У С) = go [0,8- е-*» sin (40г + 53°10')] 1 (Q. 170. Система автоматического регулирования описывается уравнением (а0р3 + щр2 +. а2р + а3) у (t) = (b0p + bt)g (t), (1) где а0 = 5 • Ю-4 сек2, а, = 0,105 сек, а2 = 2,16, а3 = &i =" =,65,3 сек'1) Ь0 = !■, 16.- . ._■ •
1ГОГ § 4.1. КЛАССИЧЕСКИЙ МЕТОД РЕШЕНИЯ УРАВНЕНИЙ 137 Найти переходный процесс для двух случаев: ■ 1. При включении системы после ее предварительного рассогласования на величину х0. ,2. При задающем воздействии в виде единичной ступенчатой функции у (t) = 1 (t) и нулевых начальных условиях у_о = #10 = у"0 = 0. 1. Решение для случая 1. Характеристическое уравнение, соответствующее (1), имеет при заданных коэффициентах вид 0,0005р3 + 0,105р2 + 2,16р + 65,3 = 0. Корни уравнения (2) могут быть найдены каким-либо из известных методов. Эти корни равны р, = — а = — 180 сек~\ 1 Pa. з = - У ± А = - Ю ± /25 сек'1. ) (3* Выходная величина системы, характеристическое уравнение которой имеет один вещественный и пару комплексных корней, имеет вид у (t) = Ae~at + Ве~* sin (М + 6). (4) Начальные условия равны 4/(0) = %, £/(0) = 0, #"(0) = 0. (5) Из (4) находим lf(t) = -aAer* + Ве~У1 [К cos (M + б) - \ sin {M -Ь6)], J/" (/) = а?Ае~м + Ве~* [(y2 - К2) sin (Л/ + б) - -2vA.cos(M + o)l. Из выражений (4) —(6) получаем систему уравнений для определения постоянных интегрирования А, В, б: A + Bsin6 = #„, — ct/4 + jBX cos б — \B sin б = 0, а2Л + В (y2 - Я2) sin б - 2y^ cos 6 = 0. После подстановки a, y. А, согласно (3) найдем А = 0,0246%, В = \,13у0, б =.59*50'. (8) (6) (7)
13S ГЛ: 4: ПОСТРОЕНИЕ: ПЕРЕХОДНЫХ' ПРОЦЕССОВ- Ц71 Подстановка. (8)> в (4). дает решение задачи: У (*) = У о [0,0246е-18И + 1,13e-Iorsin (25Z + 59°50')]. Этот результат можно получить непосредственно из (2)- и (5); если воспользоваться приложением 10. 2. Указание к решению задачи для второго случая. Начальные условия, имеющие место непосредственно после приложения ступенчатого воздействия, могут быть определены при помощи приложения 9. Ответ. y(t).= [1 + 0,054Le-180' - l,054l£-IM sin.(25/ + 88°15')] 1 (*)■ 171. Найти переходный процесс в системе, данной в предыдущей задаче, при управляющем воздействии, нарастающем по линейному закону g(t) = at\{t). Укаа-ание. Частное решение дифференциального уравнения системы, т. е. вынужденную составляющую переходного процесса, следует искать в виде ув = Ь + ct. Ответ. y(t) = a[t~0,000302e-18w - - 0,0392е-10' sin (25* - 23°30') - 0,01532] 1 (t). 172. Система автоматического регулирования описывается уравнением (aoP + aJyW^boPgit). (1) Найти переходный процесс в системе при помощи интеграла Дюамеля для двух видов управляющего воздействия: 1) g{t) = atl(t)., (2) 2) е(*Н&(<г*-<г**)1(0 (3) при нулевых начальных условиях. Решение, для случая g(t) = at\(fy.
172] § 4:f. КЛАССИЧЕСКИЙ МЕТОД РЕШЕНИЯ УРАВНЕНИЙ цдд Интеграл Дюамеля может -быть записан .в виде у it)=-е (о).* (0 + J" е*Ы Щ - т) dx, (4) о где h (t) — переходная функция системы. Для определения h(t) найдем ..реакцию системы на единичное ступенчатое воздействие, т. е. решим уравнение {a0p + al)y(t) = b0pl(;t) (5) при .нулевых начальных условиях. В соответствии с уравнением (5) найдем Ууст = 0. ' (6) Используя приложение 9, найдем также С учетом (6) и (7) решение уравнения (5) имеет вид у (/) = Ае~ + уус, = Ае~^ = 1 (*)-£ e~T, (8) гдеГ = -^. а, Таким образом, переходная функция системы равна A(0 = A.e-h(/). (9) Для линейного управляющего воздействия (2) имеем 8,Ш=а. (10) Подставим (9) и (10) в (4): t y(t)^ja^e~~^dx. (11) о Интегрируя-уравнение (11), находим ответ для первого случай задачи:
140 .'" ГЛ. 4. ПОСТРОЕНИЕ -ПЕРЕХОДНЫХ ПРОЦЕССОВ Ц73 Ответ для случая апериодического управляющего воздействия (3): t_ . '" t-b° (r-grT)e-rt-(д~дгТ)е-<>т + (д-Ле т 173. Найти переходный процесс в системе, описываемой уравнением■ ■:. (aoP + ai)y(t) = b0g(t), при затухающем колебательном задающем воздействии g{t) = ce-rt sin Ш и нулевых начальных условиях. Указание. Предлагается применить интеграл Дюамеля. Ответ. , (-^ - г) e~rt sin Qt - SUTrt cos Qt + Qe~T г[(т.-')+0*] где T-f. § 4.2. Применение изображений Лапласа и Карсона — Хевисайда 174. Передаточная функция разомкнутой системы автоматического регулирования равна Г(р)=Р(1 + 7» = рц+олр) • (1) Найти переходную А(0 и весовую № (0 функции замкнутой системы. Решение. Передаточная функция замкнутой системы, с учетом (1), равна ф/рч;>ЯМр) ■ К ■ ... 20 (2) ы^ i + W(p) ~ гР2 + р + /с o,ip2 + p + 20' w
J74] § 4.2. ИЗОБРАЖЕНИЯ ЛАПЛАСА И КАРСОНА - ХЕВИСАИДА j4j Переходная функция h(t) представляет собой реакцию системы на един-ичное ступенчатое воздействие 1 (•£).■ Изображение Y(p) выходной величины y{t) замкнутой системы при задающем воздействии g(t), изображение которого равно G (р), представляет собой при нулевых начальных условиях произведение У(р) = Ф(р)-р(р). Изображение единичной ступенчатой функции по Кар- сону — Хевисайду равно 1, а по Лапласу —. Поэтому переходная функция h(t) системы может быть получена как результат обратного преобразования по Карсону — Хевисайду передаточной функции . замкнутой системы, т. е. выражения (2), либо как результат обратного преобразования по Лапласу произведения 7Ф^=Р(0.1Р2 + р + 20)- <3> Для перехода от изображения (2) или (3) к искомому оригиналу h(t) необходимо знаменатель изображения разложить на множители. Для этого приравняем знаменатель (2) нулю, Тр2 + р + К=0 или ОД/;2+ /? + 20 = 0, (4) и найдем корни полученного уравнения (4): Pi= - V + A= -5+ /'13,2 сек-1, р2 = - у - /Я. = - 5 - /13,2 сек'1. J (5) Далее можно записать знаменатель выражения (2) в виде 0,lp* + p + 20 = 0,l(p-Pl)(p-p2) = .=0,l[p-(-Y + A)][p-(-Y-A)] = = 0,1 [(р + Y)2 + Щ = 0,1 [(р + 5)2 + 13,22].. (6) Теперь вместо (3) получим 1фЫ = 20 д 20° . (7) р \р> 0,1р[(р + 5)2+13,22] р[(р + 5)2 +13,2*] v '
142 ГЛ. 4. ПОСТРОЕНИЕ ПЕРЕХОДНЫХ ПРОЦЕССОВ V[J74 Из таблиц изображений функций по Лапласу (см. приложение 1) подбираем формулу, соответствующую выражению (7): = h ,А 1 е-* sin (Л/ - ф), ^^arctg-^. (8) При подборе формул следует иметь в виду, что в имеющихся справочниках эти формулы располагаются в порядке возрастания степени полинома от р в знаменателе изображения. Для случая вещественных и для случая комплексных корней всегда предусматриваются отдельные формулы. Так, если бы корни знаменателя изображения (2) оказались вещественными, следовало бы вместо формулы (8) взять табличную формулу р(Р + а)(р + $) • пр^а—р\«*е f,b )• где киР — абсолютные значения корней. Сопоставляя (7) и (8), получаем оригинал выражения (7), т. е. переходную функцию системы, А(0 = Ф(р) = " Ыпк* ~ шуУ+ .з,2* е~ы sin (13>2'+69° 1У>]' <* или Л (0 = [1 — 1,068в-и sin (13,2/ + 69° 15')] 1 (t). (9) Замечание. Следует обратить внимание на вычисление угла я]) по формуле (8), так как знаки в формулах для я])., типичных для "подобных выражений, записываются своеобразно. Знак числителя в выражении для тангенса ty является знаком синуса ■ф, а знак знаменателя является знаком косинуса ty. Таким образом, формула для ■ф содержит указание на квадрант, в котором находится этот угол. Это позволяет освободиться от двойственности в ответе для ty, обусловленной совпадением тангенсов двух углов, отличающихся на п. 13 2 В рассматриваемом примере, rfletgij>= ^—=—2,64, из двух возможных значений ty, равных —69° 15 и
1761 § 4-2. ИЗОБРАЖЕНИЯ ЛАПЛАвА' И KAPGOHA—ХЕВИСАЙДА (43 4-110°45', следует взять второе число, так как выраже- Я 13 2 ние ф = arctg--=— = arctg -^ указывает, что угол находится во втором квадранте. В результате из формулы (8) следует sin (M - я!>) = sin (13,2* - 110°45') - - sin (13,2* + 69450, что и учтено при записи выражения (9). Функция- веса w (t) системы может быть найдена как производная переходной функций (9) по времени. Функция веса может быть найдена и непосредственно по передаточной функции (2), как ее обратное преобразование по Лапласу, H0 = r'[O(P)l = L-[wfp + 20]. = L-'L 200 1 ( L(p + 5)2+l3,22 J. \W) либо как обратное преобразование по Карсону — Хеви- сайду произведения н у^> 0,1р2 + р + 20 (р + 5)2+13,22 * У > Подбираем из таблиц изображений функций"тто Лапласу формулу; соответствующую (10): Согласно (7), (10) и (12) получаем функцию веса системы tw(0=l5,15e-asinl3,2* 1 (t). (13) 175. Для замкнутой системы автоматического регулирования, -данной в предыдущей задаче, найти закон изменения выходной величины y(t) при отсутствии задающего воздействия, начальном рассогласовании у(0) = у0 и нулевой начальной скорости. Решение. Согласно уравнению (2) предыдущей задачи, дифференциальное уравнение замкнутой системы имеет вид {Tp2 + p + K),y{t).-KgV), 0)
J44 ГЛ. 4. ПОСТРОЕНИЕ. ПЕРЕХОДНЫХ ПРОЦЕССОВ [175 где g (t) — задающее воздействие. Для получения из (1) изображения выходной величины y(f) необходимо использовать операторные выражения для производных с учетом начальных условий. Запишем эти выражения по Лапласу, ■. полагая, что Y{p) есть изображение функции у (О py(t)-y'U)^pY(p)-y(0), I P*y(t) = y"(t) = p2Y(p)-py(0)-y'(0). ) {-} Здесь у(0) и у' (0) — начальные значения выходной величины и ее производной. Из (1) и (2), учитывая, что g (t) = 0, получаем Tp*Y (р) - Тру (0) - Ту1 (0) + pY (p) - «/ (0) + KY(p) = 0, или У(гЛ-. (ГР+1)У(0) + 7У(0) /ох' У\Р)~ Тр2+р+К • W Подставляя значения начальных условий у(0) — Уо и г/'(0) = 0 и коэффициентов уравнения Т = 0,1 сек и }*С = 20 сел:-1, получим У(п\-. (°-1Р+1)Уо (0.1р+1)Уо _^ (р + 10) {/о ,.* 1 кю 0,1р2 + р + 20. 0,1 [(р + 5)2 + 13.22] (р + 5)2 + 13,2я * к ' Подходящая табличная формула (по Лапласу) (р + 6) (р + уУ+л' - Г ^ ~ V)2 + A2 e^ (W + *), (5) ^ = arctgy^. Из изображения (4) на основании формулы (5) получим У (0 - Уо ]А(10~1^2+ '3,2а g'M sin (13,2f + 69° 150 или у {t) = .%1,06&г5< sin (13,2/ + 69° 15')- Замечания. Применение табличных формул типа формулы (5) не является единственным способом перехода от изображения функции к ее оригиналу. Возможно, например, использование теоремы разложения.
|7S) . § 4.2. ИЗОБРАЖЕНИЯ ЛАПЛАСА И КАРСОНА - ХЁВИСАЙДА < НБ До перехода к оригиналу y(t) можно произвести проверку правильности изображения У(р) по некоторым признакам. Возможна, в частности, проверка изображения по его размерности. Изображение по Карсону— Хевисайду какой-либо функции, например у (О, оо Y(p) = p\ y(t)e-Pfdt, (6) 6 имеет ту же размерность, что и оригинал y(t). Это видно, например, из того, что изображение ступенчатой функции по Карсону — Хевисайду равно самой функции, т. е. Al(t)^A при t^O. Из выражения (6) слё* дует, что аргумент р изображения имеет размерность время'1. Размерность изображения функции по Лапласу Y(p) = L[y(t)]= \y{t)e-*dt (7) о равна размерности оригинала, умноженной на время, т. е. отличается от размерности изображения (6) по Карсону — Хевисайду множителем время. Применим эти. сведения о размерностях к проверке изображения по Лапласу (3) координаты у исследуемой системы. Правая часть выражения (3) должна иметь размерность произведения координата X время. Учитывая, что размерность р — это время'1, находим, что все слагаемые числителя выражения (3) имеют размерность координаты, а знаменателя — время'1. Следовательно, проверка по размерности дает положительный результат» Перейдем к другим видам проверки изображения. Непосредственно по выражению (3) можно найти начальное значение оригинала y(0) = limpY{p). (8) р-»оо Применяя (8) к (3), получаем у (0) = у у (0). По выражению (3) можно найти также предел оригинала y{t) при t—>-oo, если этот предел существует, по формуле lirri у (t) = Mm pY(p). . (9) (->оо р-»0
146 ГЛ'. 4. ПОСТРОЕНИЕ ПЕРЕХОДНЫХ ПРОЦЕССОВ - ЦП Признаком наличия указанного предела оригинала является расположение всех полюсов изображения F(<p) только в левой полуплоскости комплексного переменного р, т. е. вещественные части всех корней знаменателя функции Y(p) должны быть отрицательными. Для. выражения (3) это условие выполняется. Применяя (9) к (3), находим у(°°) = -^-=0, что заведомо правильно, так как из физических соображений следует, что в рассматриваемой задаче установившаяся ошибка равна нулю. Указанные виды проверки полученного изображения дают лишь необходимые условия правильности результата; однако практически эти условия часто являются и достаточными. 176. Передаточная функция разомкнутой системы равна w yfJ> (i + тФ) (1 + г2р) (1- + о,2р) (1 + o.oip) ' Найти переходную функцию замкнутой системы. Ответ. h (/) = (0,750 + 0,341е-ш - 1,091 е~ш) 1 (t). 177. Для системы предыдущей задачи найти закон движения при отсутствии , задающего воздействия при начальных условиях у(0) = уо и y'{G) = yrw Указание.. Решение является суммой двух слагаемых, одно из которых пропорционально у0, & другое — у'0; эти слагаемые удобно найти отдельно и результаты сложить. Ответ. у (t) = г/о [1,455-2Ы - 0,455e-w] + 0,0182^[е"25' - <Г8М] • 178. Для замкнутой следящей системы с передаточной функцией (см. задачу 174) I m/ ч 20 1 W[p)= o,lp2+p + 20 найти закон движения при задающем воздействии в виде ступенчатой функции gol (t) и при начальных условиях у(0) = Уо и #'(0) = 0. -
J80J § 4.2. ИЗОБРАЖЕНИЯ ЛАП ЛИСА И КАТКОВА -ЖЕВИСАПДА J 47 Ответ. y(t) = g0[l- 1.06~8e-«sln(13,2f+ 69° 150] + + y0l,068e-* sin (13,2* + 69° 150. I 179. Для следящей системы, которая в разомкнутом состоянии имеет передаточную функцию w (р)= £- = 2* . W) р (J + Тр) р (1 + 0,0067р) ' найти выходную величину y(t) при задающем воздействии-в виде ступенчатой функции g(t) — gG\(t) и при начальных условиях у (0) — у0 и у1(0) — у'0. Ответ. i у(1) = ЫI - 1 >333е-30' + О.ЗЗЗе-120'] + + у0 [ 1,333е-зш - 0,333e-12KJ + 0,011ly'0 [е-3® - е~^].' 180. Найти -закон изменения выходной величины y(t) замкнутой следящей системы при. ступенчатом задающем воздействии 1(f) и нулевых начальных условиях. Передаточная функция разомкнутой системы ХР(гЛг- !/<"(1+Ггр) _ 500(1+0,03р) ,,. w yfJ) p(I +Г,р)(1 +Г.Р) p(l+0,lp)(l+0;006p)" Kl} •Решение. Находим передаточную функцию замкнут той системы W(P) __ ' К(\ + т2Р) _ Ф(Р) = 1С« I СПЛ (2) Ц-^(р) р(1 + Т1Р)(1+Т3р\ + К(1+Г2р) 15р + 500 . О.ОООбр3 + 0,1 Обр2 + 16р + 500 Изображение по Карсону — Хевисайду искомой реакции системы на ступенчатое воздействие имеет вид Х(р) = Ф(р). (3) Далее, независимо от предполагаемого способа перехода от (3) к оригиналу, необходимо н#йти корни знаменателя изображения (2), т.е. корни уравнения О.ОООбр3 + 0,106р2 + 16р + 500 = 0. (4)
148 ГЛ. 4. ПОСТРОЕНИЕ ПЕРЕХОДНЫХ ПРОЦЕССОВ 1180 (5) В результате расчета, который здесь не приводится получаем следующие корни уравнения (4): Pl = _ 39,2 сект*, р2 = (-68,8 + /128,5) сек'1, р3 = (- 68,8 -/128,5) сек'1. Если теперь знаменатель изображения (2) представить, с учетом (5), в виде произведения О.ОООбр3 + 0,1 Об/?2 -И 6р + 500 = = 0,0006 (р + 39,2) Цр + 68,8)2 Ч- 128.52], то перейти к оригиналу можно при помощи таблиц изображений. Здесь используем другой способ перехода к оригиналу — при помощи теоремы разложения. Пусть искомая функция y(t) имеет следующее изображение по Кар- сону — Хевисайду: Y(p)- В(р) ... b0pm + blPm-l + D{p) + Ьщ-ip + bn a0pn + «iPn l + + an^p + atl (6) причем m ^ n и уравнение D (p) = 0 не имеет нулевых и кратных корней. Тогда согласно теореме разложения оригинал y(t) может быть найден по формуле y(t)- Pk, • В(0) ■ЖО) + A PbD' „Pk* (7) .,рп — корни уравнения, a D'(p)= где ри . •^jjD(p). В соответствии с (2) и (3), запишем Y{p) = 15р + 500 О.ОООбр3 + 0,1 Обр2 + 16р + 500 25 00Q (р + 33,3) р3 + 176,6р2 + 26 700р + 833 000 Сопоставляя (8) и (6), находим В(р) = 25 000(/? + 33,3), В (0)- 833 000, D (р) = рг + 176,6р2 + 26 700р + 833 000, D (0) = 833 000, D' (р) = Зр2 + 353/? + 26 700. (8) (9)
180] § 4.2. ИЗОБРАЖЕНИЯ ЛАПЛАСА И КАРСОИА - ХЕВИСАРЩА 149 Согласно теореме- разложения (7) получим ^ 25 000 (р.+ 33,3) „, «(0=1 +У—, 2 ;h—-—^ePk\ (io) itxр» (Зр*+ 353^+26 70°) Вычислим отдельно слагаемые, находящиеся в (10) под знаком суммы. При /?]=—39,2 сек~х получим — В(Р') рР,* = ~ 147,500 р-39,2^ = 091 fip-39,2( l\\\ р,£>'(р,) -39,2-17430 Ь v,-.ioe . UU Прйр2=(-68,8 + /128,5) сек-^Нбе'118010' сек"1 получим ВЫ pPj = 2,5-Ю4,133,5в'"105°25' -(.68.8+л28.5)* _ раД' (р2) 146е'"118°10' - 3,4 • 10V166°53' = 0,672е' <154°8')e-68.S/e/128,5t = 0,672e' d28,5<+154-8') е-ЬЬ,Ы, (12) При р3 = (-68,8 -/128,5) сек-1*- 146в'11в"10г сек-1 получим ffij еРз< = 0»672в~' (128.5i+154° 8') е-68,8^ . ( f g) Выражение (13) записано без расчета, непосредственно по виду выражения (12), так как корни р2 и р3 сопряженные, а коэффициенты в выражении (10) чисто вещественные. При этих условиях комплексные выражения (12) и (13) заведомо являются сопряженными. Если бы все корни уравнения (4) были вещественными, то выражения (11) —(13) не содержали бы комплексных чисел и расчет можно было бы закончить подстановкой этих выражений в формулу (10). В данном случае выражения (12) и (13) являются комплексными, поэтому их следует преобразовать. Применяя к сумме сопряженных выражений (12) и (13) формулу Эйлера е'а + е->'а ■ 2 = cosa, получаем _ 0,672е~68-8' \е' 0^,5t+l5V в') _|_ е~\ (128,5f+154° 8')] __ ' ." = = l,345e-e8-wcos(128,5* + 154° 80 - = - l,345e-68.8'cos(128,5^-25D52/). (14)
150 ГЛ. 4. ПОСТРОЕНИЕ ПЕРЕХОДНЫХ' ПРОЦЕССОВ [181 Подстановка функций (11) и (14) в формулу (10) дает реакцию системы на ступенчатое воздействие 1(0: y(t) = [1 + 0,216*г39-2' — 1,345е-68-« cos (128,5*-25° 520] 1 (*)• (15) Отдельные слагаемые этого уравнения и кривая y(t) построены на рис. 108. g(t)='[l*O^JM39'z4.3iSe'emcos{m,5t-Z5''SZ')}m) 0,01 0,02 0.03 004 0.05 О.Ов OJJ7 O,08.t.zeit ^[l-l,345e-Bmcosa28,5t-Z5°5Z')]l(t) Рис. 1Q8. Переходный процесс в следящей .системе с .астатизмом первого порядка ,при ступенчатом ' задающем воздействии. 181. Для замкнутой следящей системы, приведенной в предыдущей задаче, найти в общем виде изображение Y(p) по Лапласу и Карсону — Хевисайду выходной величины y{t) через изображение G(p) задающего, воздействия при ненулевых начальных условиях у{0)=у0, «/'(0) = У>у"(0) = ^'. Ответ. Изображение по Лапласу Y(p) = L[y(t)] = = [(15р + 500),G (р) + (6 • Ю-4?2 + О.Юбр + 16) у0 + + (6 •■:10"V+ 0.Ю6).^о + 6 • 1<ГУ] X X [6 • 10~У Н- 0,106р2 + 16р + 500]"1.
И4} § 4.2. ИЗОБРАЖЕНИЯ ЛАПЛЛГА: И КАРСОНА—Х^ВИСАЙДА 151 Изображение по Карсону — Хевисайду y(jp) = [(15p + 500)G(p) + p(6.10-V2 + O,106p + 16)y0 + • + р(6 • 10~V + 0,106)^ + 6- 10~ХJ X X [6 • Ю-У + О.Юбр2 -f 16р -f500]~'. 182. Найти закон движения системы, приведенной в задачах 180 и 181, при отсутствии задающего воздействия- и начальных условиях у{0) — уъ г/'(0) = 0 и Ответ. y{t) = уо [1.221в^*и-+-0,ЗЗБе-«* sin (128,5/ - 41° 45')]. 183. Следящая^ система, приведенная в задаче 180* имеет в разомкнутом состоянии передаточную функцию ,р,„ч К(1+ТгР) 500(1 +0,03р) w УР) р (1 + TlP) (1 + Т3р) р (1 + 0.1 р) (1 + О.ООбр) • Найти выходную величину y(t) замкнутой следящей системы при задающем воздействии в виде, импульс* ной' функции A6(t) при нулевых, начальных условиях; 6(f)—единичная импульсная, функция. Найти функцию веса w(t) системы. Ответ. y(t) = A [— 8,46е-39-2' + 196,4е-68-8' sin (128,5* + 2° 30')]? 184. Для замкнутой следящей системы, передаточная функция которой в разомкнутом состоянии равна ш (D) — Е — ?й найти, выходную величину y(t) при линейном задающем воздействии g(t) = atl(t) и нулевых начальных условиях. Решение. Передаточная функция замкнутой системы равна (Ъ{ \ — _EiEL_ 3600 m W(p) — i + Г (р) ~ р2 + 150р + 3600 • . . к '
152 'ГЛ. 4. ПОСТРОЕНИЕ ПЕРЕХОДНЫХ ПРОЦЕССОВ " (184 Изображение задающего воздействия по Лапласу С{р) = ^г. (2) Согласно (1) и (2) изображение выходной величины по Лапласу равно У(Р) = ф.(р)С(Р)-рЧр2+^+зш). (3) Для отыскания оригинала выражения (3) можно использовать теорему свертывания, согласно которой t y(t)=]"xl(f)x2(t-x)dx, (4) о если Y(p)^Xl(p)X2(p) (5) хЛ*)ФХЛр), (6) x2(t)=x2{p). . m В соответствии с (5) изображение (3) следует разбить на два сомножителя с таким расчетом, чтобы произведение их" оригиналов легко интегрировалось. Выберем эти сомножители следующил* образом: VI \~ ' 3600 а_ КР' р (р2 + 150р + 3600) ' р ' т. е. у / \ = 3600 3600 ,ft. 1 (Р) р (р2 + 150р + 3600) р (р + 30) (р +. 120) ' ^ Х2{р) = ~. ^ (9) Знаменатель выражения (8) разложен на множители обычным путем. Для выражений (8) и (9) подбираел* подходящие формулы из таблицы изображений по Лапласу 1 • ' ■ а-—I—, (Ю) р(р + а)(р + р) • ар 7^Н0-~ (И)
1851 § 4.2. ИЗОБРАЖЕНИЯ. ЛАПЛАСА» II KAPGOHA— ХЕВИСАПДА 153 Теперь из. (6) —(11) найдем - х, (0 = (1 - 1,333е-зм + О.ЗЗЗе-120') 1 (/), (12) x2(t) = al{t). (13) Подставим оригиналы (12) и (13) в формулу (4) теоремы свертывания: t у,(/)= J [1 - l,333e-30t + 0,333e->^]. [a\{t-x)\dx. (14) о Интегрируем (14): у (t) = а \х + 0,0445<гзот - 0,00277е-,20т)[. Отсюда искомое решение задачи: y{t) = a(t + 0,0445e~3W - 0,00277<rIZ°f - 0,0417) 1 (t). Задающее воздействие g (t) и выходная велич«н-а# (t) построены на рис. 109. дт уШ а ОТО 0,08 00 ОМ ОМ г / > ф / /у / 7 /у * n'-rTiT .. ...I i— \ j '.—1 О 0,02 ОМ 0.00 0,10 ОМ С,сек Рис. 109. Переходный процесс в следящей системе с астатизмрм первого порядка при линейном г" задающем воздействии. 185. Передаточная функция разомкнутой следящей системы равна ■ • ,. W/ ( \ — К ^ 24 • {р) р(1 + Тр) р (1 + o.OOWo) '
154 i Г*: 4.i ПОСТРОЕНИЕ ПЕРЕХОДНЫХ5 ПРОЦЕССОВ! ГШ Найти ошибку x{t) = g(t)^-y(t) замкнутой следящей- системы, при:нулевых начальных условиях для задающего воздействия двух видов: 1) при ступенчатом воздействии g(t) = gol(t)\ 2.) при задающем-.* воздействии, нарастающем по линейному закону g{t) = at\ (t). Указание. Передаточная функция следящей системы относительно ошибки равна , 1 Ф* (/>) = • 1 + W (р) Ответ. 1) х(0 = gb(1.333e-30f —0,333erI200 1 (*), 2) x {t) = a (0,0417 - 0,0445е~зм — 0,00277e-"°0 1 (/). Ошибка>для,-обоих-.случаев.;построена, на. ррс. ПО. 1,0 0,8 ■ х а 0,04 ом 0,0 -0,4 0,2 к \ \ \^- ' "~ '/^Л<^_.. О О Q.OZ ОЛВ- 008 0,10 0,12 t,cen Рис. НО. Графики ошибки следящей системы с астатизмом первого порядка при. ступенчатом (кривая 1)-и линейном (кривая.: 2) задающих воздействиях. Правая шкала оси ординат-относится к" кривой 1{ левая шкала — к; кривой12. 186. Следящая система, схема которой дана на рис; 111, имеет- в ■ разомкнутом■ состоянии передаточную функцию
1881 § 4.2: ИЗОБРАЖЕНИЯ ЛАПЛАСА И КАРСОНА - ХЕВИСАЙДА 155 Система состоит из.двух звеньев, между которыми прикладывается возмущение f'{t). Найти выходную величину У (t) Для ступенчатого возмущения /4*)='foV(0 пРи отсутствии задающего воздействия g(t) и нулевых начальных условиях; К = kik2 = 24 сек'1, Т = §,7 мсек, k2 = 0,01 в^сек'1. Последний коэффициент записан am к, 1+Тр гш р ' Рис. 1U. Структурная.схема следящей системы к задачам 186 и 188. в предположении, что координаты у(1) и g(t) безразмерные, а входные величины второго звена, в том числе возмущение f(t), имеют размерность напряжения. Ответ. у (t) == 10"Vo [4,17 - 4,45e_3w + 0,278е"12Ш] 1 {t). 187. Следящая система состоит из двух звеньев, показанных на рис. 111; W(p) k,k2 100 p(l + Tp) p(l+0,025p) ' На вход второго звена действует возмущение в виде импульсной функции f (t) = Лб (t); , задающее воздействие g(t) отсутствует, начальные условия нулевые. Найти выходную величину y(t) замкнутой следящей системы. Ответ. 1/(0 = M1.053e-20*sm(60^-7l°34') 1 (t). 188. Передаточная ^функция .разомкнутой .'.следящей системы равна
156 ГЛ. 4. ПОСТРОЕНИЕ ПЕРЕХОДНЫХ ПРОЦЕССОВ 1183 где /С=.400 сек'2, Т = 0,01 сек. Найти выходную величину y(t) замкнутой системы при ступенчатом \ задающем воздействии g(0 = goHO и нулевых начальных условиях. Ответ. У М = go [1 + l,053e-2W sin (60* -71° 34')] 1 (/). 189. Даны две следящие системы, имеющие в разомкнутом состоянии передаточные функции: 1) wap)- ■*' 2) w2(p)- Pd + TlP) > KAL+Лр) где /С, = 100 сек-1, Г, = 25 мсек, К2 = 4000 сек'2, Г2 = = 10 мсек. Найти выходные величины y(t) и ошибки хд) а цтз омш о.ом от 0.010 олов аот лорг о -0002- t,cen ^=l(tt[0№0memsw(B0t-36~S0')} ^4(t)D.OiB7e'msineOt 002 004 '0,10 0J2 OJt 0.16 aiBt.ce/f Рис.- 112. Ошибки при задающем воздействии g (/) = = al • 1 (t) для следящих систем с астатизмом первого порядка xt (t) и с астатизмом второго порядка х% (/). x(t) = g(t) — y(t) замкнутых следящих систем при линейном управляющем воздействии, g (t) — at\ (t), и нулевых начальных условиях. Построить . на одном графике кривые ошибок для этих систем.
192! § *-8" ИЗОБРАЖЕНИЯ ЛАПЛАСА ft КАРСОНА — ХЕВИСАЙДА 157 Ответ, Ул (t) = a [t — 0,01 — 0,0167е-20' sin (60* — 36° 50')] 1 (/), Xl (t) = a [0,01 + 0,0167e-2W sin (60* - 36° 50')] 1 (Q, y2{t) = a[t —0,0Ше~*>(sin№t] l (t), *2(f) = aO,0167e-2Wsin60f 1 (t). Кривые Xi(t) и xz(t) построены на рис. 112. 190. Замкнутая система автоматического регулирования описывается уравнением (0,1479р4 + 3,7р3+ 15,61р* + 17,9р + 20)у(*) = .=(17,9p + 20)g(/). Найти выходную величину y(t) при ступенчатом задающем воздействии g{t)*=goHt) и нулевых Начальных условиях. Ответ. #(0==Ы1 + 1.456e-0■s'sin(l,2/-72°) + -f 0,398е-« — 0,019е-20'] 1 (t). 191. Найти выходную величину y[t) системы, данной в предыдущей задаче,' при отсутствии задающего воздействия и начальных условиях у(0) — уо, у'(0) = Уц, Ответ. '0(О = #о[1.2О2е-о-5'(1.2* _ 45°) + 0,155е~« - 0,005-2И] + +^[1,112e-°-5'sin(l,2jf - 4°50') + 0,099е-«-0,005е-м<] -f +< [0,283е-°-5' sin (1,2t — 19° 30') + 0,096e-«-0,002e-20<] + + ^"[0,288e-0-3'sin(l,2/~22° 15') + 0,114e-«— O.OMe-20']. 192. Найти переходную h(l) и весовую w(t) функции системы, передаточная функция которой равна где п — целое положительное число. Указание, Следует воспользоваться теоремой свертывания.
158 ГЛ. 4. ПОСТРОЕНИЕ ПЕРЕХОДНЫХ ПРОЦЕССОВ Ц8Э Ответ. А (*)■=* w(t) = а" М. an~kkl К («-1)1 ft=0 м-l -at t e . § 4.3. Приближенные методы расчета переходных процессов А. Использование вещественных частотных характеристик [31, 32] 193. По вещественной частотной характеристике Р (<о) системы регулирования {рис. 113, а) построить кривую переходного процесса при единичном,ступенчатом воздействии и нулевых начальных условиях. Решение. Кривая Р(<а) приближенно заменяется несколькими трапецеидальными кривыми с таким расчетом, чтобы сумма ординат трапеций равнялась ординате вещественной частотной характеристики Р (to). В данном случае можно взять четыре трапеции, показанные на рис. 113, б; одна из них положительная5 остальные отрицательные. Каждая трапеция должна иметь типовой вид, показанный на рис. 113, в; тогда она полностью определяется тремя числами: частотой среза сос, коэффициентом наклона % = (ajd)c и высотой г. Трапеции на рис. ИЗ, б имеют параметры, указанные в табл. 1. Таблица I № трапеции ¥ = — * сос юС) сект1 г 1 0,62 16 1,62 2 0,31 5,5 -0,09 3 0,46 ■61,5 -0,24 4 0,70 28,5 -0,29 Далее следует воспользоваться таблицами функций h[t0).
ffaj - • ■ —«n w\ го 30 40 -% SO 60 ш-4и 1.2 ЦВ Ofi 0 -0.4 L a) 5>- 'сея PflM Рис. 113. Приближенная замена вещественной частотной характеристики суммой трапецеидальных функций частоты и получение кривой переходного процесса.
160 ГЛ....А, ПОСТРОЕНИЕ ПЕРЕХОДНЫХ ПРОЦЕССОВ 1Щ Функция h{t0) представляет собой кривую переходного процесса системы, вещественная частотная характеристика которой — единичная трапеция, имеющая г = + 1 и сос= 1 сек'1. Табличные функции h (t0) приводятся для различных коэффициентов наклона 0<:%<:1, причем допускается интерполяция, если % лежит между двумя табличными значениями. Сокращенная таблица таких функций дана в приложении 35. Возьмем таблицу h(t0) функций для % = 0,62 (коэффициент наклона трапеции 1) и выпишем ряд значений времени t0 и функции h(t0) (см. первые две строки В табл. 2). Для получения точек кривой у (t) переход* }гого процесса, соответствующего неединичной трапеции, Каждое значение функции h{tG) следует умножить на высоту трапеции г, а время t0 разделить на частоту среза (ос, т- е. В третьей и четвертой строках табл. 2 даны числа t и yi(t) для трапеции 1. Аналогично получаем г/2(0. #з(0 и %(0 Для остальных трапеций (см. таблицы 3—5). По данным таблиц 2—5 на рис. 113, г построены графики y\(t), #г(0. y&{t) и y4[t). Складывая ординаты этих кривых с учетом их знаков, получаем на рис. 113, г кривую"у(t) переход? ного процесса в заданной системе при единичном ступенчатом воздействии. На рисунке указана также величина yyct= lim y{t). t-> 00 В случае неединичного ступенчатого воздействия #(0 = &о1.(0 ординаты кривой y(t) следует умножить на g0. 194. По вещественной частотной характеристике системы регулирования (рис. 114, а) построить кривую переходного процесса y(t) при задающем воздействии g'(0 = g'oHO и нулевых начальных условиях. Ответ. Кривую Р(а) можно заменить двумя трапециями, показанными на рис. 114, а пунктиром. Данные трапеции 1: х = 0>78> сос = 79 сек'1, г = 0,688; данные трапеции 2: % = 0,84, coc = 95 сек'1, г = 0,2.
§ 4.3. ПРИБЛИЖЕННЫЕ МЕТОДЫ РАСЧЕТА 161 Таблица 2 Трапеция 1 'о А Со) /, сек 01 «) /о Л Со) t, сек yAt) 0 о о 0 4,4 1,15 0,275 ,186 0,2 | 0,4 | 0,8 | 1,0 | '1,6 2,6 3,0 0,10 0,20" | 0,40 0,50 0,75 1,04 1 1,11 0,0125 0,025 0,050 | 0,0625,' 0,100 0,162 | 0,188 0,17 | 0,33 0,65 0,81 1,21 1,68 1,80 4,8. 5,4 I 6,0 | 7,0 7,8 | 9,0 1,12 1,07 | 1,01 | 0,95 | 0,94 0,96 0,300 | 0,337 0,375 1 0,438 |' 0,488 | 0,562 | 1,82 | 1,73 1.64 | 1,54 | 1,52 1,56 -| 1 4,0 1,16 I 0,250 1 1,88 10 1;00 0,625 1;62 Таблица 3 Трапеция 2 t,ceK 0 0,109 0,218 0,364| 0,54б| 0,72в| 0,822| 1,09 1,27. № V) 0 -0,022|-0,043 -0,067.1-0,08б|-0,096 -0,098 -0,096 -0,094 Таблица 4 Трапеция 3 /, сек й(0 /, сек ЫО 0 1 0 0,065 -0,271 0,0065 0,0163 0,026 | 0,0325 -0,043 1-0,108 | -0.163 -0,194 0,0813 1 0,0976 | 0,114 I -0,269 | -0,254 -0,242 | 0,0488 -0,25 0,13 -0,235 Таблица 5 л сек У At) t, сек Трапеция 4 0 0,0 н| 0,028 0,042 0,070 0,105 0 | -0,064 -0,122| -0,467| -0,267 -0,328 [ 0,210 0,246 | 0,281 | 0,316 | 0,351 0,133 -0,339 0,386 0,178 -0,314 0,458 -0,284 [-0,27 1-0,27 1-0,284 |-0,296 | -0,3021 -0,29Q
162 ГЛ. i. ПОСТРОЕНИЕ ПЕРЕХОДНЫХ ПРОЦЕССОВ [195 По этим трапециям построены кривые y^t) и y2{t) на рис. 114, б; на этом же рисунке дана искомая функция у (t) для случая go=l. При g0 Ф 1 ординаты кривой y(t) следует умножить на g0. ' КО си,^ а) 0,02 Ofit 0,00 0,08 0,10 OJZt.cen б) Рис. 114. Вещественная частотная характеристика Р (о) и кривые переходного процесса х (t) к задаче 194. 195. Построить кривую y{t) переходного процесса замкнутой системы при ' задающем воздействии g{t)—l(t) и нулевых начальных условиях. Передаточная функция разомкнутой системы W(p) = К(1 + ТзР) (-1 + 2Г,р + ГУ)(1 + Т2р) (I + TiP) 5(1+0,03р) (—1 + 0,2р + 0,01р2) (1 + 0,05р) (1 + 0,006р) У к аз а н и е. Возможно использование результатов решения задач 73 (Б) и 79.
-"§ 43. ПРИБЛИЖЕННЫЕ :№ГОДЫ РАСЧЕТА ут г.4 гл 1.Б 12 ав ОА О Л у<Ь 0,7 ffycm -Дв б) Рис. ]15. Вещественная частотная характеристика и кривая переходного процесса к задаче 195.
164 ГЛ. 4. ПОСТРОЕНИЕ рЕРЕХОДНЫХ ПРОЦЕССОВ Г«В9 Ответ. См. рис. 115,-б. Кривые г/1,2,3.4(0 построены по четырем трапециям, показанным на рис. 115, а. 196. Построить кривую у (г) переходного процесса замкнутой системы при задающем воздействии g{t)=l{t) и нулевых начальных условиях. P(W) 08 0,4 О -04 ■\ /оо \фо - 300 400 ' ' -- -^ 500 i "■ш уШ IS и 0,8 0,4 О ~Ofi а) Рис. 116. Вещественная частотная характеристика и кривая переходного процесса к задаче 196. Передаточная функция разомкнутой системы W(p)~ — К(1±1?Р) 500(1+0,03р) р(1 + Г,р) (1 + Гзр) р (1+0,1 р) (1+0,006/)) • Указание... Возможно использование результатов решения задач 67 и 77. Ответ. См. рис. 116, б. Кривые #|,2,з(0 построены по трем трапециям, показанным на рис. 116, а.
Htfl "'' § 4.3. ПРИБЛИЖЕННЫЕ МЕТОДЫ РАСЧЕТА 165 ■ Б. Использование сопрягающих частот л. а. х. [32] 19.7. Передаточная функция разомкнутой системы раВНа ■ Г(р)= *(. + г,р) W W р(1 + Г,р)(1 + Г3р) ' (О где К = 500 сек"1, Г, = 0,1 сек, Г2 = 0,025 сек, 7/3 = = 0,0025 се/с. Построить приближенную кривую ошибки x(fy = g(t) — y(t) системы при единичном ступенчатом воздействии gU)=l(0 и нулевых начальных условиях. Построение произвести по сопрягающим частотам лога» рифмической амплитудной частотной характеристики, t Ж) уф ' М 0,6 Ofi 0.2 О ~02\ \\ xpt) Л v \ * \ * \\ \\ Ц01 дог к " 003 Ц04 ' 005 0,06 0.071,сек 0) Рис. 117. Л. а. х. и кривая переходного процесса к задаче 197. Р е.ш е н и е. Л. а. х. системы построена на рис. 117, а. Эта л. а. х. удовлетворяет условию, состоящему в том, что'протяженность ее участка, пересекающего ось частот с наклоном —20 дб/дек, должна составлять не менее одной декады; поэтому построение искомой- кривой по сопрягающим частотам л. а. х.. является . возможным.
'166 -v- - ГЛ. 4. ПОСТРОЕНИЕ ПЕРЕХОДНЫХ ПРОЦЕССОВ f!97 Определяем частоту среза л. а. х. непосредственно т по л. а. х. или по формуле g>c = K-jt-, следующей из рисунка; ©с = 125 сек'1. В соответствии с методом использования сопрягающих частот л. а. х., отбрасываем в л. а. х. всю ее часть, лежащую правее частоты среза, и заменяем ее горизонтальным участком, совпадающим с осью частот. Этой новой преобразованной л. а. х. соответствует передаточная функция ^М- p(1+V„) " '. (2) или W (р) = асКТ* }. = г'р(гГ+р) (р + со2) (р + сос) (р +40) (р + 125) ,„ p(p + o>i) P(P+Ю) ' ^' где «!= 1/Г,, со2= 1/?V Формулам (2) и (2') соответствует преобразованная передаточная функция системы относительно ошибки Of (ri\ - ' = р (р +10) - И) ^ IW - w* (р) (р +40) (р + 125) • _ к°> Учитывая изображение Лапласа G(p) — -~ для воздействия g (t) = 1 (t), находим изображение Лапласа для первого приближения x\(t) функции x{t) ш=ФЧР)о(Р)-(р+^(1;+ту . (4) Из таблиц изображений по Лапласу находим подходящую формулу: (5) р + 6 . (6-a)e-af-(6-P)e-pf 0> + о)(р + р)- P-a Формулы (4) и (5) дают ответ для первого приближения ошибки системы: х\ (0 = 1,353<г1Я - 0,353е-4М. (6) Эта функция построена сплошной линией на рис. 1.17, б.
198T '§ 4.3. ПРИБЛИЖЕННЫЕ -МЕТОДЫ РАСЧЕТА 167 Для получения второго приближения x\{t) искомого решения следует ординаты кривой x\{t) умножить на поправочный коэффициент р в диапазоне Т3 < t < Т2, т. е. 0,0025 сек < t < 0,025 сек. Этот коэффициент определяется из формулы WO0-I i + wWlp-^ или, согласно (1) и (2), (р + 40)(р+125) Р = р(р +10). 500(1 + 0,025р) 1+ р(1+0,1р)(1+0,0025р) р=/125 I (/125 + 40) (/125 + 125) (/125 + 400) I . .„- _ I 50 000 (/125 + 40) + /125 (/125 + 10) (/125 + 400) | 1>4ео« Второе приближение решения построено на рис. 117, б пунктиром. На этом же рисунке крестиками отмечены точки, относящиеся к точному решению. В. Использование нормированных кривых для минимально-фазовых систем с типовыми л. а. х. [2, 4, 5] 198. Передаточная функция разомкнутой следящей системы равна W(n\- K-(l+Tg>) _ 100(1+0,160р) КР) р2 (1 + Г3р) (1 + 7» р2 (1 + 0,024р) (1 + 0,004р)' Построить график выходной величины y{t) при ступенчатом ' ' задающем воздействии g(t) = g0l(t), g0 = 10 град и нулевых начальных условиях. Решение. Строим логарифмическую амплитудную характеристику заданной системы (рис. 118, а). Согласно приложению 19 находим, что эта л. а. х. является ти- повой_симметричной, тип 2-1-2-3. Базовая частота ©о^ & YК = )Л00 = Ю сек'1. По приведенной в приложении 19 формуле находим показатель колебательности системы m — l 5,7—1 ' '
168 ГЛ. 4. ПОСТРОЕНИЕ ПЕРЕХОДНЫХ ПРОЦЕССОВ [198 где т ~ ■ 2 г, Т, + Т. 0,16' 0,024 + 0,004 5,7. »=з При М ~ 1,4 искомая кривая y(t) переходного процесса должна быть построена как промежуточная между 1,дб ' 40- 10 \ Юград ■ ■ ш 6) 0.3 t, /М сен Рис. 118. Л. а. х. и кривая переходного процесса к задаче 198. • нормированными кривыми,.приведенными для М = 1,3 и А1 = 1,5 в приложении 20, рис. П. 12. При переходе от
2011 § 4.3. ПРИБЛИЖЕННЫ» МЕТОДЫ РАСЧЕТА '' ■ [69 нормированной кривой -~{а>й() переходного процесса к Яо y{t) следует абсциссы нормированной кривой разделить на ©о= Ю сек'1, а ординаты умножить на g0 = 10 град. В результате получаем кривую y(t), построенную на рис. 118, б. 199. Передаточная функция разомкнутой системы равна w / ч = . 250(1+0,024р) W КР} (1 + 0,2р)* (1 + 0,0024р) (I + О.ОШбр) • Построить кривую y(t) переходного процесса в замкнутой системе при ' задающем воздействии g {t) = 1 (t) и нулевых начальных условиях. Ответ. Кривая y{t) переходного процесса может быть получена приближенно из нормированной кривой переходного процесса для симметричной л. а. х. при М = 1,4; базовая частота % = 79 сек"1. 200. Передаточная функция разомкнутой системы равна vgri„\- 400(1+0,04р) w УИ' р (1 + 0.1/)) (1 + 0,003р) (1+ 0,0008р)2 " Построить кривую переходного процесса y(t) в замкнутой'системе при ступенчатом задающем воздействии g(fy~gol(t)> go = 0>5 и нулевых начальных условиях. Ответ. Кривая y{t) может быть приближенно построена по нормированной кривой — (ш0/) для М = 1,3; Йо базовая частота ш0 = 63;2 сек'1. 201. К следящей системе, имеющей в разомкнутом состоянии передаточную функцию W{p>~ рЧ\+Пр)' К =^400 сек~2, Г2 = 0,078 сек, 7'3 = 0,020 сек, прикладывается при нулевых начальных условиях задающее воздействие g(t)*=a-t-l(t), a = 20 град/сек. Построить график ошибки воспроизведения этого . 'задгк>щего воздействия.
JfQ ГЛ. -t. ПОСТРОЕНИЕ ПЕРЕХОДНЫХ ИРОЦЕССОВ Г2М •Решение. Строим л. а. х. системы {см. Li(to) на рис. 119, а). Л. а. х. относится к типу 2-1-2... (см. L,BS 40 го I 2 f 4 6 10 щг20се1Гу№. 1 а) а),сен сен Рис. 119. Логарифмические амплитудные характеристики и кривая ошибки воспроизведения линейного задающего воздействия к задачам 201 и 202. ■последнюю строку табл. 1 приложения 19). Базовая частота л. а. х. ю0 = У К, — j/400 = 20 сек'1. Показатель колебательности М= "±1 = 41=1,7, ■" т—\ 2,9 ' '
§ 4.5. ПРИБЛИЖЕННЫЕ МЕТОДЫ РАСЧЕТА 171 где Искомая кривая ошибки системы определяется нормированной кривой ошибки воспроизведения линейного задающего воздействия, приложение 20, рис.. П. 14 (часть Б, случай 1). Искомая кривая ошибки x(t) (см.. рис. 119,6) получается из нормированной кривой для М — 1,7 делением абсцисс последней на величину шо = 20 сект1 и умножением ординат на отношение о/со0 = 10/20 = 0,5 град. Ошибка стремится к нулю, поскольку система обладает астатизмом второго порядка. 202. „К следящей системе, имеющей в разомкнутом состоянии передаточную функцию w W) р(1+Г,р)(1+Г»р)* где К =133 сек~\ Т = 0,333 сек, Т2 = 0,078 сек, Гз = 0,020 сек, прикладывается при нулевых начальных условиях задающее воздействие g(t) = a -t- l(t), a = = 20 град/сек. Построить график ошибки воспроизведения этого воздействия. Ответ. Приближенное решение имеет вид графика на рис. 119, б, полученного из нормированной кривой приложения 20, соответствующей л. а. х.' системы Lz(a>), изображенной на рис. 119, а. Установившаяся ошибка равна лгуст = -jT = -jgg- = 0,075 град = 4,5 уел. мин. 203. Передаточная функция разомкнутой системы равна ;'500(1+0,03р) W(p): р (L + 0,1р) (1 + ОЛЮбр) 1. Использовать нормированные кривые переходных процессов для построения графика выходной величины y(t) замкнутой системы при единичном ступенчатом задающем воздействии и нулевых начальных условиях.
172 ГЛ. 4. ПОСТРОЕНИЕ ПЕРЕХОДНЫХ ПРОЦЕССОВ f204 2. Решить эту же задачу точно (классическим или операторным методом), а также используя вещественную частотную характеристику системы. Построить все три решения на одном графике. Указание. Во второй части задачи можно воспользоваться решениями задач 180 и 196. ую 1,Е- 0,8 - Ofi - О 0,01 0,02 0,03 0,04- Ц05 0,06 t,CBK Рис. 120. Кривые переходного процесса к задаче 203, полученные тремя способами: по нормированным кривым -^— (co0i!) — сплошная линия, с помощью So точного решения — крестики и по вещественной частотной характеристике — кружки. Ответ. На рис. 120 построена кривая y(t), найденная приближенно по нормированным кривым переходных процессов, при а>0 = 70,7 сек~1 и Л1 = 1,5. Точки, относящиеся к точному решению, отмечены крестиками, а точки, полученные по вещественной частотной характеристике, кружками. Г. Построение кривой переходного процесса графическим методом 204. Построить график выходной величины y(t) в системе, описываемой уравнением. flo-§- + «i# = W) или 2-g- + 20t/ = 5g(0, (1) где 'задающее воздействие g(t) задано графически на рис. 121, а. Размерности y(t) и g(t) одинаковы. Начальное условие у (0) = —2. Использовать графический метод Д. А. Башкирова [3].
£041 § 4.3. ПРИБЛИЖЕННЫЕ МЕТОДЫ РАСЧЕТА 173 г dU Решение. Запишем (1) в виде Т~ + У = gx(J), где постоянная времени Т = а0/а1 = 2/20 = 0,1 сек, а возмущающее воздействие gi {t) = (b0!a,) g {t) = 0,25g (t). Построим на рис. 121, б две системы координат: t, y{t) y(t) 6 и г д -г _ ~м fit л сг £—ЩШ 9 с« ,Ю 0, cL / се Ш г5 ^L \а &-, 3 £в__ '/>- * 6 ч& цг d, ym 41 т 41 dm Q3 t.ce*. б) Рис. 121. Построение кривой переходного процесса у (t) графическим методом. и ^ &i(0> имеющие одинаковые масштабы, причем оси времени обеих систем совмещены, но начало Oi отсчета gi(t) сдвинуто вправо относительно начала О отсчета y(t) на величину Т. Построим согласно формуле для gi(t) и рис.. 121, с функцию gt(t). Выбираем шаг интегрирования At = T/n, где п — целое число. Возьмем At = 0,020 сек и разделим.графики
174 ГЛ. 4. ПОСТРОЕНИЕ ПЕРЕХОДНЫХ ПРОЦЕССОВ [205 н-а. рис. 121,6 на участки ло 0,02.0. сек. На графике функции g, (?) отметим точками du d2, d3, ... значения этой' функции, имеющие место в середине каждого участка. Отложим на графике у it) начальное значение i/(0) = —2 и соединим прямой линией полученную точку Ci с точкой d'\. Пересечение прямой cxd\ с абсциссой конца первого участка дает вторую точку с2 искомой кривой. Проводя прямую с2Й2. получаем точку с3 на пересечении этой прямой с абсциссой конца второго участка и т.д. Искомая функция y(t) определяется как плавная кривая, соединяющая точки си с2, с3, ... 205. Построить график выходной величины y{t) в системе, описываемой уравнением a0-Jf + ctiy = b0g{t), а0=1 сек, ^ = 20, 60=12 при i/(0) = 0,25 и задающем воздействии g(t), задан- yft) 0,3 о,г 0.1 -0.1 ■25 ■50 75 100 125 150 175 ZOO 1.ЖВК Рис. 122. Кривая переходного процесса к задаче 205. ном в виде таблицы .задающее воздействие имеет размерность выходной величины) /, мсек g(.t) t, мсек em 0 0 60 +0,900 5 —0,300 «5 +0,968 10 —0,466 75 + 1,000 15 —0,584 85 +0,950 20 —0,640 1С0 +0,759 25 —0,637 115 +0,564 30 —0,559 125 +0,475 35 -0,350 140 +0,387 45 +0,300 160 +0,350 50 +0,575 oo +0,334 Ответ. См* рис. 122.
206] ГЛ. 4. ПОСТРОЕНИЕ ПЕРЕХОДНЫХ ПРОЦЕССОВ 175 206. Построить график выходной величины в системе, описываемой уравнением d2y l° dt2 + al-^ + a2y = b0g(t), а0 = 0,048 сек2, ai = 0,4 сек, . а2= 10, Ь0 = 5 при #(0) =—1.5, уг(0) = 75 сек ' и задающем воздействии g (t), показанном на рис. 123, а. Рис. 123. Управляющее воздействие g (t) и выходная величина y(t) системы задачи 206. Указание. Заданное уравнение следует привеста к виду Ответ. См. рис. 123, б.
ГЛАВА S ОЦЕНКА КАЧЕСТВА РЕГУЛИРОВАНИЯ § 5.1. Определение точности при наличии задающего воздействия 207. Передаточная функция замкнутой следящей системы имеет вид ф lp) ^ boPm + ftlPm""' + ■ • • + Ьщ-iP + Ьт а0рп + a,bn~l + ... + ап-\р + «в Каковы условия получения: 1) астатизма нулевого порядка; 2) астатизма первого порядка; 3) астатизма второго порядка? Ответ. , 1) Ьтфап; 2) Ьт = ап; Ьт^фап.и 3) bm = an, bm-\ = c„_i, от^2:Фо-п-2- 208. Передаточная функция разомкнутой следящей системы рис. 124 имеет вид w(p)= А°рт + A,pml + • • •+ Ат-'р+Ат В0рп + В]рп-*+...+Вп-1р + Вп ' Каковы условия получения: 1) астатизма нулевого порядка; 2) астатизма первого порядка; 3) астатизма второго порядка? Ответ. 1) ВпФ0; 2) Я„ = 0; . .. . 3) Вп = 0 и £„_1 = 0. - " . .
Sill § 5.T. ТОЧНОСТЬ ПРИ ЗАДАЮЩЕМ ВОЗДЕЙСТВИИ 177 209. Передаточная функция разомкнутой следящей системы рис. 124 имеет вид is Г(р)= p(l + TlP)(l + T2p)' Определить первые три коэффициента ошибки, а также добротность по скорости. Решение. Находим передаточную функцию относительно ошибки: <Мр)= i + w(P)~ р(1 + 7-,р)(1 + 7-2р) p(l + TlP)(l + T2p) + K 7-+<&Z*. W(p) —р4^- Рис. 124. Следящая система. Разлагаем это выражение в ряд делением числителя на знаменатель К + Р + (Г, + Г2) р' + Т,Т2р> г + а-.+мрз + г.пр» . ~~ 1 « ■ Г| + 7"г , , Т\ Т? . р+жр+—— р1+-1гр ■ -(Ii+ri_^)p,+(rIr,-ii±Ii)p.-JfLP. Далее можем написать тождество Отсюда получаем коэффициенты ошибок: с2 с0 = 0, сх = -дг [сек], 2 Добротность по скорости: ^(7\ + Г2-^)[СеК2]. ^0 = 7" = ^ tceir']. 210. Для предыдущей задачи определить численные значения коэффициентов ошибок, если К = Ю0 сект1, ^1 = 0,01 сек и Г2= 0,005 сек. Ответ, с, = 0,01 сек и -j^0»00005 сек2- 211. Определить величину установившейся ошибки для предыдущей задачи при движении следящей системы со скоростью Q = 12 град/сек.
178 ГЛк -В- аЦЕНКА1КАЧЕСТВА, РЕГУЛИРОВАНИЯ [2«2 Ответ. еУст = -^ = Ф = 0,01 • 12 = 0°,12 = 7',2. 212. Передаточная функция замкнутой системы (см.. рис. L 24)., имеет вид w O.OOIjd3 + 0,502р2 + 6р + 200 * Найти установившееся значение ошибки (после затухания переходного процесса) при изменении входной величины по закону g{t) = 5 + 20t+10t2. Решение. Находим передаточную функцию-относительно ошибки: ft, I V- 1 сып\..- 0,001р3 + 0,502р2 + р WAPI- 1 ^ЛР) - о,001р3 + 0,502р2 + 6р + 200"' Делением числителя на знаменатель (см. задачу 209) находим коэффициенты ошибок: с0 = 0, ^="200 сек и -Т = 0,00236 сек2. " Далее находим производные - g' (0 = 20 + 20/, £"40 = 20, Выражение для ошибки имеет вид х (0 = c0g (t) + clg' (t) + Ц- g" (0 = 2 20 + 20< 200 + 20 • 0,00236 = 0,1472 + 0,R 213. Передаточная функция разомкнутой системы (см. рис. 124)' имеет вид wr/..i_. 50(l+0,15p) WyP>~ p»(i + 0,02p) ' Определить первые три коэффициента ошибки, а также добротность по скорости и добротность, по ускорению.
grtl ' § 5.Ь ТОЧИОСТЬ-ПРИ 'ЗАДАЮЩЕМ -ВОЗДЕЙСТВИИ 179" Ответ. с0 = 0, с, = 0, -^- = 0,02 сек2; добротность по скорости Ка-*00, добротность по ускорению /Се = 50 сект2. 214. В статической системе регулирования {рис. 125, а) передаточная функция разомкнутой системы имеет вид К / W W = (1 + Г,Р)(1 + ад ' Г^, ч Определить коэффициент передачи ■§(р) = т неединичной обратной связи, при котором система приобретает т, 'У Wfp) $($)-*>■ У а) Рис. 125. Статическая система с в.еединичкой обратной связью. астатизм первого порядка, и передаточную функцию разомкнутой эквивалентной системы с единичной обратной связью (см. рис. 125, б). Решение. Передаточная функция замкнутой системы равна [2] К T&pt + iTt + TJp + l + mK (О Условие отсутствия статической ошибки. Ф (0) = 1 или К — 1 + тК, откуда raft-1 ... , _ J_ К К Тогда передаточная функция замкнутой системы приобретает вид Ф (р) = т.т^, + {Tl + T2)p + K' №
180 .. ГЛ. 5. ОЦЕНКА КАЧЕСТВА РЕГУЛИРОВАНИЯ I2IB а эквивалентная передаточная функция разомкнутой системы с единичной обратной связью равна ш 1 \ Ф{р) * - «а W" W - 1 _ ф (р)_ (г, + т2) р + r,zv р (1 + тэР) • где добротность по скорости К Ка- Г, + г, и эквивалентная постоянная времени т TJi 1э Г, + г2 ' 215. Для предыдущей задачи определить первые два коэффициента ошибки в двух случаях: 1) общий коэффициент усиления прямой цепи стабилен (К = const); 2) общий коэффициент.усиления прямой цепи нестабилен {К Ф const). Решение. В случае К. = const имеем из (2) передаточную функцию по ошибке: Разлагая ее в ряд делением числителя на знаменатель (см. задачу 209), находим .коэффициенты ошибок: л Т, + Т2 с0 = 0 и с, = к . В случае К Ф const примем К.— Ко + ^К (будем полагать, что -гг-<1, а коэффициент передачи цени обрат- ной связи га = 1 — -~\ • Передаточная функция замкнутой системы (1) в этом случае приобретает вид. ф(.р) = *°±М Г,7-2рг + (Г, + Г,) р + /Со + А/С - -%±-
2161 § 5Л. ТОЧНОСТЬ ПРИ ЗАДАЮЩЕМ ВОЗДЕЙСТВИИ 181 Передаточная функция по ошибке. TtT^ + i^ + TJp—^. ф1р) = 1 - Ф(р),= ^-^F r.TV + (^i + Т2) р + Ко + АК - -^ Разлагая ее в ряд, получаем А/С А/(_ (^i + Т2) (Ко + А/С) Т1 + Т. (*+«--£)• 1 Ко 216. .Определить передаточную функцию неединичнои обратной связи -ф(р), при которой в статической системе регулирования устраняются статическая и скоростная ошибки. Структурная схема системы регулирования с неединичной обратной связью приведена на рис. 125, а. Передаточная функция равна is ^(р)== (1 + г1Р) (1-ы2Р) • Решение. Передаточная функция замкнутой си-* стемы с неединичной обратной связью в" общем случае имеет вид У thlnV-.' Wip) - АтРт + Ат-1Рт~> + ...+А1Р+А0 G W\p) l+${p)W{p) B„p" + B„_lP"-'+ ... +BlP + B0' Статическая ошибка обращается в нуль при А = В0. При выполнении дополнительного условия Л,-Я, в .системе устраняется скоростная ошибка. В рассматриваемой задаче ликвидация статической и скоростной ошибок может быть достигнута при ввеч Дении в цепь обратной связи фильтра с передаточной функцией •Ф(Р): *ое_ 1 + т2р
182 ГЛ. 5. ОЦЕНКА КАЧЕСТВА РЕГУЛИРОВАНИЯ [217 Тогда передаточная функция замкнутой- системы принимает вид 21 = Ф(„) = !^> = = /С(1+тгР) Т{Г2%2р* + (TiTs + Г,т2 + Г2т2) р2 + (Г, + Г2 + т2)р + 1 + /ffeoc * При «Ос— д^ И Кг2 = Г, + Г2 + т2, г, + г, t2 = /С— 1 система будет обладать астатизмом второго порядка, Статическая и скоростная ошибки обращаются при этом в нуль. 217. Для статической системы (рис. 125, а) с пере*- даточной функцией разомкнутой системы W(p) К (\+TlP)(\ + T2p) и передаточной функцией цепи обратной связи ty{p) = m, выбранной так, чтобы получить астатизм первого порядка, определить два первых коэффициента ошибки, если Г, = 1 сек, Т2 = 0,02 сек и К = 1000 ±50. Решение. На основании формул, полученных в задаче 215, имеем ДК _ 50 _^-к.|П-б со ^ ЙГ •" к1 шоо2 10" Tv+T2 1+0,02 , пг> 1П-з ClRi-^="Iooo"=:1'02'10 сек' 218. В статической системе регулирования (рис. 126) передаточная функция разомкнутой системы имеет вид W{p)= (i+7»(i + r,p)(l+T2p) •
12191 " § 8.1, ТОЧНОСТЬ ПРИ ЗАДАЮЩЕМ ВОЗДЕЙСТВИИ 183 •Определить коэффициент передачи п масштабирующего устройства в выходной или входной цепи, яри котором система приобретает астатизм первого порядка относи- -тельно управляющего воздействия. 1 л W(p) ф п У Рис. .126. Статическая система с масштабированием. Решение. Передаточная функция замкнутой системы с учетом масштабирующего устройства Ф ^ = To'fiT^ + {TDTl + Т0Т2 + Т{Г2) р* + (Г, + Tt + Tt)p + K' Условие получения астатизма первого порядка откуда -имеем 1+К п- К 219. Для предыдущей задачи определить передаточную функцию разомкнутой эквивалентной системы без масштабирующего устройства. Ответ. W ( \- Ф(р) -, Ка ' wsKP) 1_ф(р) pd+ap + bp2)* где эквивалентная добротность по скорости ' К Ка- коэффициенты Гь+^ + Ts [сект1], т0т{ + т0т2 + т{гЛ Т0 + Тг+Тя » Ь=^ r0 + Ti+T2'
181 ГЛ. 5. ОЦЕНКА КАЧЕСТВА РЕГУЛИРОВАНИЯ [820 220. Для системы комбинированного управления (рис. 127) определить условия получения астатизма третьего порядка и коэффициент ошибки с3. На рис. 127 обозначено: ЧЭ — чувствительный элемент, состоящий из 48 ъ J =^ в \ —\- .— —»■ // L. п- Р- /// i 1.. . з р„ тг А Г~\ . к8 ЛКЬ-Р * l_t- в, ъ о. в - -л ф _ ■ —,— i \ С «! /7, А ' - " Н /■>£ /Г/Г ТгЩр Т7 Щр *> > -1 fcfr a fr —"1 ' ' Л/ * Л/ *■ #£ -чу ^ ? г -~ У 7+Тур —*• Д Р0?ЬР) Рис. 127. Система комбинированного управления. двух потенциометров IJ1 и П2 задающей и принимающей осей, Д — исполнительный двигатель, Pv и Рч — \>е~ дукторы, ТГ-— тахогенератор, КК—корректирующий контур, 0i — угол поворота задающей оси, G2 — угол поворота принимающей исполнительной оси, 0 = 0, — 02— рассогласование. Исходные данные: /г, = 1 в/град = 57,3 в/рад — крутизна чувствительного элемента; k2 = 25 — коэффициент усиления по напряжению предварительного усилителя основной цепи; k3 = 4 — коэффициент усиления по напряжению оконечного усилителя; /г4 = 27,3 об/в мин = = 2,86 рад\в сек — коэффициент передачи исполнитель-
2201 £ 5.L ТОЧНОСТЬ ПРИ ЗАДАЮЩЕМ- ВОЗДЕЙСТВИИ 185 ного двигателя; /г5 = 7Г=Тобо"~ коэффициент передачи редуктора Р{, k6 = 0,055 в мин/об = 0,525 в сек/рад — коэффициент передачи тахогенератора; k7 — коэффициент усиления по напряжению предварительного усилителя в цепи коррекции, k& = i2 = 500 — коэффициент передачи редуктора Р2; Ту = 0,005 сек — постоянная времени усилителя; 7^ = 0,1 сек — постоянная времени исполнительного двигателя; Тх = RtC и Г2 = „ _|_^ С — постоянные времени пассивного дифференцирующего контура. Искомыми параметрами являются k7, 11 Т, и То в, ,1? 1 <* 1 9 * И£#Д— вг Рис. 128. Преобразованная структурная схема системы комбинированного управления. Решение. Преобразованная структурная схема рассматриваемой системы изображена на рис. 128. Передаточные функции частей основной цепи: W*W= P(l + Typ)\l+rAp)' Передаточная функция цепи коррекции: Ф(гН WbP тт\(аХТт*)- Передаточная функция замкнутой системы: ф (р) - е^ G») - w to + <Р 0») ^» (р) (1) (2) в, (р) 1 + г о») где . передаточная функция исходной разомкнутой с» стемы ^(р)=г1ыг2(/?) = к p(Jl + V)(i + r,p) * (3)
186 'ГЛ. 5. ОЦЕНКА КАЧЕСТВА РЕГУЛИРОВАНИЯ (220 Общий коэффициент усиления: „ и и-и и г. 57,3-25-4-2,86 ,а . „ ^ К =>kik2kbkikb = jqqq = 16/4 сек11. Передаточная функция относительно ошибки равна е W - е, (р) W 1 + ^ (р) * w Подстановка (1) и (3) дает vJ>e W (1 + Т2р) [ГуГдрз + (Гу + ТЯ:р2 +,Р + К] • W где Ь2 = ^у + ^д + Т2 — k^k^kiksT^, Т Условия получения астатизма третьего порядка &з = 0 и Ь2 ~ Q. Отсюда получаем два- уравнения: ■ TYfaaMsWe - 1) = Гу + 7!д., (7) В два уравнения (6) и (7.) входят три неизвестные: &7, Ti и 7V Третье, недостающее уравнение может быть получено на основании требований, накладываемых на величину последующих коэффициентов ошибки после с0, сл и с2, которые равны нулю, так как система имеет астатизм третьего порядка. Если никаких ограничений на последующие коэффициенты ошибки не накладывается, то" расчет -можно сделать на .основании следующих соображений. Для дифференцирующих пассивных контуров отно- т шение постоянных времени -~т- обычно равно приблизи- 11 Т- тельно 10. Положив в уравнении-(6)— «ТО, получаем * 2
22,-j § &.Ъ ТОЧНОСТЬ- ПРИ ЗНАЮЩЕМ ВОЗДЕЙСТВИИ 187 требуемое значение коэффициента усиления усилителя в цепи коррекции life т* _ 10-1000 .о о, Rl ~ . k3ktk5k6ks 4 ■ 2,86 • 0,525 • 500 °'°^ Из (7)- находим' требуемое значение постоянной времени! Ту + Тя _ 0,005 + 0,1 _ Tz=z ktkjtekekjkb-l ~ 4-2,86-0,5-0,525-3,34-1 ~ 0,105 ■ _ „,,_ • = 'XoZri =0,0117 сек. Кроме того, находим 7'1 = 10Г2 = 0,117 сек. При выполнении условий (6) и (7) передаточная функция по ошибке (5) приобретает вид ft, (r,\ — Ь0р* + Ь1Ръ (М WffKP) (1.+ Т2р) [TfTAf,S + {Ту + уд);р2 + р +Щ • W Делением числителя на знаменатель (8) находим коэффициент ошибки по третьей производной управляющего воздействия: 3! ~ /С ~ /С ' '9' Подстановка численных значений дает ** с3 0,005 -0,1. + 0,005 -0,0117:+ 0,1 -0,0117 . _ ,_-4 з -g-= щ =1,3-10 Се/С°» Уравнение (9) является тем недостающим уравнением, которое может быть использовано для совместного решения с уравнениями (6) и (7). 221. В системе регулирования (рис. 129) для повыт шения порядка астатизма введены два изодромных устройства, ИМ1 и ИМ2. Определить первые пять коэффициентов ошибки. Ответ. Со-U, <?,-«,. с2.-0, -б-_"Т"' "и tf •
188 ГЛ. 5. ОЦЕНКА КАЧЕСТВА РЕГУЛИРОВАНИЯ [222 222. Передаточная функция разомкнутой следящей системы имеет вид К W{p)- р (1 + 7» (1 + 7» Значения параметров К = 2D сек'1, Тх = 0,02 сек и Г2 = 0,03 сек. На вход системы поступает гармоническое воздействие с амплитудой 8Imax = 10° и периодом Тк = 7 сек. Определить амплитуду ошибки. ИМ1 имг Рис. 129. Система с изодромиыми устройствами. Решение. 1) Для точного решения найдем передаточную функцию относительно ошибки <МР) = 1 рО + т1Р)(1+пр) 1 + КЧр) р (1 + 7» (1+?»+/< После приведения подобных членов и подстановки значений параметров имеем Фе(р) 6- 10~У + 5-Щ~У + р 6 ■ 10~У .+ 5 • 10"У + р + 20 ' Амплитуда ошибки: 6шах = |Фо(М101тах. Находим модуль частотной передаточной функции относительно ошибки при со = юк = -jr- = 0,9 сек~1. * к |ФеО'о>к>1ь 6 ■ 10~4 (/ык)3 + 5 • 10~2 (>сок)2 + /сок 6 • 16 4 (/ок)3 + 5 • 10 2 (/сок)2 + jaK +20 _ [■ г- 0,004 + /0,9 I _ /" 0.0042 + О.Э2" _ n n л ё I 20+ /0,9 Г У 202 + 0,92 U'U
«231 § 5.1. ТОЧНОСТЬ ПРИ ЗАДАЮЩЕМ ВОЗДЕЙСТВИИ 189 Далее находим 6тах = 0,045 • 10 - 0°,45 - 27'. 2) Для приближенного решения находим модуль частотной передаточной функции разомкнутой системы при ю = юк: 20 А(сак) = I W (M<) I = 0>9^1+0j92.0j022^1+0j92.0(o3^ = 22'2- Амплитуда ошибки: "1 так '0 "max Л 1 тя* == _i^_ = о° 45 = 27' (<»,<) 22,2 и ,40 z' ' 223. Для- следящей системы дана л. а. х. разомкнутой системы (рис. 130). Определить амплитуду ошибки, s&r Рис. 130. Л. а. х. следящей системы. если входное воздействие изменяется по закону в( =*> = elmaxsin<oK/, где eImax=15°, а <ок = 0,2 сек'1. Решение. По л. а. х., изображенной на рис. 130, определяем значение модуля в децибелах при частоте © = ©,. = 0,2 сек'1. L (сок) = 20 lg А (<йк) = 45 дб. Далее находим lg Л (<ок) = 2,25. По логарифму определяем модуль: Л(©«)=10?.25 = 168.
190 "ГЛ. 5. 'ОЦЕНКА 'КАЧЕСТВА- РЕГУЛИРОВАНИЯ 1224 Амплитуда ошибки: о 6' max 15 «о yyoq _ ,pv о «max ~ А ((Вк) ~ 168 - U .ИВУ - О ,3. 224. Решить предыдущую задачу, если: О 6l max = 5°, С0К=0,1 \\сек\ 2) в,.пих = -10°, <ок = 0,8 Цсек\ 3) eIraax = 30D, сок = 0,4 1/сек; Огеет: 1) 0',88; 2) 14',2; 3) 21',2. . . 225. Передаточная функция разомкнутой следящей системы имеет вид W(t})= К(\+тлР) WKP> pCl + TlP)(l+Tsp)' где К = 200 сек-1, Ji = 0,5 сек, Т2 = 0,1 сек и Г3 = 0,01 сек. Определить фазовую ошибку при воспроизведении гармонического входного сигнала с амплитудой 6imax = 20° и периодом Тк= 1 сек. Решение. 1) Для точного решения находим частотную передаточную функцию замкнутой системы при о = сок = т=- = = 6,28 1/сек. 'к дц:г-\-__ W(M = К(1+!ч>кТа) U к/ 1 + W (;сок) jcoK (1 + /сОкГ,) (1 + /шкГ,) + К (1 + /««ТУ) /Г(1+/сокГ2) ^_ ш| (г, + у,) + J [Щк (1 + Кт2) _ шз г,Г,] Подстановка численных значений параметров дает откуда ф (/«»«)=жтИ = 1;09 - А0325- q>=-arctg-^p-=-arctg0,03~ - Г,7. 2) Для приближенного решения примем, что в районе частоты входного воздействия частотная передаточная функция- разомкнутой системы имеет вид К W(j®) /ю(1+/шГ1)
!. ТОЧНОСТЬ ПРИ ВОЗМУЩАЮЩЕМ ВОЗДЕЙСТВИИ Ш„ При значении <а = юк передаточная функция относительно ошибки может быть принята равной Л ,. ч .1 /oK(l+j(oKri) . ФеОюк) « №• (/шк)' _ К ' ОТКУДа , „ K9R ф—1ттпкг= -1= - W" - 0.0314 р«а = = - 1°,8. 226. Определить фазовую ошибку для предыдущей задачи, если: 1) 7/к = 10 сек] 2) 7/к = 2 сек. Ответ. 1) -0°,18; 2) -0°,9. § 5.2. Определение точности при наличии возмущающего воздействия 227. Для следящей системы, изображенной на рис. 24 (задачи 41 и 42), определить установившееся значение моментной ошибки, если момент нагрузки на ис- полнительной оси составляет М = 200 Г • см, а к. п. д. редуктора равен 0,8. Решение. Добротность по моменту рассматриваемой следящей системы (см. задачу 42) составляет /См ?= 1700 Г'см . угл.мин Отсюда находим моментную ошибку: о ^ М„ М _ 2000 _ ., .7 м Ки пКи 0,8; 1700 1**/' 228. Решить предыдущую задачу, если задан момент нагрузки на оси двигателя Мнд = 5 Г-см. Решение. Определяем добротность по моменту, отнесенную к оси двигателя: К = -*•«- = 170° =17 Г'см мд i 1000 ' углмин ' где I— 1000 — передаточное отношение редуктора. М.О- ментная ошибка: В = -Мнд. = — = 2' 95 ■- Амд li<
192 ТЛ. 5. ОЦЕНКА КАЧЕСТВА РЕГУЛИРОВАНИЯ 1229 229. Определить моментные ошибки для следящих систем с астатизмом первого порядка при следующих исходных данных. 1) Добротность по скорости Ка = 200 сек'1, передаточное отношение редуктора i = 500, скорость холостого хода двигателя пхх = 6000 об/мин, пусковой момент Мп=100 Г-см, момент нагрузки, приведенный к оси двигателя, М„д = 30 Г-см. 2) Ка = 500 сект1, i= 10000, пхх = 7500 об/мин, Мп = 300 Г■ см, Мш =150 Г- см. Ответ. 1) ем = 3440як„.. М» 3440-3,14-6000 30 2) е 30i/CQ Мп 30 • 500 • 200 3440-3,14-7500 150 100 = 6',5. = 0')27. 30 ■ 10000 • 500 300 230. На рис. 131 изображена' структурная схема системы регулирования. Значения параметров kt = 10, . * hT,p —ф— -1 *2 Р(^гр) JC Рис. 131. Структурная схема к задаче 230. fe2 = 2 сек 1, Г, = =0,1 сек, и Т2= 1 сек. Возмущающее воздейг.твие меняется по закону f = fmax sin <dKt, где /max= 15 и toK=5 сек'1. Определить амплитуду ошибки хтах. Решение. Передаточная функция по возмущающему воздействию в замкнутой системе равна ф/(р)= Wf(P) T+wJpT P{l + T2p) 14- ktk2 p (1 + TlP) (1 + T2p) k2 (1 + T,p) P (I + TiP) (I + T2p) f kxk2
2321 § 5.2. - ТОЧНОСТЬ ПРИ .БОЗМУЩА*МЦ£М ЬСИДКтл ш141 IH'd Амплитуда- ошибки: .*«.*= |Ф/(/«>«) !/<"«= kJmixVl + vlrf У [*,*, - «%(Г, + T2W + юЦ (1 - о>2 Г,ГЙ)2 Радстановка числовых значений дает #тах 2- 15]Л + 5я-0,12 У [10 • 2 - 5? <0,t + I)]2 + 52 (1 - 52 • 0,11)2 ■ = 2,65. 231. Внешняя характеристика генератора (зависимость напряжения на его зажимах от тока нагрузки) изображена на рис. 132. Наклон характеристики .составляет р = 0,1 в/а. Генератор имеет статическую систему стабилизации напряжения с общим коэффици- У6 ентом усиления по разомкну- 120 той цепи К = 200. Определить установившуюся ошибку при скачке нагрузки А/п= 100 а, Решение, 80 - 4/0 - __ РАЛ, 0,1 - 100 1+К 1+200 0,05 в. OW 60 120 7601, а Рис. 132. Внешняя характеристика генератора. 232. В системе стабилизации температуры печи в качестве чувствительного элемента используется тер- \мопара. При отключенной системе регулирования внешнее возмущение вызывает отклонение температуры от "заданного значения Дт0 = 200' С. Определить установившееся отклонение температуры, если используется .ЫЬтема регулирования с передаточной функцией разомкнутой системы Щр) = К ^де К = 500. \ Решение. (1 + 7» (1 + TiP) At = Лт„ 200 1 + К 1 + 500 0,4° С.
J94 ГЛ. 5. ОЦЕНКА КАЧЕСТВА РЕГУЛИРОВАНИЯ {233 § 5.3. Корневые методы оценки динамических свойств , 233. Даны характеристические уравнения систем регулирования: 1) р3+14р2 + 53р+130 = 0, 2) р3+11р2 + 51р + 41=0, 3) р3 + 2,5р2 + 27р+13 = 0, 4) рл 4- 7р3 + 418р2 + 1220р + 808 = 0, б) р4 + Зр3 + 5,5р2 + 6р + 2,5 = 0. Определить корни уравнения, степень устойчивости h, колебательность ц и затухание г\ системы. Указание. При вычислении корней целесообразно воспользоваться методикой, изложенной в книге [23]. Ответ. 1) Pi — ~ Ю. сек'1, р2,3 = ( — 2 ± /3) сек'1, ./i=2 сек'1, }х = -|=1,5, ч=» 1-е » =98,5%;. •2) р, = - t се/с"1, .р2;3 = (-5 ± /4) Сек'1, Л=1 сек'1, ц = 0,8, г] = 99,96%; 3) р,= — 0,5 сек'1, ,р2.з.= (— 1 ± /5) сек'1, /г = 0,5 сек'1, ц = 5, г\ = 71,5%; 4) р, = - 1 сек'1, р2= -2 сек-1, p3i4.= (-2 ± /20) сек"1, ■ /г= 1 се/с-1, ц = 10, ^ = 47%; 5) р!= — 1 сек'1, р2~ — 1 се/с-1, р3.4 = ( —0,5±/1,5) сек'1, /г = 0,5 се/Г1, ц = 3, т] = 88%. . ' 234. Даны "характеристические уравнения систем регулирования: 1) р3 + 4р2 + 41р + 64 = 0, 2) р3 + 14р2 + 144р + 1000 = 0. Пользуясь диаграммой Вышнеградского, определить без отыскания корней затухание и степень устойчивости.
S9ftl § :5.3. КОРНЕВЫЕ МЕТОДЫ 195 решение. 1) Используем подстановку р = V64 q = __4я. Тогда уравнение (1) после деления на 64 приобретает вид <73 + <?2 + Jnb+l=-0. 41 Параметры Вышнеградского А = 1 и £ = ^-=-2,56. По диаграмме Вышнеградского с- нанесенными линиями равного затухания (приложение 7) находим т) = 70%. По диаграмме Вышнеградского с нанесенными линиями равной степени устойчивости (приложение 8) находим относительную степень устойчивости А0 = 0,25. Далее определяем абсолютное значение степени устойчивости h = 4А0 = 1 сект1. Ц) -п = 75%, Ь = 2 сек^. 235. Дана передаточная функция разомкнутой системы с астатизмом первого порядка" / ^M-Wttw- (1) Определить соотношение между добротностью по скорости Кй и постоянной времени Т, при котором затухание за .один период будет не меньше заданного значения т). Решение. Находим характеристическое уравнение системы l + W{p) = 0 или, после подстановки (-1), Корни этого уравнения где
196 ГЛ. 5. ОЦЕНКА КАЧЕСТВА РЕГУЛИРОВАНИЯ [288 Колебательность связана с затуханием ;за одим период, зависимостью In — Далее находим , ■|== УЧ/СаГ - 1. (5) Совместное решение (4) н (5) дает искомое условие. . КиГ<~, ^-^- + 0,25. (6) (1пт^г) 236. В системе с передаточной функцией разомкнутой системы постоянная времени Т = 0,1 сек. Определить допустимое значение добротности по скорости, соответствующее получению затухания за один период г] = 90%, т) = 95%, i] = 98% и 11=100% (см. предыдущую задачу). Ответ. /Са = 31,1 сек~\ ^а = 13,6 сек'1, Ка = 8,7 сек~\ /Си = 2,5 сек~1. 237. Передаточная функция разомкнутой системы с астатизмом второго порядка имеет вид W(p)=K*ll + Tp). (1) Определить соотношение между добротностью по ускорению Ке и постоянной времени Т, при котором затухание за один период будет не меньше заданного значения 1]. Решение. Находим характеристическое уравнение замкнутой системы l + W{p) = 0 ИЛИ . : Р2-г/<е7> + /<е = 0. . . = (2)-
§ 5.3. КОРНЕВЫЕ МЕТОДЫ - 197 Корни этого уравнения Колебательность Pl,2~~ 2 ке ■■- ■■■ 4 = )/-ш:-1- = ~а±/р. (3) (4) \£ Используя соотношение между колебательностью и затуханием И=—^т—, (5) in—5— 1-11 находим окончательно *V<7 ТТ+°Д (6) 6 (lnxhr) или КеТ*>: ^ —. (7) + 0,25 238. В системе с передаточной функцией разомкнутой системы ' р- добротность по ускорению /Се=100 сек~2. Определить минимальное значение постоянной времени' Т, соответствующее получению затухания за один период т) = 90%, ■*!;== 95%, т) = 98% и 11=100% (см. предыдущую задачу). ..Ответ. Т = 0,069 сек, Т = 0,086 сек, Т = 0,107 сёк, Г = 0,20 сек. 239. В статической системе регулирования передаточ- на.я функция разомкнутой системы имеет вид W{p)~ К (l + Tep)(l + TlP) Постоянные времени равны Т0 = 1 сек и Т\ = 0,5 сек. определить допустимое значение общего коэффициента
198 ГЛ. 5. ОЦЕНКА КАЧЕСТВА РЕГУЛИРОВАНИЯ 12'0 усиления К, при котором затухание за один период будет не меньше t] = 90.%. Ответ. И1пТГода) J "■jfcjjfg-l, -Т „+0,261-1-8,5. § 5.4. Оценка по кривой переходного процесса 240. Замкнутая система регулирования описывается дифференциальным уравнением iflQf? + aip+Y)y = {a1p + \)g. (1) Определить величину перерегулирования в предположении, что корни характеристического уравнения комплексные рив «= т а ± /Р, для случая отсутствия задающего воздействия g = 0. Начальные условия у = Уо и г/ = 0 при / = — 0. Ответ. Переходный процесс определяется выражением У = ^0^""°' (cos Р< + -f sin pr j — *- Уо /а2 + р2-^ sin (р* + arctg -&-). (2) При исследовании на экстремум можно получить его первое значение а я Ут = — Уое J ' = — УФ "■ • (3) Отсюда искомое перерегулирование: а Уо /^ (4) 241. Для предыдущей задачи определить условие отсутствия перерегулирования. Ответ, р = 0, что соответствует выполнению условия a0<0,25fl£
§ 5.4. оценка по кривой переходного процесса jgg 242. Для задачи 240 определить соотношение коэффициентов, при котором перерегулирование будет а = == 10%, 0 = 20%, <7 = 509*. Ответ. a0 = 0,72af, а0= 1,22а*; а0 = 5,25а?. 243. Для системы регулирования, дифферерциальное уравнение (1) которой приведено в задаче 240, опреде* лить перерегулирование при подаче на вход ступенчатой функции g0l (t), если до приложения входного воздействия система находилась в покое. Ответ. Переходный процесс определяется выражением У = go [l - e~at (cos pf - jsin pf)]. Исследование его на максимум дает Отсюда определяется перерегулирование с = где- = ехр[ ^p-H-J, ц = 1 = ! A *SL-1 ? « J/ 4 а\ 244. Для соотношения коэффициентов а0 и fli, соответствующих получению перерегулирования а = 0%, 0=10%, о = 20% и сг = 50%, при согласовании из не* подвижного положения, (см. задачи 241 и 242), определить величину перерегулирования при подаче на вход ступенчатого воздействия g(t) — g0-l{t) и произвести сравнение величин перерегулирования. . Ответ. Значения перерегулирований сведены в таблицу: Вид движения ™-2 00 = 0,720' о0 = 5,25о. ^ргласованне из. неподвижного положения . . . . Отработка единичного ступенчатого воздействия 0% 13,5% 10% 25% 20% 32% 50% 55%
200 ГЛ. 5. ОЦКШ£А." КАЧЕСТВА РЕГУЛИРОВАНИЯ 1245; 245. Передаточная функция замкнутой системы регулирования, имеет вид . / ч_ СЦр+ 1 V[P>- asP3 + a2P2 + alP+l ' На вход системы поступает воздействие типа единичной ступенчатой функции 1 (t). Посредством построения переходного процесса определить перерегулирование и время переходного процесса при следующих значениях коэффициентов: 1) аг = 10,33 сек, а%= 0,01 сек2, ал= 1,58 • Ю^ се/с*; 2) а, = 0,415 сек, а2 = 0,04 сек2, а-л = 0,002 се/с3; 3) а, = 0,087 сек, а2 = 0,0025 сек2, а3 = 0,435 • 10~4 се/с;!. Ответ. Г) а =13,8% tn = 0,775 сек; 2) а = 26,5%, /„=1,17 сек; 3) о = 37,2%, *п = 0,27 сек. 246. На рис. 133 изображена вещественная частотная характеристика замкнутой системы. Определить ориентировочные значения перерегулирования и времени переходного процесса. Решение. Интервал существенных частот для вещественной характеристики юс = 20 се/с-1. Это дает время р(а)) переходного процесса или 0,157 сек <tu< 0,628 сек. ы>сён Перерегулирование а< Рис. 133. Вешсстицшшя частотная <18%. характеристика. Для более точного расчета необходимо обратиться к кривым, данным в приложении 12. Коэффициент наклона вещественной характеристики (см. рис. 133) составляет к = 0,4. Это Дает о =10% и ta = ='-щ = 0,35 сек.
24») § 5.5. ИНТЕГРАЛЬНЫЕ ОЦЪНКИ 20 { 247. Определить перерегулирование и время переходного процесса для вещественной частотной характеристики, изображенной на рис. 134. Р(СО) Z0 30 40 50 Щ сек Рис. 134. Вещественная характеристика. Решение. Высокочастотную часть характеристики, соответствующую Р(ю)<0, можно отбросить, так как Лп1п<0|2. Тогда перерегулирование в системе равно а< l,18Pnlax-P(0) 1,18-1,2-1 Р{0) 1 -0,41 = 41%. Время переходного процесса tn> 4- = -^-=0,0628 сек. (Ос «3U Для более точного расчета необходимо обратиться к кривым приложения J3. В результате их использования определяем Зя = 3-3,14 <ос 50 о = 23% t„ = 0,18 сек. § 5.5. Интегральные оценки 248. Передаточная функция разомкнутой следящей системы с астатизмом первого порядка имеет вид W № = ~р{1 + Т1р)Ц + Тяр) ' ■ <- При значениях постоянных времени Т{ = 0,02 сек н Г2 = ■=0,04 сек определить значение добротности по скорости, соответствующее. минимуму квадратичной интегральной оценки при отработке ступенчатого воздействия ё(()=*
202 ГЛ. 5. ОЦЕНКА КАЧЕСТВА РЕГУЛИРОВАНИЯ [248 Решение. Передаточная функция замкнутой системы ^КР) l + W(p) /Г0 + Р + (Г1 + Г2)р« + Г1Г2р»- Изображение по Лапласу выходной величины имеет вид X (рУ- Ка 1 Ка + р + ^ + Т^Г + Т^? р 1 а^ + а^р-^- а$Р + а3р3 р В соответствии с приложением 16 находим значение интегральной оценки / = ДрАо 2о£д гдеВ0 = 62 = ^, а0 = Я£ А = А0 = а0 0 0 «1 а0 0 - «2 а. - а0 — а2 й. - «о и -«з «2 0 -«8 а2 Значения определителе? а0(а,а2-«оаз)> й?а2-й100й3^йо4 В результате имеем 1 2Кп I -Ь 1 (T1 + T2f л]' 2 Г1+Гат/Гв7'1 Для получения минимального значения интегральной оценки приравняем нулю производную 1 Tfafr-f-Ttf dl l Г dKQ~T[^- ■4Г«У 1-Q /Г| (Г. + Г.-лв откуда оптимальное значение добротности ri + r, 'TiTt + iTi+TjYTfa'
£49] ■§■■5.6. ИНТЕГРАЛЬНЫЕ ОЦЕНКИ 203 Подстановка численных значений постоянных времени дает Kqz=, °'06 = 24 сек-\ 249. Передаточная функция разомкнутой системы имеет вид WKP} p(l+Tp)' При фиксированных значениях Т — 0,1 сек и добротности по скорости /Со = 20 сек~1 определить значение коэффициента Ки определяющего уровень сигнала по первой производной, соответствующее минимуму квадратичной интегральной оценки при подаче на вход управляющего воздействия в виде единичной импульсной функции g 00 = 6(0. Решение. Передаточная функция замкнутой системы равна ф{ \ W{P) ^Q + ^lP ф{р) i + w(p) *e + (i + *,)p + V • Изображение входного воздействия G(p)=l. Изображение выходной величины к(рЬф<р)0(р)- Ka+KlP Ка+ (1+/С,)р + Гр2- Значение квадратичной интегральной оценки (см. приложение 16) В,А,+В2Д2 * ""'"' о"""* - 2«g& Коэффициенты равны а0 = Ка, а1 = 1 + /С, и а2=Т.
2G4 ГЛ. 5. ОЦЕНКА КАЧЕСТВА-РЕГУЛИРОВАНИЯ 1Й0' Значения определителей «о - «2 О О а, . О О — а0 а2 а0 «! О О а0 О О О а2 «о - «2 «1 О а, а0 = с0«,а2 = /CQ (1 + /Q) Т, = а1а2 = ^Г. А2 = О -' а0 О Далее находим / = 1<1Ф + *?*а *£• /cQf + к? 2/С|Л'и(1+/С,)Г 2(1+ /С,) Г Для нахождения минимума/ приравняем нулю производную -Т7Т- = 0. В результате имеем *K* + 2Kl-KQT=0, откуда /с, — \ + У\ + кат. Подстановка численных значений^дает Л", = - 1 + /1+20-0,1 = 0,73. 250. Передаточная функция разомкнутой системы имеет вид При фиксированном значении постоянной времени 7'= 0,2 сек определить оптимальное значение добротности по скорости, соответствующее минимуму интегральной оценки вида- /= |(л-2 + т2а-2)Лс 0)
2501 ..§ Б;5,- ИНТЕГРАЛЬНЫЕ'ОЦЕНКИ- 205 при подаче на вход единичного ступенчатого воздействия g(0—И') лля значений постоянной времени экстремали т = 0, т = 0,1 сек, х = 0,5 сек ит=1 сек. Решение. Разбиваем интеграл (1) на два интеграла /=А, + /2= f x*dt + T*fj?dt. о о Находим передаточную функцию замкнутой системы Ф(Р) = Ко + р+Тр* Изображение выходной величины при G(p) = —равно у(р)=Ф(Р)о(р)=Ка+к;+Тр2^ Б- соответствии с приложением 16 находим в0Д0 Значения коэффициентов: Значения определителей: - а., (2) Д = «о 0 : Й0Й1 = ^G > Л„ а, 'о «7 + а0ал = 1 + /СНГ. а, а0 а, Подставляем найденные значения в (2); получим , Kl(l+Kf?) 1+«0Т .2#Q#S2 2Kb Для нахождения /2 определяем изображение скорости изменения выходной величины «о />п/>) = Ка + Р+Гр»
2©6 ГЛ. 5. ОЦЕНКА КАЧЕСТВА РЕГУЛИРОВАНИЯ _^ №1 В соответствии с приложением 16 находим В.А, >h '1Ц| 2а2д ' (3) где Bl = b\ = K?Q, а определитель А,- Далее имеем а0 а, О ап Д.- -*8-K£- ^2 2- 2 Результирующее значение интегральной- оценки равно (4) l+KaT Kat2 / = /l + /a= о^ + 2/tQ Для нахождения оптимального значения4К& приравняем нулю первую производную (4): ■ dl dKa = 0. После дифференцирования имеем откуда оптимальное значение добротности по скорости Ка = ~. Численные значения Ка-* °°, Ка=*10 сек'1, /Сй = 2ге/с_1 и Ks=l се/с-1. § 5.6. Частотные оценки динамических свойств 251. На рис. 135 изображена амплитудная частотная характеристика замкнутой системы. Определить показатель колебательности. Ответ. Ф (/ев) I max _ .3,2 м |Ф(0)| = 1,6. \
§ Б.6. ЧАСТОТНЫЕ ОЦЕНКИ ДИНАМИЧЕСКИХ СВОЙСТВ 207 252. На рис. 136 изображена амплитудно-фазовая характеристика разомкнутой следящей системы. Она может быть построена по.таблице Re W (/о) Im W (/ш) -2 -4,95 -1,75 -1,8 -1,5 -1,75 -1,25 -1,6 -1 -1,4 -0,75 -1,05 -0,5 -0,85 -0,25 -0,65 0 -0,55 Определить ' показатель колебательности "замкнутой системы. — С- о т го зо w w,^ Рис. 135. Л. ч. х. замкнутой си- Рис. 136. А. ф. х. разомкнутой стемы. системы. ""* Решение. Для нахождения показателя колебательности необходимо определить параметры окружности, которой касается амплитудно-фазовая характеристика. Параметры окружности связаны с показателем колебательности формулами А ~~ М2 - 1 и Ь ~ М2 - 1 ' где R — радиус окружности, а С — сдвиг центра окружности влево от начала координат. В результате подбора определяем, что касающаяся окружность соответствует М = 2, R С = ± Построение сделано пунктиром на рис. 136.
208 ГЛ. 5:-ОЦЕНКА КАЧЕСТВА РЕГУЛИРОВАНИЯ (253 253. Передаточная функция разомкнутой следящей системы имеет вид Определить соотношение между добротностью по скорости Ко. и постоянной времени, при котором' система будет иметь показатель колебательности не более заданного значения М. . Решение. Передаточная функция замкнутой системы равна rrw у..' W(p) - Ка W{P> l+W(p) Ка + р+Тр*- Частотная передаточная функция замкнутой системы запишется в виде Ф (/со) = -р—г- ^ • Kq +1® ~ ®т Ее модуль равен |ф(/ю)|=— в . Исследование на максимум этого выражения дает значение показателя колебательности | Ф (/со) Ux = ^r^C-r = М (при КиТ > 0,5). Из последнего выражения находим м£ + м Ум2 -1 254. Решить предыдущую задачу, если передаточная функция разомкнутой системы имеет вид w(P) = KA\rp\ где Ке — добротность по ускорению, а х — постоянная времени корректирующего контура. Ответ.
5'.6.. ЧАСТОТНЫЕ. ОЦЕНКИ ДИ ИЯМИЧЕСКИХ СВОЙСТВ 2вЭ • 255. На рис, 137 изображены логарифмические частотные амплитудная и фазовые характеристики (л. а.х. и л. ф- х.) разомкнутой системы. Определить показатель колебательности замкнутой системы. Ш), дб 3D 20 W -W0°0 -135° -90°- 0°- Рнс. 137. Л. а. х. н л. ф. х. разомкнутой системы. Решение. Для нахождения показателя колебательности необходимо таким образом построить запретную зону для фазовой характеристики, чтобы фазовая характеристика касалась этой зоны. Построение запретной зоны делается в соответствии с приложением 14, где приведены необходимые запасы по фазе в функции модуля в децибелах для различных значений показателя колебательности. В результате подбора определяем, что показатель колебательности /VI = 1, 2. Построение запретной зоны показано пунктиром на рис. 137. 256. Построить л. а. х. и л. ф. х. и определить показатель колебательности, если передаточная функция разомкнутой системы имеет вид l)W(pl-m{l+0'l7Sp)- р2(1+0,035р) 2) W{p)= -Ж'+О.бМ 3) W(p) = 4) W\p) = р2(1+0,03р) ' 400(1+ 0.1 р) . р(1+р)(1+0,013р) ' 1000(1+0,05р) р(1+0,4р)(0,013р)
210 -Г'Л. 5„ ОЦЕНКА КАЧЕСТВА РЕГУЛИРОВАНИЯ [2S7 Ответ. 1) М = 1,5; 2) Af = l,l; 3)Af=l,3; 4) Af = 1,7. 257. Передаточная функция разомкнутой системы имеет вид -'Со w(p) = —z—* . pJlV + Tw) i = l Определить условие, при котором показатель колебательности замкнутой системы не будет превышать единицу, если число постоянных времени является произвольным, т. е. п — произвольное целое число. Ответ. <- п i=i 258. Для амплитудной частотной характеристики замкнутой системы (см. рйс. 135) определить полосу пропускания системы. Ответ. соп = 30 сект1, fn = 4,8 гц. . 259. Для л. а. х., изображенной на рис. 137, определить ориентировочное значение полосы пропускания. Ответ. В первом приближении полосу пропускания замкнутой системы можно принять равной частоте среза л. а. х. разомкнутой системы. В результате имеем ©п ^ ©ер = 13 сек'1 или /п =? 2,1 гц.
ГЛАВА 6 СИНТЕЗ ЛИНЕЙНЫХ СИСТЕМ §6.1. Выбор параметров САР по требуемой точности 260. Для системы стабилизации углового положения некоторого тела (рис. 138) выбрать такое значение коэффициента передачи по углу k^, чтобы при внешнем Рис. 138. Блок-схема системы стабилизации углового положения тела. возмущающем' моменте МУ) = 1Щ отклонение угла *J> не превышало допустимое значение -фд0п. Уравнения отдельных звеньев системы имеют вид! 1. Уравнение регулируемого объекта I Ei ' ' dt m. упр + Щ, где / — момент инерции тела, -ф — угол поворота тела, 'ф —его угловая скорость, mynp — управляющий момент со стороны исполнительного органа системы стабилизации, пг0 — внешний возмущающий момент.
212 гл. с еинтЕз -линейных систем 1яви 2. Уравнение исполнительного с усилителем-преобразователе.и органа совместно m упр — f^H.o^» где klu 0 — коэффициент передачи исполнительного органа и усилителя-преобразователя.- 3. Уравнение регулятора (принятый.закон управления)- и = — (А^лр + kqty). Решени е. Уравнение замкнутой системы стабилизации может быть, записано в виде dt m o> откуда 4>д бн.обф Ь > ^о 261. Определить требуемое значение общего коэффициента усиления К для системы регулирования температуры (рис. 139) из условия обеспечения нужной точности регулирования в' установившемся режиме. Отклонение регулируемой величины Ф измеряется с помощью термометра сопротивления, включенного в мостовую схему. Напряжение- и с диагонали моста поступает на балансный усилитель Ус, который управляет двигателем Дв. Через редуктор Р двигатель приводит в движение регулирующий орган. Регулирующий орган воздействует на объект за счет изменения величины регулирующего воздействия Y- ' . Уравнения звеньев имеют вид. 1. Уравнение регулируемого объекта, Рис. 139. Система регулиро ваиия температуры.
и § 6.1. ВЫБОР ПАРАМЕТРОВ ПО ТРЙБУЕМОЯ ТОЧНОСТИ £{3 где Ti [сек] — постоянная времени объекта, й, и kb-~ коэффициенты передачи, f - возмущающее воздействие. 2 Уравнение чувствительного элемента — моста с термометром сопротивления и = кф, где. k2 [в/град] - коэффициент передачи. 3. Уравнение привода вместе с усилителем {l+T2p)py^k3u, где Т2 [сек] — электромеханическая постоянная времени, k3[l/сек] — коэффициент передачи. Решение. Общий коэффициент усиления разомкнутой системы должен выбираться из условия идоп, где f — значение скорости возмущающего воздействия; ■&ДОП-— допустимое значение ошибки в установившемся режиме. . 262. Определить требуемое значение коэффициента усиления для тахометрического привода постоянной ип П— ; У Ус „ Г^ Дв и^>=^ 1 •• 1 Рис. 140. Тахометрическнй привод постоянной . скорости. скорости (рис. 140). Допустимая ошибка скорости вращения при моменте нагрузки по валу двигателя Л4н = 0,2ЛТкз не должна превышать 0,1% от скорости холостого хода. Решение. Ошибка регулирования Дй состоит из Двух слагаемых4 1 а. ± т-?£& л*. /О) AQ=. I+WO»)' l+W(p)
214 • ГЛ. 6. СИНТЕЗ ЛИНЕЙНЫХ СИСТЕМ ' |2«2 где й3 — заданная скорость вращения привада, №(р)—■ передаточная функция разомкнутой системы, WM(p) — передаточная функция разомкнутой системы по моменту нагрузки. Первое слагаемое соответствует ошибке из-за выбранного закона регулирования, второе слагаемое определяет составляющую ошибки, вызванную действием момента нагрузки Мн. Если учитывать постоянную времени усилителя Ту и электромеханическую постоянную времени двигателя Ти, то W(P): K (1 + 7» (1 + 7» W-Ap)- Ik. Мкз (1 + Тур) ' ■ где Мкз — момент короткого замыкания, развиваемый двигателем, Q3 —скорость холостого хода двигателя, К— общий коэффициент, усиления разомкнутой системы. Выражение для ошибки регулирования Дй принимает вид [Гм7,ург + (ГМ + Гу)р+1]£23 AU- ТмТуР* + (тм + Ту)р+1+К± м > н на** (2) *- тмтуР* + (ты + ту)р + 1+к' ^ Обычно настройка системы осуществляется таким образом, чтобы ошибка регулирования была наименьшей. Этому условию соответствует такая настройка, при которой в статической системе устраняется статическая ошибка от закона регулирования. Для ликвидации статической ошибки коэффициент передачи главной обратной связи должен отличаться от единицы и быть равным Кос — —^~» (у).. т. е. в системе регулирования должна иметь место неединичная обратная связь.
2631 § 6.1. ВЫБОР' ПАРАМЕТРОВ ПО ТРЕБУЕМОЙ ТОЧНОСТИ 215 Чтобы обеспечить требуемую точность поддержания скорости вращения при моменте нагрузки МН = 0,2МКЗ, коэффициент усиления разомкнутой системы К должен быть выбран из условия '" ' М« " Дй = ^^, (4) от^уда м_к_т_ К-^М™ Qo 0.2-0.001 100 «* Л ~ Ли ~~ 0,001 ~ УУ- W 263. Определить местоположение л. а. х. разомкнутой "следящей системы из условия, чтобы ошибка слежения не превышала Фщах^ 1'.5 при изменении входного воздействия по гармоническому закону #1 = 6l max Sill d>J, ГДе 01 max = 25°, 1л «)к = -=- = 6,28 1/cerc. Структурная схема следящей системы показана на рис. 141, а. Решение. Ошибка слежения, вызванная изменением входного воздействия, равна 1 11 + w (;<o) i 0iraax при со = сок, (1) где W (/to) — частотная передаточная функция разомкнутой системы. Так как обычно в современных следящих системах | W (;сок)|^>1, то можно воспользоваться приближенной зависимостью А. ~ 0i max /24 I W (M() I .(: .Разрешая выражение (2) относительно \W(jaK)\, получим требуемое значение модуля частотной передаточной функции Ттах
216 ИЛИ ГЛ, 6. СИНТЕЗ ЛИНЕЙНЫХ СИСТЕМ ©in L,t = 201g|W'(/«>)| = 201g^ I2S4 (4) По формуле (4) в логарифмической системе координат % { W(p) Ъ а) б) Рис. 141. а) Структурная схема слё^ дящей системы; б) построение контрольной точки Ак. (рис. 141, б) построена так называемая контрольная точка Ак; "l max . юк = 6,28 1/се«, 20 lg- ■ 20 lg Щ^ = 60 до. ■ ° 1,5 Требуемая точность слежения будет достигнута, если Л. а. х. системы будет лежать выше точки Лк, в пределе пересекая ее (рис. 141, б.) 264. Определить запретную область для л. а. х. разомкнутой следящей системы из условия, чтобы ошибка слежения не превыщала.'&шах^ Г,0 при изменении
2<M| $ 8.1. ВЬГВЭР• ПЛ^-AMBTfС)В ПО ТРЕБУЕМОЙ ТОЧНОСТИ 2(7 входного воздействия с максимальной скоростью (£-4:0град/сек и максимальным ускорением е=66 град/сек2. Решение.-В тех случаях, когда закон изменения входного задающего воздействия неизвестен, расчет мЪжно вести на эквивалентное синусоидальное воздействие [4]. Параметры эквивалентного режима определяются по формулам ю„ "l max " 1,5 сек ', _L = J0= 26" 7 < 1,5 (1) Здесь <вк — угловая частота колебаний эквивалентного Синусоидального воздействия, 6, тах — амплитуда колебаний эквивалентного синусоидального воздействия, U6 80~' во 40 го ' 0 -го <?^^ 'ЪъъЛддб/Зек i fc . $ 4J* 1 ■ Щ •О ч> ^Ъ^дб/дек ■ %>5^СТ >о Ч«Ч4 Рис. 142. Построение запретной области Координаты контрольной точки Лк (см. предыдущую задачу) равны (рис. 142) g>k = -£- = 1,5 сек \ ^ = 20 ^-^- = 20^-^ "шах CUma 20 [а 1600 ~ 63 дбГ Для построения всей запретной области найдем геометрическое место контрольных точек Ак[, соответствующих
218 ГЛ. 6. СИНТЕЗ ЛИНЕЙНЫХ СИСТЕМ 1265 двум случаям: 1) когда Q максимально, а е убывает до нуля, 2) когда е максимально, a Q убывает до нуля. В первом случае геометрическим местом точек будет являться прямая, проходящая с наклоном — 40 дб/дек через точку Лк. Во втором случае — прямая с наклоном —20 дб/дек (см. рис. 142). Для обеспечения требуемой точности слежения л. а. х. разомкнутой следящей системы не должна заходить в запретную зону, ограниченную этими прямыми. ■ 265. Для следящей системы, структурная схема которой показана на рис. 141, а, построить низкочастотную LM °>'Ш Рис. 143. Построение низкочастотной части желаемой л. а. х. часть желаемой л. а. х. и определить требуемую величину общего коэффициента усиления из условия обеспечения необходимой точности слежения. Система обладает астатизмом первого порядка. Требования, предъявляемые к точности слежения, те же, что и в задаче 264. Решение. Для облегчения задачи демпфирования системы л. а. х. должна располагаться как можно левее. Предельно допустимое смещение л. а. х. влево ограничивается запретной зоной по точности. С этой точки зрения целесообразно, чтобы низкочастотная ветвь желаемой л. а. х., имеющая наклон —40 дб/дек, проходила как можно ближе к линии запретной зоны (рис. 143), т. е. чтобы ю0 = со0к. и Ti=l/<oK. Однако первая асимптота.л. а. х., имеющая наклон —20 дб/дек, должна проходить выше границы запретной области 3 дб (см. рис. 143),
§ 6.1. ВЫБОР ПАРАМЕТРОВ ПО ТРЕБУЕМОЙ ТОЧНОСТИ 219 Если эту асимптоту продлить до пересечения с нулевой- осью, то точка пересечения ©а даст значение общего коэффициента усиления разомкнутой системы (добротности по скорости Ко). Согласно рис. 143 имеем т/п Q 1/и 40-600 n.nn _, /Cq = V 2 а— = 1,41 : = 3400 сек1. Базовая частота , = 1/1,41 * =1,19]/-^ = 1,19 j/6G- 60 = 71,3 сек-1. 266. Определить требуемую величину общего коэффициента усиления разомкнутой следящей системы. Следящая система обладает астатизМом второго порядка. Остальные данные те же, что* в задаче 143. -Ответ. Общий коэффициент .усиления разомкнутой системы добротность по ускорению КЕ'— 3600 сек-2. 267. Для следящей системы, обладающей астатиз- мом первого порядка, определить параметры низкочастотной части желаемой л. а. х. из условия обеспечения требуемой точности слежения без учета и с учетом .момента нагрузки. Максимальная скорость слежения Q = 24 град/сек, максимальное ускорение е=20 град/сек2, допустимая "величина ошибки ,&гаах = 0о,1. Момент нагрузки, приведенный к валу двигателя, М„ = 2 Гсм = = 19,6 • Ю-5 к • м. Жесткость механической характеристики р = -jr— = к ' = 52,3 1/Г • см • сек (где Q0 = IrlQ О/,О ' IV — 5000 об/мин — скорость холостого хода, М0 = 10 Тсм=?= ■= 9,81 • 10~4н • м — пусковой момент двигателя). Передаточное отношение редуктора /=1000. Решение. 1. Момент нагрузки отсутствует (см. задачу 265). Тогда Г, = —= —=1,2 сек, (1) Kq„Y2~ =-^41^ = 338 ceK-im (2) °тах u>l
220 ~ " ГЛ. б. СИНТЕЗ ЛИНКИМЫХ СИСТЕМ' [267 2. Двигатель нагружен моментом Мя = 2 Гсм. Демпфирование системы осуществляется- по первому ме1- тоду [4], т. е. введением обратных связей, охватывающих двигатель, или производных от угла рассогласования. Тогда Г|Я1аД=1,2 сек, (3) к- !/о & , РМн _ооо , 104,6-57,3 /Cq==]/2^^' + '5^~338+ o,i.)ooo = = 338 + 60 = 398 сект1, (4) где ftmaxi —ошибка, приведенная к валу двигатели. 3. Двигатель нагружен, Мн = 2 Гсм. Демпфирование системы осуществляется по второму методу [4], т. е. введением инерционности в усилительный канал. Тогда Г, < 0,236 ^"'а*+>>а ^ #мГ е о. РМ„ * где хтм = -ТГ-.— моментная ошибка, приведенная к неполна' нительной оси. Если общий коэффициент усиления выбрать равным ' ДЪ-~ = -^- = 240 сек-1. ТО РАГн 104,6 -.57,3 _ „0 „ - KQi 240-1000 ,U ° /(О,! +0,025)3 0,025 /20 Г, <0,236- „_Т,7^-= 0,0935 сек. Если величину общего коэффициента усиления увеличить, например, до Ка = 300 сек'1, то А - Р^" '04,6-57,3 ПрПО ■ м /Гц/ ~ зоо • юоо ~ и >uz и . ij^0j236^—- 7=:— = 0,1.1 сек. ■ 0,02 К 20 '■ ■ '
m\ § 6.1. ВЫБОР ПАРАМЕТРОВ ПР. TPBGJ'EMOn ТОЧНОСТИ 22;1 Для иллюстрации на рис. 144 построены л. а. х. соответствующие трем рассмотренным случаям. Рис. 144. Низкочастотные ветви желаемой л. а. х.: 1) без учета момента нагрузки; 2) с учетом момента нагрузки прн первом методе демпфирования; 3) с учетом момента нагрузки прн втором методе демпфирования. 268. Для системы регулирования построить низкочастотную часть желаемой л. а. х., если известно, что при изменении входного задающего воздействия по закону #! = Э1гаах sin сокг, где 6imax = 30°, со,.-: -|^=12,56 сек~1, допустимая ошиока слежения не должна превышать по А© фазе Аф^10, по амплитуде й ^1%. Система обла- дает астатизмом первого порядка. В области низких частот передаточная функция разомкнутой системы аппроксимируется выражением W (/со)= *и ico(l+/co7,i)" Решение. На рис. 145, а показана векторная диаграмма ошибок. Ошибка слежения
222 гл. 6. синтез линейных систем [гее где Фа — синфазная составляющая ошибки, fyp —квадратурная составляющая ошибки. Фазовая ошибка , КМ A(p = arctg[0[_0A[, относительная амплитудная ошибка АО _ | О, | - | ft2 | о lmax Чипах Если считать, что при частоте <ок модуль | W(}a>K)\'^> 1, то фазовую ошибку можно подсчитать по приближенной формуле Дф~тЛ- Im-e,max- - Wk lmax ^04) Ко а относительную амплитудную ошибку —по формуле д# lmax lmax -"lmax Re в, lmax <£г, w 04) Ко К Задание величины фазовой и относительной амплитудной ошибок определяет предельное левое положение атщ^я Рис. 145. а) Векторная диаграмма ошибок; б) низкочастотная часть желаемой л. а. х. первой и второй асимптот л. а. х.: сок 12,56 ■ 57,3 = 720 сек~К Вид низкочастотной части желаемой л. а. х. показан на рис. 145, б. 269. Для замкнутой системы комбинированного управления определить уровни компенсирующих сигналов
§ 6.1. ВЫБОР ПАРАМЕТРОВ ПО ТРЕБУЕМОЙ ТОЧНОСТИ 223 по первой и второй производным от входного воздействия, при которых в системе, обладающей астатизмом первого порядка, устраняются скоростная ошибка и ошибка, зависящая от ускорения. Структурная схема замкнутой системы комбинированного управления приведена на рис. 146. Компенсирующие сигналы имеют вид _ф(р)А'1 = (т1р + т1т2р2)в'11 где Ti — отношение крутизны сигнала по первой производной от #i к крутизне сигнала по ошибке Ф, т2 — отношение крутизны сигнала по второй производной к кру* тизне Сигнала по первой производной от fy. лГ <р(р) р(М,рНЩр) « Рис. 146. Структурная схема следящей системы комбинированного управления. Решение. В системе комбинированного управления выходная величина Ф2 пропорциональна не только ошибке Ф, но и компенсирующему сигналу ф(р)#ь т. е. 0*=-1Пр)Ц> + Ф(/>)Ф|]. к где W (р) = р(1+г,р)"(1+7» ~~ передаточная функция разомкнутой системы. Ошибка в замкнутой системе равна л [1-У(р)ф(р)1<>1 v~ l+W(P) Подставляя значения W(p) и ф(р), получим _ [Г.ГгР3 + (Г, + Т2 - Кйт,т2) р» + (1 - Ках) р] О, Tftf + ft + TdpL+p + KQ
224 ГЛ. -6, СИНТЕЗ-'ЛИНЕЙНЫХ СИСТЕМ [27С При выполнении условия 1 в системе устраняется скоростная ошибка.- При дополнительном условии Т\ + Т2 = /Снт^г или . x^Ti + Ti обращается также в пуль ошибка, зависящая от уско* рения. Эквивалентная передаточная функция разомкнутой системы соответствует системе, имеющей астатизм третьего порядка: w (п\- ^(р)И + ф(р)1 ... W'W>- l-W{p)<p{p) ~ = Кв(1 + х1Р + х{г2р2) _ Г{Г^ + (Г, + Tz-KqX^s) P1 + (1 - %*.) Р ~* ■J\T2p* ■ ~ш § 6.2. Алгебраические методы выбора параметров САР 270. Система регулирования напряжения с угольным регулятором (рис. 147) описывается уравнением третьего порядка [(1 + 7» (1 + 7» (1 + 7» + k0kp) Au = = (TlP + l)(l+T2p)fU), где Г0 = 0,02 сек — постоянная времени- генератора (объекта регулирования), &0 = 36 в/ом — коэффициент передачи генератора, Т{ — постоянная времени чувствительного элемента (обмотки электромагнита), Г2—постоянная времени регулятора (угольного столба), kp = = 0,405 ом/в — коэффициент передачи регулятора. Выбрать варьируемые параметры системы регулирования Ти Т2 так, чтобы обеспечить степень устойчивости Л013*0,4 при колебательной форме переходного процесса. Решение. Обратимся к диаграмме Вышнеградского (приложения 7 и 8). Характеристическое уравнение
270] § 6.2. АЛГЕБРАИЧЕСКИЕ МЕТОДЬк ВЫБОРА ПАРАМЕТРОВ 22S (1) системы регулирования имеет вид a0ps + ахр2 + а2р + а3 = О, где D е а0 = Т^Т{Г2, сц — TqTi + Т{Г2 + Т0Т2, a2=To + Tl + T2, a3=l+W Приведем его к нормированному виду <73 + Aq* + Bq + 1 = О, A fl' Т0Т, + Т0Т2 + Т^2 л — з ~ з В = -J = -3 (2) (3) Va04 VW2O + W — параметры Вышнеградского. Решить задачу можно было бы, задавшись предварительно значениями А и В (например, А = 4 и В = 3), Рис. 147. Система регулирования напряжения с угольном регулятором. Удовлетворяющими поставленным требованиям. Однако такой путь определения 7Y и Т2 связан с. решением
226 ГЛ. 6. СИНТЕЗ ЛИНЕЙНЫХ СИСТЕМ [270 системы двух кубических уравнений. Проще можно найтн значения Т\ и Т2 методом последовательных приближений, задаваясь их численными значениями и наблюдая О 7 83456789 70 7/ 12 Рис. 148. Построение траектории на диаграмме Вышнеградского. траекторию точки с координатами А и В на диаграмме Вышнеградского. Подставляя заданные значения параметров в (3), получим расчетные формулы для вычисления А к В: л 0,2(Г, + Г2)+ЮГ,Г2 в_ 0,02 + Г, + Г2 ^ до
2«1 § 6.2. АЛГЕБРАИЧЕСКИЕ МЕТОДЫ» ВЫБОРА ПАРАМЕТРОВ 227 Результаты вычислений А и В по формулаи (4) при изменении Т2 и при Ti~0,0l сек приведены ниже. Та, сек А в 0,1 1,8 0.8 0,2 2.1 1,1 0,5 2,8 1,9 о,? 3,3 2,4 I 3,5 2,8 2 3,8 4,4 На диаграмме Вышнеградского (рис. 148) построена траектория точки G(A, В). Из построения следует,' что для обеспечения поставленного требования Л0> 0,4 достаточно1 при 2"i == 0,1 сек выполнение условия 0,5<Г2<1,8. - (5) Это условие может быть выполнено соответствующей настройкой демпфера регулятора. Если бы траектория G не попала в желаемую область диаграммы Вышнеградского, то нам пришлось бы изменить значение Tt и' аналогичным образом найти новую, смещенную траекторию, проходящую через нужный участок диаграммы. Изменения параметров Т\ и Т2 следует вести, согласуясь с технической возможностью обеспечения задаваемых значений. 271. Для следящей системы, схема которой приведена на рис. 149, определить требуемые значения коэффициента усиления усилителя ky и коэффициента передачи по петле обратной связи k0 при заданных значениях общего коэффициента усиления системы /<а = 500 сек'1 и степени затухания переходного процесса т) = 98%. Передаточная функция разомкнутой системы с учетом тахометрической обратной связи имеет вид К где Гм= 0,03 сек — электромеханическая постоянная времени двигателя, К = &,&тр&у£дв — общий коэффициент
228 ГЛ. в, СИНТЕЗ ЛИНЕЙНЫХ СИСТЕМ l»t усиления системы без учета влияния обратной связи, &. = 0,1 в/рад — крутизна чувствительного элемента, ^дв = ttff" рад/сек • в — коэффициент передачи двигателя, ku = kykRBk0 с, kTp = 3—коэффициент трансформации входного трансформатора, k0 c — коэффициент передачи цепи обратной связи. &■■ LXttO Вх.тр. О/ Рис. 149. Следящая система с тахиметрической обратной связью. Решение. Найдем передаточную функцию замкнутой системы относительно ошибки %(р) = p + ^-H + ke) *■ м ' м а) Характеристическое уравнение замкнутой системы при этом равно />2 + В.р + В2 = 0,. (2) Где £i = y-(l + *o) и В2 = -£-. При степени затухания т_ = 98% должно выполняться условие M^)fi' (3) или
6.2. АЛГЕБРАИЧЕСКИЕ МЕТОДЫ ^ВЫБОРА ПАРАМЕТРОВ 229 ОТКуДа / 2 , л\ h (Я2+ 4) ,. , .2 Общий коэффициент усиления системы Ка связан с коэффициентом усиления К соотношением К _ я2+ 4 !0 ^а ^ та:=-ш\т(1+ *<>>• <4) Из этого равенства следует, что *о = ^ТТ^~1- (5) Численные значения коэффициентов равны *° = 1Ц4^500-1==16'3> /С = Ка (1 + *ь) = 500 (1 + 16,3) = 8700 сек~\ ь = К = 8700,°'55 = 1617 272. Пользуясь методом стандартных переходных характеристик [2,15] (см. приложение 18), выбрать параметры системы ■ регулирования так, чтобы время затухания переходного процесса было /^1,5 сек, а величина перерегулирования а <110%. Передаточная функция разомкнутой системы имеет вид WW>- рЧ1 + :г2р) ' где KR — общий коэффициент усиления разомкнутой системы по ускорению, Т\ и Т2 — постоянные времени. '■■ Решение. Соответствующая стандартная передаточная функция имеет вид (приложение 18) ! ' «о 7 6>3 \ в,з^ч-юз=5т(-ьур;
230 ГЛ. 6. СИНТЕЗ ЛИНЕЙНЫХ СИСТЕМ Г273 Приравнивая ее заданной передаточной функции, получим условия для выбора параметров: KR 5,1 * 6,3 5,1ш0 Для того чтобы время затухания переходного процесса не превышало заданной величины, необходимо, чтобы т 9 п _1 «о = 7" ^ 1J = 6 сек » где т — время переходного процесса. Тогда Яе = |г = 7,05 сек-2, Г, =-£?. = 1,05 сек, Г2 = j~ = 0,032 сек. Рис. 150. Корневой годограф - к задаче 274. Таким образом, передаточная функция разомкнутой системы должна иметь вид 7,05 (1 + 1,05р) /ш го юо ' 1*1 X 1 1 Н» |<* -6 -5 -4 -3 -2 -13 W(p)- р2(1+0,0326р) 273. Пользуясь методом стандартных переходных характеристик (приложение 18), выбрать параметры следящей системы так, чтобы общий коэффициент усиления разомкнутой системы по ускорению был бы Ке^ 100 се/Г2, а величина перерегулирования а^10%. Общий вид передаточной функции разомкнутой системы тот же, что в задаче 272. Ответ. TWnW Ю0(»+0,28р) W{P) р2(1+0,0087р)' 274. Для системы автоматического регулирования, имеющей в разомкнутом состоянии передаточную
tfSl § 6.3- ЧАСТОТНЫЕ МЕТОДЫ ВЫ*ОРА ПАРАМЕТРОВ 231 ФУНКЦИЮ к k к где f = 1 сек, Т2 — 0,25 сек, k = уу-, построить корневой годограф. Ответ. Корневой годограф состоит из трех ветвей, поскольку знаменатель функции W(p) имеет третью степень. Вид годографа показан на рис. Г50. § 6.3- Частотные методы выбора параметров САР. Расчёт последовательных корректирующих устройств 275. Построить желаемую л% а. х. и произвести выбор последовательного корректирующего устройства для си» стемы автоматического регулирования, если передаточ* ная функция разомкнутой системы при отсутствии корректирующего устройства имеет вид W(p) = Kq Р(Р + TiP) (1 + Пр) (1 + Tsp) (1 + 7» ' где Г, = 0,1 сек, Т2 = 0,02 сек, Т3 = 0,01 сек, ТА = 0,005 сек. Система регулирования должна быть астатической системой первого порядка и удовлетворять следующим показателям качества: а) коэффициент ошибки по скорости Cj = -2qq сек, б) коэффициент ошибки по ускорению С2 = 0,06 сек2, в) перерегулирование or при единичном ступенчатом управляющем воздействии не должно превышать 30%, г) время переходного процесса t„ при единичном ступенчатом управляющем воздействии не должно быть больше чем 0,8 сек при числе колебаний, не превышающем двух. Решение. В соответствии с методом [30, 32] на рис. 151 построена исходная л. а. х. L нескорректированной системы с коэффициентом усиления 7(q, равным требуемому /Со = J- = 200 сек~\
232 ГЛ. fi. СИНТЕЗ ЛИНЕЙНЫХ СИСТЕМ .[£73 Затем по заданным показателям качества построена желаемая л. а. х. L>K. Первая сопрягающая частота желаемой л. а. хч согласно пункту б), определяется из следующего приближенного выражения: v <°i те /- „ = хлк опп «* 0,08 сек-1. Cg/Cfj 0,06 - 200 Для обеспечения условия пункта в) достаточно, чтобы Рис. 151. Логарифмические амплитудные и фазовые частотные характеристики к задаче 275. желаемая л. а. х. Лж имела запас устойчивости по модулю ±16 дб'и по фазе 45° (рис. 152). Найдем теперь значение частоты -срезе.-«с. 'Пользуясь приложением 12,13, получим при а = 30%, что соответствует РЯ1ах= 1,3, 11,5 . , .' ■ _, • сос *» -——- *« 14 сек . Проведем через точку юс прямую с наклоном 20 дб на декаду. Пересечение этой прям,ой со второй асимптотой
Wrei 6.3. ЧАСТОТНЫЕ МЕТОДЫ ВЫБОРА ПАРАМЕТРОВ 233 желаемой л, а. х., имеющей наклон 40 дб на. декаду, даёт вторую сопрягающую частоту, со2= 1,3 сек'1. В рассматриваемом примере сос/со2> 10, что вполне допустимо. Таким образом, желаемый- вид 1ж при со<сос найден. Перейдем к выбору формы LM при со>сос, обращая особое внимание на то, чтобы на каждом из участков наклон желаемой л. а. х. возможно меньше отличался от наклона исходной л. а. х. Попытаемся удовлетворить заданным условиям качества, ограничившись разностью наклонов между 1ж и L, не превышающей 20 дб на декаду. Тогда 1ж должна иметь, как это ясно из рис. 151, со1рягающие частоты. сй3 = 50 сек'1-, ш4 = 100 сек*1, соответствующие сопрягающим частотам исходной л. а. х. Начиная с частоты (о5 —200 сек~1, желаемая л. а. х. совпадает с исходной л. а. х. Желаемая. передаточная функция имеет вид 40 б, % Pirc. i52. Кривые для выбора запаса устойчивости но ми- дулю I и но фале [Л,.. W^ip)- Ч1+тй •('+Tfe№+*)(l + i8r)(l+w)f Запас устойчивости определяется видом логарифмических характеристик в области средних частот, т. е. в интервале со2 <; со ^ со3. Проверим, имеет ли полученная 'л*, а. х. Ьж требуемый запас устойчивости по фазе при LK= 16 дб (со = со2), 0 дб (co = coL.) и 14 дб (со = со3). Согласно рис. 151 при L?IC = 16 дб со = 2 сек'1 и Ч>(2)= - 90- arctg — + arctg-jj- - 121°. Этр: соответствует запасу по фазе =-■: |i* 180° -F -ф == 180° -121° = 59°.
234 ГЛ. 6: СИНТЕЗ ЛИНЕЙНЫХ СИСТЕМ' IS78- При Lx— — 14 дб, о = 50 сек ', ф (56) = -90 - arctg-Ц - arctg -^ - 2 arctg -^ = -190° и соответственно ц.= 180°— 190°= — 10°. При Ьж = 0, со = сос = 14 сек'1, чр(14) = - 90°-arctg-щ-Ч-arctg-^--arctg■g^- - arctg —- - 2 arctg -^ = - 108°, и соответственно ц= 180°- 108° = 72°. Из трех полученных значений ф(со) только второе не укладывается в требуемые пределы. Это может привести к незначительному увеличению абсолютного значения | Pmin I по сравнению с принятым (| Рт1п [ = Ртах — 1 = =0,3), что, как известно [30, 32], несущественно. Поэтому найденная л. а. х. LXi может использоваться для синтеза корректирующих средств. Tf~ icq n " Вычитая ординаты L из орди- Рис. 153. Пассивное „ ^ . , fir... интегро-дифференци- нат желаемой л. а. х. 1ж (рис. 151), рующее звено. получим л. а. х. последовательного корректирующего устройства, которая на рис. 151 не показана. В рассматриваемой задаче в качестве корректирующего устройства нужно использовать пассивное интегро-дифференцирующее звено (рис. 153), передаточная функция которого имеет вид т in\ .\1+"Ш\1 + То) = (1+0,77р)(1+0,1р) W*W) i „м „ \ (| + 12,5р)(1+0,005/»)-* (I+w)(1+wJ Для проверки полученных результатов построим фа-- зовую характеристику я|)ж(ю) (рис. 151), а также, пользуясь номограммой приложения 11, определим вещественную частотную характеристику Т5 (со) замкнутой системы (рис. 154, а). Применяя метод трапецеидальных
$ 6.3. ЧАСТОТНЫЕ МЕТОДЫ ВЫБОРА ПАРАМЕТРОВ 235 характеристик, построим график переходного процесса (рис. 154, б). Переходный процесс в системе удовлетворяет заданным показателям качества. Рт t,cet< Рис. 154. с) Вещественная частотная характеристика замкнутой системы; б) график переходного процесса. • 276. Произвести выбор последовательного корректирующего устройства для системы автоматического регулирования. Передаточная функция разомкнутой системы без коррекции имеет вид W <Р) = (1 + 2» (1 + ВД (1 + Тгр) ' где Ti~0,05 сек, Т2=.0,1 сек, Г3 = 0,2 сёк. Скорректированная система должна обеспечивать следующие показатели качества переходного процесса при ступенчатом управляющем воздействии: а) перерегулирование о<С20%; б) время затухания переходного процесса 4<|0,6 сек при числе колебаний я^З; в) установившаяся ошибка Д не должна превышать 3%. Решение. Произведем выбор корректирующего устройства при помощи амплитудно-фазовых характеристик [I, 36]. Для получения установившейся ошибки в 3% необходимо, чтобы коэффициент передачи системы был не ниже К - '~д _ 1-0.03 _ or, А~ Д ~ 0,03 ~**'
ГЛ Г, СИНТЕЗ ЛИНЕЙНЫХ СИСТЕМ fi76 Дли : построения амплитудно-фазовой характеристики ско/рректированной системы нужно выбрать соответствующую форму вещественной частотной характеристики Р(со). ет го 40 60 ш,-^ Рис. 155. а) Вещественная частотная характеристика; б) гра- •' фик Q = /(—-I; в) мнимая частотная характеристика. Исходя из заданных показателей качества, а также задаваясь коэффициентом наклона х = 0,7, найдем, пользуясь номограммой (см. приложение 14), значения вещественной частотной характеристики Р (со), обеспечивающей нужные качественные показатели скорректированной системы. Для а = 20% и Ршх—1,0 находим- Pmia — 0,3;
8761 $ 6.3. ЧАСТОТНЫЕ МЕТОДЫ ВЫБОРА ПАРАМЕТРОВ 237 запас устойчивости по модулю Л/? = 55%, запас по фазе Дф = 40°, а также *„ = —-• При заданном времени ре- <о„ гулирования получим интервал положительности (0„ 3,8зт 3,8тс /.. 0,6 20 сект На основании значения сои и параметров, положенных в основу номограммы приложения .14, построена вещественная частотная характеристика Р(ю) (рис. 155, а). К 32 Начальная ордината Р (0) = -j+j^ = ТТ32 = С*'97, (od = х(опО,7 • 20 = 14 сек~1, (ол = 7i.(o„0,5 • 20 = 10 сек'1, 4 coe = x'efflfc0,5 -10 — 5 сек"1, а2 = 25 сек~г, щ = -г1 = -^ = 42 сек'1, о»! = A;,a>o = 0,7 • 42 = 29 сек-1. При помощи графика Q = / (—}, составленного для трапеции высотой, равной единице (рис. 155, б), йостроена мнимая частотная характеристика замкнутой системы (рис. 155, в). По характеристикам Р (со) и Q (со) легко построить амплитудно-фазовую характеристику скорректированной системы f36|. Эта характеристика построена на рис. 156, а по данным табл. 1. На рис. 156, б пунктирной линией изображена амплитудно-фазовая характеристика нескорректированной системы (табл. 2). Таблица 1 Амплитудно-фазовая характеристика скорректированной системы «а, сек~1 1])с((й) 2(1 0,37 -116° 25 0,20 -130° 30 0,17 -150° 40 0,11 -160° 50 0,06 -18.1"
238 ГЛ. 6. СИНТЕЗ ЛИНЕЙНЫХ СИСТЕМ 127в Таблица' 2 Амплитудно-фазовая характеристика нескорректированной системы <о, сек ' R (ш) 15 4,6 -164° 20 2,3 -185° 25 1,15 -200° 30 0,99 -208° 40 0,5 -220° 50 0,23 -231° Модуль и аргумент амплитудно-фазовой характеристики корректирующего устройства получатся из харак- - 1т -Ш-OJ, ' /о*40 Ьи=30- ta=Z5\ e>=20f 0.1 Re -0.1 -0,2 -0,3 Jm 1 • О Re / -г -1 -1 -г б) Рис. 156. Амплитудно-фазовые характеристики: а) скорректированной системы; б) нескорректированной системы; в) корректирующего устройства. теристик нескорректированной и скорректированной систем Rk (со) 幫т = Ш1. е/[Фс<»>-*<«»]. Данные вычислений сведены в табл. 3.
2771 § 6.3. ЧАСТОТНЫЕ МЕТОДЫ ВЫБОРА ПАРАМЕТРОВ 239 (0 Rk Ук Таблица 3 Амплитудно-фазовая характеристика корректирующего устройства сек"1 (со) (ю) is 0,23 33° 20 0,16 69° 25 0,174 70° 30 0,17 58° 40 0.22 60° 50 0.26 50° По найденным значениям можно построить ампли» тудно-фазовую характеристику корректирующего устрой* ства. Дальнейшее решение задачи состоит в том, чтобы выбрать такой тип корректирующего контура, ампли* тудно-фазовая характеристика которого наиболее близко совпадает с расчетной амплитудно-фазовой характеристикой корректирующего устройства. Предположим, что в области низких и высоких частот амплитудно-фазовые характеристики скорректированной и нескорректированной систем должны совпадать. Тогда в .качестве корректирующего контура следует выбрать пассивное интегро-дифференцирующее звено с передаточной функцией тт. 1„\ _ (1 + 7W>) (1 + Т3кр) WAp)-(\ + tiKP){1 + TiKPy Амплитудно-фазовая характеристика этого контура представляет собой окружность с центром в точке 0\ (рис. 156, в). Взяв для любых четырех точек значения модуля RK или фазы я[)к, найдем постоянные времени TiK=l,85 сек, Гак=0,18 сек, Г3к=0,08 сек, Г4„=0,02 се/с. 277. Определить передаточную функцию последова» тельного корректирующего устройства для следящей системы, передаточная функция которой имеет вид W (Р) = Р2 (1 + 2» (1 + Т2р) (1 + Т3р) ' где Г, = 0,04 сек, Т2 = 0,01 сек, Г3 = 0,002 сек. Следящая система должна иметь астатизм второго порядка
240 ГЛ.. 6. СИНТЕЗ ЛИНЕЙНЫХ СИСТЕМ [278 и удовлетворять следующим показателям качества: а) общий коэффициент усиления по ускорению /СЕ^ 100 сект2; б) перерегулирование а <|30%; в) время затухания переходного процесса f„^ <|0,45 сек. Ответ. wk(p)=- _ (1+0,25р)(1+0,04р) ' (1 + 0,0029р) (1 + 0,00066р) ' График переходного процесса построен на рис. 157. 278. Произвести динамический синтез следящей системы по следующим качественным показателям: ошибка .vmax^0,1 град при максимальной скорости слежения- Qimax = 20 град/сек и максимальном ускорении eImax = 5 град/сек2; запас устойчивости оценивается по показателю колебательности М ^ 1,5. Передаточная функция исходной нескорректированной системы имеет вид где 7=0,1 сек. Решение. Определяем запретную область по точности (рис. 158): Рис. t,ce«- 157. График переходного процесса к задаче 277. <»к = "imax Л! "lmax *max 20_ ,1 ■■ 200 сек' «-s-/^SL-■/^г■"7•07c^,• Желаемую л. а. х. системы Ьж в области низких частот формируем из двух отрезков прямых, имеющих наклон — 20 дб/дек и — 40 дб/дек с точкой излома на частоте 1 ю„ = = 0,25 сек~\ Для обеспечения требуемой точности Лж приподнимаем над запретной областью на 3 дб, т. е. необходи-
S7«l 5 6.3. ЧАСТОТНЫЕ МЕТОДЫ ВЫБОРА ПАРАМЕТРОВ Ш мое значение общего коэффициента усиления определяется -из условия Ла-1,41%= 1,41 -^i. = 282 сек'1 #тах И , ю0=1,19(о0. к =1,19-1/-^«^ = 8,42 сек1. У #тах \ Определяем постоянные времени Т2 и Г3 (см. рис. 158): /з- А - м+1 U'U4/ С6К- Передаточная функция скорректированной системы, т. е. L дб Р 50 6Л чп Си W ~iu *ъ^ м V I / А % Xi s=r—- "•^ еГ~-~ wr i fifanT ' %, Nfc ^ / ^ПКУ ш ф ъ% / \h \ 4* щ "ф |ц i_ lUI'—J i/ 1 i" ~- \ ^ >j ;--^J (И/^ ~1 J i i j -1 i i i i Г;-':- '■ . "~-~^_ . too ---^ ччЛ _ (У Рис. 158. Л. а. х. к задаче 278. соответствующая желаемой л. а. х., может быть записана в виде WCK[p)= p (1 + Г,р) (1 + ГзР) • Для коррекции системы следует использовать пассивное интегро-дифференцирующее звено с передаточной функцией
242 ГЛ. 6. СИНТЕЗ ЛИНЕЙНЫХ СИСТЕМ (279 Схема этого звена и его частотные характеристики показаны на рис. 159. Рис. 159. Интегро-дифференцирующее звено и"его частотные характеристики. 279. Определить последовательное корректирующее устройство и рассчитать необходимый коэффициент усиления усилителя k2 для следящей системы, структурная -к &г 43 )-[^Т--и-л u,=H}ff ИУ Эй» Рис. 160. Схема следящей системы. схема которой изображена на рис. 160. На схеме обозначено: Д — двигатель, КУ — корректирующее устройство, Р — редуктор, ЧЭ — чувствительный элемент, определяющий рассогласование, РМ — рабочий механизм, fy и #2 ~~ углы поворота задающей и исполнительной осей. Исходные данные: I) крутизна чувствительного элемента ki = 10 мв/угл • мин = 34,4 в/рад;
§ 6.3. ЧАСТОТНЫЕ МЕТОДЫ ВЫБОРА ПАРАМЕТРОВ 243 2) передаточное отношение редуктора « = 3500; 3) максимальная скорость слежения Q = 5 град/сек 300 угл. мин/сек; 4) максимальное ускорение в = 2 град/сек2 — 120 угл. мин/сек2; \'- Б) максимальная ошибка &тах = Г 6) максимальное напряжение . выхода усилителя 7) максимальная скорость. двигателя при полном открытии усилителя Qamax = 6000 об/мин = 630 1/сек; 8) пусковой- момент М0 = 100 Г • см — 9,81 • Ю_3к • м; механические характеристики двигателя совместно с усилителем представляют собой параллельные прямые; 9) момент нагрузки на оси двигателя Мн= 10 Г • см = = 9,81 • Ю-4 н-м; 10) момент инерции, приведенный к оси двигателя8 • / = 0,018 Г • см ■ сек2 = 17,6 • 10~8 кг • м2; 11) постоянная времени усилителя Гу = 0,02 сек; 12) показатель колебательности М^1,50 Решение. Передаточная функция разомкнутой системы при отсутствии корректирующих устройств равна произведению передаточных функций звеньев: Коэффициент передачи двигателя равен йд max " 630 Коэффициент наклона механических характеристик двигателя вместе с редукторлм о __ Ро йд тах 630 » Ш0 = 3500 • 100 = = 1,8 • 10" \/Г • см • сек = 6,3 угл • мин/Г • см • сек. Постоянная времени двигателя Уд = Ро/= -§§■• 0,018 «0,1 сек..
244 ГЛ. fi. СИНТЕЗ ЛИНЕЙНЫХ СИСТЕМ [879 Для определения необходимого значения общего коэффициента усиления (добротности) по скорости Kq построим запретную зону для низкочастотной части л. а. х. (см. задачу 264). Контрольная частота оо,< = -gj- = у = 0,4 сек"1. Ордината контрольной точки *■ = 20 .*■££?*£ пп. 3002 +6,3-10-300 сп ,,. 20 lg ;—77^ ■ = 59 дб. 'П1.1Х6 — I • 120 Предельное значение добротности по скорости а + рл4„ _ 300 + 6,3- ю /<Q = 1 363 сек'1. По этим данным построена запретная область (рис. 161). la тольЖ Рис. 161. Л. а. х. к задаче 279.' Проверим возможность работы следящей системы без корректирующих звеньев. Так как первая сопрягающая частота л. а. х. рередаточной функции (1), равная <й|=-^- = 10 сек'1, значительно больше контрольной ' д частоты ©„=.0,8 сек'1, то можно в качестве окончательного значения: добротности по скорости принять
79flw § 6.3. ЧАСТОТНЫЕ МЕТОДЫ ВБ1ЕОРА ПАРАМЕТРОВ 245 значение, равное 363 сек'1. Соответствующая л. а. х. типа 1—2 — 3 показана на рис. 161. Допустимая сумма постоянных времени v 1 M* + mYm*-1 _ ZT = %. 2 1 1,52+1,5]Л,52-1 363 -0,0054 сек. В действительности сумма имеющихся постоянных времени 1iT = TA + Ty = 0,10 + 0,02 = 0,12 сек. Таким образом, убеждаемся, что без корректирующих устройств система не будет обладать требуемым показателем качества. Рассмотрим возможный способ улучшения динамических свойств системы при помощи последовательных звеньев [4]. При введении в прямой канал пассивного звена, содержащего инерционную часть, необходимо так сформировать желаемую л. а. х., чтобы пик ошибки в районе перемены знака скорости не превосходил заданного максимального значения '©max- Найденному значению добротности по, скорости Ко = 363 сек-1 соответствует добротность по моменту KQ 363 Кы = -g- = -gg- ~ 57,5 / • см/угл • мин. • ■ Моментная ошибка ' А _.Л*н _ 10 0,174'. м Кы 57,5 , Допустимое значение большей постоянной времени f, = 0,236 П*Ж+*Л _о,236 ^^ -0,16 сек. ®ЫУ7 . 0,174 V 120 По добротности Кв. = 363 сек~г и постоянной времени ^1^=0,16 сек можно построить - низкочастотную часть
246 ГЛ. «. СИНТЕЗ ЛИНЕЙНЫХ СИСТЕМ {£79-. л. а. х. (рис. 161). Базовая частота л. а. х. Сформируем теперь низкочастотную и высокочастотную части желаемой л. а. х. типа 1—2—1—2. По.базовой частоте определим требуемое значение второй постоянной времени: *■« - -к Vw^t = 4^/35 - °'0365 сек- Третья постоянная времени равна т_ t умт-Р t yi,5(i,5-t)_ ls Wo M+l. _47,5 1,5+1 -U,UU/dce/C. По этим данным построена вся желаемая л. а. х. Lx. Л. а. х. корректирующих средств получается в результате вычитания ординат располагаемой л. а. х. из ординат желаемой л. а. х. Эта разностная' л. а. х. также изображена на рис. 161. Из вида этой л. а. X; следует, что последовательно корректирующее устройство должно состоять из: . I) пассивного интегро-дифференцирующего звена g передаточной функцией , (1 + ГдР) (1 + Т2Р) wK(P) (i + tiP)(1 + TbP) ' где постоянная времени Тв определяется из известного свойства интегро-дифференцирующего звена ТпТ2 0,05 ■ 0,0365 7-0 = -^--—5^ = 0,0114 сек; . 2) идеального дифференцирующего звена с передаточной функцией wK2(P)=i + ryP, 3) комбинации пассивного дифференцирующего звена и линейного усилителя с общей передаточной функцией 1 л. Т „ ~-(1 + Твр) тот I \ ' + *вР « *В WksW- 1 + Гзр -«у 1 + Гзр •
§ 6.3. ЧАСТОТНЫЕ МЕТОДЫ ВЫБОРА ПАРАМЕТРОВ 247 Идеальное дифференцирующее звено может приближенно создаваться за счет использования тахогене- паторов на задающей и исполнительной осях, включен-* ных встречно и вводящих производную от угла рассогласования. В случае включения сигнала от тахогенератороз. в том же месте, куда включен и сигнал от основного % * &*'* \Лр КУ *~и.=ктгр& О Рис. 162. Схема с тахогенераторами для получении звена чистого дифференцирования. чувствительного элемента (рис. 162), требуемая крутизна напряжения каждого тахогенератора равна &тг = kxTy = 10 • 0,02 = 0,2 мв ■ сек/уел • мин = = 0,37 в • сек/об = 0,06 в • сек. Пассивные звенья должны вводиться в прямой канал усиления и могут состоять из R С-цепей. Коэффициент усиления усилителя с учетом дополнительного усиления, необходимого для работы пассивного т Дифференцирующего звена ky — -^-, равен fe»-^-- Кй1 0,0114 363-3500 Txkl = 0,0073 34,4 • 5,73 = 9850. В случае невозможности устпт-',*,'"ч "тахогенераторов Для введения производной от утл., рассогласования Можно изменить вид требуемых кэр^л-^ующих средств.
248 - - ГЛ. 6: СИНТЕЗ ЛИНЕЙНЫХ' СИСТЕМ fJ7» Как видно из рис. 161, идеальное дифференцирующее звено получилось вследствие того, что высокочастотная асимптота исходной л. а. х. имеет большой наклон по сравнению с наклоном желаемой л. а. х. Для устранения этого можно изменить вид желаемой л. а. х. в высокочастотной области, перейдя от л. а. х. типа 1—2 — 1—2 к л. а. х. типа 1—2—1-^3, но с- тем же запасом устойчивости. Высокочастотная часть последней л. а. х. ноказана на рнс. 161 пунктиром. Ей соответствует передаточная функция ~ . WKP> p{\ + TiP)(\+T,pf с Постоянная времени 7"5 определяется как Т* = ы; V М+х = Т "= 0>0036 сек' Л. а. х. корректирующего устройства показана для этого случая на рис. 161 также пунктиром. Из рассмотрения этой л. а. х. видно, что корректирующее устройство последовательного типа должно состоять из трех пассивных звеньев: интегро-диффереицирующего звена и двух пассивных дифференцирующих звеньев в комбинации с линейным усилителем с общей передаточной функцией (1+Гдр)(14-Г£р) 14-7> I 4- Тур WAP)- {i+Tlp).(l+rBp) " \+Тър ' 1+Т5р~ (1+Гдр)(14-У»(1 + 7» ■{1 + Тф)(1+Тьр)* Коэффициент усиления ky будет выше, чем в предыдущем случае, ТУТВ 0,02 • 0,0114 , .у Ц 0,00362 Общий коэффициент усиления усилителя оказывается также значительно больше, '"" " Кы . 363-3500 . : **.rb/&=17>5-swm-поооо. ,.
§ 6.4. РАСЧЕТ ДОПОЛНИТЕЛЬНЫХ. UbPAitibiA (..вязни 24У Возможен и иной выбор высокочастотной части желаемой л. а. х., в частности, соответствующей передаточной функции вида К0_{\ + Т2р) . W^ " р (1 + Tip) (1 + Ttp) (1 + Т5р) • При этом Г4 ф Т5, но их сумма по-прежнему должна быть равной _ г , г _ 1 Ум(М~\) § 6..4. Расчет дополнительных обратных связей и прямых параллельных корректирующих связей 280. Произвести расчет дополнительной обратной связи для следящей системы задачи 278. Решение. Найдем дополнительную обратную связь WoAp)' эквивалентную пассивному интегро-дифферен- цирующему звену WBKy{p\ полученному при решении задачи 278. Предположим, что дополнительная обратная ' связь охватывает часть системы с передаточной функцией Гогда Wm,y(p)Wc(p) . (1+Г»р)(1 + Гр) (14-7»(1 + Г3р) kocP* С + Тгр) (1,+ Тр) kc 1 + 7'гр' il+Ttp)(l+Ttp) ' р(\+Тр) где Kc,- Г, + Г3-Г2-Г Возможная схема осуществления этой обратной связи .показана на рис. 163. 281. Произвести выбор параметров жесткой тахеометрической обратной связи для следящей системы, схема которой приведена на рис. 164. Исходные данные те же, что и в задаче 279, кроме Тл = 0,05 сек.
250 ГЛ! 6. СИНТЕЗ ЛИНЕЙНЫХ-СИСТЕМ [281 Р е ш е.н и е. Передаточная функция разомкнутой системы с учетом действия тахометрическои обратной связи примет вид где /Chq = • KQ р(\+ар + Ь(?) ' -новое значение общего коэффициента усиления по скорости (добротности по скорости), а = тд + ту тлту . и о = т—т коэффициенты эквивалентного 1 + *ос 4 -я % \ р чэ Н—в~ liftoff f О' >}0 Рис. 163. Схема дополнительной обратной связи к задаче 280. Рис. 164. Схема следящей системы с жесткой тахометрическои обратной связью. звена второго порядка, koc = kjtsk„ — коэффициент усиления канала обратной связи, krr—крутизна тахогене- ратора и масштабирующего устройства в цепи обратной связи. Для обеспечения необходимого запаса устойчивости, оцениваемого величиной показателя колебательности, следует соблюсти неравенство [2] мг + м У~м' hlli. = а< 1 1 К н£2 (2) Нужно учесть также то обстоятельство, что введение жесткой тахометрическои обратной связи изменяет в (1 + koc) раз наклон механических характеристик исполнительного двигателя. Требуемое значение добротности по скорости при учете более жестких механических
8811 ■ § в.4. РАСЧЕТ ДОПОЛНИТЕЛЬНЫХ ОБРАТНЫХ СВЯЗЕЙ ggl характеристик равно о . №_ ''max т^ KvQ = а • (3) _ Решая два последних уравнения совместно, можно определить требуемое значение коэффициента усиления канала обратной связи. Qmax (Гд + Ту) fe°c = 2цАтах + + Гу)2 , РМ.(Гл + Гу) где 300-0,7 . .. /* 3002 • 0.072 6,3-10-0,07 1п 2-1,96-1 + V 4-1,962-1 Н ШП = Ш» M2 + mVW^~\ 1,52+1,5У1,52"1 , пе \ >■ Ц = 2 = 2 = 1 ,УЬ. Требуемое значение добротности по скорости (3) К*а = р±-^ = 306 с&к-К Допустимая сумма постоянных времени (2) Vt 1 1,52+1,5]Л,52-1 nnnR„ „ 2лт = -ш~—2——=0>0064 сек- Имеющаяся эквивалентная постоянная времени Гд+Гу 0,07 ' ■ а — . , ,— = , , ,л = 0,0064 сек. 1 + k0 с 1 + 10 Таким образом, задачу выбора параметров цепи обратной связи можно считать решенной. Передаточная функция разомкнутой демпфированной системы примет вид *hq 306 W(p) = p{l+ ap.+ be?) p(l + 6,4• 10~3р + 9,1 - 10"V) "
252' ГЛ. 6. СИНТЕЗ ЛИНЕЙНЫХ СИСТЕМ [282д ■ В заключение определим требуемый коэффициент усиления усилителя и требуемую крутизну тахогенератора. Общий коэффициент усиления разомкнутой цепи, следящей системы при отключенной обратной связи должен быть равен /С'.а = /С..и (1 + /г9с) = 306 • 11 ** 3360 сек'1. Коэффициент усиления усилителя £,= K'„Qt 3360-3500 Ai*s 34,4 • 5,73 = 59 500. Требуемое значение крутизны тахогенератора с учетом масштабирующего устройства тг fe,ft. 3-10 в • сек. ■2к3 59 500 • 5,73 Большое значение коэффициента усиления усилителя является недостатком рассмотренного варианта введения обратной связи. 282. Произвести выбор параметров гибкой тахометри- ческой обратной связи для следящей системы, схема *J $ ut=f(fff Р 0- >р] сг ИЗ \Ю Рис. 165. Схема следящей системы с гибкой тахометричоской обратной связью. которой приведена на рис. 165. Исходные данные те же, что в задаче 279. Крутизна тахогенератора /гтг = 0,05 в'сек. Решение. В связи с тем, что используется демпфирование по первому способу [4] (см. § 6.1), желаемую л. а. х. Ьж можно так сформировать, чтобы ее первый излом совпадал с контрольной частотой точки Ак
S 6.4. РАСЧЕТ ДОПОЛНИТЕЛЬНЫХ ОБРАТНЫХ СВЯЗЕЙ 253 (рис. 166). При этом л. а. х. должна быть поднята выше запретной зоны на 3 дб. Требуемое значение добротности по скорости будет Кти = V2 Ка — 1,41 - 363 = 512 сел"1. Рис. 166. Логарифмические амплитудные характеристики к задаче 282. Это вызывает необходимость иметь коэффициент усиления усилителя K.rUi 512JJ5W **=А^=зм.5,таГ9100- Базовая частота желаемой л. а. х. ©(,= I/ ТГ = ]/^^tTi= 1/0,4-512=14,3 сек"1.' Вторая постоянная времени желаемой л. а. х. -l -./: Допустимая сумма постоянных времени, соответствующих сопрягающим частотам, правее ч.астоты среза [2, 4],
254 ГЛ. 6. СИНТЕЗ ЛИНЕЙНЫХ СИСТЕМ [282 равна 2 X Уи<:17П-4^П^Г,} -0,024 сек. ©о М+1 14,3 1,5+Ь ' Сформулируем желаемую л. а. х. таким образом, чтобы ее высокочастотная" асимптота имела одинаковый наклон с высокочастотной асимптотой располагаемой л. а. х. L. В данном случае этот наклон составляет 60 дб/дек. Тогда в высокочастотной части желаемой л. а. х. можно иметь двойной излом при частете со5 = -=-. Соответствующая постоянная времени должна быть равна У. Т 0,024 Г5 = ^- = -2- = 0,012 сек. Для упрощения корректирующих средств можно продолжить участок с единичным наклоном желаемой л. а. х. до совпадения высокочастотных- асимптот Ьж и L, что показано на рис. 166 пунктиром. Это даст некоторое увеличение запаса устойчивости. Постоянная времени, определяющая двойной излом желамой л. а. х., может быть определена непосредственным измерением сопрягающей частоты. Она равна Г6 = 0,009 сек. Построенной таким образом желаемой л. а. х. соответствует передаточная функция разомкнутой системы mi у- *iQ(1 + raP) _ 512(1+0,1%,) W{P) p(l + TlP)(l + Tep)2 p(l+2,5p)(l+0,009)2 ' В дальнейшем мы будем ориентироваться на этот более простой случай. На рис. 166 построена л. а. х. корректирующих устройств последовательного типа LK, полученная вычитанием ординат L из ординат Ьж. Она соответствует последовательному включению интегро-дифференцирую- щего и дифференцирующего звеньев с передаточной функцией wn3(p) (1 + 7»(1 + гвР)2 Полученная передаточная функция играет вспомогательную роль, так как по условию задачи коррекция
§ 6.4. РАСЧЕТ. ДОПОЛНИТЕЛЬНЫХ ОБРАТНЫХ СВЯЗЕЙ 255 системы должна осуществляться обратной связью, а не последовательными звеньями. Поэтому, пользуясь ею, необходимо вычислить эквивалентную обратную связь. Передаточная функция корректирующего звена в цепи тахогенератора может быть определена по формуле woc(P) WB3(p)Wc(p)t где Wc(p)= (i+'гурИН-Гдр) -передаточная" функция части системы, охватываемой обратной связью. В результате подстановки значений Wc(p) и Wn3(p) имеем W0. с (Р)= [(Ti + 2П - Т2 - TR - Ту) р + ' +(Гб+2Г,Гб-Т2ТЛ-Т2ТУ- TRTy)p2 + {TiT%-Г2ГдГу)р3] X X[k2k3(l + T2p)]-K Такое звено является,физически нереализуемым, так как степень многочлена в числителе выше степени многочлена знаменателя. Однако можно попытаться применить какое-либо физически реализуемое звено с передаточной функцией, близкой к желаемой. В качестве физически осуществимой передаточной функции можно взять, как первое приближение, функцию w°*yP> kzkz (1 + Т2Р) R°-c 1 + T2P> где ^oc=igg£- Эта передаточная функция может быть реализована при помощи тахогенератора постоянного тока, простого делителя и дифференцирующей /?С-цепи с постоянной времени Г2 = 0,12 сек. Потребный коэффициент передачи в цепи обратной связи 2 5 -5 °- с = k™ ' km = 0,12 ■ 9100*. 5,73 • 0,05 = 40'5 ' 10 * Специальный делитель может и не устанавливаться, но в этом случае место включения обратной связи в усилители должно быть выбрано таким образом, чтобы
25G ГЛ. 6. СИНТЕЗ ЛИНЕЙНЫХ СИСТЕМ 1282 от этого места до выхода усилителя коэффициент усиления по напряжению составлял k'2 = k2ko. с = 9100 • 40,5 ■ 10"' = 3,6. ' Таким образом, в первом приближении передаточная функция корректирующего звена в цепи обратной связи должна быть Го.сЫ = 40,5.10-5т^. Проверим теперь применимость этого звена для достижения нужных динамических качеств для формирования л. а. х. желаемого вида. Передаточная функция разомкнутой системы с учетом гибкой тахометрической обратной связи примет вид /Со (1 + Т<,р) W (п) = — — wCK\V) р(\+аф + а2р* + агр3) ' где а, = Ту + ТА + Т2 + k2k3k0. J2 = Ту + Тд + Т2 + Г, = 2,74 сек, а2 - ТуГд + ТуТ2 + TJ2 = 1,64 • 10"4 сек2, а3 = TyTJ2 = 2,4 • 10~4 сек3. Раскладывая знаменатель полученной передаточной функции на множители, получаем W (п) = — — - = W™W> p(l + Tp)(l+ap + bp*) = 512(1 + 0,12р) р (1 +2,74р) (1 + 0,6 • 1СГ2р + 0,88 • I0"V) ' В низкочастотной части эта передаточная- функция практически совпадает с передаточной функцией, соответствующей желаемой л. а. х. Ьж. Небольшое отличие имеется только в величине постоянной времени Г, = 2,74 сек, формирующей первый излом л., а. х. В высокочастотной области условие ограничения суммы постоянных времени выполнено, так как а=0,6-10" сек, а по условию допустимая сумма постоянных времени составляет 2 Т = 2,4 ■ Ю-2 сек. Проверка того, чтобы пик амплитудной характеристики колебательного звена не заходил в запретную
'3S31 S М- РАСЧЕТ ДОПОЛНИТЕЛЬНЫХ ОБРАТНЫХ- СВЯЗЕЙ 257 область для высокочастотной части л. а. х., т. е.. . . modi W(/(о) |<rj^j, подтверждает допустимость подобной аппроксимации. При желании можно получить более точное совпадение полученной передаточной функции с желаемой в низкочастотной области и устранить неравенство Тф'1\. Для этого необходимо уточнить значение коэффициента koc и выбрать его. равным ■ Аос = -р" *«* = -Ц- • 40,5 • 10"5 = 37,4 ■ 1 (Г\ Тогда аналогичным образом можно получить скорректированную передаточную функцию разомкнутой системы в виде 512(1+0,12р) W,AP)< р (1 + 2,5р) (1 + 0,65 • I0"2/; + 0,95 • 10~У) 283.- Определить вид и параметры обратной связи для электрогидравлической следящей системы, блок- схема которой приведена на рис. 167, а. На рис. 167, а обозначено: А и Б — части гидрорегулятора, ГУ — гидроусилитель, ПД — приводной двигатель, РМ — рабочий механизм, УД— управляющий двигатель, ЧЭ — чувствительный элемент (тахогенератор), Р — редуктор. Передаточная функция разомкнутой системы имеет вид W(p) К* р(1+7»(1+'ЛмР) где Ке — добротность по ускорению, Та = 0,05 сек — электромеханическая постоянная времени управляющего двигателя, Тгм~0,02 сек — гидромеханическая постоянная времени гидрорегулятора. Система должна иметь добротность по ускорению Ке^2Ъ сек'2 и показатель колебательности М ^1,8. Обратная связь охватывает управляющий двигатель и усилитель. Решение. На рис. 168 построена л. а. х. исходной системы L„ при значении Кв = 25 сек~\. Там же построена желаемая л. а. х.. .Ьж, отвечающая всем требованиям качества, предъявляемым к системе.
258 ГЛ. 6. СИНТЕЗ ЛИНЕЙНЫХ СИСТЕМ Рассмотрим порядок определения вида и параметров обратной связи. С учетом дополнительной обратной связи передаточную функцию разомкнутой системы Wm(p) можно записать в виде W(P) - WK(P): \ + Wx(p)'Woc(p) ' где Woc(p)— передаточная функция цепи дополнительной обратной связи, №х(р) — передаточная функция- ff, -и % чэ а) н--~ рм $2г- иг=кгд ^Лу Pi $ % ■)-&i- рм -0- б) > тгЛ Рис. 167. Схемы электрогидравлической следящей системы. части системы, замыкаемой обратной связью. Из приведенного выражения следует, что л. а. х. цепи обратной связи Loc можно определить по известным л. а. х. Lm и Ln в следующей последовательности [4], [28]:
§ 6.4. РАСЧЕТ ДОПОЛНИТЕЛЬНЫХ ОБРАТНЫХ СВЯЗЕЙ 259 1) из л. а. х. исходной системы Ьи вычитается желаемая л. а. х. Ьж, т. е. определяется л. а. х. Lu соответствующая передаточной функции 1 + Wx (p) Woc (/?); -^ ? ^ \ \ к\гж W 1 а Wj *■» - и " Jm NJ «fey ,-' - \s , 10 \'' K\ \ ■■~ff JL -L 1Д \ TM */,~ - \Аг S^/ !^ lYV f a>, V! £ s '" \ 4* \ r"' \ \ \ V v s 4щк ^ жГ ^- >..^ Л < \ \ !>\ f \\ 1 V, и \ \ \ \ \ \ \ T- *V Il 1 i / "icsx ШЮ "V -■% Рис. 168. JI. a. x. к задаче 283. 2) по виду л. а. х. Lx строится л. а. х. L2, соответствующая передаточной функции Wx(p)Woc{p); 3) из л. а. х. L2 вычитается л. а. х. той части системы, которая замыкается обратной связью Lx, в результате чего определяется л. а. х. цепи обратной связи Loc.
2§0 ;' ГЛ.. В.. СИНТЕЗ-ЛИНЕЙНЫХ СИСТЕМ , . [од ■В решаемой задаче разностная л, а. х. Ь.л.будет полностью располагаться под осью нуля децибел, что вызовет нарушение условия минимальности фазы при переходе к л. а. х. L2 [4]. Поэтому прежде следует увеличить коэффициент усиления исходной системы настолько, чтобы разностная л. а. х. L\ полностью лежала выше оси нуля децибел. На рис. 168 построена л. а. х. исходной системы с увеличенным коэффициентом усиления Д^~ а'0 =3600 сек~'г, которая обозначена Lp. Там же изображена разностная л. а. х. /„|, полученная вычислением желаемой л. а. х. 1ж из располагаемой л. а. х. Lp. Для определения л. а. х. L2 обратимся к таблице преобразования л. а. х. (приложение 25, п. VII). Поскольку обратная связь охватывает управляющий двигатель а усилитель, то Соответствующая этому выражению л. а. х. Lx построена на рис. 168. ^Вычитая из ординат л. а. х. L2 ординаты л. а. х LK, строим искомую л. а. х. Loc, по виду которой может быть записано выражение для передаточной функции цепи обратной связи где * 1 гр ___ ,1 ггл I Полученную передаточную функцию можно легко реализовать, включив в цепь обратной связи тахогенера- тор и пассивное звено указанного типа (рис. 167, б). 284. Определить вид обратной связи для системы, рассмотренной в задаче 283, в предположении, что цепь обратной связи охватывает часть усилителя, т. е. Wx (p) = kK. Остальные данные те же,, что в предыдущей задаче. Ответ. Передаточная функция цепи обратной связи имеет вид ' XV/ (,Л — feocO +Т3р) оЛ'Р)~(1 + ТгР)(1 + Тмр)'
r$S5l § 6-*- РАСЧЕТ ДОПОЛНИТЕЛЬНЫХ. ОБРАТНЫХ СВЯЗЕЙ 261 .-.- 285. Произвести выбор параметров прямой параллельной корректирующей связи для системы автоматического регулирования, структурная схема которой —i-®- ff, U а) г* ff г *, р —G^— \ Wfp) Ъ Рис. 169. а) Структурная схема системы с введением прямой параллельной корректирующей связи; б) л. а. х. к задаче 285. ' показана на рис. 169, с. Передаточная функция разомкнутой исходной системы имеет вид Г0>) = р(1 + 7>р)(1 + 7др)'
262 ГЛ. 6. СИНТЕЗ ЛИНЕЙНЫХ СИСТЕМ ГЕ83 где Ка =500 сект1, Гд = 0,08 сек, Гу = 0,02 сек. После введения прямой параллельной связи система должна обладать астатизмом второго порядка, иметь добротность по ускорению Кв = 100 сек~2 и показатель колебательности А1^1,5. Решение. Передаточную функцию разомкнутой системы с учетом введения прямой параллельной связи представим в виде 157 ( о) = i— -пс—•'- где Ке = Kaknc. Реализация прямой связи, осуществляющей введение сигнала, пропорционального интегралу от ошибки (рассогласования), возможна путем использования интегрирующего привода. На рис. 169, б изображены л. а. х. исходной системы L„, желаемая л. а. х. Ьж и л. а. х. прямой связи Lnc. Требуемое значение коэффициента передачи по цепи прямой связи km определяется из условия Ke = *l = KQkBC или . Ке 100 „,, . fe- = ^==900- = 0'11 СвК К Отношение -~ целесообразно выбрать равным — . Отсюда ft, «Jhl» ML «о,018. <о2 ь Для выполнения этого условия придется намеренно уменьшать коэффициент передачи первого звена, охватываемого прямой параллельной связью. Одновременно во столько же раз увеличивается коэффициент передачи другого звена, входящего в прямой тракт усиления, чтобы сохранить постоянной величину Ка- Введением интегрирующей связи удается приблизить л. а. х. исходной системы L„ к желаемому виду Ьж только в области низких и частично средних частот (Ьж).
«861 § 6.5. СИСТЕМЫ КОМБИНИРОВАННОГО УПРАВЛЕНИЯ 263 . Окончательное приближение л', а. х. системы к желаемому виду может быть достигнуто путем коррекции л. а. х. системы в области средних и высших частот применением последовательных корректирующих звеньев либо с помощью эквивалентных им прямых или обратных связей. § 6.5. Расчет систем комбинированного управления 286. Определить требуемый уровень компенсирующего сигнала по первой производной от входного воздействия, при котором устраняется скоростная ошибка системы ?(Р) •% У х Ц(Р) -©* Ъ(р) !/ Рис. 170. Структурная. схема системы комбинированного управления. (рис. 170), звенья точные функции: которой имеют следующие переда- Ф (Р) ='т,р, WM- k. Р(1 + ТР)' где ki = 10 в/град, &2 = Ю град/сек, Т = 0,02 сек, т [сек] — коэффициент, определяющий уровень компенсирующего сигнала. Решение. Передаточная функция замкнутой системы относительно ошибки имеет вид .1 - W* (Р) ф (р) 1 + W, (р) Wa (р)' откуда ФАР) (1) <М/>) = Трг + р — к2тр V + P + Мг' (2)
264' 'Fiflv 6. СИНТЕЗ ЛИНЕЙНЫХ. СИСТЕМ ■ • [287 :■ Условие устранения скоростной ошибки k2X — 1. Следовательно, требуемый уровень компенсирующего сигнала равен ■ 1 р, , в сек fi2 град 287. Определить требуемые уровни компенсирующих сигналов по первой и второй производным от входного воздействия для. следящей системы комбинированного управления (см. рис. 170) с передаточными функциями: №,(/?)= 1, W*M = р (1+7» (1+7»' где 7/, = 0,05 сек, Т2,= 0,002 сек. Система должна обеспечивать слежение с ошибкой л;тах^0°,1 при максимальной скорости слежения Qmax = 150° сек-1 и максимальном ускорении. е1пах = 750 град/сек2. Показатель колебательности М -^ 1,5. Решение. На рис. 171 построена контрольная точка с координатами ' в г -1 юк = -q — 5 сек 1 и • ■■■■ Q2 1502 ^ = 20,В1й^£гвя20|ВаГ7вб-'50^ ...Если провести через эту контрольную точку низкочастотную асимптоту л. а. х., соответствующей передаточной функции исходной системы, то требуемое значение добротности по скорости составит *шах и>• Однако известно, что при заданном значении показателя колебательности М наименьшее допустимое значение добротности по' скорости при отсутствии каких-либо
гт- 6.5. СИСТЕМЫ КОМБИНИРОВАННОГО УПРАВЛЕНИЯ 265 корректирующих средств составит {4] М2+М VW^X 1,52 + 1,5 У'йР^Т /(« = ■ 2 (Г, + Т3) 2 (0,05 + 0,02) = 40 сек~1. Если ввести сигнал первой производной от управляющего воздействия, следящая система приобретает свойства системы с астатизмом второго порядка. /й-гщЬ Рис. 171. Л. а. х. к задаче 287. Требуемое значение добротности по ускорению равно егаах '50 ке -*"тах 0,1 == 7500 сек~2. Требуемое значение добротности по скорости при этом для исходной системы определяется по формуле Ка = (Г, + Т2) Ке = 0,052 • 7500 = 390 сек'1, т. е. Дй получается уже значительно меньше, чем Ка. При введении дополнительно второй производной требуемая добротность по третьей производной Ятах 0,1 = 37500 сек~я.
266 ГЛ. 6. СИНТЕЗ ЛИНЕЙНЫХ СИСТЕМ [288 Требуемая добротность по скорости может быть опрет делена по выражению Приравняв /Си" добротности по скорости, которую можно иметь без корректирующих средств (/Сп = 40 сек'1), получим требуемое значение постоянной времени т3: KQ - 7-,7-2Xv 40 - 0,05 • 0,002 • 37500 3 Тз (Г, + Т2) Ку ~ (0,05 + 0,002) • 37500 ~ lb,b ' U CeK" Постоянные времени, определяющие уровни вводимых сигналов, находятся из условий компенсации т = -&- = -77Г = 0,025 сек, Afl 40 т2 = Tj + Т2 + т3 = 0,05 + 0,002 + 0,018 = 0,070 сек. Таким образом, передаточная функция компенсирующей цепи должна иметь вид ( \ плос . °>025 • 0,070р2 Ф(Р) = 0-025Р+ 1+0.0018Р • Л. а. х. системы, соответствующая найденным параметрам, показана на рис. 171 (нижняя л. а. х). 288. Определить требуемый уровень компенсирующего сигнала, пропорционального первой производной от входного воздействия, ф(Р) = Т1/>. и произвести расчет других необходимых корректирующих средств для следящей системы, передаточная функция которой в разомкнутом состоянии имеет вид (см. рис. 170) где Тд = 0,\ сек — электромеханическая постоянная времени двигателя, Гу = 0,05 сек — постоянная времени усилителя. Система должна обладать астатизмом вто-
i.5. СИСТЕМЫ КОМБИНИРОВАННОГО УПРАВЛЕНИЯ 267 обеспечивать слежение с ошибкой ъ, ^^ угл. мин при максимальной скорости слежения „ого порядка и <2 „ = 30 град/сек Q и максимальном ускорении етах = = 30 град/сек2. Запас устойчивости определяется показателем колебательности М<1,5. Решение. Определим вначале желаемую эквивалентную передаточную функцию разомкнутой системы. Рис. 172. Л. а. х. к задаче 288. Первая асимптота л. а. х. представляет собой прямую с наклоном 40 дб/дек. Положение ее определяется базовой частотой (рис. 172) ю0 = ^ " /fe = V^T2 = 30 сек' Для получения запаса устойчивости, соответствующего показателю колебательности М,. передаточная Функция для области средних частот должна иметь вид [4] w3Ap)- иб(1 + Г,р) р2(1+Г2р)
268 "■,д '": ГЛ. В. СИНТЕЗ'ЛИНЕЙНЫХ СИСТЕМ ' $й88 где «О *= ^ = 900 Се,С2' Тг = — V тг-1 = 4г l/r^-f = 0,0575 се«. ^ = ^х| Г, = Н£х4 0,0575 = 0,0115 сек. ©о А1-1 _ 1,5-1 , М+1 1~ 1,5+1 Желаемая передаточная функция замкнутой системы равна л („У ^эж(р) - КвП + ^р) При введении компенсирующего сигнала передаточную функцию замкнутой системы можно представить в виде Сравнение приведенных выражений дает или • г, = Т, = 0,0575 сек, что определяет требуемый уровень компенсирующего сигнала. Далее, имеем Кг , ■ KeTlP Фж(Р)> KE + KJlP + p2+Tsp^ ■ Ke + KeTlP + p* + T2P = Ф,(/?) + Ф2(р). Желаемая передаточная функция исходной следящей системы равна J_ W*KP) 1-ф.(р) /,, 1 , т* л j_ Г. _ , 17,4 ' рЦ+ар + Ьр2) р(1+0,0193р + 0,00022р2) ' Передаточная функция нескорректированной системы имеет вид w(n\= Kq ■ •' = — WW . р (1 + ТлР) (1 + тур) -р[1 + (Гд + 7-у) Р + УдГур2] *
1.5. СИСТЕМЫ КОМБИНИРОВАННОГО . УПРАВЛЕНИЯ ..2g9 Сравнение двух последних выражений показывает, ч,то для получения равенства W& {p) =*= W (р) необходимо выполнение следующих условий: Ка = 17,4 сек-11, Тя + Ту = 0,0193 сек, TJy = 0,00022 сек2. Выполнение первого условия не представляет трудностей, так как добротность по скорости Ка, ■представляющая собой общий коэффициент передачи разомкнутой системы, может быть принята любой. Выполнение второго, и третьего условий требует введения корректирующих звеньев, снижающих коэффициенты при р и р2 в квадратной скобке выражения для W(p), так как без корректирующих звеньев Тл + Ту = 0,15 сек TJY = 0,005 сек2. Это может быть сделано введением жестких обратных связей, охватывающих усилитель и усилитель совместно tfp Рис. 173, Структурная схема системы к задаче 288. с двигателем (рис. 173). В этом случае передаточная функция разомкнутой цепи вместе с обратными связями будет равна *«</» = ■ *° T + l l+fc. + v )р l + kl+'ki ■]■
270 ГЛ. 6. СИНТЕЗ ЛИНЕЙНЫХ СИСТЕМ Сравнивая последнее выражение с выражением для Wx(p), получаем Тл + Ту + ktTR TJy = а = 0,0193 сек, • = 0,00022 сек2, l + kt + k2 откуда находим требуемые коэффициенты усиления по первой и второй петлям обратных связей (см. рис. 173) /г1==1,9 /г2 = 22,5. § 6.6. Расчет последовательных корректирующих контуров, работающих на несущей частоте 289. Выбрать схему и параметры звена переменного тока, л. а. х. которого по огибающей соответствует С ■R, Ъ\ Рис. 174. а) Л. а. х. реального дифференцирующего звена; б) схема резонансного RLC-звена. дифференцирующему звену (рис. 174, а) с передаточной функцией W^P)-Tl(i+T2py где 7^ = 0,08 сек, Г2 = 0,01 сек. Несущая частота со„ равна 3140 сек'1. Решение. Передаточную функцию реального дифференцирующего звена переменного тока по отношению к частоте огибающей Q можно для достаточно узкой полосы частот Q записать в виде WQQ)=>G0 1 + /ЙГд 1 + jQr„G0 °° 1+jQx
2891 § б-6" ПОСЛЕДОВАТЕЛЬНЫЕ КОРРЕКТИРУЮЩИЕ КОНТУРЫ 271 По условию задачи 00==-^-= ojb1T = 0Л25 и Та~ = Г, = 0,08 сек. В настоящее время получили распространение следующие виды дифференцирующих звеньев: а) двойные или параллельные ТЧэбразные RC-звенья; б) мостовые 7-образные #С-звенья; в) мостовые RC- и LC-звенья; г) резонансные RLC-звенъя. Рассмотрим возможность применения резонансного RLC-звена (рис. 174, б). Передаточная функция такого звена по огибающей имеет вид Hl*R* 1+/Й2Я.С ^ Ri + Rz Для нашего случая 2/?jC = Тл, 2RtC ft +ft — TAG0, R* G0 и co„ = Rl + Ri ° H VTL азом, получаем четыре \ :стными: 0,08 = 2RlC, 0,01V= 2/?,C Таким образом, получаем четыре уравнения с четырьмя неизвестными: v R2 0,125 = —^—, 3140= ' Ri + R2 ' Ri + R*' Vic' Величина сопротивления R2 обычно бывает задана (равна входному сопротивлению последующего устройства). Пусть R2 равняется 100 ком. Определим Rt: 0,1257?,+ 0,125- 100=100, #1 = 0^5 = 700 ком- Определим теперь емкость конденсатора С: п 0,08 0,08 пг.с_ , C = W=2T0J=°>057^- В заключение найдем индуктивность L: L = —5— =• 5 = 1,8 гн. of С 3140 - 0,057
272 ГЛ. 6. СИНТЕЗ ЛИНЕЙНЫХ СИСТЕМ R,. 0—1—II—1 fir 0- Ъ Сз Л\ 1 290. Определить параметры двойного Г-образного звена (рис. 175), работающего на несущей частоте сои — 2nfn — 3\4 сек~К Остальные условия те же, что в предыдущей задаче. Решение. Для определения параметров звена обратимся к таблице, приведенной в приложении 22. ■ По условию задачи произведение Таш„ = 25. Путем интерполирования данных можно найти Go, соответствующее полученному произведению ГД<0И. Коэффициент G0, определенный таким образом, оказывается равным 0,02. Таким образом, полученное значение G0 по сравнению с его заданным зна- _0 чением можно уменьшить; Это в свою очередь при сохранении Г).= ГД=0,08 сек может привести к уменьшению постоянной времени Т2 до значения = 0,08-0,02 = 0,0016 сек-. Рис. 175. Схема двойного Г-образного /?С-звена. Обычно уменьшение постоянной времени Т2 не свя- 8ано с ухудшением динамических свойств корректируемой системы. Перейдем к определению параметров двойного Г-образного звена. Пусть Ci = C2 = Ca = C = 0,5 мкф. Тогда (см. приложение 22) n a 0,394-10е пкпг. /?, = -7Г7Г = ,lyl.ng » 2o00 ом, \*г /?2 /?Я со„С 1 314-0,5 106 2со,,Са 1 2-314-0,5-0,394 10° V2 ч>„С 1,41.314-0,5 8000 ОМ, 4500 ом. ~?91. Определить параметры двойного Г-образнОго звена. Постоянная времени 7'д = 0,0047 сек. Несущая частота ©„ = 23xfH = 3140 сек~\ Ct = С2 = С = 1 л/с0. . Ответ. G0 = 0,034, /?,.= 134 ши, 7?2=380 <ш, /?а=225ол«.
ГЛАВА 7 СЛУЧАЙНЫЕ ПРОЦЕССЫ В ЛИНЕЙНЫХ СИСТЕМАХ § 7.1. Вычисление корреляционных функций и спектральных плотностей 292. Определить корреляционную функцию /?(т) и спектральную плотность S(<o) для величины, изменяющейся по гармоническому закону х*=А sin (f>t + -ф). Проверить, что интегрирование спектральной плотности по всем частотам, а также значение /?(0) дают средний квадрат (в данном случае ои равен дисперсии) рассматриваемой величины. Амплитуда' Л =10 и угловая частота р = 2 сек'1. Решение. Корреляционная функция + т Я(т)=Пт 4r f x(i)x(t + x)dt = -т ъ = ~ J Л2 sin (р/ + -ф) sin (р/ + Рт + г]з) dt = ~- cos рт, о где r0 = ~g-. Подстановка исходных данных дает #(t)~=50cos2t, а также /?(0) = 50.
274 ГЛ. 7. СЛУЧАЙНЫЕ ПРОЦЕССЫ В ЛИНЕЙНЫХ СИСТЕМАХ [283 Спектральная плотность может быть вычислена на основе интеграла Фурье [2]: со оо S((o)= J Я (т) *г''вт = Г -у- е-'шт cos рт dx = —оо _ —оо оэ л2 Г = -g- cos ют cos рт dx = —оо оо л2 Г = -^- [cos (ю — р) т + cos (ю + р) т] dx = —оо =-^f-[6(ft,-p)+6(©+p)], где б (ю — р) и 6 (ю + Р) — единичные импульсные функции, расположенные при частотах со = р и ю = — р. Интегрирование спектральной плотности по всем частотам дает + оо 1 2я j S(e»)de> = -£- J [6(co-p) + 6(© + p)]dco. Интегралы от единичных импульсных функций равны единице, + оо [ 6(ft>-p)rfft)= J" 6(ft> + p)dco=l. —оо —оо Поэтому в результате получаем +.оо J S(©)dft, = ^ = -^- = 50. + оо 1 2я 293. Для стационарного случайного процесса, имеющего постоянный спектр в полосе от — юп до + юп (рис. 176), вычислить среднее значение (математическое ожидание), средний квадрат (момент второго порядка) и дисперсию, а также найти аналитическое выражение й построить график корреляционной функции.
288) § 7.1. ВЫЧИСЛЕНИЕ КОРРЕЛЯЦИОННЫХ ФУНКЦИЙ 275 -' Решение. Среднее значение случайной величины равно нулю х — 6, так как спектральная плотность не содержит при (0 = 6 особенностей типа импульсной функции (дельта-функции). В результате дисперсия равна среднему квадрату случайной величины D = х2 — х2 = х2 ■■ о\ где а —среднеквадратичное отклонение. Далее находим +4, ■■Л-5Г JsMrf»-i J "*»- JVAco 2я где Аю = 2а>п — полоса частот (в радианах в секунду). Последняя, формула может быть записана также в следующем виде: S(tt» x2 = D = NAf, ~0>п О - йш ■ т N -w'k .г Дш где А/ = -Kz— полоса частот (в герцах). Среднеквадратичное значение случайной величины *ск = о = V~N Vbf. . Корреляционная функция может быть определена на основе интеграла Фурье [2]: -&-OCJ СО Я(т) = -^ j 5,(ю) е^ dft) =-^ J S(c»)cos<i>Td(» Рис. 176. Белый спектр в ограниченной полосе частот. ИЛИ *>„ iw-i Г N , N cos сот dco = — sin юпт. я J зтт о График корреляционной функции изображен на рис. 177. Значение корреляционной функции при т = 0 равно R (0) = lim — sin ю„т = -^г" = D.
2 76 ГЛ. 7i: -СЛУЧА-ЯНЬШ- ПРОЦЕССЫ В- ЛИНЕЙНЫХ СИСТЕМАХ [29S„ 294. Для предыдущей -задачи определить-нормированное значение спектральной плотности и корреляционной функции. Rtt) х,сен Рис. 177. Корреляционная функция к задаче 293. -Цв-0,4-Ц2 О 0,2 0,4 Ofiz.ceK Рис. 178. Корреляциониаяфунк- ция- экспоненциального вида. Ответ. Нормированная спектральная плотность при соп <; ю ^ G>n равна Нормированная корреляционная функция R (т) sin шпт р(т) = D (0ПТ Значение р (т) при т = О р(0) lim ——— = 1. т->П «"пТ 295. В результате обработки осциллограммы стационарного случайного процесса с математическим ожиданием (средним значением), равным нулю,, получено выражение для корреляционной функции #(т) = £е-^Ч где D ■= 100 — дисперсия и у, = 5 сек'1 — параметр затухания. Корреляционная функция построена на рис. 178. Определить спектральную плотность и построить ее график. Решение. Спектральная плотность может быть найдена согласно интегралу Фурье [2] 5 (со) = J R (т) e-i»x их = | Der*'х Уах dx.
S96J " § 7-'- ВЫЧИСЛЕНИЕ КОРРЕЛЯЦИОННЫХ ФУНКЦИЙ 277 Последний интеграл для удобства желательно разбить на два: 5(») где [о °° П 2ц£> 2Г£> |i2 + ш2 I + согГ* » Г = -- = 0,2 сек. Подстановка численных значений дает о/ \ 40 5(й)= 1+0,04а/• Спектральная плотность построена на рис. 179. 4 6"-тт Рис. 179. Спектральная плотность, соответствующая корреляционной функции на рис. 178. 296. Решить предыдущую задачу, если рассматриваемый стационарный случайный процесс имеет среднее значение (математическое ожидание) х = 5. Построить графики корреляционной функции и спектральной плотности. Ответ. Средний квадрат случайной величины ^= D + х2 = 100 + 52 = 125. Корреляционная функция R (т) - De~» i ч! + г2 = 100<г5' * i + 25.
278 ГЛ. Т. СЛУЧАЙНЫЕ ПРОЦЕССЫ В ЛИНЕЙНЫХ СИСТЕМАХ (297 Спектральная плотность ■ S(tt)-2««(a>) + T^5.-l67u(o>)+7+^45?, где б (со) — единичная импульсная функция. Графики построены на рис. 180. -Ofi-Dfi-QZ О 0£ Dfi 0,61.сеи S(Q»\ « 0а.ТШ Рис. 180. Корреляционная функция и спектральная плотность к задаче 296. 297. В результате обработки осцилограммы -случайного стационарного процесса с математическим ожиданием, равным нулю, получено выражение для корреляционной функции: R(x) = Der»[x \cosfr, (1) где D — 40 — дисперсия, ц = 0,5 сек-1 — параметр затухания (коэффициент нерегулярности) и р = 2 сек'1 — резонансная частота. Корреляционная функция изображена на рис. 181. Найти аналитическое выражение и построить график спектральной плотности. Ответ. Спектральная плотность ^m)a^[i.H(i-B)' + (Р-ю)2 ' ц2 + (р + а)2 ]•
298) § 7.1. 'ВЫЧИСЛЕНИЕ КОРРЕЛЯЦИОННЫХ ФУНКЦИЙ 279 После подстановки численных значений 20 . . 20 S(©) ■ + 0.25 + (2 - со)2 "•" 0,25 + (2 + со)2 ' График спектральной плотности изображен на рис. 182. Rm Рис 181. Корреляционная функция нерегулярной качки. S«o) Рис. 182. Спектральная плотность нерегулярной качки. 298. Для аппроксимации формулы корреляционной функции по исходным данным предыдущей задачи принято более точное выражение #.(т) = De~»'х i (cos рт +1- sin 0 | т |). Найти спектральную плотность для этого случая. Ответ. 5.(°>~р>Ь 2Р-С0 + 2Р + со -]- + (Р~со)2 ^ ц2 + (Р + со)2_ LO,25 + (2-co)2 + 0,25 + (2 + to)2 J *
280 "Ч'Л. 7. СЛУЧАЙНЫЕ ПРОЦЕССЫ В ЛИНЕЙНЫХ СИСТЕМАХ (299 299. Стационарный случайный процесс на входе следящей системы имеет вид, представленный на рис. 183. Среднеквадратичное значение рассматриваемой величины хск = 2. Средняя протяженность участка х = const составляет Т = 10 сек. Определить корреляционную функцию и спектральную плотность. xtt) U t.CBII Рис. 183. Типовой входной сигнал следящей системы. Решение. Корреляционная функция может быть найдена по выражению _ R (т) = Fp, + х2Ръ (1) где я2 —средний квадрат, а х2 — квадрат среднего значения случайной величины, Рi — вероятность нахождения перемножаемых ординат случайного процесса на интервале х = const, т. е. вероятность отсутствия изменения скорости на отрезке времени т; Р2—\ — Р\ — вероятность наличия изменения скорости на отрезке времени т. Так как для рассматриваемого процесса х = 0, то x2='D и формула (I) приобретает вид ** '■/?(т) = 0/>-,. (2) Вероятность появления изменения рассматриваемой случайной величины на малом интервале времени Лт может быть принята пропорциональной значению Лт и „ Лт _ ■ .. равной -у-. Вероятность отсутствия изменения случайной величины будет 1 =-. Вероятность отсутствия изменения величины на отрезке времени т равна произведению вероятностей p^l-A^V (3)
SMJ'" ' S''7J; ■6Ь1ЧИСЛЕНИЕгКОРРЕЛЯШТОН^ШХ-Ф'УИК1Н1И : 281 Искомая вероятность Pt может быть найдена как предел выражения (3) при Ат->0: Т _ X Pi = Iim 1 т" = е Так как Р\ {%) = Р\{ — т), то в результате получаем корреляционную функцию в виде R{r) = De r = 4е~ Спектральная плотность [2] -0.1 1т| S(co) R (т) e-'ax-dx 2 ГО 80 I + со2Г2 1 + 100ш2 " 300. Решить предыдущую задачу, если известно, что участки х>0 н х<0 чередуются и изменение О 1,сек lUL д 1 t,ce/e Рйе. 184. График процесса к за- Рис. 185. Последователь- даче 300. иосгь импульсов. значения всегда сопровождается изменением знака. График, такого процесса изображен на рис. 184. Ответ. I х I -2 J-^- -ЧХ2 Itl R(x) = De г =Ае , ,,, ч TV 40 &(©) = ■ , ?'2 , 1+ -г- СО2 I + 25<о2 • 301. Определить спектральную плотность последовательности равноотстоящих положительных импульсов, имеющих одинаковую- ширину и случайную амплитуду (рис. 185), при следующих исходных данных: период следования импульсов Т = 0,1 сек; ширина импульса
282 ГЛ. 7. СЛУЧАЙНЫЕ ПРОЦЕССЫ В ЛИНЕЙНЫХ СИСТЕМАХ [3С1 "уГ = 0,01 сек, что соответствует скважности у = 0,1; среднее значение амплитуды импульса х = 20j_ среднеквадратичное значение амплитуды импульса Yx2 = л;ск= = 25. Решение. Представим функцию х (t) в виде суммы периодической составляющей X\(t), состоящей из последовательности импульсов с постоянной амплитудой, равной х (рис. 186, а), и случайной составляющей Хг(0, состоящей из последовательно- сти импульсов со случайной амплитудой и средним значением, равным нулю (рис. 186,6). Периодическую составляющую разложим в ряд Фурье, *i(0= S Cke'~¥~, (1) xt LXUJ t,ceH a) 3% О U JL t.cen 6) где Си — комплексное число. Амплитуды гармоник Рис. 186. Составляющие последовательности импульсов. ku sin kny (2) что при подстановке исходных данных приводит к равенству Ак = А-к= Мsin0,314ft . Это дает следующие значения амплитуд гармоник: Л0 = 2, Л6=1, Л12 = 0,31, Л, = 1,98, Л7 = 0,73, Л13 = 0,39, Л2=1,86, Л8 = 0,46, Л14 = 0,43, Л, = 1,7, Л9 = 0,21, Л15 = 0,42, Л4=1,51, Л10 = 0, Л16 = 0,38, Л =1,27, Л„ = 0,17, и т. д. Спектральная плотность для периодической соста- ляющей (1) может быть записана в виде (см. задачу 292) SM = 2**t S *(°—^ (3) ft=—оо 2nft \ Т )
„ § 7Л. ВЫЧИСЛЕНИЕ КОРРЕЛЯЦИОННЫХ ФУНКЦИЙ 283 - представляет собой линейчатый спектр. Он изображен на рис. 187, а, причем площадь импульсной фуик- А. ции, равная 2п-~-, показана условно в виде амплитуды конечного по высоте импульса. а) ■14,,.,II V-sooA-uoo-* Sz(w) Z-W7' mill -500- A-sao-A a>,Tk сек Ф Рис. 187. Составляющие спектральной плотности к задаче 301. Значение амплитуд гармоник (2) может быть также найдено на основе преобразования Фурье от одиночного импульса высотой х и продолжительностью уТ. Изображение Фурье для такого импульса равно xe-№dt = x- -/covr /ю Модуль этого выражения \Ft (/со) | = 2х sin щТ (4) Амплитуда /г-й гармоники может быть получена 2nk из формулы (4) для частоты сой подстановкой со = —~- и
284-' гл1. г случайные - процессы в линейных-системах ^и делением полученного значения на период следования Т: А„ (. "Ink \ I ('-HI X sin fety Это выражение совпадает с (2). Спектральная плотность случайной составляющей может быть найдена из общего выражения для спектральной плотности случайной величины S(<D) = 'lim -±-\F(ju>)?, r0->w"o которое в рассматриваемом случае превращается в выражение 52((о) = ^|,Р2(/(о)|2, где F2 (/ю) представляет собой изображение Фурье одиночного импульса, среднеквадратичное значение которого равно а=\х2 — х2. Аналогично формуле (4) можно записать соуГ \F2{ja)\ = 2о sin - со (5) • •■ Отсюда- находим спектральную плотность случайной составляющей . „ . , ауТ 52(со) = г^И-. (6) ■Подстановка численных значений дает с , . 9000 siп2 0,005(0 . ч ' со2 Спектр является непрерывным. Он изображен . на рис. 187, б. По своей форме он подобен огибающей линейчатого спектра, так как значения спектральной плотности также пропорциональны квадрату модуля изображения одиночного импульса (4). '■' 302. Спектральная плотность скорости входного сигнала следящей системы (рис 183) может быть
7.1. ВЫЧИСЛЕНИЕ КОРРЕЛЯЦИОННЫХ .ФУНКЦИЙ = ,gg5 представлена в виде ^ Si(«>)= l + JT2 ■ (1) где Dq = иск — средний квадрат скорости. Момент нагрузки на исполнительной оси постоянен по величине Ш = Мн — const), а знак его меняется вместе с изменением знака скорости исполнительной оси. Считая приближенно, что знак момента меняется вместе со знаком входной скорости, определить корреляционную функцию для момента нагрузки 52((о), а также взаимные корреляционные функции для входной скорости и момента нагрузки Sl2(u>) и 52|(со). Считается, что входная скорость изменяется по закону с нормальным распределением. Решение. Спектральная плотность момента нагрузки может быть получена из спектральной плотности скорости входного сигнала (1), если в ней заменить средний квадрат скорости на средний квадрат момента М2 = Ми, ■S2(a) = j^^. Взаимную спектральную плотность можно подсчитать по взаимной корреляционной функции, определяемой как среднее по времени или среднее по множеству +т Rl2{%) = lim 4f \ Q(t + t)M(t)dt~Q(t + x)M(t). Вероятность нахождения il(t + x) w M(t) в одном интервале (см. задачу 299) равна 111 а вероятность нахождения в разных интервалах 1т[ ра=,1-р = 1-е ■''. При нахождении скорости и момента в разных интервалах среднее от их произведения равно нулю. . ,
286 ГЛ. 7. СЛУЧАЙНЫЕ ПРОЦЕССЫ В ЛИНЕЙНЫХ СИСТЕМАХ [303 При нахождении Q и М в одном интервале знак момента равен знаку скорости. Произведение скорости на момент будет при этом всегда положительным. При этом, поскольку величина момента постоянна, момент можно вынести из-под знака осреднения: Q{t + x)M (t) = MHQ (t + т) = MHQC1 где Qc — среднее по модулю значение скорости. Для нормального распределения Sc = Qck]/-|=0,8Qc Таким образом, имеем взаимную корреляционную функцию i -г 1 m Rl2{x)^MHQcPl=MAe T =0,8MHOCKe r, (2) Спектральная плотность находится, как изображение Фурье, из выражения (2): с . , 2гмнас _ 1,бгмноск /« Аналогичным образом можно найти, что R2i (т) = R^it) и S21(co) = 512(o)). § 7.2. Прохождение случайного стационарного сигнала через линейную систему •яг 303. Система слежения за звездами состоит из фотоэлемента, безынерционного усилителя, фяльтра (апериодического -звена первого порядка) и исполнительного устройства в виде гироскопа или тахометрического привода (идеального интегрирующего звена). Помеха на выходе фотоэлемента может быть принята в виде белого шума со спектральной плотностью S((.o) — N. Показать, что среднеквадратичное значение случайной ошибки системы не зависит от постоянной времени фильтра. Решение. Передаточная функция разомкнутой системы имеет вид > W(p)= ,* . , ^' р(1 +Тр) '
в 72 ПРОХОЖДЕНИЕ СЛУЧАЙНОГО СИГНАЛА 287 S04] я ■■ ■ К [сек'1] — добротность по скорости, Т — постоянная времени фильтра. • Передаточная функция замкнутой системы равна ■ФЫ—Г(р) >- * &КР)- 1 + Г(р) тр*+Р+кш Спектральная плотность ошибки примет вид Se(со) = I Ф(/«) Р5(со) = lT(.JlNja + Kli • Интегрирование спектральной плотности ошибки по всем частотам (см. приложение 17) дает средний квадрат v +00 ошибки fi2__L Г K*Ndu> KN ,fA. где эквивалентная полоса пропускания белого шума Как видно из полученных выражений, среднеквадратичная ошибка не зивисит от постоянной времени фильтра. 304. Для системы слежения за звездами (см. предыдущую задачу) среднеквадратичное напряжение шума фотоэлемента i7CK = 6 в в полосе частот А/=10000 гц- (± 5000 гц). Крутизна характеристики фотоэлемента £фЭ=10 мв/угл. мин. Определить допустимое значение общего коэффициента усиления (добротности по скорости К), при котором среднеквадратичное значение случайной ошибки не будет превышать 1 угл. мин. Решение. Представим напряжение шумов фотоэлемента в виде эквивалентного среднеквадратичного Углового сигнала на входе, 6СК = —— = =- = 600 угл. мин. А:фэ Ю ■ ю-3 • Уровень белого шума на входе S„ (со) = N = -~ = -jQQQJj- = 36 {угл • мин)2/гц.
2S8 ГЛ. 7. СЛУЧАЙНЫЕ ПРОЦЕССЫ В ЛИНЕЙНЫХ СИСТЕМАХ ' [зес В задаче 303 было определено, что среднеквадратичное значение ошибки составляет 0 = У-7 KN_ 2 Отсюда находим значение общего коэффициента усиления К < -^ = Ц£- - 0,055 сект1. 305. Даны передаточные функции разомкнутой' системы регулирования с астатизмом первого порядка: 1) U? (/>) = -£-, 2) Г(р)= К 3) W{p) = К р(1+Г1Р)(1 + Г2р) Вычислить эквивалентную полосу пропускания белого шума замкнутой системы, если добротность по скорости составляет К = 10 сект1, а постоянные времени Г| = 0,1 сек и Г2 = 0,05 сек. Ответ. г) zv = 4=-r = 5 ^ 2) Af-^—T- = 5 гц, 3) А^ = 'о/. ^УТТ=пЛ ю-о,1-о,о5Т^У'5 г^- Г Г. + rJ Л 0,1+0,05 J 306. Даны передаточные функции разомкнутой системы регулирования с астатизмом второго порядка: 1) v(p)=^i^, ^ ww р2 (1 + Гр)" Вычислить эквивалентную полосу пропускания белого шума замкнутой системы, если добротность по ускоре-
308, § 7-2. ПРОХОЖДЕНИЕ СЛУЧАЙНОГО СИГНАЛА 289 нию К = Ю сек"2, а постоянные времени т = 1 сек и Т = 0,5 сек. Ответ. 1) bf = —!Г-- 2-1 - = 5,5 гц, ., 1+/Ст2 1 + 10-Р ,, 2) ^=2(т-г)=ТТГ^адГ=11 гг<- 307. На входе системы регулирования действует помеха, имеющая спектральную плотность с / \ 2Г"Р" 4 1+со27^ Определить коэффициент сглаживания системы, равный отношению среднеквадратичного значения помехи на входе к среднеквадратичному значению ошибки ' к — р" А-сгл ff > и среднеквадратичное значение ошибки о. Передаточная функция разомкнутой системы . W{p) = f. Численные значения коэффициентов: /С = 0,5 сек-1, ап=10, Гп = 0,1 сек. - Ответ. /Ссгл = /T+If = /l+^V - V2T - 4,6; V0 = ^n_= 10'=2,18. , /Ссгл 4,6 308. Решить предыдущую задачу, если передаточная ■.функция разомкнутой системы где Г, = 1 сек.
290 ГЛ. 7. СЛУЧАЙНЫЕ ПРОЦЕССЫ В ЛИНЕЙНЫХ СИСТЕМАХ [30S Ответ. 0,5 • 0,1 К 1 / Тп . 1 ., 1 / 0.1 ... . Асгл- у Г, + Гп КТп У 1+W -1^20,1 =4,5, On _ Ю _ о 99 /Ссгл ~ 4,5 £***' 309. На' входе следящей системы действует полезный сигнал, скорость которого изменяется в соответствии с рис. 183. Спектральная плотность, записанная для скорости, имеет вид 2TDa Sq (и) = J + ш27-2 « где Da — Qck — дисперсия скорости. Среднеквадратичное значение скорости QCK = 2 град/сек. Среднее время продолжительности одного интервала Т — 1 сек. Определить среднеквадратичную ошибку, если передаточная функция разомкнутой системы имеет вид Добротность по скорости составляет К — 25 сек~1, а постоянная времени Т = 0,05 сек. Решение. Передаточная функция относительно ошибки равна гт» (,-л - ' _ р(1 +TiP) WeW- i + w(p) ~ TlP* + p + K- Спектральная плотность ошибки с / ,_im/- чвЯп<т> . 2ГРа(1+ш2Г^) - be(a)-\^(ja>)\ —^2- - (I + a2T2} j Ti (/m)2 + ita + Kp- Приведем ее к виду, удобному для интегрирования (см. приложение 17), _ — rf (/<в)2 + 1 Se (со) = 2TDa | TTt (j<s>y + (Г + Г,) (/ш)2 + (1 + КТ) /<в + /С |2 ' Интегрирование по всем частотам дает средний квадрат ошибки _ е2=2ГАз/3,
•§"7.2. ПРОХОЖДЕНИЕ СЛУЧАЙНОГО СИГНАЛА 291 где интеграл [-T^(ja)+l]da ' 2я J | ТТ1 (/<»)8 + (Г + Г,) (/(й)2 + (1 + КГ) /со + К |* ' — СЮ в соответствии с приложением. 17 равен Значения В коэ результате h- /3 = — aib0 + a0b1 ■ 2а0 (a0as - ффициентов: «0 = Gl = G2 = G3 = имеем V- TTU h T + Ti, bi l+KT, b2 к. аф2 a3 _ „ n \ Ob a3 a,a2) , = 0, =■-71, = 1, ^ 7Ч-У, + Д7^ ' IT J_ T J_ fc"7'2\ Окончательно В -l/^Mr + r. + KTl) CK |/ к (r + т, + кт*) Приближенное выражение для среднеквадратичной ошибки имеет вид еск = у -^^г = -f- = is = 0°.°8 ~ 4'А 310. На входе системы регулирования действует помеха с корреляционной функцией Rn (т) = £)„вПЧ ' I (cos рт + ^ sin p | т |)
292- ГЛ. 7. СЛУЧАЙНЫЕ ПРОЦЕССЫ В ЛИНЕЙНЫХ СИСТЕМАХ Is» и спектральной плотностью С I \— л ^ Г 2ft - со 2(3 + со ] Численные значения коэффициентов: Du = < = 100, и = 0,4 сек-1 и . р = 5 сек-1. Определить коэффициент сглаживания, равный отношению среднеквадратичного значения помехи на входе к среднеквадратичной ошибке на выходе системы к «r-^s Асгд ~ ' и среднеквадратичную ошибку а. Передаточная функция разомкнутой системы W(p) = £, где добротность оистемы по скорости К = 0,1 сек'1. ■ Ответ. Коэффициент сглаживания к Z'_ .__ P2J-u2 ' ■ _ Асгл / 5* + 0,4* _16>7> 5 °-1 V Л 52 • 0,1 52 Среднеквадратичная ошибка (Гп 10 п „ 0 =■ -7?-°- = TFT = °>6- /(„л 16,7 311. Для аппроксимации корреляционной функции предыдущей задачи используются две формулы: /?(*) = Ое-"1*1 стерт; (1) /?(T) = 0<r*lT»£cospr + -j|-sinp|T|). (2) Этим корреляционным функциям соответствуют спектральные плотности:
im § 7.2. ПРОХОЖДЕНИЕ СЛУЧАЙНОГО СИГНАЛА 293 для формулы (1) S (со) = [xD [м* + (ш_р)2 + M» + (m + p)2j. (3) для формулы (2) \ _ it П Г 2Р - а . j_ 2P + C0 1 . S.(co)- p W[(12 + (t0_p)s + ц2 + (ю + р)2 J* ^ Определить дисперсию скорости для формул (1) и (2), .'. Ответ.. 1) Da-> оо; 2) Da = (ц2 + р2)D. 312. На вход системы с передаточной функцией в момент времени / = 0 поступает случайный центрированный стационарный сигнал с корреляционной функцией /?,(T) = D1e^lTi. (2) Определить изменение дисперсии выходной величины во ■ времени D2 (t), а также дисперсию выходной величины в установившемся режиме. Исходные данные: К — 10, Г =10 сек, Dy—l и ц = 0,О5 се/с-1. Решение. Значение дисперсии на выходе может быть определено по формуле * ' t D2(t) = f w (s) ds J w (tj) Rt (tj - s) rftj, (3) о о где w (s) и да (tj) представляют собой функцию веса системы w (t) при замене t = s и f = tj. Для передаточной функции (1) функция веса будет w{t) = aKe~ai, где а = Т~1 (см. приложение 1). Следовательно, ■ t f ^2 (t) = Щ^. Г «-« rfs J e^e-n l n-« I rftj = ,. о о =*°Щг- J e-bs-ds j. e^^g+i* oi-?) dr\+ [ <r*"1£-'A <i-s>dTj . (4)
294 ГЛ. 7. СЛУЧАЙНЫЕ ПРОЦЕССЫ В ЛИНЕЙНЫХ СИСТЕМАХ [312 Вычисление интегралов дает при а Ф ц S ц —а о t S t s t Ds (0 = 4?- J e"as (7i + h) ds = _„2n f 1 , e~zat 2<Г<а^Ч , В установившемся режиме Формула (6) может быть получена также из спектральной плотности входного сигнала —оо —оо Спектральная плотность выходного сигнала S2 (©) = | W (/со) Р S, (со) = ,„'. *У' . .„ . Интегрирование спектральной плотности 5г(а) по всем частотам дает — оо —с» * В соответствии с приложением 17 получаем При Подстановке числовых значений получаем D2(/) = 66 + 200е-°-а - 266е-°-1И, Д>(°о) = 66.
§ 7.3. ОПТИМАЛЬНЫЕ СИСТЕМЫ 295 313. Решить предыдущую задачу, если передаточная функция системы соответствует идеальному интегрирующему звену с передаточной функцией w &)■=£-, где tfi = 0,l сект1. Ответ. D2(t) = 2KiDy [^ - (1"^Г°] = 0,4/ - 8 (1 - е-о.°*), £)2(оо)—> об. § 7.3. Оптимальные системы 314. Передаточная функция разомкнутой системы регулирования имеет вид Щр) = ^, где К= 100 сек-2 —общий коэффициент усиления разомкнутой цепи, а т — постоянная времени корректирующего устройства. На входе системы действует полезный ре- bt2 гулярный сигнал вида g = at + -5-, где а= 100 град/сек и 6=10 град]сек2, и помеха, представляющая собой белый шум со спектральной плотностью Sn(a>) = N — = 0,2 град2/гц. Определить значение постоянной времени корректирующего устройства, соответствующее минимуму среднеквадратичной ошибки в установившемся режиме, а также значение среднеквадратичной ошибки. Решение. Установившееся значение ошибки от полезного сигнала хс = C\g + -у g = ct (a + bt) + -j-b, где сг и с2 — коэффициенты ошибки. На основании разложения передаточной функции относительно ошибки
1-296 ГЛ. 7. СЛУЧАЙНЫЕ;-ПРОЦЕССЫ В ЛИНЕЙНЫХ СИСТЕМАХ :{S!S в степенной ряд имеем С[ = 0 и ~ф — ~}(- В результате регулярная составляющая ощнбки _^ л:с-г ^ или . *с2=|г. (1) Средний квадрат случайной ошибки (см. приложение 17) равен -f ею — оо K2N. 2л Г [~ т2 (j-ш) + 1} da> (l+Ki2)N , J .|(/ю)2.+ /Ст/Чй + К|2 2т ' ^' а средний квадрат, суммарной ошибки Для отыскания минимума последнего выражения приравняем • нулю первую производную по постоянной времени корректирующего устройства 2Ят2-(1+/(т2) = 0, ш откуда получаем 1 I л, Г/С I 10!) Среднеквадратичная ошибка определяется из (3): A™ К НЮ2 + 2-0,1 °'Z ' '* • 315. Решить предыдущую задачу, если передаточная функция разомкнутой системы имеет вид IV/ I \ ^(1 + Т/1) *НрН>(1+7»' где /С =100 сек-2 и Г = 0,05 с<?/с.'. ,
. -S ГЛ.-ОПТИМАЛЬНЫЕ СИСТЕМЫ 297 Ответ. .- Х=Т + ~\^Т2 + х = °'05 + ^°'052 + °-01 = °-16 сек- Среднеквадратичная ошибка _, / б2 (1+/Ct2)JV *ск = |/ "JC5""*" 2 (Т - Г) _,/ Ю2 (1 + 100-0,16г)-0,2. ,OQ У 1002 + 2(0,16-0,05) .' Ь 316. Решить задачу 314 в предположении, что можно изменять как значение постоянной времени корректирующего устройства х, так и общий коэффициент усиления К. Решение. Дифференцируя выражение (3) в - задаче 314 по т и по К и Приравнивая частные производ-.. ные нулю, получаем '-тт- "> -|J + f.-l>. (2) Подставляя (1) в (2) и решая последнее уравнение,, имеем Кош = У -^г = У -^г- = 21 сек~2- Постоянная времени корректирующего звена 1 • I Г К Г 21 0,218 сек. Среднеквадратичная ошибка определяется из (3) задачи 314, / 10* ' (1 +21 -0,218») 0,2 '= ,0 П7 *ск— у 212- 2-0,218 ' ' 317. Передаточная функция разомкнутой системы регулирования имеет вид
298 ГЛ. 7. СЛУЧАЙНЫЕ ПРОЦЕССЫ В ЛИНЕЙНЫХ СИСТЕМАХ [317 где К — общий коэффициент усиления, а Г, — постоянная времени. Передаточная функция замкнутой системы равна На входе системы действуют помеха в виде белого шума со спектральной плотностью S„{a) = N и полезный сигнал со спектральной плотностью Sc(w)=tS%- Между помехой и полезным сигналом отсутствует корреляция. Исходные данные: Т\ = 0,1 сек, Тс = 20 сек, D = = 100 град2 и N = 0,01 град2\гц. Определить оптимальное значение общего коэффициента усиления К0т> соответ-, ствующее минимуму среднеквадратичной ошибки, и среднеквадратичную ошибку при К —Кот- Решение. Составляющая среднего квадрата ошибки» определяемая помехой (см. приложение 17), равна K2N da KN |Г,0©)г + ](о + К|! (1) Составляющая среднего квадрата ошибки, определяемая полезным сигналом на входе (см.^приложение 17), :_L Г 2я J -t-oo a? (I+<&■*) 2TCD |r,0<»)2 + /№ + K|2 1 + cA* = 2TeD + oo i Г [rf (/©)«-(/©)«] и© 2я J \TlTc(jti>)3 + (Tl + Tc)(i&)2 + {l+KTc)i& + K[2 — oo = dt1 + tc + kt1t, tx + tc + kt2. Результирующий средний квадрат ошибки & = е2п + егс = ™- + в Tl + Тс+^'^ . (3)
sisl .§ 7:3. ОПТИМАЛЬНЫЕ СИСТЕМЫ 299 При минимизации среднеквадратичной ошибки необходимо приравнять нулю производную от последнего выражения по коэффициенту усиления. В результате имеем N />rc(7*-_7f) 2 (7-, + 7-с + /СГ2с)2 '0. Решение последнего уравнения дает оптимальное зн;> чение коэффициента усиления 2D (Tl- Г2) Гс+Г, Ао — " ' -V NTi Определяем численное значение оптимального коэффициента усиления _ /2-100 (20°-0,1г) _ 20 + 0,1 _ , Аопт— |/ 0,01-203 202 * Среднеквадратичная ошибка на основе (3) равна о ,/30-0,01 , 1ПП 0,1+ 20,+зо-о,1-Ж,. По0 Иск у 2 -г lw 0.1+20 + 30-202 ~ ' ' 318. Для предыдущей задачи определить передаточную функцию системы регулирования, соответствующую теоретическому минимуму среднеквадратичной ошибки, и определить значение последней. Решение. При условии физической реализуемости системы регулирования искомая частотная передаточная функция замкнутой системы может быть представлена в виде ф (>•*>=w- (1> Знаменатель (1) определяется из равенства : ф (/ю) V О) = Sc (о) + Sn (со), (2)" где -ф* (/со) является комплексно сопряженной ф (/<■>)• Для нашего случая о / ч „ , ч 2Гггу 27-cD + W(l+co2r2)
300 ГЛ 7. СЛУЧАЙНЫЕ ПРОЦЕССЫ В ЛИНЕЙНЫХ СИСТЕМАХ 1ЯШ< Разложим последнее выражение иа комплексно сопря* женные множители 2ТСР + N (1 + со2Г^) _ (1 + jaco) (1 - jaa) , l+ctf?7* . (1+у7»(1-/7>) Отсюда имеем at2. где Далее находим выражение Sc (m) 2TCD (1 - ;Тссо) 2ГС£> 1 ■ф (со) (1 + «Рт2.) У А (1 - /асо) К"л (1 + /7» (1 - /аш) " Р'азложим последнее выражение иа простые дроби ■Sc(co) = 2ГСР г тс . 1 о 11 ■ф* (/со) /Л L7"c + o: 1+/Тссо Тс + а 1-jaa}' Функция В (/со) определяется членами разложения, соответствующими полюсам Sc(fi>), лежащим в верхней полуплоскости. В результате имеем ВЦ<*)=2™ J' ' ., (5) У А 1 с + а 1 + /7 ссо ■Искомая частотная передаточная функция замкнутой системы (1) равна К'Ш> ф(;со) А Тс + а l+jaa' {U' Определяем численные значения коэффициентов а = ?'<= /Wjhw = 20 /2.20-100 + 0,0. = °'°32 С™> 2Т2сО 2Т2Г> 2Т2П . , А(Тс + а) (2TcD + N)(Tc + a) 2T2D ' Окончательное . выражение для передаточной функции
glg. $7.3. .ОПТИМАЛЬНЫЕ СИСТЕМЫ - 301 замкнутой ^системы примет вид ф0»>-Т+7>. (7) где Т = 0,032 сек. Этой передаточной функции соответствует передаточная функция разомкнутой системы w\P) , _ ф (р) Гр - р , (») где /С = -у- = 31 сект1 — общий коэффициент усиления разомкнутой системы (добротность по скорости). Передаточная функция относительно ошибки Фе(р)=1-Ф{р) = т^г. (9) Спектральная плотность ошибки. S,(©) = I % U<o) I2 Sc (©) +1 Ф (/со) I2 S„ (со) = 1 + оэ2У2 l + cu"Y; 1 + ю2Г2 ' Интегрирование (10) по всем частотам дает средний квадрат ошибки — oo Среднеквадратичная ошибка равна ft л/~ tD jl. N ./0,032-100 , 0.01 no cK Bc«~ J 17ГГ + -if - V "2оПмй2- + 2-ода ~и -bb-
ГЛАВА 8 СИСТЕМЫ С ПЕРЕМЕННЫМИ ПАРАМЕТРАМИ § 8.1. Построение переходных процессов 319. Определить весовую функцию системы, движение которой описывается дифференциадьным уравнением a0~ + {a\ + bt)x = l!(t), (1) где а0 = 1 сек, а° = 0,5 и Ъ = 0,2 сек~1, при поступлени'и на вход единичной б-функции f(t) = 6(t — Щ в произвольный момент t — ■&. Начальное условие х — 0 при t — 0. Решение. В выражении (1) сделаем подстановку / (/) = б (t — ■&) и разделим все члены на а0, их а?+ 6(0 б (*-■&) — + ' х = ■ (2). или *jL + P(t)x = Q{t). (3) Далее находим I S(t)=\p(t)dt=[^~-dt~ J •! "о = -£- V - <*) + 4" (*2 - «2) = <*(' - 0) + p (*2- П
320] § 8.1. ПОСТРОЕНИЕ ПЕРЕХОДНЫХ ПРОЦЕССОВ 303 Весовая функция t t _ е-о (<-»)-Р (Я-©2) Г 6 (* ~ в) е+а (<-©)+р (<*-*>) ^ ^) При вычислении последнего интервала необходимо воспользоваться известным свойством 6-функции + оо J 6(t-o)f(t)dt = №. Тогда из формулы (4) имеем до (t - ft, ft) = — е~а <Н»-Р <*!-»2>. (5) Подстановка числовых значений дает w(t-ft,ft) = е-0-5 <ИЧ-о.1 <*2-»2). (6) 320. Для весовой функции предыдущей задачи построить графики: 1) нормальной весовой функции при "■О = 2 Сек в виде до (t — ft, ft) и в виде до (т, ft), где т = t — ft, 2) сопряженной весовой функции при £ = 5 сек в виде до (t — ft, ft), т. е. в зависимости от смещения ft, и в виде до (в, / — в), где 0 = t — ft — реверс-смещение. Ответ. 1) Нормальная весовая функция . до (i- ft, ft) = е-°-5 t'-«-o.i с2-») при f > ft = 2 сек. График изображен на рис. 188, а. Переход к времени х = t — ft, дает ДО (Т, ft) = e-0.5l-0,lT (T+29) _ e-0,9T-0,lt» при т ^> Q. График изображен на рис. 188, б. 2) Сопряженная весовая функция ДО {t — ft, ft) = е~Ь-5 ©-ft)-0.1 (25-fl2) _ е0,5 (»-5)+0,1 (»*-25) при ■в^^ = 5 сек.
304 ГЛ. 8. СИСТЕМЫ С ПЕРЕМЕННЫМИ- ПАРАМЕТРАМИ [32.1 График изображен на рис. 188, в. Переход к реверс-смещению 6 = < — -& = 5 — f> дает w (в, t - в) = е-'-5 е+0-' № при в > 0. График изображен на рис. 188, г. о) \w(t-i,i) 8 =2сен S сен С W 0,8 ЦБ 0.4 02 12 3 4 w(B,t-e) г) \ t=5ceti 1 ^Ч , i сел в 2 3 4 5 сен 0 1 2 3 4 сен Рис. 188. Графики к задаче 320. 321. Методом последовательных приближений построить переходный процесс в системе, описываемой дифференциальным уравнением d x dx dt2 dt (1) при поступлении на вход в момент времени t = ■& = 1 сек ступенчатой функции g{t) = g0l (t — ■&). Значения коэффициентов: а0=1 сек2, а,\(t) = (0,9 + 0,It)сек, а2 = 0,16 и go = 1,6. Начальные условия нулевые. Решение. Замораживаем переменный коэффициент дифференциального уравнения (1) в момент времени / = ■&= 1 сек. В.результате получаем а1 {^}=а\ + 0,1 = 1 сек.
,. § .8.1. ПОСТРОЕНИЕ ПЕРЕХОДНЫХ- ПРОЦЕССОВ 305 Уравнение (1) записываем в виде a0^ + ^^)^ + a2x = g(r)-0,lx^. (2) Первое приближение находится из дифференциального уравнения fl0 $- + a'l (*) % + a2xt = gl) 1 (т), (3) т = * - ■». Используя преобразование Лапласа, находим изображение искомой величины |W а0р2 + а,(#)р + а2 р(р2 + р+ 0,16) _ 1,6 _ Ш ■ 13,3 3,3 р (р + 0,8) (р + 0,2) р р + 0,2 р + 0,8 - Переход к оригиналу (см. приложение 1) дает, лг, (т)= 10(1 - 1,ЗЗе-°-2г + 0,ЗЗе-°-8т). Поправка x2(t) находится в результате решения уравнения оо^ + аМ^ + а^^-ОЛх^. (4) Подстановка в (4) числовых значений коэффициентов, а также найденного первого приближения дает -^ ■-****+ 0,16лг2 = 0,027т (e-0-8t - е-°-2т). (5) rfr2 d% Изображение искомой поправки v {гл = 0,027 Г 1 1 ] 2УН' Р2 + р + 0,16 L (Р + 0,8)2 (р + 0,2)* J 0,027 0,027 (р + 0,8)3 (р + 0,2) (р -К0,2)3 (р + 0,8) (6) Переход к оригиналу можно сделать, воспользовавшись' интегралом свертки. Для этого запишем оригиналы
306 ГЛ. 8. СИСТЕМЫ С ПЕРЕМЕННЫМИ ПАРАМЕТРАМИ следующих изображений: I =ie-0.8T р + 0,8 р + 0,2 • 1 . т2' (р+ 0.8)3 1 {р + 0,2)3 • 2 Интеграл свертки е-0.8т 0,2т Т .2 ж2(т) = 0,027 J ~- e-°-8t' • erwv-^dxt - о - 0,027 I ~ е-0-2*' • в"0-8 <т-т'> tft, = о = 0,027 J -J- e-°-6t'-°.2T tfr, - 0,027 J -J- е0-6т'-°-8т dr,. о о Вычисление интегралов дает x2 (т) = 0,075 [е-0-2* (т - 0,3т2)— e-°-8t (т + 0,3т2)]. Таким образом, второе приближение дает х{т) = дг, (т) + х2 (т) = 10 (1 - 1,ЗЗе-°-2т + 0,ЗЗе-°-8т) + + 0,075 [e"°'2t (т - 0,3т2) - е-°-8т(т + 0,3т2)]. Сравнение л:2(т) и *i(t) показывает, что вычисление следующей поправки л:3(т) не является необходимым. 322. Параметрическая передаточная функция замкнутой системы регулирования имеет вид Ф<Р'*-р + а+аЫ + сР- М Определить переходную функцию системы при поступлении входного воздействия g(t) = g0l(t).
3231 § 8.T. ПОСТРОЕНИЕ ПЕРЕХОДНЫХ ПРОЦЕССОВ 307 Решение. Изображение по Лапласу входной величины G(p) = f. Изображение выходной величины равно у(р, о-ф<р. o-G(P)-p(p+;+g°6f+c,2r (2) Зафиксировав в выражении (2) время t — const, на основании приложения 1 находим оригинал, y(t)- ago a + bt + ct2 Г J _ e-(a+bt+ct') П 323. Построить графическим путем переходный процесс в системе, описываемой дифференциальным уравнением dx a0(t)-w + alx = f1(x). dt (1) - График изменения входного воздействия /, (t) изображен на рис. 189, а. График изменения коэффициента <i сен Рис. 189. Графики к задаче 323. qa(t) дан на рис. 189, б. Коэффициент at = 2. Начальное значение х = х0 = 1,5 при t = 0. Решение. Все члены уравнения (1) разделим на щ, T{t)^- + x = f{t). (2) Здесь г(о = a0(t) /(0 = МО
308 ГЛ. 8. СИСТЕМЫ С ПЕРЕМЕННЫМИ ПАРАМЕТРАМИ При решении уравнения (2) графическим путем «постоянную» времени Т {t) считаем постоянной на интервале t, t + At и равной Т it + -~-1. Формула для решения в этом случае имеет вид А/ f(/+^)-*(0 А,. '(/+ 2 ЬЧ) + - м (3) Процесс построения сводится к следующему. На" рис. 190 нанесены f(t) и Т(t). Шаг времени выбираем t.cea- Рис. 140. Построение переходного процесса к задаче 323. ~А/ = 0,5 сек. Из точки Е кривой f(l), взятой в середине первого интервала At, откладываем по горизонтали отрезок ЕМ = Т 1 — 1, величина которого берется равной ординате точки Н кривой Т (t), т. е. тоже в середине первого интервала А/. Полученная точка М соединяется прямой линией с заданной начальной точкой процесса А. В результате получается 'новая точка В искомой кривой хИ). Аналогично берется ордината точки /, откладывается в виде отрезка FN и проводится прямая NB, дающая новую точку С решения х{1) и т. д.
S241* у ft 2. ОЦЕНКА УСТОЙЧИВОСТИ РР.ГУЛМРОБАТШЯ 309 6 8 2. Оценка устойчивости и качества регулирования 324. Система регулирования описывается дифференциальным уравнением a^ + ^^F + ^m^r + ^y^b^it). (1) Значения коэффициентов: а0 = 0,1 сек3, а, = 4,2 сек2, Дй(/) = (72 — 0,Н) сек, g3 = 40° и &о = 400. Оценить приближенно устойчивость системы, если время работы ее Т г= 100 сек. ■ Решение. Рассмотрим систему с замороженными коэффициентами при ^ = 0 и при t = T — 100 сек. В этих случаях характеристическое уравнение, соответствующее исходному дифференциальному уравнению (1), будет 0,1р3 + 4,2р2 + 72р +400 = 0, (2) 0,1р3 + 4,2р2 + 62р +400 = 0. (3) Для уравнения (2) находим корни: р, = 10 сек'1, р2_з = (—16±/12) сек'1. Степень устойчивости г\ = | р, j = = 10 сек-1. Время переходного процесса /n»s3ii-1 = 0,3 сек. Для уравнения (3) корни равны р, = — 25 сек'1, Р2-з = (~ 8,8 ± /8,7) сек~х. Степень устойчивости ц = = 8,8 сек'1. Время переходного процесса tn<3i\~1 = = 0,34 сек. За время переходного процесса коэффициент a2{t) изменяется на величину Да2«0,1 '0,34 = 0,034, что составляет приблизительно 0,05%. Следовательно, система может рассматриваться как квазистацибиарная. Оценка устойчивости может быть сделана по замороженным коэффициентам характеристического уравнения. Применяя критерий Гурвица, имеем аха2 (t) > йо^з- Подстановка числовых значений дает 4,2 (72 -0, И) > 40. Последнее неравенство выполняется при любом вре-. мени, лежащем в пределах 0=^/^100 сек. Следовательно, система устойчива.
310 ГЛ'. 8. СИСТЕМЫ С ПЕРЕМЕННЫМИ ПАРАМЕТРАМИ [Щ, 325. Дана весовая функция квазистационарной системы w(t-Q, ■&) = е~а <е°-&) ('-*>, где вь = 20 сек, а = 5 сек-2, ^ — текущее время, отсчитываемое от момента включения системы, Ь — момент поступления входного импульса. Определить устойчивость системы. Ответ. Нормальная весовая функция затухает и система является устойчивой в пределах времени 0 <^ < вь = 20 сек. При t>Q0 любое малое'возмущение на входе может вызвать неограниченное возрастание выходной величины. 326. Параметрическая передаточная функция замкнутой системы имеет вид где а =10 сек~л, b = 0,1 сек~2 и с = 0,01 сек~3. Определить коэффициенты ошибки c0(t), c1(t) и c2(t). Решение. Находим передаточную функцию замкнутой системы относительно ошибки: фхО>,о-1-ф(Р,о-р^+"6Т^,. (2) Разлагая последнее выражение в ряд по степени оператора р, получаем ^ , ,._ U + ct2 ар ар2 Wx{p, l)- a + bt + ct2 •+- (a + bt + ct2)2 (a + bt + cl2)3 "•"••• Отсюда можно определить коэффициенты ошибки bt + ct2 0,U + 0,0U2 - Со(*) = a + bt + ct2 10 + 0,lt + Ofilt2 ' г Н\^ а = ■ 10 IV/ (a + bt + ct2)2 (W + 0,lt + 0,0U2)2 ' c2(t) _ a 10 2 {a+bt + ct2)3 ~ (10+0.U +Ofilt2)3 ' 327. Объект регулирования совместно с исполнительным органом описывается дифференциальным
tUff vf-8.fi. ОЦЕНКА УСТОЙЧИВОСТИ РЕГУЛИРОВАНИЯ 311 уравнением dt2 ■ регулируемая величина, (I) ГЛе V — регулируемая величина, X — g — У — ОШИбкЗ, а — входное воздействие,- £>0 = 100 сек'2 и &t = Q,l сек~3. Считая систему квазистационарной, определить необходимые корректирующие устройства, чтобы в пределах времени работы системы 0<itf< L000 сек замкнутая система имела бы показатель колебательности, не превышающий значения М = 1,5. Задачу решить методом замораживания коэффициентов. Решение. Передаточная функция исходной разомкнутой системы с замороженным коэффициентом равна W0{p) = bo + bit- к (2) Л. а, х. представляет собой прямую с наклоном —40_дб/де/с (рис. 191). Базовая частота л. а. х. to0= VK- Рис. 191. Л. а. х. к задаче 327. Использовав л. а. х. типа 2—1 — 2 (см. приложение 19), получаем желаемую передаточную функцию разомкнутой системы №ж(р) = Р2 (1 + TiP)
312 ГЛ. 8- СИСТЕМЫ С ПЕРЕМЕННЫМИ ПАРАМЕТРАМИ [8*28 Постоянные времени равны т _ ± i/ЗГ Т = Тг(М-\) = 1 УМ (М-1) *3 М+.1 сов М+1 Передаточная функция корректирующего звена выражается в виде UV (р) 1 + ТгР WK3(p) = W» (Р) 1 + 7'зР Подстановка исходных значений дает требуемые законы изменения постоянных времени /: 100 + O.U ' Ть 5 У 100 + О.Н ' При / = 0 значения постоянных времени То = 0,173 сек и Г3 = 0,0346 сек. При /=1000 сек, 7,2 = б,123 сек и 7^3 = 0,0246 сек. 328. Определить передаточную функцию объекта совместно с исполнительным органом по данным предыдущей задачи методом замороженных реакций. Решение. 1. Замораживание весовой функции. . В уравнении (1) предыдущей задачи необходимо положить A-(/) = 6(i —#). ' Тогда t ^-J*(&o+M)*('-<>)<tf=&o + M>. (0 у = ш„(/ - О, О) = J (fr0 + 6,0) <tf = = (&о+М)('-#) = (&о+М>)*- <2> Зафиксировав в последнем выражении # = •&„ = const, получаем w0{x) = {b0+ bi&0)x: Переход к передаточной
.13281 § 8-2- 0UeHKA УСТОЙЧИВОСТИ РЕГУЛИРОВАНИЯ 313 функции объекта дает W0(p) = L [(&„ + Mo) т] = ^^ = jri (3) Эта передаточная функция совпадает с полученным в задаче 327 выражением (2). Поэтому использование замороженной весовой функции в данном случае не дает ничего :нового по сравнению с методом замороженных коэффициентов. 2. Замораживание переходной функции. ' В выражении (1) задачи 327 положим x(t) = 1 (t — ■&). Тогда, положив t — ■б, = т, имеем т ■ -g- = J [Ьо + &. (* + т)] f (т) rfr = bux + Ь,«т + -^, "(4) о t у = А„ (* -*,#) = J (&ит + 6,<к + -^) dx = о _ 60тг . Ь,Ъх2 b.,xs /кч --2~_+-2— + -Т~- (5) Зафиксировав смещение Ф ='■&„ = const, получаем за-- b т2 b "б1 т2 мороженную переходную функцию /?0 (т) = -~ Ч—L|—(- - Ъ Xs Ч—!g-. Продифференцировав ее' по т, получаем замо- b т2 роженную весовую функцию к>0(т) = {b0+ b$Q)x-\—L—. Передаточная функция объекта w0 (p)=l [a?0+Mo).*+-bf'\- _ bg + b^o ]± 6,(l-!-7-0p) /ft. где Г0 = ' ' ' ' меняется в пределах от 1000 сек при *о = 0 до 2000 сек при #„.= 1000 сек. ч
ГЛАВА 9 СИСТЕМЫ С ЗАПАЗДЫВАНИЕМ И С РАСПРЕДЕЛЕННЫМИ ПАРАМЕТРАМИ § 9.1. Системы с временным запаздыванием 329. Структурная схема автоматической системы имеет вид, показанный на рис. 192. Определить, при какой величине общего коэффициента усиления разомкнутой системы K = kxk2 замкнутая система устойчива при любых значениях по- стоянной^времени Т и времени запаздывания т. Ответ. К^\. Рис. 192. Структурная схема 330. Для системы авто- к задачам 329 и 330. матического ' управления, структурная схема которой показана на рис. 192, определить критическое время запаздывания тк. Общий коэффициент усиления разомкнутой системы К. = kxk2- Постоянная времени Т = 0,5 сек. Решение. Частотная передаточная функция разомкнутой системы равна где K = kxk2. Частоту среза сос, при которой а. ф. х. разомкнутой системы пересекает окружность единичного радиуса, найдем из условия |Г(/сос)|=1. (1) Из уравнения (1) получим ">\1 г к, /*Тр ¥tfi
«331 § **■ СИСТЕМЫ С ВРЕМЕННЫМ ЗАПАЗДЫВАНИЕМ 315 Критическое время запаздывания тк найдем из условия равенства фазовой частотной характеристики разомкнутой системы при частоте со = сос величине —я •ф («с)= — arctg (осГ — (остк = — л. Из последнего уравнения находим я — arctg ысТ п — arctg VK2 — 1 ~ п , 0 *--: = Vim т = 0'18сек- 331. Передаточная функция разомкнутой системы автоматического управления имеет вид pV + tp) ' где /С = 20 сек'1 — общий коэффициент усиления разомкнутой системы, Т = 0,1 сек — постоянная времени. Последовательно ъ канал управления включается звено чистого запаздывания, имеющее передаточную функцию е~хр, где т —время запаздывания. Требуется найти Критическое время запаздывания тк, при котором зам-' кнутая система автоматического управления заходится на границе устойчивости, и частоту незатухающих колебаний сок. Ответ. -юк=12,5 сек~г, тк = 0,11 сек. ^332. Решить предыдущую задачу, если постоянная времени Т = 0. 4 Ответ. сок = 20 сект1, тк = 0,78 сек. 333. Передаточная функция разомкнутой системы имеет вид С помощью логарифмических частотных характеристик определить критическое время запаздывания тк, если коэффициент усиления разомкнутой системы К = 30 сек~1, постоянные времени. Т^ = 0,025 сек и Г2 =* = 0,2 сек. Решение. Асимптотическая л. .а. х. системы приведена на рис. 193. Частота среза разомкнутой системы
316 ГЛ. В. СИСТЕМЫ С ВРЕМЕННЫМ ЗАПАЗДЫВАНИЕМ [3SV юс= 12,6 сек 1. Фазовая частотная характеристика при частоте со = сос и критическом времени запаздывания тк ^—i—со. сек ■ Рис. 193. Асимптотическая л. а. х. к задаче 333. должна пересекать линию "ф= —л. Поэтому •ф («с) = - у - arctg сосГ! - arctg <ясТ2 - состк = Отсюда находим п ■ Ц- arctg ШсГ, — arctg сос^ т„ = - «Ос 4,8 • 10"* сек. 334. Определить критическое время запаздывания тк системы, структурная схема которой приведена на '—<НДг1 ^ г*-у Рис. 194. Структурная схема к задаче 334. рис. 194. Общий коэффициент усиления разомкнутой системы /С = kik2k3 = 5, постоянные времени апериодических звеньев Г, = 5 сек и Т2 = 0,4 сек. Ответ. тк=1,4 сек.
335! 9.1. СИСТЕМЫ .С ВРЕМЕННЫМ ЗАПАЗДЫВАНИЕМ 317 335. Передаточная функция разомкнутой системы имеет вид где /С = 2 сек'1, Т = 0,1 сек. Для достижения устойчивости системы последовательно в канал управления вводится звено чистого запаздывания с передаточной функцией е~хр. Определить, при каких значениях времени запаздывания т замкнутая система устойчива. Р ё ш е и и е. Результирующая передаточная,функция разомкнутой системы W(p) = W0(p) е~тР = р(1 + .f2pl) . Асимптотическая л. а. х. системы приведена на рис 195. Замкнутая система будет устойчивой, если oj.ce/f' Рнс. 195. Асимптотическая л. а. х. и л. ф. х. к задаче 335. л. ф. х. пересекает линию i|) = — я в диапазоне частот К-ъ--~г. Критические значения времени запаздывания тк находим из следующих уравнений: я ь, ■ п_ 1_ П. Отсюда fк тлх = -^ = 0,79 сек, т,( n,i„ = j T = 0,16 сек. Замкнутая система.устойчива, если выполняется следующее неравенство: 0,16 сек <т<0,79 сек.
318 ГЛ. 9. СИСТЕМЫ С ВРЕМЕННЫМ ЗАПАЗДЫВАНИЕМ 336. Передаточная функция разомкнутой системы имеет вид" W(p)- Ке~хР ту+т1Р+1' где К = 0,5, Т2 = 1 сек2, Тх = 0,25 сек. Определить устойчивость замкнутой системы при следующих значениях времени запаздывания т: а) т = 0; б) т = 0,3 сек; в) т = 2 сек; г) т = 5 сек. Ответ: а) Система устойчива; б) система устойчива; в) система неустойчива; г) система устойчива. 337. Структурная схема автоматической системы приведена на рис. 196. Коэффициенты передачи звеньев соответственно равны kx = 1 сек~1, kz= 0,125, k3=l. Постоянная времени Т = 2 сек. Время запаздывания ff —»-®-<- * ~®~ кц 1+Тр к3е*Р X «HI ' -у Рис. 196. Структурная схема к задаче 337. т = 0,2 сек. Определить устойчивость системы по критерию Наиквиста. Размыкание системы произвести в точке х (см. рис. 196). Ответ. Система устойчива. 338. Передаточная функция* разомкнутой системы с запаздыванием имеет вид w кр} Р*(1 + Пр) е ' где Ti = 0,5 сек, Т2 = 0,2 сек, т = 0,3 сек. Определить значения общего коэффициента усиления разомкнутой системы К, при которых замкнутая система устойчива. ' Решение. Фазовая частотная характеристика системы определяется следующим выражением: -ф (со) = — 180° + arctg соГ, - arctg соГ2 - сот -^~ (1)
§ 9.1. СИСТЕМЫ С ВРЕМЕННЫМ ЗАПАЗДЫВАНИЕМ 319 приведена на рис. 197. Замкнутая система устойчива, если л. а. х. пересекает линию L = 0 левее точки пересечения -ф(со) линии !])==-- 180°. В критическом случае л а. х. пересекает линию L = 0 на частоте со_л. Провопим' асимптотическую л. а. х. так, чтобы она пересекла град вб со,се/Г Рис. 197. Асимптотическая л. а. X; и л. ф. х. к задаче 338. линию L = 0 на частоте ю_я. Точка пересечения низкочастотной асимптоты л. а. х. с осью частот равна УКК = 3,5 сект1. Отсюда находим, что /fK=3,52= 12,2 сек'2. Асимптотическая л. а. х. в точке излома примерно на 3 дб отличается от действительной л. а. х. Поэтому окончательно получаем Кк= V% ' 12,2= 17 сек'2. Замкнутая система устойчива при 0</С<17 сек'2. 339. Передаточная функция, разомкнутой 'системы имеет вид где К= 10 сек'1, 7, = 0,05 сек, т —время запаздывания. Определить допустимое значение времени запаздывания тд, при котором показатель колебательности системы не превышает Af=l,l. Решение. Показатель колебательности системы не превышает заданного значения М, если выполняется следующее условие [2]:
320 ГЛ. 9. СИСТЕМЫ С ВРЕМЕННЫМ ЗАПАЗДЫВАНИЕМ . |я<о Из неравенства получаем выражение для допустимого времени запаздывания М2 + М Ум2 - 1 - 2КТ 2/С 0,036 сек. 340. Определить запас устойчивости по фазе и частоту среза разомкнутой системы при условиях предыдущей задачи. Значения коэффициентов: общий коэффициент усиления К — Ю сек-1, Т = 0,05 сек, % = 0,036 сак. Ответ. Запас устойчивости по фазе ц = 42,5°. Частота среза сос— 10 сек~1. ы.сен -№* 001 0.02 ОМ 006 ОМ t.cert Рис. 198. Вещественная частотная Рис. 199. Переходная функция характеристика к задаче 341. к задаче 341. 341. Построить переходную функцию системы, передаточная функция которой имеет вид Ф(р) = Ке~хр Р + Ке~хр ' где К = 40 сек'1, т= 12,5- 10~3 сек. Решение. Амплитудно-фазовая характеристика системы равна Ф ,. ч _ Ке~'аг К (cos сот — /' sin сот) /со + Ke~lm> /со + К (cos «от — / sin «от) Вещественная частотная характеристика соответственно равна г>/\ г> «т» / - \ К'2 — К<а sin ют . Р (со) = Re Ф (;со) = -т^-,—ъ—^—- . (1)
342J' § 9.2. СИСТЕМЫ С РАСПРЕДЕЛЕННЫМИ ПАРАМЕТРАМИ 321 Вещественная частотная характеристика* построенная по выражению (1) для /С = 40 сек~1 и т= 12,5 ■ Ю-3 сек, приведена на рис. 198. По вещественной частотной характеристике методом трапеций построена переходная функция (рис. 199). § 9.2. Системы с распределенными параметрами 342. Структурная схема системы автоматического управления гидротурбиной имеет вид, изображенный на рис. 200. v, чд, ф и р — соответственно относительные -91 Щ(Р) JL Щр) ■ & ИЬгГр) ■v9 Рис. 200. Структурная схема системы управления гидротурбиной. величины момента нагрузки турбины, момента, развиваемого турбиной, угловой скорости вращения турбины и перемещения регулирующего органа f8]. Передаточная функция трубопровода, полученная с учетом волновых явлений [8], равна у Передаточная функция турбины W0{p)= l+°T ■ , передаточная функция безинерционногорегулятора W0{p) = = £-, 6 = 0,05, Т0 = 6 сек, £, = 1, v = 0»05. Определить критическое время запаздывания т,„ соответствующее границе устойчивости системы. Решение. Передаточная функция разомкнутой системы равна тиг/ \____£о___ 1 - 2у th хр 1 -2у th тр ■ ЛИ> 6(1 + 2»' 1+ythTp"" в(1 + ВД(1+у1Ьтр) * Характеристическое уравнение замкнутой системы записывается в виде • - 6 (1 + Т0р) (1 + v th хр) + 1 - Эу th хр « 0.
322 f ilV 9- СИСТЕМЫ С ВРЕМЕННЫМ ЗАПАЗДЫВАНИЕМ [343 е*Р _ e-ip После замены thrp на ———— и несложных преобразований характеристическое уравнение приведем» к следующему виду 1 - 2у + (1 + у) б + (1 + у) ЬТ0р + + [1 + 2Y + (1 - v) б + (1 - v) ЬТф] е-*Р = 0. Эквивалентная передаточная функция разомкнутой системы (эквивалентность понимается в смысле тождественности характеристических уравнений замкнутой системы) равна W (п\ - ^+2у + (1-у)б + (1-у)6Г0р 2тр w*w 1-2Y + (1+Y)6 + (1+Y)6r0p 1+0..22Р 2tp 1,ВД 1+0,38р е Эквивалентная частотная передаточная функция разомкнутой системы примет вид Частота среза, соответствующая модулю (1), равному единице, равна &с = У 0,382-'(0,22-1,46)2 = 5j3 CgK '* При т = тк фазовый сдвиг на частоте © = сос должен равняться — зх. Поэтому arctg (0,22 • 5,3) - arctg (0,38 • 5,3) - 2 • 5,3тк - - я. Из последнего уравнения получаем п - arctg (0,22 • 5,3) + arctg (0,38 ■ 5,3) _ n 07 тк — g. 5 3 u>z' 6ere' 343. Найти значение для Критического времени запаздывания системы, рассмотренной в предыдущей задаче, если Wo (р) = j^,. Г0 = Ю сек, б - 0,05, Y = 2.
?f?) § 9.2. СИСТЕМЫ С РАСПРЕДЕЛЕННЫМИ ПАРАМЕТРАМИ 323 Ответ. 6T0arctg 2УТу 2у2— 1 2)^2 0,12 сек. 344. Определить частоту среза разомкнутой системы и запас устойчивости по фазе для системы, рассмотренной в задаче 342. Передаточная функция объекта трубопровода W. передаточная _ 1 - 2у tfa -ср регулятора Wp(p) = l+Ythrp ■ ftp (1 + Tg>) функция приведенного передаточная функция Значения коэффициента+ВД тов: коэффициент передачи турбины &0 = 20; постоянная времени турбины Т0 = 31,5 сек; время запаздывания т = 0,95 сек; у = 0,03; коэффициент передачи регулятора &р = 0,77 сек'1; постоянные времени регулятора Tt *= 12,5 сек, Т2 = 0,48 сек. Ответ. Частота среза разомкнутой системы хшс =» = 036 сек'1, запас устойчивости по фазе и = 66°. Г к, сем Рис. 201. Переходный процесс к задаче 345. 345, Построить при помощи вещественных частотных характеристик переходный процесс в системе, рассмотренной в предыдущей задаче, при подаче на вход системы возмущающего воздействия в виде единичной' ступенчатой функции. Ответ. График переходного процесса приведен на рис. 201.
ГЛАВА 10 ИМПУЛЬСНЫЕ СИСТЕМЫ § 10.1. Дискретные функции и уравнения импульсных систем 346. Вычислить г-преобразование для функции времени f(t) = a0 + alt + a2f2, определенной для /!>0. Период дискретности Т<> = 0,1 сек. Значения коэффициентов: а0 = 1, а{ = 2 сек'1 и а2 — 4 сек~2. Решение. В соответствий с приложением 2 имеем г, ч аог . «lV . а2Т%г(г + 1) z~l ' (г-1)2 ' . (г-1)3 ' Подстановка числовых значений дает р(гЛ_. г , 0^' ■ 0,04г(г+1) г^> г-1 7" (г-1)8 "■"- (г-1)3 ' 347. Вычислить г-преобразование для функции времени, изображение Лапласа которой Решение. Раскладываем изображение на простыв дроби К . К КТ{ р(1+Г,р) р 1 + Г.р' В соответствии с приложением 2 имеем FM- Кг Кг _ K(\-d)z Гу) г-1 г-й ~~-- (z-l)(z-d) » ■ -1st.' где tf = e г', а Го —период дискретности. •
jSOj § 10.1. УРАВНЕНИЯ ИМПУЛЬСНЫХ СИСТЕМ 325 348. Вычислить z-преобразование для функции времени, изображение Лапласа которой К L{f(t)}- рЧ1 + т1Р) ■ Исходные данные: К = 2 сек~л, Г, = 0,1 сек, период дискретности Го = 0,5 сек. Ответ. pi \ кт°г KTiz ) кт^г = г °'2 °'2 ^'г' (г-1)2 z-\'+'z-d (г-1)2 z-1 + z-0,0067 ' -It где d = e r- =e~5 = 0,0067. 349. Вычислить г-преобразование функции времени f(t) — A sin (at = 10 sin a>t в трех случаях: 3) (й = -=т-. ■"о Oreer. 1) F(z) = 7z z2-l,4z+l * 10z ■ ■ z2 + 1 ' 3) F(z) = 0. 350. Дано z-преобразование дискретной функции времени * F{z)- (г-1)5 ' где Го —период дискретности. Определить исходную функцию времени в точках:t = nT0 (n = 0, 1, 2, ...). Решение.-Деление числителя на знаменатель дает бесконечный ряд (ряд Лорана)
326 ГЛ. 10. ИМПУЛЬСНЫЕ СИСТЕМЫ [351 Отсюда можно получить f(nT0) = anT0 = at\t_nTo. 351. Дано г-преобразование дискретной функции времени t^z>- (г_1+а)(г-1) • Разложением в ряд Лорана построить исходную функцию времени в точках * = п7\> (п = 0, 1, 2, ...) для трех случаев: 1) а= 1, 2) а=1,8 и 3) а = 0,5. Ответ. Графики построены на рис. 202. 352. Дано г-преобразование дискретной функции времени / F{z) = -z—, , , ng . (1) *- v ' г2 — l,5z + 0,5 v ' Найти исходную ре- Рис. 202. Дискретные функции шетчатую функцию вре- временн. мени разложением на простые дроби. Решение. Находим корни уравнения z2-'l,5z + 0,5 = 0. Значение корней гх = 1 и г2 = 0,5. Далее представляем F{z) в виде суммы простых дробей F (Z\= (г-1) (г-0,5) ^ 2 \7=Т ~ z-0,5/ ' ^ Первое слагаемое в правой части (2) соответствует оригиналу 1 (пТ0), а второе — е~апТо, причем d = е~аТ" = z2 (см. приложение 2). Поэтому можно записать для оригинала f {пТо) = 2 [1 (пТ0) - е-апТ°] = 2 (г? - г£) = 2 (l - 0,5"). 353. Найти г-преобразование для функции времени. / (0 = а0 + otf + а2*2, / да да /7 fb>V i \ i \ i \ i \ 1 Г * 'I /а=0,5 * \ т0 гт, А 1 \ ЗТ0 4Т0
ass] § 10.1. УРАВНЕНИЯ ИМПУЛЬСНЫХ СИСТЕМ 327 определенной при t !>0 в дискретные моменты времени i~{n + е) Т0. Исходные данные: а0 = 1, щ = 2 сек"', й2 = 3 се/с-2, 7,о=1 сек, е?=0,5. Огеег. й0г , щТ0г , wiBro2 , 2a2Toz ., f (^*=-7=T+ (Z-l)2 + 2-1 + (2-1)3 + а2(1+2е)Г^г а2е2Т%г 2,75z 82 6z (Z-l)2 ' (2-1) (2-1) ' (2-1)2 ^ (2-1)3 ' 354. Найти ш-преобразование функции времени f(0 = ao + «^ + «2^2» определенной для t^0. Период дискретности JT0 = 1 сек,' 00= 1» сц — 1 сек-1 ий2=1 сек-2. Решение. В соответствии с приложением 2 нахо* дим ^-преобразование исходной функции для дискрет», ных моментов времени пТ0 (п = 0, 1, 2, ...): Р( \- a°Z aiT°Z Vpz(z+1) t{?>- z-1 + (z-l)'2 + (z-l)3 ' Используя подстановку г — у^—■; получаем floU+ш) а^О-ш2) а2Г2(1-.щ2) F И- 2ш + 4^ + to3" = 1 + ю 1 — ш2 ■ 1 — ш2 1 + ш + да2 + а>3 ' "1 7Z3 Г 2ш 4эд2 4ш3 . 4ш3 .355. Дано ш-преобразование дискретной функции времени „*, ^ а^(1-ш2)ш F {W) = 4^ ' где а = Ъ сект2, а 7,0=1 сек. Определить исходную дискретную функцию времени. Решение. Используя подстановку w=. ., , на* ходим г-преобразование функции времени: ,,-, °r»['-(Sf)'1 -♦<«+■> '(si)8 ~ <г-"*'
328 ГЛ 10. ИМПУЛЬСНЫЕ СИСТЕМЫ [358 В соответствии с приложением 2 имеем 356. На рис. 203 изображен "импульсный фильтр. Импульсный элемент ИЭ генерирует прямоугольные импульсы относительной продолжительности уТ0, где у = ■=0,05, а период повторения Т0—1 сек. Передаточная „, функция непрерывной части ^о= /, .г:,л » U) к Г+Т,р Т„ ■ I t*T,p [ yv° (1 + Тф)> Рис. 203. Импульсный гДе *С=1°» а Т{= 0,5 сёк. фильтр к задаче 356. Определить передаточную функцию фильтра совместно с импульсным элементом, считая, что последовательность импульсов на выходе импульсного элемента может быть заманена последовательностью 6-функций. Решение. Передаточная функция равна оо Щг) = уТъ S о>о(kT0)'z* = yT0Z {w0{kT0)}. (2) fe=0 Зцесь w0{kT)~ весовая функция непрерывной части Wq{1) при замене t = kT0, z = ePT\ a Z{w0(kT0)} представляет z-преобразование весовой функции. Для рассматриваемой непрерывной части весовая функци я . ^0(/) = Г'{т^} = |-еЧ . (3) В соответствии с приложением 2 находим ■ „Ik где d = е т>. В результате получаем искомую передаточную, функцию .. • • .. ^-^•г^г (5) После подстановки числовых значений имеем 0,5 г — е * г—0,135
§ 10.1. УРАВНЕНИЯ ИМПУЛЬСНЫХ СИСТЕМ 32» 357. Построить амплитудно-фазовую частотную характеристику для импульсного фильтра, приведенного в предыдущей задаче. ' решение. В выражении (6) предыдущей задачи необходимо сделать подстановку z = е/0)Г° = cos соГ0+ + jsintaT0. В результате получается частотная передаточная функция фильтра W(e"»T°) ■ „/<»*•„ с/а>Г„_0)135; Модуль этого выражения и фаза cos юГ0 + j sin оГр cos to7'0 ~ 0,135 + / sin юГв Vl + 0,135s - 0,27 cos юГ0 ■ф = (йТ0 — arctg sin оГр cos (оГ0 — d ' А. ф. х. представляет собой окружность (рис. 204). При оэ = 0, а также при соГо^^тт, где п — натуральное число, модуль и фаза составляют \mW А0- 1 1 l—d 1-0,135 1,15 При (о7,о = (2п—1)л модуль и фаза ' 1 1 Л,= l+d 1+0.135 •ф==± 180°. 0,88 Re**' Рис. 204. А. ф. х. к задаче 357. Центр окружности смещен вправо от начала координат на величину С = ~~ = 0,136, R = -~t=lfil. 358. Найти частотную передаточную функцию импульсного фильтра задачи 356 (см. рис. 203) в зависимости от абсолютной псевдрчастоты.. ) " - *
830 гл. ю. импульсные системы [Я59 Решение. В передаточной функции (6) задачи 356 перейдем к ^-преобразованию посредством подстановки 1 + w z =-rJ—• 1 — W В результате имеем w и —jr- • l_d + {l+d^w = \КТ0' 1 -f w Г, (1 — rf) , . ,. Т0 х ' 1 + w cth -z£- Zl i (1) 4 2 c\T Перейдем к абсолютной псевдочастоте A.=-~-tg ——■ Т посредством подстановки ш. = /-у-Л: vKT 1+У-^Я ^^)-TIfb)--TT7fer- <2> Здесь введена эквивалентная постоянная времени T, = %-cth-^.. (3) Подстановка числовых значений дает Га = 0,5 cth 1 — .«= 0,66 сек. Передаточная функция равна w(m 0,05-10-1 1+/0.5Я __ 1,15(1+/0.5Я) w ЦЛ) — 0>5(1 _ 0135) • j + .QmK -~ j + jQmK . 359. Решить задачу 356 при условии, что относительная продолжительность импульса у = 0,5. Решение. В рассматриваемом случае изображение Лапласа импульса, генерируемого импульсным элементом при.подаче на его вход единичного сигнала, будет равно 1 _ p-PYft \.e-Ptdt=~ . ™-J Приведенная передаточная функция непрерывной части Vu (Р) "= ^0 (Р) Рп (/>) °= р (! + г,р) (1 - g"PVn)-
0, § тол. уравнения импульсных систем 331 Дискретная передаточная функция В соответствии с приложением 2 находим Kd-d)z K[ z zdP \ Kd{d-i-l) W(z) — ~(z-l)(z-d) z\z-\ z-dj ■ z-a (4) -Is. где d — e~T> =0,135, a o=l—y. Подстановка числовых значений дает W^^T^S- (5) 360. Для импульсного фильтра, изображенного на рис. 205, построить логарифмические амплитудную и фазовую характеристики. Пере- даточная функция непрерывной "3 части %(Р) FK T Wl Рис. 205. Импульсный Исходные данные: К — 100 сект1, фильтр к задаче 360. Г0 = 0,05 сек, Ti — 0,2 сек и ■у = 0,1. Считать, что последовательность импульсов на выходе импульсного элемента ИЭ может быть заменена последовательностью 6-функций. - Решение. Дискретная передаточная функция импульсного фильтра равна W(z) = yT0Z{W0{p)} = yTQz{T^f^}. (1) В соответствии с приложением 2 находим I pV + TiP) 1~с\ р— t + r.P / — ~к ( г г \ — K0—d)z ,„. "~AU-I z~d)~ (z-l)(z-rf)' W
332 ГЛ. 10. ИМПУЛЬСНЫЕ- СИСТЕМЫ 13(10 где d = e Ti = е-0-25 = 0,78. Подставляя (2) в (1), имеем (3) W(o*\ - vT0K(l-d)z WW- (Z-\)(z-d) • Используем подстановку z = -j 1 + w W(w) = yToKjl-w*) 2wll+wcth^-\ (4) Переходим к абсолютной псевдочастЬте подстанов- т КОЙ да = /'-у-Я Здесь Те ■^-cth 2 23", W(jk): то _ (<^4) уКи+№ jk(l+jlTa) ' Т0 0,6 *5) 2 th 2-0,115 = 0,217 сек. 2Т, X lU) К'=4,6сен~' J ' -40с&Г 100 сен Рис. 206. Л. а. х. и л. ф. х. к задаче 360. Подстановка числовых значений дает W(A)- 10(1 + 0'02—^ (6) /М1+/0,217Л) ■• Логарифмические амплитудная и фазовая характе ристики должны строиться по выражению . Х(Я) = 201ё|Г(/Я)| = 201ё-МШв1£1, - ; . •»J)(X)=arg-W'0^)=-90o-ar.ctg0,'2l7A.
ш. §. ЮЛ. УРАВНЕНИЯ ИМПУЛЬСНЫХ СИСТЕМ 333 Асимптотическая л. а. х. и л. ф. х. построены на рис. 206. ' 361. Построить логарифмические амплитудную и фазовые характеристики для импульсного фильтра, изображенного на рис. 205, если передаточная функция непрерывной части К Wo(P)= p(l + TlP).(l + T2p)- 0> Исходные данные: /С.== Ю сект1, Т0 = 0,1 сек,Ti = 0,25 сек, 7'2 = 0,01 сёк и у = 0,01. Считать, что последовательность импульсов на выходе импульсного элемента может быть заменена последовательностью б-функций. Решение. Разложим выражение. (1) на простые дроби: К _£ Ktf р (1+г,р) (1 + ад ~ р (г, - ту (.1+ад + + (Ti-TJU + TipJ' W В соответствии с приложением 2 находим дискретную передаточную функцию где 7* - ' Т\ 2| Г,~Т2 ~i(' °~ Г,-^ ~ Ту" ' _!». - -Is. d{ = е r' . и d2=e Ta. Формула (3) может быть представлена в виде Пг)~уТоК[^Г)Р-Н+%-1^я]-. <4> V Используем подстановку г-= . . ,
334 ГЛ. 10. ИМПУЛЬСНЫЕ СИСТЕМЫ 1361 Перейдем к абсолютной псевдочастоте заменой т а> = /-7г-А. В результате l-Af ^.(l+A^cth^.) + : ь Так как Ть < 2Г, и Т% > 2Г2, то Ц- cth и do ~ 0. 2-7У Т„ Cth^.^1 2Г, ■ 8сен ' Рис. 207. Л. а. х. и л. ф. х. к задаче 361. При учете этих зависимостей получаем W(jh) = yT0[((l+jxQ) l~jk~t , т2 /WoU+^r,) /А. (I + /ХГ.) Подстановка числовых значений дает (5) №(А) = [1+/0.004Х-0,015(/Я)2} _ jk (1 + /0,25*,) • ~ 1+0.0I5A,2 *** /Л(1+А25Я) (1 + /0,122Я) (1 — /0.122Я) jX0+jO,25K)
863] § ЮЛ. УРАВНЕНИЯ ИМПУЛЬСНЫХ СИСТЕМ 335 Асимптотические л. а. х. и л. ф. х. изображены на 207. ^ИС362. Импульсной системе регулирования (рис. 208) соответствует передаточная функция разомкнутой системы (см. задачу 360) РГМ \T6K(l-d)z_ 0,11* w\z) — (z_ 1)( ■ d) (z- I)(z-0,78) ' где д- = 100 сект1, То — 0,05 сек, Ту = 0,2 сек, \ ■■ d==:e'T:==e-0^=0,78. JL& (1) 0,1 и «*** . У —о о— т0 —> к р(1+Т,р) У Рис. 208. Импульсная система регулирования. Определить передаточную функцию замкнутой системы и передаточную функцию относительно ошибки. Ответ. Передаточная функция замкнутой, системы W (г) yT0K{l-d)z Ф(г): 1 + W (г) (г•- I) (г - d) + yT0K (1 -d)z, 0,1 \z ~ (z— 1) (z — 0,78) + 0,1 \z Передаточная функция по ошибке (2) <М*)' 1 (z-l)(z-rf) 1 + W (г) ~"" (z - 1) (z - d) + v^oK (1 - d) z (z - 1) (z - 0.78) ~~ (z-l)(z-rf) + 0,llz (3) 363. Передаточная функция замкнутой импульсной системы (см. рис. 208) ф ,;_Д 0.1 lZ ^ W KZ) ~ (г - 1) (г - 0,78) + 0,1 lz ' . На вход системы поступает ступенчатая функция g{t) = rf8 gol (0- Найти z-преобразование выходной величины v(z) и ошибки К (г).
336 тл, ю. импульсные системы 1зв4 Решение. Изображение входной величины в соответствии с приложением 2 будет равно Изображение выходной вличины У(»)-Ф,МО М- и,_ ,ц,-й{н?o,iu](*- ,)- Изображение ошибки Х(2) = Фл(г)0(2) = [1-Ф(2)]0(г) = ■ go (г-0,78)z (г-I) (z-0,78)4-0.I12' . 364, Найти разностное уравнение, связывающее входную и выходную величину импульсной системы регулирования (см. рис. 208). Передаточная функция замкнутой системы -,, ч_ 0,11 г W^Z) (г - 1){г - 0,78) + 0,11г ' Решение. Изображения входной и выходной величин связаны передаточной функцией К(г) = Ф(г)С(2) = аи*0(*) ^ _ 0,llzG(z) 0,11 г~'О (г) z*— 1.67г + 0,78 1— 1,67а:-1+0,78гта ' Перепишем эту формулу в ином виде: (1 - l,67z-' + 0,78z-2)Y (г) = 0,1 lzr'G (z). Отсюда может быть получено разностное уравнение у [п] - 1,67у [п - 1] + 0,78# [/г - 2] = 0,1 lg [п - 1 ]. § 10.2. Устойчивость и качество импульсных систем 365. Передаточная функция замкнутой импульсной системы регулирования ^\г> 2»-1.67г + 0,78" Определить устойчивость системы.
S671 § J0.2. УСТОЙЧИВОСТЬ И КАЧЕСТВО ИМПУЛЬСНЫХ ' СИСТЕМ 337 I Решение, Характеристическое уравнение системы ^-1,672+0,78 = 0. Находим корни Zl 2 = 0,835 ± ]Л0,8352 — 0,78 = 0,835 ± /0,292. Модуль корней | z,. 21 = ]^0,8352 + 0.2922 = 0,88 < 1. Система устойчива. Звб. Определить наибольшее значение общего коэффициента усиления для системы импульсного регулирования,' рассмотренной в 'задаче 362, соответствующее границе устойчивости. . . Решение. Характеристическое уравнение системы может быть получено из формулы (2) задачи 362: (z-l)(z-d) + yT0K(l-d)z = = z*-[l + d-\T0K{l-d)]z + d = 0. Перейдем к to-преобразованию подстановкой z = = . _ ^-. В результате получим характеристическое уравнение в другом виде, l2.+ 2d-yT0K(\-d)]w2 + 2(l-d)w + \T0K;{l-d) = 0. Условия устойчивости: 2 + 2d-\T0K{l-d)>0, (1) d<\, (2) Y7VC(l-rf)>0. (3) Критическое значение коэффициента усиления может быть определено из формулы (1) К = 2('+<*> ^ М! ^3240 сек-' - р уТо (1 - d) 0,1 • 0,05 - 0,22 °^U СвК ш 367. Характеристическое уравнение импульсной системы регулирования v 5z3+2z24-3z+l=-0.
33$ ;'■' РЛ- 10. ИМПУЛЬСНЫЕ СИСТЕМЫ [ЗИ Определить устойчивость системы. Решение. Используем подстановку г= _w ■. Тогда получим 5(1 + wf + 2(1 + ш)2(1 - ю) + + 3(1 + ш)(1 — йу)2 + (1 — ау)3 = 0. Приведение подобных членов дает 5ву3+13ву2+11ву + 11=0. Применим критерий Гурвица 13- 11—5-11=88>0. Система устойчива. 368. Определить устойчивость системы, характеристическое уравнение которой z2-hz2 + z + l=0. Ответ. Система неустойчива. 369. Передаточная функция замкнутой системы -1,52 + 0,6 ' Определить первые коэффициенты ошибки с0 и с{ при 7'0 = 0,1 сек. Решение. Находим передаточную функцию относительно ошибки: Ф (z) = 1 - Ф (z) = z*~ 1,52 + 0'5 Заменяя в последнем выражении г=1, что соответствует р — 0, получаем с0 = Фж(1) = 0. Для нахождения коэффициента ошибки с, продифференцируем передаточную функцию Фх(ерТо): dOx(epT") (2Гог-1^Го)М. dp (2»--1,5г+а(6)«*- ,
371] § 50.2. УСТОЙЧИВОСТЬ И" КАЧЕСТВО ИМПУЛЬСНЫХ СИСТЕМ 339 При подстановке в последнее выражение, z = 1 получаем Cl " (1-1,5 + 0,6)* b7 о -".О сел- 370. В импульсной системе регулирования коэффициенты ошибки равны с0=0, Ci = 0,01 секи с2=0,05 сек2. Определить ошибку в дискретные моменты времени t= = пТ0 {п — 0, 1, 2, ...) при поступлении на вход Системы сигнала g(t) — aQt-\-a{t2, где о0 = 5 сек'1 ио, = 2 сек^2. Ответ. Значение ошибки в дискретные моменты времени равно x(nT0)=clg(nT0)+-%g(nT0) = = cI(fl0 + 2a,n7,0) + -f-.2^ = = 0,01 (5 + 4пТ0) + 0,05 • 2 = 0,15 + 0,04иГ0. 371.. Определить запас устойчивости по модулю и- по фазе, а также показатель колебательности импульсной системы регулирования, передаточная функция которой в разомкнутом состоянии приведена в задаче 362 i Is. где Y = 0,1, Г0 = 0,05 сек, К =100 сек"1, d = e г'= = e-o.25 = 0>78> a J, = o,2 сек: Решение. Перейдем к частотной передаточной функции подстановкой z~-r——, а затем хю=-\%,-^- (см. задачу 360), твппх. Ю[1+0.025гЯ21 ... WW~ А(1+;0,217Л) ' {1> • Определим частоту среза для передаточной функции (1) из равенства 10{1 + 0,0252я^1 =1 (2) AcpYl +0,217%
340Г'l1 ГЛ. 10. ИМПУЛЬСНЫЕ СИСТЕМЫ [373 Приближенное решение (2) дает Запас по фазе для (1) равен ц = 180° + ф = 90° - arctg 0.217Я. Запас устойчивости по фазе при Я = Яср равен ц, = 90° - arctg 0,217 • 6,8 = 34°. Фазовый сдвиг достигает значения гр = — 180° прн Я-»-оо. Поэтому запас устойчивости по модулю о : 1 _ 0,217 _ 35_ I" | W{j°°)\ Ю-0,0252 Для определения показателя колебательности найдем частотную ' передаточную функцию замкнутой системы w\]Л) - l + w (j%) _ 0j2]7^2 + /я + 10 ' Модуль последнего выражения равен 1 ' 1^( 10 — 0,217Я.2)2 + Я.2 * ' Исследование на максимум выражения (3) дает значение показателя колебательности Атах = М = 1,56. • 372. На вход импульсной системы с передаточной функцией в замкнутом состоянии ^^' 2*-1Дг + 0,4 поступает единичная ступенчатая функция g (t)~ l(t). Построить переходный процесс для выходной величины у(пТ) и определить время переходного процесса. Период дискретности 7'о = 1 сек. 'Решение. Изображение входной, величины равно C?(z)—jrr. . (1)
3721 § 10-2. УСТОЙЧИВОСТЬ И КАЧЕСТВО ИМПУЛЬСНЫХ СИСТЕМ 341 i Изображение выходной величины примет вид Н*) = Ф(г)С(гН(г_1)(Л1,3* + о.4Г <2) Найдем корни характеристического уравнения z2—1,32 + 0,4 = 0. • Решение дает zx = 0,8 и z2 = 0,5. Представим выражение (2) в виде ^HtVt^ + т^Ы- Нахождение коэффициентов разложения на простые дроби дает Л=1, В = — 1,67 и С = 0,67. В результате имеем Для приведения (3) к табличному виду (см. приложение 2) положим йх = е~а'г» = zx = 0,8 и й2 = е-а*т° = = z2 = 0,5. Отсюда находим а, = ~ In тпг = 1 ■ 0,223 = 0,223 сект1, 1 о U,o а2 = ~ In ^ = 1 • 0,693 = 0,693 сек-1. 1 о ",о Таблица 3 п 0 1 2 3 4 5 6 7 8 , —1.67-0,8" -1,67 -133 -1,06 -0,85 -0,68 -0,54 -0,43 -035 -0,28 0,07-0,5" 0,67 0,33 0.16 0,08 0,04 0,02 0,01 . 0 0 У (пТо) 0 0 0,1 0,23 0,36 0,48 0,58 0,65 0,72 п 9 10 11 12 13 14 15 16 —^ч —1,67-0,8" -0,22 -0,18,, -0,14 -0,11 -0,09 -0,07 -0,06 -0,05 0,67-0,5" 0 0 0 0 0 0 0 0 J/(nr0) 0,78' 0,82 0,8> 0,8Э 0,91 0,93 0,94 0,95
342 • ГЛ. 10. ИМПУЛЬСНЫЕ СИСТЕМЫ [37S В соответствии с приложением 2 получаем искомую дискретную функцию времени у (ПТ0) = 1-1,67е-а'"г° + 0,67е-а*пГ° = = 1-1,67z? + 0,67z£ = 1 ^- 1,67 • 0,8" + 0,67 • 0,5". (4) Для построения переходного процесса удобно воспользоваться табл. 1. Вычисление у(пТ0) производилось в таблице до тех пор, пока ошибка не стала равной 5%. Время переходного процесса составляет при этом ^п = 167/"0 =16 сек. 373. Решить предыдущую задачу разложением изображения в ряд Лорана. Решение. Найденное - изображение выходной величины (2) разложим в ряд Лорана делением числителя на знаменатель 0 1г I z3 — 2,3г2 + 1,7г— 0,4 О',}г - 0,23 + 0,17г-' - Q,Q4z-2 |o,Iz-2+0,23z-3+0,36z-4'+... О.гЗ-ОЛГг-' + ОДОг"2 0,23 - 0,53г-' + 0,39г~2 - 0,092г-3 О.Збг"1 -ад5г~2 + 0,0922"3 Коэффициенты при г-" у частного представляют собой значение выходной величины у(пТ0)- Таким образом, при га = 0 и п—\ имеем у(0) = у{Т0) = 0. Далее получаем у(2Т0) = 0Л, у(ЗТ0) = 0,23, у(4Т0) = 0,ЗЬ. Продолжая деление дальше, получим числа, совпадающие с приведенными в табл. 1.
РАЗДЕЛ II НЕЛИНЕЙНЫЕ СИСТЕМЫ АВТОМАТИЧЕСКОГО РЕГУЛИРОВАНИЯ ГЛАВА 11 \ СОСТАВЛЕНИЕ УРАВНЕНИЙ НЕЛИНЕЙНЫХ, СИСТЕМ § 11.1. Уравнения нелинейных следящих систем i 374. Составить дифференциальные уравнения и структурную схему электромеханической следящей системы, схема которой изображена на рис. 209. На схеме обозначено: ■&!, -&2 — углы поворота командной и исполнительной осей, '& = '&1 — ■&% — рассогласование (ошибка), # , , 4 г- 1 4 Р *Д, ,_ <?Д ■у "jm 4=£ чз Щ= рм i=^0=# д ®—* > а3 ТГ РУ Рис. 209. Электромеханическая следящая система. ЧЭ — чувствительный элемент (датчик угла рассогласования), У — линейный усилитель, РУ — релейный усилитель, Д — двигатель, Р — редуктор, ТГ — тахогенератор, РМ — рабочий механизм (объект). Исходные данные: крутизна статической характеристики чувствительного элемента kx = 1 ejepad=57,3 в/рад, коэффициент усиления линейного усилителя . k2 = 2,5, постоянная времени линейного усилителя 7Л = 0,05 сек, максимальное напряжение на выходе релейного усилителя
344 ГЛ. II. СОСТАВЛЕНИЕ УРАВНЕНИЙ НЕЛИНЕЙНЫХ СИСТЕМ [374 U3 max = с = 110 в, крутизна статической характери- ■2 в • сек передаточное от- стики тахогенератора &4=10 рад ^Зтах, из ношение редуктора i = 1000, скорость холостого хода двигателя п0 = 6000 об/мин, пусковой момент двигателя Мо=Ю0Г' см, момент инерции всех вращающихся ча- _£ q i ™х стей, приведенный к валу дви гателя, / = 0,008 Г • см • сек2. Влиянием статического момента нагрузки и переходных процессов в цепи якоря двигателя можно пренебречь. Статическая характеристика релейного усилителя изображена —на рис. 210. Зона нечувствительности Ъ = 0,25 в. Решение. По заданной принципиальной схеме составляем дифференциальные уравнения звеньев системы. 1. Уравнение чувствительного элемента Рис. 210. Статическая характеристика релейного усилителя. «I = kxb, ■& = ■&, — ! (О где И] —напряжение на выходе чувствительного элемента. 2. Уравнение релейного усилителя (TiP+\)u2 = k&i, и = щ (2) где «2 ~ напряжение на выходе усилителя, нтг — напря- d жение тахогенератора, Р — -ут-. 3. Уравнение релейного усилителя запишем в следующем виде: u3 = F(u2), (3) где и3 — напряжение, на выходе усилителя, Fiu^) — нели-. нейная функция, заданная статической характеристикой (см. рис. 210). 4. Уравнение исполнительного двигателя. Согласно исходным данным влиянием статического момента нагрузки и переходных процессов в цепи якоря двигателя можно пренебречь. Поэтому дифференциальное
S74| ' § ИЛ. УРАВНЕНИЯ НЕЛИНЕЙНЫХ СЛЕДЯЩИХ СИСТЕМ 345 уравнение двигателя (см. гл. I) можно записать в виде {Тыр+1)р^я = к3и3, (4) где #д_ угол, поворота вала двигателя, Тм — электромеханическая постоянная времени, k3 — коэффициент передачи двигателя. F(u2) Wh к, -+&>-*- Чтг M+L 1Г "з к. WhP% О* "*Р *5=Т 1 Рис. 211. Структурная схема следящей системы. Электромеханическая постоянная времени двигателя (см. гл. 1) Ос . этпс п „по 3,14-6000 п л- - = ' — = 0,008 ■._ 1П^ га 0,05 сек> 7 ы J М„ J 30ЛГ, 30- 100 Коэффициент передачи двигателя £. = ■ 2» JltlQ = ^ 3,14-6000 £/зтах _-30£/зтах 30-110 5; Уравнение тахогенератора 6:Уравнение редуктора *2 = ^Д. 5,73 рад (5) (6) где fe5 = YI=-JQQ0-= 0,001 — коэффициент передачи ре* ■дуктора. По уравнениям (1)—(6) составляем структурную схему системы (рис. 211). Согласно этой схеме дифференциальное уравнение линейной части системы, записанное относительно входной величины нелинейного звена ~и2, имеет вид <?■&■+ЩТыр+Г) р^-* - == kikz{Тыр + 1)f&i - k2k3(k1k5 + к^р)us. (7)
346 ГЛ. П. СОСТАВЛЕНИЕ УРАВНЕНИЙ НЕЛИНЕЙНЫХ СИСТЕМ (S75 После подстановки численных значений параметров получим (0,0025р3 + 0,1р2 + р)и2 = = (7,1V + 143р) О, - (0,143р + 0,82) иг. (8) Уравнение линейной части системы дополняется уравнением нелинейного "звена (3) «з = F ("2). 375. Составить дифференциальные уравнения электро-" механической следящей систе'мы с электромагнитными муфтами и логическим устройством. На схеме системы (рис. 212) обозначено: fy, ft2— углы поворота командной ЗМ ч « =£= «? S=£: РМ Zd~ ТГ §=##{ "t. *£** ' ^тг /У |W#-=# л 1 Рис. 212. Следящая система с электромагнитными муфтами трення. и исполнительной осей, Ь = f}, — f>2 — ошибка системы, ЧЭ — чувствительный элемент, ЛУ — логическое устройство, ЭМ — электромагнитные муфты, Д — приводной двигатель, ТГ — тахогенератор, РМ — рабочий механизм. В этой системе приводной двигатель вращается в одном направлении с постоянной скоростью. Реверсирование исполнительной оси осуществляется переключением муфт согласно командам логического устройства. Для построения логического закона управления (рис. 213) ис-1 пользуется напряжение щ, пропорциональное ошибке системы ■&., и напряжение ытг, пропорциональное скорости вращения исполнительной оси &2- Исходные данные: вращающий момент приводного двигателя, приведенный к исполнительной оси, Мо = = 10 Г • см; момент инерции всех вращающихся частей,
sWl § 11.1. УРАВНЕНИЯ НЕЛИНЕЙНЫХ СЛЕДЯЩИХ СИСТЕМ 347 приведенный к той же оси, 7 = 100 Г • см • сек2; параметры логического устройства (пересчитанные в угол рассогласования и угловую скорость) Ьх = 0,2°, Ь2 = == 0 1 град/сек. Статическим моментом нагрузки и влиянием переходных процессов в электромагнитных муфтах трения можно пренебречь. Решение. Запишем закон равновесия моментов, приведейных к исполнительной оси (статическим моментом нагрузки пренебрегаем): J^-M, (I) Ф=0 -Л1 о Ф=+/ L 1 Ф=-Г р Ф=0 где М — вращающий момент. Уравнение управляющего устройства, включающего чувствительный элемент, тахоге- нератор, логическое устройство и электромагнитные муфты трения, имеет вид М*=М0Ф(®,Ь2), (2) где Ф(&, ^ — нелинейный логический закон, реализуемый в управляющем устройстве и заданный графически на рис. 213. Из уравнений (1)и (2) и рис. 213 следует М0 -М0 Рис. 213. -Статическая характеристика логического • устройства. й2Ф2 dV>. (3) при ■&< — Ьи Ъ2<Ь2у При ®>ЬЬ ,&2> — Ь2, 0 в остальных случаях. Подставим в уравнения (3) численные значения параметров системы. Получим дифференциальные уравнения движения системы 0,1 при ft< — 0,2°, Ъ2<0,\ град/сек, . 0,1 при ft>0,2°,! Ь2> — 0,1 град/сек, (4) 0 в остальных случаях. 376. Составить дифференциальные уравнения и структурную схему следящей системы с переменным демпфи- d;2
348 ГЛ. II. СОСТАВЛЕНИЕ УРАВНЕНИЙ НЕЛИНЕЙНЫХ СИСТЕМ [з7б рованием (рис. 214). На схеме обозначено: fh,<te— углы поворота командной и исполнительной осей, Ф — -&, ~_ — Фг — угол рассогласования (ошибка), ЧЭ — чувствительный элемент (датчик угла рассогласования), РМ — рабочий механизм, У\, У2— усилители, ТГ\, ТГ2— тахо- генераторы, Д — двигатель, Р — редуктор. // > VA_aJ La-aJ La_aJ Рис. 214. Следящая система с переменным демпфированием. Местная обратная связь в этой системе образуется тахогенераторами ТГ\ и ТГг,- которые включены последовательно и встречно. Разность напряжений этих тахо- генераторов щ — и5 складывается с. напряжением ии снимаемым с чувствительного элемента. При большом угле рассогласования ■& напряжение «4 > «5, так как ток в обмотке возбуждения тахогенератора ТГ2 пропорционален углу •&. Поэтому суммарный сигнал на входе усилителя ^1 Иг > «ь что обеспечивает большую скорость нарастания процесса. При малом угле рассогласования «4 < «5 и напряжение на входе усилителя У( «2 < "ь Поэтому система работает с пониженной скоростью, что исключает перерегулирование. При составлении уравнений системы статическим моментом нагрузки и влиянием переходных процессов в цепи якоря двигателя и в обмотке возбуждения тахогенератора ТГг можно пренебречь'. Усилители У4 и Уг считаются ■безынерционными. Решение. Составляем дифференциальные уравнения звеньев системы. 1. Уравнение чувствительного элемента «, = &,#, # = #, — #2, (I)
•576) § I1.I. УРАВНЕНИЯ НЕЛИНЕЙНЫХ СЛЕДЯЩИХ СИСТЕМ 349 где k\— крутизна статической характеристики чувствительного элемента. 2. Уравнение усилителя У\ ы3 == k2u2, и2 = щ + щ — «5, (2) где Ug _ напряжение на выходе усилителя, кг — коэффициент усиления. 3. Уравнение двигателя {Тыр+\)р\ = к^ (3) где Ад—угол поворота вала двигателя, Тм и-&з"~электР°" механическая постоянная времени и коэффициент передачи двигателя. % «^ 9-0 г л, "i ^ V ^J * ч ■\% кг \+3 9 » * Рис. 215. Структурная схема следящей системы- с переменным демпфированием. 4. Уравнение цепи нелинейной обратной связи, состоящей из тахогенератора ТГ2 и усилителя У2, и4 = ktpufo-n, (4) где k4 — коэффициент пропорциональности. 5. Уравнение тахогенератора ТГ\ иъ = k5p%, (5) где kb— крутизна статической характеристики тахогенератора. 6. Уравнение редуктора b2 = k6\, (6) где k6 — коэффициент передачи редуктора. По уравнениям (1) — (6) составляем структурную схему системы (рис. 215). На этой схеме знаком м
350 ГЛ. II. СОСТАВЛЕНИЕ УРАВНЕНИИ НЕЛИНЕЙНЫХ СИСТЕМ 1377 обозначено множительное устройство, выполняющее операцию умножения двух переменных согласно уравнению (4). В соответствии со структурной схемой нелинейное дифференциальное уравнение всей системы имеет вид [7>2 + (1 + k2k3k5) p + kxk£zk6\ О* - - kikzk3k4p (■&, - ■в'г) #2 = ktkji3kg&i. (7) § 11.2. Уравнения нелинейных систем стабилизации 377. Составить дифференциальные уравнения и структурную схему системы автоматического регулирования температуры, схема которой изображена на рис. 216. Рис. 216. Система автоматического регулиро-1 вания температуры. На схеме обозначено: ОР — объект регулирования, ЧЭ— чувствительный элемент (мост с термосопротивлением), РП — поляризованное реле (усилитель), Д.— двигатель, ОВ — обмотки возбуждения двигателя, Р — редуктор, РО — регулирующий орган (заслонка), ПОС — потенциометр обратной связи. Исходные данные: объект представляет собой апериодическое звено первого порядка с постоянной времени Т0 = Ш сек, коэффициент передачи объекта и регулирующего органа й0=10 zpadfpad, коэффнциент.передачи
§ а£. УРАВНЕНИЯ НЕЛИНЕЙНЫХ СИСТЕМ СТАБИЛИЗАЦИИ 351 -аи>с£ max "max (т>ф aw Рис. 217. Статическая характеристика поляризованного реле. чувствительного элемента ft, = 0,25 а-в/град, коэффициент передачи двигателя k2= 2 рад/в • сек, передаточное отношение редуктора i~ 1000, коэффициент передачи цепи обратной связи йос = 2,5 а-в/рад. Статическая характеристика поляризованного реле изображена на рис. 217. Ампер-витки срабатывания реле awcp = 0,5 а-в, максимальное напряжение на выходе релейного усилителя [/„,„1=110 в. Влиянием статического момента нагрузки, переходных процессов в обмотках поляризованного реле и постоянными времени двигателя Тя и Ти можно пренебречь. Решение. По заданной прин- Ut ципиальной схеме составляем дифференциальные уравнения звеньев системы. 1. Уравнение регулируемого объекта (r0p + l)flv, = fc0<j>, (1) где #а — фактическое значение температуры объекта, Ф —угол поворота регулирующего органа. 2. Уравнение чувствительного элемента aw^kft, # = #,-#2, (2) где #1 — заданное значение температуры объекта, ft — ошибка системы. 3. Уравнение релейного усилителя u = F(aw), aw = awl — awoc, (3) где F (aw) — нелинейная функция, заданная статической характеристикой (см. рис. 217). 4. Уравнение двигателя постоянного тока pa = k2u, (4) где а —угол поворота вала двигателя. 5. Уравнение редуктора <р = k3a, где ks~ — ~ коэффициент передачи редуктора. (5)
352 ГЛ. 11. СОСТАВЛЕНИЕ УРАВНЕНИЙ НЕЛИНЕЙНЫХ СИСТЕМ [378 6. Уравнение цепи обратной связи awoc = &осф, (6) где аииол — ампер-витки обмотки обратной связи. .. Структурная схема системы изображена на рис. 218. ±-®L к, F(aw) т<^ц uf 1 ос L аюы 4 ее «о МП Рис. 218. Структурная .схема системы автоматического регулирования температуры. В соответствии со структурной схемой дифференци- гльное уравнение линейной части сиетемы можно записать в следующем виде: (TGp +l)paw = kip&i - k2k3 [koc (T0p +l) + k0ki\u. (7) = Подставим в уравнение (8) численные значения параметров. Получим (I О/?2 + р) aw = 0.25/7U, - (0,05/7 + 0,015) и. (8) Уравнение линейной части (7) дополняется уравнением нелинейного звена (3) u — F{aw). 378. Составить дифференциальные уравнения системы регулирования скорости вращения турбогенератора (рис. 219). На схеме обозначено: со — отклонение угловой скорости вращения вала турбогенератора от номинального значения, г\ — координата положения муфты центробежного механизма, с — перемещение штока золотник-а, | — перемещение цилиндра гидравлического двигателя и связанной с ним заслонки. Исходные данные: объект (турбогенератор) можно рассматривать как апериодическое звено первого порядка с коэффициентом передачи k0 и постоянной времени 7ф чувствительный элемент представляет собой
3781 § 11-2. УРАВНЕНИЯ НЕЛИНЕЙНЫХ СИСТЕМ СТАБИЛИЗАЦИИ 353 ■апериодическое звено второго порядка с коэффициентом передачи fct и постоянными времени Т{ и Т2, k2 = ас . ВС ли и статическая характеристика гидравли- АВ ъ вс k*~ AB ческого двигателя задана графиком (рис. 220). -АА Ъ,Ъг Рис. 219. Система регулирования скорости вращения турбогенератора. Рис. 220. Статическая характеристика гидравлического двигателя, Решение. Запишем дифференциальные уравнения звеньев системы. 1. Уравнение регулируемого объекта (турбогенер атора) (Го/? +1) © = £„!. (I) 2. Уравнение чувствительного элемента (центробежного механизма) (7-,/? +1) (Г^ + 1)Ч = fe,o- (2) 3. Уравнение золотника. Для составления уравнения золотника зададимся направлением отсчета всех координат так, как показано на рис. 219. При произвольном перемещении точек А, В и С получим - a = —k2ii)-k3l. (3) 4. Уравнение гидравлического двигателя запишем в следующем виде: pl = F{a), (4)
354 ГЛ- 11. СОСТАВЛЕНИЕ УРАВНЕНИЙ НЕЛИНЕЙНЫХ СИСТЕМ ГЙ» Где jF (о) — нелинейная функция, заданная статической характеристикой (рис. 220). По уравнениям (1) —(4) определяем дифференциальное уравнение линейной части системы {T0p+l)(Tlp+l){Tsp+l)o-. — (kJilka + kJl. (5) Это уравнение дополняется уравнением нелинейного звена (4) pl*-F(o). (6) 379. Составить дифференциальные уравнения системы автоматического регулирования давления (рис. 221). Лежача Утета FW^k Слаб Рнс. 221. Система автоматического регулирования давления, Рис. 222. Статическая характеристика струйного усилителя и гидравлического двигателя. На схеме обозначено: / —объект; 2 —мембранный измеритель давления (чувствительный элемент); 3 — струйный усилитель; 4 — гидравлический двигатель; 5 — заслонка (регулирующий орган). Исходные данные. 1, Уравнение регулируемого объекта v (T0p+l)pK = ktfp, где рк — отклонение давления, ф — угол поворота заслонки. 2. Уравнение чувствительного элемента
3.80] §.П.2. УРАВНЕНИЯ НЕЛИНЕЙНЫХ СИСТЕМ СТАБИЛИЗАЦИИ 355 3. Статическая характеристика струйного усилителя и гидравлического двигателя изображена на рис. 222. Ответ. Дифференциальное уравнение линейной части системы (7> +1)(ТУ + Т2р + 1) о = - Vi<p. . Уравнение нелинейного звена py = F(o). 380. Составить дифференциальные уравнения интегрирующего привода {рис. 223) при учете нелинейности *-£ Jf щ (Оц ТГ "гтс% Рис. 223. Интегрирующий привод. типа насыщение в усилителе (рис. 224).' На схеме обозначено: У — усилитель, Д — двигатель, ТГ — тахо- генератор. Исходные данные: коэффициент передачи двигателя k2 = 4 рад/в • сек, электромеханическая постоянная времени двигателя Тм — 0,1 сек, крутизна статической характеристики тахогенератора U, '/max Рис. 224. Статическая характеристика усилителя. ' &ТГ=1(Г в • сек/рад, максимальное напряжение на выходе усилителя (/2гаах== = 120 в, ширина зоны линейности статической характеристики усилителя 6 = 0,1 е. Ответ. Дифференциальное уравнение линейной части системы (0,1р + 1)« = (0,1р + 1)«г Уравнение нелинейного звена u2 = F(u\. • 0,04 и2.
ГЛАВА 12 ТОЧНЫЕ МЕТОДЫ ИССЛЕДОВАНИЯ УСТОЙЧИВОСТИ И АВТОКОЛЕБАНИЙ § 12.1. Метод фазовых траекторий 381. Исследовать процессы в электромеханической следящей системе с электромагнитными муфтами трения и нелинейным логическим устройством (см. рис. 212) в режиме стабилизации. В качестве исходных данных принять данные задачи 375. .Решение. В режиме стабилизации угол поворота командной оси #i = 0, Ь2= — ®. -Прн этом уравнение всей системы можно записать в следующем виде (см. задачу 375): 0,1 при #<-0,2°, Ь<0,] град/сек, (1) -0,1 при О>0,2°, Ь>- 0,1 град/сек, (2) 0 в остальных случаях., (3) Найдем уравнение фазовых траекторий для области / (рис. 225). Для этого введем новые переменные .x = ft и y = -fif и уравнение (I) запишем в виде -£-0.1. (4) Для исключения времени / разделим это уравнение на -ут=*У- Получим М^М. (5) dx у ИЛИ ydy = 0,\dx. (6) d2b di2
8821 § 12-f- МЕТОД ФАЗОВЫХ ТРАЕКТОРИЙ 357 Проинтегрировав уравнение (6), получаем уравнение фазовых траекторий //2 = 0,05а- + С„ (7) где Ci — произвольная постоянная. Уравнение фазовых траекторий для областей // и /// получаются аналогичным образом: tf = - 0,05* + С2 - (8) и • ■ -, У = с* (9) Уравнения (7) и (8) представляют собой уравнения парабол, симметричных относительно оси х, параметр Рис. 225. Фазовые траектории следящей системы к задаче 381. которых /7 = 0,025. Уравнение (9) представляет собой уравнейие прямых линий, параллельных оси х. Вид фазовых траекторий изображен на рис. 225. Зададимся начальными условиями процесса. Пусть при7 = 0, х = ■& = 0, у = Ь = 0,2 град/сек. По виду фазовой траектории для заданных начальных отклонений мцжно установить, что переходный процесс заканчивается менее, чем за один период, после чего в системе устанавливаются ^автоколебания. Амплитуда угловых колебаний Оо^ 0,25 и амплитуда колебаний скорости сц,=0,1 град/сек легко определяются по предельному циклу. ,, 382. Исследовать процессы в электромеханической следящей системе с электромагнитными муфтами трения,
358 ГЛ. 12. ТОЧНЫЕ МЕТОДЫ ИССЛЕДОВАНИЯ УСТОЙЧИВОСТИ [383 -Ь О W) Т рассмотренной в задаче 375, при условии, что вместо нелинейного логического устройства в системе используется релейный усилитель. Статическая характеристика F(p) этого усилителя изображена на рис. 226. Значения параметров релейного усилителя: 6 = 0,2°, с=\. Ответ. Фазовые траектории си- , стемы представляют собой параболы, уравнения которых tf^OfiSx + d, f = - 0,05л: + С*. Рис. 226. Статическая характеристика нелинейного звена к задаче 382. Линиями переключения привода являются прямые АВ и CD (рис. 227). При любых начальных условиях изображающая точка удаляется от начала координат. Следовательно, система неустойчива. 383. Исследовать процессы в системе регулирования Температуры (см. рис. 216) при отключенной местной Рис. 227. Фазовые траектории следящей системы к задаче 382. обратной связи. В качестве исходных данных принять данные задачи 377. Решение. В режиме стабилизации температуры можно принять ■&! —0, f>2= — О. При этом уравнения
$ "12.T. МЕТОД ФАЗОВЫХ ТРАЕКТОРИИ 359 звеньев системы можно записать в следующем виде (см. задачу 377): 1. Уравнение объекта регулирования (7-0/7+ 1)#=-£0<р. 2. Уравнение чувствительного элемента awt — kft. 3. Уравнение усилителя (при k0. с — 0) u = F(aWi). 4. Уравнение двигателя постоянного тока ри = k2u. 5. Уравнение редуктора Ф = k3a. (1) (2) (3) (4) (5) Учитывая, что ток в обмотке поляризованного реле пропорционален отклонению температуры ■&, а скорость d<p отклонения регулирующего органа -£■ пропорциональна напряжению и, в качестве входной величины нелинейного звена (поляризованного реле) можно при- соп нять ft, ав качестве выходной — о>л ■О dt (рис. 228). На этом рисунке Ъ = OKicp _05_ _ _(d(P\ _ ft," "* 0,25 -*> a°~\dt)max ~ = fe^^max=0.22 рад/сек. В соответствии с уравнением объекта регулирования (1) и статической характеристикой нелинейного звена (см. рис. 228), уравнения всей системы можно записать в следующем виде! (70р + 1)#=-£оФ, (6) + ©о при О > + Ь, /ир= 0 при | •&!<&, (7) ; I — ©0 ПрИ ^ < — fa Рис. 228. Статическая характеристика нелинейного звена к задаче 383.
360 ГЛ. 12. ТОЧНЫЕ МЕТОДЫ ИССЛЕДОВАНИЯ УСТОЙЧИВОСТИ 1383 Решив уравнения (6) я (7) совместно, получим (Т0р + 1) рЪ = - Mo при «■ > + Ъ, (8) (Т0Р + DP» = &о<»о ПРИ *< ~ ь> (9) (Т0р+1)рЪ = 0 при |#|<6. (10) Рассмотрим уравнение (8): Введем-обозначения х = Ъ, ~тр — У и уравнение (11) перепишем следующим образом: Г0-§г + У=-*ЬРЬ. (12) Для исключения времени из уравнения (12) разделим его н а -зт- = #.. Получим dy 1 kB(u0 ... . л Ах ~ Т0 Т0у * или после разделения переменных ■ dx=-T0dy- Tff" d#." (13) и ^ у + коЩ " у ' Проинтегрировав уравнение (13), получим уравнение фазовых траекторий х = — Т0у + Toko&o In (у + *b©o) + Ci при *> + &• (И) Проделав аналогичные операции с уравнениями (9) и (10), получим для них х= - Т0у - Toko&oЩу — k<fi>0) + С2 при х<- Ь, (15) х^-Т0у + С3 при \х{<Ь. (16) Поставив в уравнения (14) —(16) численные значения- параметрбв, получим '*=- 10# + 221п(2,2 + #) + С, при.х> + Ь, (17) х = - 10#-221п(2,2-#) + С2 при х<-Ь, (18) *=-10# + С3 при \х\<Ь. (19)
S-I2.I. МЕТОД ФАЗОВЫХ ТРАЕКТОРИЙ 361 - По уравнениям (17) —(19) на рис. 229 построен фазовый портрет всей системы. Там же выделена фазовая Рис. 229. Фазовые траектории системы регулирования температуры к задаче 383. траектория, соответствующая начальным условиям: при £==()■& = 5,7°, 0 = 0. По виду фазовой траектории можно установить, что процесс в системе заканчивается немного больше, чем за один период колебаний. Переходный процесс в системе может закончиться в любой точке отрезка АВ. ■ 384. Исследовать процессы в системе регулирования температуры, рассмотренной в задаче 383, при условии, что поляризованное реле обладает статической характеристик кой, изображенной на рис. 230. Численные значения параметров: Ь => 2°, щ = 0,22 рад/сек. Ответ. Уравнения фазовых траекторий в области I, и в области // (рис. 231) следующие: х = - Юу +.22 In (2,2.+ у) + С,, х = - 1 Оу - 22 In (2,2 - у) + С2. -ь со0 1 0 ? i (Оо ъ Рис. 230. Статическая характеристика нелинейного ввела к задаче 384.
362 ГЛ. 12. ТОЧНЫЕ МЕТОДЫ ИССЛЕДОВАНИЯ УСТОЙЧИПОСТИ [38S Переключение привода происходит на прямых АВ и CD. В системе при любых начальных условиях устанавливаются автоколебания. Амплитуда колебаний температуры й0 я* 5°, амплитуда колебаний скорости изменения температуры а$ » 1,2 град/сек. 40 ВГ §jp,zpad Рис. 231. Фазовые траектории системы регулирования темпе- • ратуры к задаче 384. а) р , м 0 щ "г У 0 б) h -ъ М0 ■ 0 F(L ') М0 ъ Рис. 232. Система стабилизации ИСЗ и статическаи характеристика управляющего устройства к задаче 385. 385. Исследовать устойчивость системы стабилизации искусственного спутника Земли (ИСЗ), схема которой изображена на рис. 232 а, б.
§ 12.T. МЕТОД ФАЗОВЫХ ТРАЕКТОРИЙ 363 На схеме обозначено: РО - регулируемый объект (ИСЗ), ЧЭх и ЧЭ2 — чувствительные элементы (датчики угла рассогласования ■& и угловой скорости ■&), У О- управляющее устройство (совместно с исполнительным органом), М — стабилизирующий момент со стороны исполнительного органа, щ и «2~напРяжения на. вых°Де чувствительных элементов. Исходные данные: момент инерции ИСЗ / = 5000 Г -см- сек2, максимальное значение стабилизирующего момента М0 = 500 Г - см, коэффициент передачи датчика угла pf ■ # Рис. 233. Статическая характеристика чув- рассогласования kx = 1 в/град, КОЭф- ствительного элемента фициент передачи датчика угловой к задаче 385. скорости k2 = 1 в • сек/град, ширина зоны нечувствительности управляющего устройства Ъ = = 0,2 в, ширина зоны нечувствительности ЧЭ2 (рис. 233) d = 0,l град/сек, временное запаздывание в управляющем устройстве т = 0,3 сек. Сопротивлением среды вращению ИСЗ можно пренебречь. • Решение. Уравнение объекта запишем в виде dt2 = -М. О) Уравнение управляющего устройства M = F{u), (2) где F {и) — нелинейная функция, заданная'статической характеристикой (см. рис. 232, б). Суммарный сигнал на выходе чувствительных элементов ЧЭ\ и ЧЭ2 u = ui + u2 = kl® + F1{$), (3) где F (■&) — нелинейная функция, статическая характеристика которой изображена на рис. 233. В соответствии с рис. 233 выражение (3) можно записать в следующем виде: ,., ( fe,0 при |<4<d, * kft + k2(6 - d) при ®>d, (4) I kfi + k2{p + d) при #<-d. u = \
364 ГЛ. 12. ТОЧНЫЕ МЕТОДЫ ИССЛЕДОВАНИЯ УСТОЙЧИВОСТИ |38В Переключение исполнительного устройства при отсутствии временного запаздывания (t = 0) согласно у, град/сен х.град Рис. 234. Фазовые траектории системы стабилизации ИСЗ к задаче 385. рис. 232, б происходит при и= + Ъ и и=-.— Ь, или с учетом (4) ■■'*' . при |*|<d, (5) ■& = 0= •&== 0 = ь — ъ — — ъ — ъ *1 Mfl-d) 1 ft2 (* + <*) *i J ■ г . d) } при 0>d, при ft ^ — d. (6) (7)
jBjJ § 12.1 МЕТОД ФАЗОВЫХ ТРАЕКТОРИИ 365 Если принять, что в течение времени т движение ИСЗ происходит с постоянной скоростью, и обозначить х = ■&, #/ = ■&, условия переключения исполнительного органа можно записать в виде х=-|- + ту = 0,2 + 0,Зу, ] 1 ь \ при |y|<d = 0,l, (8) Ь 1 ПРИ £- + т«/ = 0,2-0,30 j ^f-^^ + ^O.S-O^. I ДС-—ij- £ + Tj/=-0,l-0,7yj 0) *> МН-О.-.. no «-Л ПРИ '<-* = = -0,1. (10) a; = - ^ - -^р-+ту= -0.3-0.7y i По формулам (8) —(10) на рис. 234 построены линии переключения (ломаные ABCD и A'B'C'D'), которые разделяют фазовую плоскость на три области. Согласно (1), (2) и рис. 232, б уравнения всей системы будут равны rf2ft dt2 0 = — 0,1 для области- /, 0 для области //, ■ (11) + —^ = 0,1 для области ///. у/А После замены х = Ь, у — -^- разделим уравнения (11) на ~тг~У- Получим у dy = — 0,1 dx для области /, dy = 0 для области //, ydy= + 0,1 dx для области ///,
366 ГЛ. 12. ТОЧНЫЕ МЕТОДЫ ИССЛЕДОВАНИЯ УСТОЙЧИВОСТИ 1386 откуда после интегрирования найдем уравнения фазовых траекторий у2 + 0,2* = Cj для области /, у = С2 для области //, у2~0,2х = С3 для области ///. (12) Первое и третье уравнения (12) представляют собой уравнения парабол, оси которых совпадают с осью х. Второе уравнение (12) есть уравнение прямой. Фазовый портрет системы изображен на рис. 234. Процесс в системе является расходящимся при малых начальных условиях и затухающим при больших начальных условиях. Фазовые траектории сходятся к предельному циклу, свидетельствующему о наличии в системе автоколебаний. Амплитуда угловых колебаний а$ *» 0,3°, амплитуда колебаний скорости a*«0,14 град/сек. 386. Для системы, структурная схема которой изображена на рис. 235 а и б, построить фазовый портрет методом изоклин. *) "cZTp * л, 6) *г U Jbf -*г Рис. 235. Структурная схема и статическая характеристика нелинейной системы к задаче 386. 2 2 Исходные данные: Т\ = 0,5 сек , Тг ■■ 1 сек, k = 1, Решение. Согласно структурной схеме уравнения замкнутой нелинейной системы могут быть записаны- в следующем виде: (Tip +T$p+ l)*2= — kc при *2>0, (Т2р2 + Tip + l) X2 = + kC При Х2 < 0. (1)
S87I § 12.1 МЕТОД ФАЗОВЫХ ТРАЕКТОРИИ 367 „ йх dx* Введем обозначения х = х2, У = ~аГ~ ~м и °ДН0ВРе" менно подставим численные значения параметров. Получим "7jf-=- 2у-2х-4 при л;>0, при х < 0. -f--2ij-2x + i (2) Для исключения времени поделим уравнения (2) на ~- = у. В результате получим dij 2y-f 2x + 4 . ,_ 1 ■лГ-= у—: ПРИ х>0' I _d£= _ 2t/ + 2x- - 4 Лс у при лг<0. 1 О) dy Положим в первом уравнении (3) —- = т, а во вто- dy ром— ~fa—nt и найдем уравнения изоклин: 2х + 4 У = У = при х > 0, п + 2 ПРИ *<9- т + 2 2х-4 (4) По уравнениям (4) для различных значений тип. строим поле изоклин (рис. 236). Наклон фазовой траектории к оси абсцисс для каждой изоклины на рис. 236 показан отрезками прямых, проведенных соответственно под углами arctgm и arctgra. Эти отрезки являются касательными к фазовой траектории. Как видно из рис. 236, при любых начальных условиях изображающая точка стремится к началу координат. Следовательно, исследуемая система устойчива. 387. С помощью метода изоклин построить фазовые траектории и исследовать устойчивость нелинейной системы, свободное движение которой описывается дифференциальным уравнением d^x ' dx -dF-°'5x-dT + x==0-
368 ГЛ. 12. ТОЧНЫЕ МЕТОДЫ ИССЛЕДОВАНИЯ УСТОЙЧИВОСТИ {387 />=-/ Л-А& jm—3 Рис. 236. Изоклины и фазовые траекторий к задаче 38о. m-ff Рис. 237. Изоклины и фазовые траектории к задаче 387.
3881 § ЛТ&.' МЕТОД А. М; ЛЯПУНОВА-А. И. ЛУРЬЕ 369 их Начальныеусловия процесса: при t = О, х = 0, -^-=2,5. Ответ. Система неустойчива. Уравнение изоклин' — х •> ~~ т — 0,5* Фазовые траектории изображены на рис. 237. § 12.2. Метод А. М. Ляпунова — А. И. Лурье 388. Исследовать устойчивость нелинейной автоматической системы, структурная схема которой изображена на рис. 238. <э~*> 11 4 к Г,р+/ U(i л F(u) У" -А Jr-L$ р&4 (1) Рис. 238. Структурная схема системы к задаче 388. Решение. Согласно структурной схеме система описывается следующими дифференциальными уравнениями: Т\Щ + И[ = — Шъ T2$2 + % = F{u), Н = М| — ^ос^2- Приведем уравнения (1) к нормальному виду. Для этого введем обозначения T]i = »i. il2 = 4. % = '9'2, o = «, F(u) = f{a). Получим 4i = 1 k • Тл Tt ЛЗт Й2=— -т-Л2 + -т-/(°)> Т)2- Гг -в ■ у2 *)з = Л2. " О = 11, - £ос%. (2)
370 ГЛ. 12. ТОЧНЫЕ МЕТОДЫ ИССЛЕДОВАНИЯ УСТОЙЧИВОСТИ [388 Система уравнений (2) совпадает с формой второго типа (см. п. 6 приложения 23) при п — 3 и ап = —~- 11 0, а31 = 0, п 1 _ 1 а12 — и, а13 = — -=- , а21 = и, а22 = —ъг-, о23= '2 1 2 %2=1, азз = 0, &i = 0, fc2 = y-. 63 = 0, с,= 1, с2=0, С3= Кос Запишем уравнения (2) в канонической форме (см. приложение 23). Для этого из коэффициентов уравнения составим определитель D{k) = 1 ■X о О 0 i 2 0 1 -Я, — Я(Я+7Г)(Л+^)» <3> и определим корни характеристического уравнения D {к) = 0 Л( = — -j~, % Я2 = "jT"' л3 = 0. Ввиду того, что в характеристическом уравнении имеется один нулевой корень, канонические уравнения записываются в следующем видег (4) лг[ = а,,.Х[ + /(0) х2 — Я2лг2 + f (о), 6 = р,л', + р2х2 — г/ (о). . Определим постоянные г, (^ и (32: г = — (с,/?, + c2b2 + c3fc3) = 0, так как bt = 0, с2 = 0, fc3 = 0. Постоянные Pj и р2 определяются по формулам (п. 10) приложения 23.
■S 12.2. МЕТОД А. М. ЛЯПУНОВА —А. И. ЛУРЬЕ 371 Так как в нашем случае с2 = 0, то согласно формуле (п. 11) приложения 23 требуется определить только Ni(A) и N3(h) по формуле (п. 12) приложения 23 N,(A) = №iM = — W3(A) = №3M = — у; 1 г2 0 1 11 0 k Ti -л 0 1 г,г, (5) Определяем D'(A): - По формуле (п. 11) приложения 23 находим (6) (7) Y2; лчад "ат*—.Г|* Г,-"-Г, и определяем Pi^iYi' Гя-Г,' Р2 - Л2\2 - — jr f^T\ • Для класса нелинейных систем, к которому принадлежит рассматриваемая система, достаточные условия устойчивости имеют вид (см. приложение 24) Г2>0 (8) и Г2> —40, (9) где Г2 = it + X ^ k + k°" ,а _ (Pi-Pa)^!-^) = _ ZkTi + kociTz-Tj) 4Л,Л2 AT 2 Условие (9) приводит к следующему достаточному условию устойчивости рассматриваемой системы: -KJ^'kTt. (Ю)
372 ГЛ. 12. ТОЧНЫЕ МЕТОДЫ ИССЛЕДОВАНИЯ УСТОЙЧИВОСТИ [389 389. Исследовать устойчивость нелинейной автоматической системы, структурная схема которой изображена на рис. 239. F(u) 1 к, Г,р*1 и, fl 1 ГгР Ь и. и 1 |г -|| "3 1 Т9р*1 ч Рис. 239. Структурная схема системы к задаче 389. Ответ. Дифференциальные уравнения системы в канонической форме имеют следующий вид: х1 = Я1д;1Н-/(о), ^ а = р^, + р2*2. где Ki = • J_ г, Pi А,2 =—^г-, а коэффициенты Р[ и р2 равны ■"3 Ь _, ki (T2fei — Т3) р2 Г,Г,(Г4-Г,)" Г, Г, (Г,-Г,)' Условие устойчивости системы Г2> — 40 (приложение 24) может быть записано в следующем виде: 390. Исследовать устойчивость системы, структурная схема которой изображена на рис. 240. Решение. Согласно структурной схеме дифференциальные уравнения системы имеют' следующий вид: Tiili + ы, = — kfi, b = F(u), и = k2Ui + 1г3щ — koc6 Преобразуем систему уравнений (1) к канонической форме, Для этого обозначим t]=*"i, 1 = 6, а = и, Да) = /7(ы) (1)
§ 12.2. МЕТОД А. М. ЛЯПУНОВА-А. И. "ЛУРЬЕ 373 и запишем уравнения (1) в виде а *? k2r\ + kzr\ - kocl, 'Полученные уравнения (2.) имеют форму (п. 12) при (2) ложенйя 24 при п = 3, а1 = у- и равенстве нулю всех остальных коэффициентов d = -~, Co = k2, с, = &3> r = koz бъд, i, */ P('IP*I) и, кзР ъЛ. у ^ uo.c y- -A Э» : / P 6 ' Рис. 240. Структурная схема системы к задаче 390. Определяем корни многочлена 1 > D (Я) = Я2 + aYK = К2 + ~К. Эти корни р авны Я, = — у-, Я2 = 0. Коэффициенты р( и fe определяем по формулам (п. 17) приложения 24. Для этого предварительно находим Д (Я) = с0 + с,Я = k2 + /г3Я, D, (Я) = Я + Gi = Я + у- , d;m=i. С учетом этих выражений = — d Ц*ч) bi ko — • Tt J_ ■Г, =44*2^-*з). Гх , = — rf- й/г-г тг 1 Г, £.& 1«2-
374 ГЛ. 12. ТОЧНЫЕ МЕТОДЫ ИССЛЕДОВАНИЯ УСТОЙЧИВОСТИ 1391 Уравнения системы в канонической форме имеют вид ki = %vxi + f (а), л x2 = f(o), J 6 = Р[Л:, + {52*2 — f-1 Достаточные условия устойчивости (1) и (2) приложения 24 в данном случае имеют следующий вид: P = |i- + r>-o или T* = — kl{k3Tl-kJ + koc>0. Из этого выражения получим окончательно достаточное условие устойчивости в виде koc>kl(k2Tl-k3) при k2Ti>k3. § 12,3. Частотный метод В. М. Попова 391. Структурная схема нелинейной автоматической системы изображена.на рис. 241. Коэффициент передачи линейной части системы и нелинейного звена.k = knkH Рис. 241. Структурная Рис. 242. Нелинейная схема системы к за- статическая характери- даче 391. стика к задаче 391. условно отнесен к нелинейному звену. Определить, при каких значениях k система будет абсолютно устойчива, если характеристика нелинейного звена расположена в секторе (0, k) (рис. 242).
S91] § 12.3. ЧАСТОТНЫЙ МЕТОД В. М. ПОПОВА 37S . . Исходные данные: постоянные времени линейной части системы Г1 = 0,5 сек, Г2 = 0,2 сек, T3 = Q,\ сек. Решение. Частотная передаточная функция линейной части системы имеет вид W № (1 + л»г,) (1 + ;соГ2) (1 + jwT3)' (1) ы-~о иЪ), Ее вещественная и мнимая части соответственно равны U (а>-\ = Re W (М = ' ~ ю* (Г'Га + Г'Гз + ТгТз) (2) * V (со) = lmW (/со) = - ю Ъ + Т>+ У +fTJfl • (3) Введем некоторые функции £/*(/(о) и 1"(/<в) следующим образом: (/*(<o) = ReW(/<D) = 1 — со2 (Tir2 + Г^з + Г2Г3) . (1 + о2Г|) (1 + со2Г|) (1 + со2Г|) * (4) 1/*(<о)=*<о1тГ(/'со) = _ - ю2 (Г, + Т2 + Г3) + (04Г1Г2Г3 (1 + со2Г2) (1 + co2rl)(l + ш2г|) * (5) По выражениям (4) и (5) построим характеристику Г(со) = /[£Г(<о)] (рис. 243) и через точку I — -г-, /0) проведем прямую Попова так, чтобы построенная характеристика целиком лежала справа от этой прямой. Согласно рис. 243 -г «* 0,08. Поэтому система абсолютно устойчива для всех нелинейных характеристик, лежащих Характеристика V* (со) =/[[/* (со)] к зада^ ' че 391.
376 ,ГЛ- 12. ТОЧНЫЕ МЕТОДЫ ИССЛЕДОВАНИЯ УСТОЙЧИВОСТИ [392 в секторе 0<&<12,5 (6) и, в частности, для характеристики релейного типа, изображенной на рис. 242. Таким образом, достаточное условие абсолютной устойчивости замкнутой нелинейной системы сводится в данном случае к выполнению необходимого и достаточного условия устойчивости замкнутой линейной системы, имеющей в разомкнутом состоянии коэффициент передачи, равный k. 392. Структурная схема нелинейной автоматической системы изображена на рис. 244. Проверить выполнение достаточного условия абсолютной устойчивости системы при следующих значениях параметров системы: Т{ = 5 сек, Г2= 1,25 сек, I, = 0,5, |2 = 0,05, коэффициент J7 U+Z£,T,p +TfpZ) (f+ЯЛр - Tjp«) Рис. 244. Структурная схема системы Рис. 245. Нелинейная к задаче 392. статическая характеристика к задаче 392. передачи линейной части системы k„ — 4, коэффициент усиления нелинейного звена ku = ~ = 0,5 (рис. 245). Решение. Коэффициент передачи разомкнутой системы k = KkH = 4 • 0,5 = 2 отнесем к нелинейному звену. Тогда частотная передаточная функция разомкнутой системы будет равна Wt(ja) = (J - ту + 2£iru») (1 - ту+2,w») (1)
« 12.3. ЧАСТОТНЫЙ МЕТОД В. М. ПОПОВА 377 Определяем функции (I - Г2юг)(1 - 7|ю2)- 4|1£27'17>2 V (о) = Re WAH ~^_т2ту + ч2Ту^_т2ту + 4ф.2^ (2) у*(ю) = №1тГл(/ю) = - 2«2 [(1 - rf«>2) g2r2 + (1— rlco2) 6, Г, j - [(1 - 7>2)2 + 4i27>2] [(1 - Г2»2)2 + 4$ТУ\ • (3) По выражениям (2) и (3) строим характеристику у1*(<о) = /[[/*(©)] (рис. 246). На вещественной оси отложим точку с координатами I—^-, /0). Через эту точку можно провести прямую Попова так, что вся построенная характеристика будет располагаться справа от нее.. Следова- ~kj '^ q\JqH19\ ffj ь>=0 Ц*(а) тельно, данная система - будет абсолютно устой- i чивой при заданном k = 2, если статическая * характеристика Рис. 246. Характеристика V *(©)=< нелинейного звена це- =flU* (и)] к задаче 392. ликом располагается в секторе (0, k). Этот сектор заштрихован на рис. 245. 393. Для нелинейной системы, рассмотренной в задаче 392, определить граничное значение коэффициента Ответ. Граничное значение коэффициента передачи 0,3 3,34. 394. Передаточная функция линейной части системы Г(р)== (7-1Р+1)(Г2р+1)* Определить условия абсолютной устойчивости.
378 ГЛ. 12. ТОЧНЫЕ МЕТОДЫ . ИССЛЕДОВАНИЯ. УСТОЙЧИВОСТИ [385 Ответ. Характеристика V*(<о) = /[U*(со)] целиком располагается в. нижней полуплоскости. Через точку I — -г-, /О] при любом 0<&<оо можно провести прямую Попова так, что вся характеристика расположится правее ее. Поэтому система абсолютно устойчива при всех k = kJkH и для всех нелинейных звеньев, статические характеристики которых принадлежат сектору (О, оо), т. е. располагаются в I и III квадрантах. § 12.4. Метод припасовывания , - 395. Исследовать переходный процесс в системе, Структурная схема которой изображена на рис. 247, а, б. а) ^ ■я, щ~п 6) -ъ с 1 0 X, 1' 'I ъ 1 Jj? Рис. 247. Структурная схема еистемы и статическая характеристика нелинейного звена к задаче 395. Исходные данные: k = 2 се/с-1, Ь = 0,5, с = 5. Решение. Согласно структурной схеме дифференциальное уравнение замкнутой нелинейной системы имеет следующий вид: x2 + kF(x2) = 0, (1) где F(x2) — нелинейная функция, заданная статической характеристикой (см. рис. 247, б), причем F(x2) -ч при при ( при F{x2)=-c\ 4 г' [ При г- х2 > + Ь, b<x2< + b, когда х2 < 0, х2<-Ь, Ь<х2< + Ь, когда х2 > 0. (2) (3)
gjSj § 12.4. МЕТОД ПРИПАСОВЫВЛНИЯ -379 Нелинейное уравнение (1) внутри участков (£) и (3) заменяем двумя- линейными уравнениями: x2+.kc = 0, (4) х2 — kc = 0, (5) или, с учетом численных значений параметров х2 + 10 = 0, (6) л:2— 10 = 0. (7) Решения уравнений (6) и (7) имеют вид x2-=-l0t + Cl, (8) х2=Ю* + С2, <9) где С[ и С2 — постоянные интегрирования. . Для определения постоянной С{ на первом участке процесса зададимся начальными условиями: при t = 0 х2(0 = х2 (0)-' Из уравнения (8) находим С, = Сп = х2(0), и решение (8) для первого участка окончательно принимает вид лг21=-10г + л:2(0). (10) Постоянную интегрирования С2 находим из условий равенства начального значения процесса на втором участке и конечного значения процесса на первом участке. При этом учитываем, что скорость на первом участке х21 — —10 < 0 и согласно (2) переход ко второму участку происходит при х21 = — Ь — 0,5 в момент времени t = tb причем из (10) , _ х2(о) + о,5 пп г'~ io ' I11* Поэтому из условия припасовывания в точке t = /] можно записать *21(*1)=-0,5 = *а(*1). (12) Из уравнения (9) с учетом (12) находим постоянную С2 для второго участка процесса С2 = С22=-015-10*1
380 ГЛ. 12. ТОЧНЫЕ МЕТОДЫ ИССЛЕДОВАНИЯ УСТОЙЧИВОСТИ [396 и окончательное выражение для решения на втором участке х22 = 10 (f-fi)- 0,5, *>*,. (13) На этом участке скорость jc22=10>0. Поэтому согласно (3) при х22 = + Ъ в момент t = t2 происходит переход на третий участок, для которого процесс описывается уравнением (8), но при новом значении по- L»_f_^ стоянкой Cl = Cis. Из -4--Д— -Д-1-7Г а~~ уравнений (13) «ЛДЛЛ/ txen t _ Wi+I _ x2(0) + l,5 /www ш ш __V—V—У—v— Условие припасовыва- ния в точке / = t2 имеет Рис. 248. Кривая переходного ВИД процесса к задаче 395. ^(у = 05 = Хя8(^ (Н) где ж23 означает решение (8) для третьего участка процесса. / Из уравнения (8) с учетом (14) находим постоянную С, для третьего участка Са = 0,5+10f2 и окончательное выражение для решения на этом участке процесса *23=-10(f-/2) + 0,5, f>*2. (15) Дальнейшее построение кривой переходного процесса производится аналогичным образом. Переходный процесс для двух значений начальных условий х2(0) = 0,75 и лг2(0) == —0,25 построен на рис. 248. Как видно из этого рисунка, в системе устанавливаются автоколебания с амплитудой А = b — 0,5 и частотой Q=-у-«31,4 сек"1. 396. Для предыдущей задачи определить амплитуду и частоту автоколебаний, если 1)А=1 сек'1, 6 = 0,5, с = 5, 2) k = 1 сек~\ Ъ = 0,25, с = 5, 3)6=1 се/Г1, 6 = 0,25, с =10.
зет) § 12.4. МЕТОД ПРИПАСОВЫВАНИЯ .#81 Ответ. 1) А = 0,5, fi« 15,7 сек"1, 2) Л = 0,25, Q« 31,4 сек"1, 3) А= 0,25, Q « 62,8 сек"1. 397. Для системы, рассмотренной в задаче 395, найти условия существования автоколебаний и аналитические выражения для их частоты и амплитуды. Решение. Дифференциальное уравнение замкнутой нелинейной системы (см. рис. 247) x2 + kF{xJ~0, (1) | при х2 > + Ъ, FOfeH + c при mi_b<Xi< + b когда i2<Qj (2) f при х2< — Ь, F(к2) = - с | при _b<X2< + bf когда ^>0j (3) внутри участков (2) и (3) заменяем двумя линейными дифференциальными уравнениями x2 + kc = 0, (4) х2 — kc = 0. (5) - Находим решения уравнений (4) и (5) х21 = — kct + Си (6) лги = kct + С2. (7) Время t на первом участке условимся отсчитывать от точки, в которой х21=+Ь. Тогда начальное условие для первого участка будет равно х21 = + b при t = 0. Используя его, находим постоянную интегрирования Cj = + b. Отсюда на первом участке х21 =-kct + Ъ. (8) Для второго участка процесса время t будем отсчитывать от точки, в которой х22 = ~ Ь. Начальное условие для второго участка будет х22 = — Ъ при / = 0. (9)
382 ГЛ. 12. ТОЧНЫЕ МЕТОДЫ ИССЛЕДОВАНИЯ УСТОЙЧИВОСТИ [397 Чтобы в системе существовало устойчивое периодическое решение (автоколебания) с периодом Г, необходимо потребовать выполнения единственного условия (так как рассматривается система первого порядка, а характеристика F(x2) симметрична относительно начала координат) *2i(t) = *22(0). (1°) Подставим в (10) значения переменных из (8) и (9). Получим hrT -■^- + Ь=-Ь. (11) Из (11) находим Т_АЬ 0 _ 2я _ nkc n . 1 ~ кс* ы~ Т ~ ЧЪ * Vl> Амплитуда автоколебаний А определяется как максимальное значение величины х21, полученной из уравнения (8) в течение полупериода колебаний. Из (8) очевидно, что «~ I ХЯ Iniax ~ I 2 ^
ГЛАВА IS ПРИБЛИЖЕННЫЕ МЕТОДЫ ИССЛЕДОВАНИЯ УСТОЙЧИВОСТИ И АВТОКОЛЕБАНИЙ § 13.1. Алгебраические способы определения устойчивости и автоколебаний 398. Исследовать устойчивость состояния равновесия электромеханической следящей системы, принципиальная и структурная схемы которой изображены на рис. 209 и 211, при следующих значениях параметров системы: kx = 1 в/град = 57,3 в/рад; k2 = 2,5; k3 — 5,73 рад/в • сек; А4=0 (скоростная обратная связь отсутствует); &5=0,001; Тх — 0,05 сек; Гм = 0,05 сек. Статическая характеристика нелинейного звена изображена на рис. 210. Ширина зоны нечувствительности & = 0,25е, U3mix = с==1Ю в. Решение. По структурной схеме (см. рис. 211) определяем дифференциальное уравнение линейной части системы (см. задачу 374) при ■0,1 (t) = 0 [TiTuP* + (Г, + Тн)р+1) ри2= - hk2k3kbu3. (1) Для нелинейного звена запишем гармонически линеаризованное выражение u3 = [q(a) + ^p]u2t (2) где согласно приложению 28 для нелинейности (см. рис. 210) <7'(а) = 0.
384 ГЛ. 13. ПРИБЛИЖЕННЫЕ МЕТОДЫ ИССЛЕДОВАНИЯ (898 Подставляя значение и3 из уравнения (2) в уравнение (1), получим линеаризованное уравнение замкнутой нелинейной системы [ТхТиР3 + (7\ + TJ.p2+p + kq(а)] и2 = 0, (4) где k = kik2k3k5 = 0,82 сек"1 — коэффициент передачи линейной части системы. Этому дифференциальному уравнению соответствует характеристическое уравнение Г,Гмр3ч-(Гд + Гм)р2 + р + ^(й) = 0. (5) Условия существования в уравнении (4) периодического решения и2 = A sin Ш (6) будем отыскивать с помощью критерия Михайлова. Для этого в характеристический полином L{p)^T1TMp3 + (Tl + TM)p2 + p + kq(a) (7) р = /о, выделим вещественнук шравняем их нулю: Х(о, й) = ^(й)-(7,1 + 7,м)ш2 = 0, подставим р = /о, выделим вещественную и мнимую части и приравняем их нулю: . (8) У(со)= ш(1 — Г,Гмю2). Из второго уравнения (8) сразу найдем искомую частоту периодического решения <i> = Q = -7=L=^= ' ' =^ = 20 сект1. (9) VtJh /0,05.0,05 v ' Подставим это значение в первое уравнение (8) и найдем выражение, -связывающее амплитуду периодического решения а = А с параметрами системы или, после подстановки числовых значений,
gjgj § I3.I. АЛГЕБРАИЧЕСКИЕ СПОСОБЫ 385 Решение этого уравнения дает два значения амплитуды Л, = 0,257 в и Л2 = 2,86 в. Для исследования устойчивости найденного периодического решения воспользуемся приближенным аналитическим условием [25], согласно которому периодическое решение устойчиво, если выполняется неравенство тт-№(ш>о- (») Из выражений (8) находим дХ _ kdqja) _ 4kc 2b2- да да а дХ д® dY да dY де> п «3 У а2 - Ь2 ' = -'2(Г, + Гм)(о, = 0, = 1-'зГ1Гмю2. а^-Ъ, Подставим выражения для частных производных в (11) и одновременно произведем замену о = Q = VTtT*' Получим условие устойчивости периодического решения в виде -^ 2*;z^(i-3)>o я аяу а3 — Ь2 или ' _ а>ЬУ2. (12) Следовательно, из двух найденных значений амплитуды периодического решения Л, = 0,257 в и Л2 = 2,86 в амплитуда Л2 соответствует устойчивому периодическому решению, т. е. является амплитудой автоколебаний. Определим амплитуду и частоту колебаний исполнительной оси системы. Частота автоколебаний одинакова для любой переменной системы, в том числе и для угла в2, и равна Q = 20 сект1. Амплитуда колебаний Л0 согласно структурной схеме (см. рис. 211) равна Ло = -А- = 1^ = 1,14°. ^ kjkt l • 2,5
386 ГЛ. 13. ПРИБЛИЖЕННЫЕ МЕТОДЫ ИССЛЕДОВАНИЯ [36» . Определим зависимость амплитуды и частоты авто- колебаний от параметров системы. Из выражений (9) и (10) видно, что частота автоколебаний Q зависит только от постоянных времени Тх и Тш а амплитуда автоколебаний, кроме того, от коэффициента передачи линейной части системы k и ширины зоны нечувствительности нелинейного элемента Ъ, причем из (10) k = Г,7>/ (а) • (_13> Из формулы (13) с учетом (12) получим критическое 1ициента передачи линейной части системы я6(7'1 + 7'м) ,j„ значение коэс Kv = 0,0057 сек -1 2сТ,Ти • Для заданных значений параметров и _ я • 0.25 (0,05 + 0,05) ft4>~. 2-1I0-0.052 399. Исследовать следящую систему, рассмотренную в задаче 398, при наличии скоростной обратной связи. Коэффициент обратной связи kA =10" в • сек/рад (см. рис. 211). Остальные параметры системы остаются без изменения. Ответ. Состояние равновесия системы устойчиво. Автоколебания отсутствуют. 400. Исследовать устойчивость состояния равновесия системы, структурная схема которой изображена на рис. 249. Если в системе D Я J. к' ? V*' х'.\ мя ** *1 '1-М * РЪР'П "■ X* X. Рис. 249. Структурная схема системы Рис. 250. Статиче- к задаче 400. екая характеристика нелинейного звена к задаче 400. устанавливаются автоколебания, то определить их амплитуду и частоту для переменной хх. Исходные данные: 7'1 = 1 сек, 7*2 = 0,01 сек, ftt=10, kz = 5 сек~\ статиче-
tf\\ § 13-.I. АЛГЕБРАИЧЕСКИЕ СПОСОБЫ 387 -2> екая характеристика нелинейного звена изображена на рис. 250. ■ Ответ. В системе устанавливаются автоколебания с амплитудой А *» 6,3 и частотой Q= 10 сек'1 Xi = 6,3sin I0t. ■ 401. Найти область устойчивого состояния равновесия и область автоколебаний и определить амплитуду и частоту автоколебаний для следящей системы, схема.которой изображена на рис. 209, при учете нелинейности типа насыщения в предварительном усилителе и отсутствии релейного усилителя и обратной связи по напряжению тахогенератора. ■ Исходные данные: постоянная времени усилителя Т{ = 0,1 сек, электромеханическая постоянная времени двигателя 7^= 1 сек, общий коэффициент передачи линейной части системы /гл = 20 сек~1, коэффициент передачи чувствительного элемента kx — 50 в/рад, статическая характеристика усилителя изображена на рис. 251. Исследование провести для й2= 1 и k2 = 2. -^®- Ъ=1в cc=arctgl<2 Рис. 251. Статическая характеристика усилителя следящей системы к задаче 401. V - й 3 ' — А, "t v- -Л "г Хз ЩрЧ) #Д *5 »г Рис. 252. Структурная схема следящей системы к задаче 401. Решение. Составляем структурную схему системы (рис. 252). Согласно этой схеме дифференциальное уравнение линейной части системы при •&i(f) = 0 запишем в виде {Тир+1)рщ= -kAu2, (1) где k^kfak^ :...'.
388 ГЛ. 13. ПРИБЛИЖЕННЫЕ МЕТОДЫ ИССЛЕДОВАНИЯ Ш Дифференциальное уравнение нелинейного звена имеет вид (Ttf+tyih-kyU!. (2) Коэффициент усиления усилителя ky является нелинейной функцией, заданной графически (рис. 251). Поэтому согласно методу гармонической линеаризации запишем для него гармонически линеаризованное выражение ky = 4(a) + ~L.P. (3) где коэффициенты гармонической линеаризации для характеристики с насыщением имеют значения (см. приложение 28) q(a) — k2 при а<[Ь, ,(«) = ^i(arcsinA + AT/T^) при а>Ъ, (4) . 9'(«) = 0. Из уравнений (1) —(3) получим линеаризованное уравнение замкнутой нелинейной системы [(^p+lHT-jp + Op + ferffaflu.-O, (5) которому соответствует характеристическое уравнение Г,Г2р3 + (Г1 + Г2)/э2 + /э + ^(а) = 0. (6) Для отыскания условий существования периодического решения ы, = A sin Ш (7) из характеристического полинома после подстановки р = /о> выделим вещественную и мнимую части и приравняем их нулю.,* Х(а)) = М(а) + (Г, + Г2)ш2 = 0, У(ш) = (й(1-Г,Г2©2). U Частота периодического решения находится из второго уравнения (8) со = Q = г^= = JL_ ~ 3,16 сек'\ (9) ■VTiTM У 1-0.1
4011 § 13.1. АЛГЕБРАИЧЕСКИЕ СПОСОБЫ •389 Из первого уравнения (8) с учетом (9) получим формулу, связывающую ' амплитуду периодического решения с параметрами системы 2k4k-2 I . Ь , Ь , /' Ь2\ (10) Для исследования устойчивости периодического решения найдем частные производные от выражений (8): (£)'-*■ ( дд(А\ ЗА -0,695, (П) Щ' «= - 20(7-, + г2) = - 2 г;±12 ■ 0*==1-злг2а2=1-з = -2. Для устойчивости периодического решения (7) требуется, чтобы выполнялось не- ' равенство I ^' ' idXV IdY V* ■ \ да ) \де>) чт иг>». или, с учетом выражений (11), Dfi дд(А) дА •°» (13) о \ as /iff is io is зм т е чягтняя пппичиппняя Рис- 253' Зависимость коэффшш- гЗ с/,час1НсШ производная ента гармонической линеаризации У. ' должна быть отри- от амплитуды колебаний к за- оА г даче 401. дательной. Для определения знака этой производной по выражению (4) для q(A) построим график (рис. 253), согласно которому ^-<0 при. А>Ь. (И)
390 ГЛ. 13. ПРИБЛИЖЕННЫЕ МЕТОДЫ ИССЛЕДОВАНИЯ Следовательно, амплитуда периодического решения =(7) будет амплитудой автоколебаний лишь при выполнении условия А>Ь. При А<Ь автоколебания в системе отсутствуют, что и понятно, так как при этом согласно рис. 251 нелинейная система превращается в линейную, которая, как легко показать, устойчива. кгр го 30 40 50 60 к, сея'' А„,рао 0.16 0,12 № 0.04 - \ ■ ! /** ; у J2 IT .ill 8 ю а ь,в Рис. 254. Зависимость изменения амплитуды и частоты автоколебаний от параметров системы к задаче 401. Определим амплитуду автоколебаний. Уравнение (10), связывающее ее с параметрами системы, является трансцендентным. Поэтому для определения амплитуды воспользуемся графоаналитическим методом. Для этого решим уравнение (10) относительно /г = £л&2: k = 27\ ,Г2 j^a resin-j + -j /-*) (15) и построим график k = k(A$) (рис. 254), где А>=£ амплитуда колебаний исполнительной оси системы (см.
4011 § 13.1. АЛГЕБРАИЧЕСКИЕ СПОСОБЫ ■391 рис. 252). Что касается частоты автоколебаний' У, то она' остается неизменной для любой переменной системы и согласно выражению (9) не зависит ot коэффициента к. Граничный коэффициент передачи системы kTV определяется из выражения (15) при А = Ь и равен ^гр Tt + T2 TJ2 1+0,1 0,1 = 11. (16) Автоколебания в системе возникают лишь при k > kTB, Нетрудно убедиться, что граничный коэффициент (16) 1из "г Рис. 255. Статическая характеристика нелинейного звена к задаче 402. #? № 0,6 0,8 /.0 tf Рис. 256. Зависимости изменения амплитуды и частоты автоколебаний от параметров системы к задаче 402. совпадает с коэффициентом передачи, найденным из условия границы устойчивости линейной системы. Но, в отличие от линейной системы, у которой за областью устойчивости лежит область неустойчивости, в системе с нелинейностью типа насыщения за областью устойчивости лежит область автоколебаний, т. е. устойчивых периодических колебаний с вполне определенной амплитудой и частотой. На рис. 254 также изображены графики, связывающие амплитуду и частоту автоколебаний с шириной зоны линейности статической .характеристики нелинейного звена b при &2=1.
392 ГЛ. !3. ПРИБЛИЖЕННЫЕ МЕТОДЫ ИССЛЕДОВАНИЯ [402 Для заданных значений параметров системы по графику (рис. 254, а) определяем частоту и амплитуду автоколебаний: Q = 3,16 сек~\ Ае « 0,022 рад при k2 = 1; Q = 3,16 сек~\ Ае « 0,044 рад при /г2 = 2. 402. Найти область устойчивого состояния равновесия и область- автоколебаний для следящей системы, рассмотренной в задаче 398, если аппроксимированная статическая характеристика имеет вид, как показано на рис. 2.55 при 6 = 0,25 в, с= 110 в. Ответ. Область устойчивости состояния равновесия, область автоколебаний в зависимости амплитуды и частоты автоколебаний от коэффиииента передачи линейной части системы показаны на рис. 256. § 13.2. Частотный метод определения автоколебаний 403. Исследовать устойчивость состояния равновесия нелинейной системы, структурная схема которой изображена на рис. 257, если заданы параметры линейной -ъо с _ i ; К Г с 1 b Рис. 257. Структур- Рис. 258. Статическая пая схема системы характеристика пели- к задаче 403. нейного звена к задаче 403. части системы k = 0,82 сек'1, Tt = 72 = 0,05 сек и статическая характеристика нелинейного звена (рис. 258), для которой & = 0,25, с =110. Решение. Построим амплитудно-фазовую частотную характеристику линейной части системы Wa{j&) 9 к 1г .
4031 § 13.2. ЧАСТОТНЫЙ МЕТОД ОПРЕДЕЛЕНИЯ АВТОКОЛЕБАНИИ 393 и годограф гармонически линеаризованного нелинейного звена -Z(a)=- ^н(а) ■ Согласно структурйой схеме частотная передаточная функция линейной части системы W» № = /а)(1+Г1/о>)(1+Г4/и>) ' ее модуль и фаза to/O + r^l + rlw2) •ф (©) = - 90° - arctg ©Г, - aretg ©7*2. После подстановки численных значений параметров получим l^^h^in^, О) ф (to) = - 90° - 2 arctg 0,05©» (2) Задаемся значениями © от 0 до оо и по формулам (1) и (2) строим амплитудно-фазовую характеристику линейной части системы Wa(ja) (рис. 259). Рис. 259. Частотные характеристики линейной- части системы и нелинейного звена к задаче 403. . Гармонически линеаризованная передаточная функция нелинейного звена согласно приложению 28 равна W,{a)-q{a) па г I £,. °>Ь.
394 Г ГЛ. 13. ПРИБЛИЖЕННЫЕ МЕТОДЫ ИССЛЕДОВАНИЯ [403 Отсюда г, / \ 1 эта2 1 — Z (а) — — , ■ . ' Wa(a) 4с VV-fc2 После подстановки численных значений параметров нелинейного звена получим -Z{a)=- — -7==L=. (3) 440 JV- 0,0625 ' Задаемся значениями а от а = b = 0,25 до °о и строим годограф нелинейного звена — Z (а) (рис. 259). В данном случае этот годограф совпадает с отрицательной вещественной полуосью и имеет две ветви. Минимальное значение модуля функции —Z(a) |Z(o)|mIn = -^=-f^f-RjG'0036 достигается при а — Ь \г2 =» 0,352. Годографы Wn(ja) и —Zip) пересекаются в двух точках. Это означает, что уравнение имеет два периодических решения, x = Ai sin Ш, 1 где согласно рис. 259 О = 20 сек'1, Л, = 0,257, Д. = 2,86.. Для устойчивости периодического решения требуется, чтобы амплитудно-фазовая характеристика линейной части системы Wn(ja) охватывала часть годографа — Z (а), соответствующую ■ меньшим амплитудам. Поэтому первое из решений (4) является неустойчивым, а второе — устойчивым. Следовательно, в системе устанавливаются автоколебания с амплитудой Л = 2,86 и частотой Q = 20 сек-1: -- х = 2,86 sin 20/.
«Я § 132. ЧАСТОТНЫЙ МЕТОД ОПРЕДЕЛЕНИЯ АВТОКОЛЕБАНИЙ 895 404. Решить предыдущую задачу, если: 1) k = 2 сек-1, Т\ = 0,05 сек, Т2 = 0,02 сек, Ъ = 0,25, с = 110; 2) k = 4 сек"1, Г, = 0,01 сек, Г2 = 0,08 сек, Ъ = 0,25, с = 110; 3) /г = 0,5 сек'1, 71 = 0,1 сек, 7^ = 0,01 сек, Ь = 0,5, с = 110; 4) /г = 2 сек'1, Г, = 0,05 сек, Г2 = 0,05 сек, Ь = 0,1, с =40; 5) k = 2 сек"1, Г, = 0,05 сек, Т2 = 0,02 сек, Ъ = 0,25, с = 11. Огеет. 1) А «4, fi«31,6 се/Г1; 2) Л ~ 5, й « 35,3 сек'1; 3) Система устойчива, автоколебания отсутствуют; 4) А « 2,55, Q « 20 сек~и, 5) Система устойчива, автоколебания отсутствуют. 405. Исследовать устойчивость состояния равновесия системы, структурная схема которой изображена на У *f Щр+1). ,\г J\ X -4 J/ -b А г— А b, fy Риг. 260. Структурная схема системы к задаче 405. х Рис. 261. Статическая характеристика нелинейного звена к задаче 405. рис. 260, если заданы параметры линейной части системы #1 = 10 сек""1, Г, = 0,1 сек и статическая характеристика нелинейного звена (рис. 261), для которой #1 = 0,1, &2 = 0,3, fc = tga = 5.
396 ГЛ. IS. ПРИБЛИЖЕННЫЕ МЕТОДЫ ИССЛЕДОВАНИЯ [4в6 Ответ. Годографы №л (/и) и — Z {а) не пересекаются. Следовательно, состояние равновесия системы устойчиво, автоколебания отсутствуют. 406. Исследовать устойчивость состояния равновесия •электромеханической следящей системы, структурная схема которой изображена на рис. 211, если заданы параметры, линейных звеньев: ft, == 57,3 в/рад, £2 = 2,5, k3 — 5,73 рад/в • сек, kA = КГ2 в • сек!рад, k5 = 0,001, Г, = = 0,05 сек, Тм = 0,05 сек, и статическая характеристика нелинейного звена (см. рис. 210), для которой Ъ — 0,25 в, U3max = c= ПО в. Решение. По заданной структурной схеме определим передаточную функцию линейной части системы ^л(р) и гармонически линеаризованную передаточную b P(hi)*t) «3 J ил rlFl a г Ml 1 ^m/ \пч %~ 1*1 *7 a, u 1 I b<n % h hp+t Ф +* ■ Рис. 262. Преобразованная структурная схема » к задаче 406. /S 1), ■■ функцию нелинейного звена Wa{a). Для этого структурную схему нелинейной системы представим в виде последовательного соединения нелинейного звена и линейной части системы (рис. 262). Согласно рис. 262 WAP)' kjk3 (ktp-f kiks) k(xp+ 1) р(7>+1)(7>+1) ~ p(TlP+l)(Tup+l) (1) где k = kikskjir, — коэффициент передачи линейной части 4 постоянная времени. системы, т. k ikb Гармонически линеаризованная передаточная функция нелинейного звена, имеющего однозначную статп-
40ft] § 13-2. ЧАСТОТНЫЙ МЕТОД ОПРЕДЕЛЕНИЯ АВТОКОЛЕБАНИЙ „897 ческую характеристику, может быть записана в. виде Wa(a)=q(a), (2) где (?(«)= ШУ * ~lf ^M' пРиложение 28)- По передаточной функции (1) определяем частотную передаточную функцию ■ fe(l+/Q>T) _ 0,82(1+0,174/0) (3) W,R № ~~~ /со(1+/аГ,)(1+/св7-м) ~ jm (1 + 0,05/го)2 » ее модуль ,»/,,л| 0,82У1+0,0305д)2 . и фазу г|з(со) = - 90° + arctg0,174c»-2 arctg0,€5©. (6) ' По формулам (4) и (5) строим амплитудно-фазовую характеристику линейной части системы (рис. 263) и годограф нелинейного звена -Z(a)=- 1_ Wa(a) яа2 ■ Ш_^Ш1оЧт п 1 4с /а*— 6 яа2 ! 440 У а2 -0,0625 (6) fta~5Q Рис. 263. Частотные харак- Ы при значениях амплитуд Ь< . „ 5 v. г, J теристнки линейной части < а < <». В данном случае этот си£темы н нелинейного звена годограф совпадает с отрица- к задаче 406. тельной вещественной полуосью и имеет две ветви. Минимальное значение модуля функции — Z (а) пЬ я -0,25 Z(a)\m 'mln ' 2с 2-110 0,0036 достигается при а = b У2~*« 0,352 е. Как видно из рис. 263, годографы WAi®) н — Z(a) не.имеют общих точек пересечения. Следовательно, состояние равновесия рассматриваемой системы устойчиво.
308 ГЛ. 13. ПРИБЛИЖЕННЫЕ МЕТОДЫ ИССЛЕДОВАНИЯ |407 .- 407. Исследовать устойчивость состояния равновесия нелинейной системы, структурная схема которой изображена на рис. 264. Параметры линейной части системы: Г] -1,0 сек, Т2 = 0,9сек, Г3= 1,1 сек, /г, = 0,5, /г2 = 5. ъ ■* г к, Т,р*1 л, V ■А \Х-9 (ггр*тр+» 4» Рис. 264. Структурная схема системы к' задаче 407. Статическая характеристика нелинейного звена изображена на рис. 265, где Ь — \, ^3 = 4. • -ь о ( 1 /Г Л \6 jc, arctgk3 -20 №ш Рис. ,265. Статическая Рис. 266. Частотные характери- характеристи'ка иелиней- стики линейной части системы ного звена к задаче 407. и нелинейного звена к задаче 407. Ответ. В системе устанавливаются автоколебания ,v'i = A sin Ш, где А^ 1,8, Q— 1,7 сек~К Амплитудно-фазовая характеристика линейной части системы \^л(/и) и годограф нелинейного звена — Z (а) показаны на рис. 266. 408. Исследовать устойчивость состояния равновесия нелинейной системы, структурная схема которой изображена на рис. 267. Параметры линейной части системы: /г,=2, /г?=10 сект1, Т = 0,02 сек, £ = 0.15. Статическая характеристика нелинейного звена изображена на рис. 268. ...
m] § 13-2. ЧАСТОТНЫЙ МЕТОД ОПРЕДЕЛЕНИЯ АВТОКОЛЕБАНИЙ 3§9 Ответ. В системе устанавливаются автоколебания х{ — As'mQt, I где А ~ 8,5, £2 « 50 сек~1. Амплитудно-фазовая характеристика линейной части системы W„(ja) и годограф нелинейного звена — Z (а) изображены на рис. 269. * -Ни—* А, •г. * •ЗСр рсг'р* г(тр*1) Рис. 267. Структурная схема системы к задаче 408. " "0 5,0 . •%г 5.0 Л/ Рис. 268. Статическая характеристика пелинейиого звена к задаче 408. Рис. 269. Частотные характеристики линейной части- системы и нелинейного звена к задаче 408. 409. Исследовать устойчивость состояния равновесия электромеханической следящей системы, рассмотренной в задаче 406, если статическая характеристика нелиней? ного звена имеет петлю гистерезиса (рис. 270). Решение. Согласно структурной схеме (см. рис. 262) передаточная функция линейной части системы при исходных данных задачи 406 равна feptp+l) 0,82 (0,174?+1) Wa{p)- p(TlP+l)(TMp-tl) р(0,05р+!)2 0)
400' ГЛ. 13. ПРИБЛИЖЕННЫЕ МЕТОДЫ ИССЛЕДОВАНИЯ 1409 Частотная передаточная функция нелинейного звена, имеющего неоднозначную. статическую характеристику, может быть записана в виде Wa(a) = q(a) + jq'(a), (2) где для релейной характеристики с гистерезиснои петлей (рис. 270) в соответствии с приложением 28 \ (3) Годограф нелинейного звена — Z (а), построенный по выражению ■у/ \ • яа /-1 f \ Ь* , . b\ (4) изображен на рис. 271. На этом же рисунке построена амплитудно-фазовая характеристика ' линейной части -д "з ЫЮВ ЪЧ&56 Рис. 270. Статическая характеристика нелинейного звена к задаче 409. Рис. 271. Частотные характеристики линейной части системы и нелинейного звеиа к задаче 409. системы й^ (/со), которая полностью совпадает с построенной на рис. 263. Точка пересечения этих двух кривых определяет периодическое решение в системе u2 = AsmQt, (5)
41Ц §13.2 ЧАСТОТНЫЙ МЕТОД ОПРЕДЕЛЕНИЯ АВТОКОЛЕБАНИЙ 401 К ■ где Л»0,73 в, Q == 100 се/с""1. Согласно рис. 271 найденное периодическое решение является устойчивым, т. е. А и Q являются амплитудой и частотой автоколебаний. 410. Исследовать устойчивость состояния равновесия системы, структурная схема которой изображена на рис. 272, если задан коэффициент передачи линейной части системы k= 10 сек~х и статическая характеристика нелинейного звена (рис. 273), для которой 6 = 0,5, с = 10. ilm W У к р * X / с -ъ 1 0 1 1 ъ 1 с Зх О -0,03-0,01 -Z(a) Л со~0] Re GJ=oo -aot -0,02 -о,оз -0.04 Ч(М Рис. 272. Структурная схема системы к задаче 409. Рис. 273. Статическая характеристика нелинейного звена к задаче 410. Рис. 274. Частотные характеристики линейной части системы и нелинейного звена к задаче 410. Ответа В системе устанавливаются автоколебания с амплитудой А = b = 0,5 и частотой Q = 255 сек'1. Амплитудно-фазовая характеристика линейной части системы Wn(j(o) и годограф нелинейного звена —Z(a) изображены на рис. 274. 411. Решить ту же задачу, если 1) 2) 3) Ответ. 1) 2) 3) - . k k k = 2 сек'1, = 1 сек~\ = 1 се/с-1, А = 0,5, А = 0,25. А = 0,5, b — 0,5, с = 5; & = 0,25, с = 5; b = 0,5, с = 5. Q = 25,5 сек~1; Q = 25,5 се/с"1; £2 = 63,7 сект1.-
ГЛАВА 14 ОЦЕНКА КАЧЕСТВА НЕЛИНЕЙНЫХ СИСТЕМ § 14.1. Исследование колебательных переходных процессов аналитическими методами 412. Для электромеханической . следящей системы, структурная схема которой изображена на рис. 2il, построить диаграмму качества переходного процесса по коэффициенту передачи разомкнутой системы k = «= kik2k3ks при отключенной местной скоростной обратной СВЯЗИ (&4 = 0). Исходные данные: Г, = 0,05 сек, Гм = 0,1 сек, ширина зоны нечувствительности релейного усилителя (см. рис, 210) 6=1 е, максимальное напряжение на выходе релейного усилителя U3max — с =100 е. Решение. Согласно структурной схеме (см. рис. 211) передаточная функция гармонически линеаризованной разомкнутой системы равна W^a' р{Т1Р+1)(Тыр+1)> (1} 4с / Ъ '> где с/{а)= — V 1 г — коэффициент гармонической 3XCL V С1 линеаризации для релейной характеристики с зоной нечувствительности (см. приложение 28). По передаточной функции (1) определяем характеристический полином замкнутой нелинейной системы ** D {р, а) = Г,Гмр3 + (Г, + Ты) pt + p + kq (a). (2) Для построения диаграммы качества в полиноме (2) произведем подстановку /? = £ + /© (£ — показатель затухания). Эту подстановку удобно выполнять путем
4121 $ 14.Г.-АНАЛИТИЧЕСКИЕ МЕТОДЫ1 403 разложения полинома D(p, а) в р'яд по степеням /ю £(£+/©) = * где индекс | означает, что в выражения для производных необходимо подставить | вместо р. Из (2) находим D № = Г,ГИ&8 + (Г, + Гм) |2 +1 + А9 (а), 3! \ dp3 k = ' м" (4) Подставим выражения (4) в ряд (3), выделим в последнем вещественную и мнимую части и приравняем их нулю. Получим Х(ш, а, I) = Т{ТЫ? + (Т1 + Ты) l2 + l + kq (a) - - [ЗГ,ГМ| + (Г,+ ГМ)| со2 = О, (5) У (со, |) = [ЗГ,ГМ|2 + 2 (Г, + Тм) I + 1 - Г,7>2]со = 0. Из уравнений (5) исключим частоту колебаний ©. Для этого из второго уравнения (5) находим 1 [ЗГ.Г „б2 + 2(Г1 + 7\Л+1]. (6) ш2 = - У^м Подставим значение о2 в первое уравнение (5). Получим TJJf + ^ + TJg + Z + kqia)- ^-fY-[STlTMl2 + 2(Ti + TM+ ^[ЗГ.Г^ + СГ.+ТЛ. (7) 'I'M Уравнение (7) решаем относительно /г. В результате находим k-jk{ik[3TiT^+2{Ti+T-n+l]x ■ " X №?£ + (Г, + Ты)\ -ТРЛ*- (Г, + 7\,)12-|}, (8)
404 ГЛ. 14. ОЦЕНКА КАЧЕСТВА НЕЛИНЕЙНЫХ СИСТЕМ 1412 В выражение (8) подставим численные значения параметров. Получим И = па* 400}fa2 - 1 (0,04|3+1,2|2+Щ + 30). (9) Задаемся различными значениями амплитуды колебаний а и при выбранных постоянных значениях показателя затухания £ строим кривые а — a (k) (рис. 275), а, град Рис. 275. Диаграмма качества переходного процесса к задаче 412. Согласно формуле (6) частота колебаний ю не зависит от k й а. Поэтому кривые со = const будут совпадать. с кривыми £ = const. Кривая % = О, со = V~rtTM 14,2 сект} соответствует автоколебаниям в системе и представляет собой зависимость амплитуды автоколебаний а»Л от коэффициента передачи разомкнутой системы k. Кривые £>0 соответствуют расходящимся колебаниям, а кривые £ < 0 — затухающим колебаниям. Область, лежащая правее пунктирной, прямой, проходящей через точку fe.=«0,5 сек""1, является областью существования автоколебаний. Область; расположенная левее этой пря-
4MI . § мл;-аналитические методы 405 мой,, является областью устойчивого равновесного состояния системы. 413. Для системы, рассмотренной в задаче 412, исследовать устойчивость состояния равновесия и определить амплитуду А и частоту Q автоколебаний, если 1) /г = 0,4 сек'1; 2) k = 2 сек~х; 3) k= 4 сек'1. Ответ. Согласно диаграмме качества (см. рис. 275) имеем 1) состояние равновесия системы устойчиво; 2) А ~8,3°, Q=14,2 ceic1; 3) /»« 17°, Q=14,2 сек'1. 414. Для системы, рассмотренной в задаче 412, при k = 2 сек'1 определить время затухания переходного процесса tu число колебаний m в переходном процессе за время th величину перерегулирования ап, если начальное значение амплитуды колебаний й0=15°. Решение. Из диаграммы качества переходных процессов (см. рис. 275) видно, что коэффициент затухания | и частота колебаний © во время переходного процесса не остаются постоянными. Выражение для амплитуды колебаний системы имеет вид г jldt а = а0е° ., (1) где а0 — значение амплитуды колебании, определяемое начальными условиями. Согласно рис. 275 при k = 2 сект1 в системе устанавливают автоколебания с амплитудой А #» 8,3 град. Из выражения (1) определяем время затухания переходного процесса tv от амплитуды а0 до амплитуды А '■-/етг- <2) an где £ (а) —аналитическая зависимость коэффициента затухания | от амплитуды колебаний а при заданном k. Зависимость Ъ,(а) имеет сложный вид (см. задачу 412) и поэтому точное вычисление-интеграла (2) затруднено.
406 ГЛ. 14. ОЦЕНКА КАЧЕСТВА. НЕЛИНЕЙНЫХ СИСТЕМ [41* Время затухания переходного процесса можно оценить приближенно следующим образом [25]. Грубую оценку времени tt производим по формуле Sep "о где |ср — среднее значение коэффициента затухания | на интервале изменения* амплитуды колебаний от а = а0 (точка М0 на рис. 275) до а = А (точка Мх на рис. 275). Если принять, что в точке М0 6^ — 1,5, а в точке Л1, 1 = 0, то |ср--^|±о==_0)75- Поэтому * - ' 1пт|-«0,79 сек. (4) 1 -0,75 '" 15 Для более точного определения времени затухания переходного процесса разобьем участок M0Mi (рис. 275) на три отрезка, M0Af2, M2M3, М3М{, и определим значение | внутри каждого отрезка: ii = go= —1,5, |2= — 1Д !з = ~ °>5- Тогда /, 1 1П.£!+ I ]П^ + ' In А, .(5) 61 а0 52 й2 S3 «S где а2 и с3 — значения амплитуд колебаний в точках М2 и М3. Подставим в (5) численные значения параметров. Получим , 1 . 14 . 1 , J0 , 1 . 8,3 „„. .Грубую оценку величины перерегулирования производим по формуле ^ср %- ~e"v (6еР<0), (6) где |ср и ©ср — средние значения величин | и а на участке М0Ми а0 — начальное значение амплитуды колебаний.
41«I ,§ 14Л. АНАЛИТИЧЕСКИЕ МЕТОДЫ 407 По формуле (6) находим (положив шср = = 12,4 сект1} 10,2+14,2 ап — 15е ш 12,4° Грубую оценку числа колебаний m за время переходного процесса производим по формуле откуда га пг "ср 2л1 In ,ср а0 (7)1 12,4 ■ 2я ■ 0,75 8,3 15 1,5. 415. Для системы, рассмотренной в задаче 412, произвести грубую оценку времени затухания переходного процесса и числа колебаний за время переходного процесса, если 1) fe = 2 сек~\ а0 = 2°; 2) k = 3 сек'1, а0=15°; .3) k = 3 сек'1, «о — 4°. Ответ. Согласно диаграмме качества переходного процесса (см. рис. 275) имеем ц 1) tv «* 0,71 сек, m ~ 2; 2) *,»0,68 сек, /и« 1,5; 3) ^^0,71 се/с, m ~ 1,6. 416. Для электромеханической следящей системы, структурная схема которой изображена на рис.211, построить диаграмму качества переходного процесса по коэффициенту передачи разомкнутой системы k ~kik2k3k5 при отключенной местной скоростной обратной связи (k4 = 0), если вместо релейного .усилителя применяется усилитель с насыщением. Статическая характеристика усилителя изображена на рис. 276. Рис. 276. Статическая характеристика усилителя следящей системы к задаче 416.
408 ГЛ. 14. ОЦЕНКА КАЧЕСТВА НЕЛИНЕЙНЫХ СИСТЕМ 1417 Исходные данные: Г1 = 0,05 сек, Гм = 0,1 сек, ширина зоны линейности статической характеристики нелинейного усилителя Ь.~ 1 в, коэффициент усиления на линейном участке fey = tga=100. Ответ. Диаграмма качества переходного процесса изображена на рис. 277. Кривые | = const и со = const совпадают. Границей областей устойчивого состояния равновесия и автоколебаний является пунктирная прямая, проведенная через точку k *« 0,3 сек~1. а, град Рис. 277. Диаграмма качества переходного процесса к задаче 416. 417. Для системы, структурная схема которой изображена на рис. 278, построить диаграмму качества переходного процесса по коэффициенту передачи k. Исходные данные: /гос = 5, Ti==T2 = 0,l сек, с =10. Решение. Согласно структурной схеме передаточная функция линеаризованной нелинейной разомкнутой системы равна (7'lp+l)[p(7'gp+l) + W<?(a)] ' (1)
4171 $ M.I. АНАЛИТИЧЕСКИЕ МЕТОДЫ. 4fl9 4с где q{a) яа — коэффициент гармонической линеаризации для идеальной релейной характеристики (см. приложение 28). б) .д а) * &- i "Jbp Wr' 2®at^ JCs, &OC nffS т О *з Рис. 278. Структурная схема системы и статическая характеристика нелинейного звена к задаче 417. По передаточной функции (1) определяем характеристический полином замкнутой нелинейной системы D\p, а) = Т{Г2р* + (Г, + Ts) p* + + (l+k0Cq(a)Tl)p + q{a){k + k0C). (2) Полином (2) запишем в виде D (/»»" а) = р3 + АУ + Агр + А3, (3) где - _ Tt + T2_ 0,1+0.1 _ рП л1 г.т. nni zu» У, Г. 1J2 0,01 Л2= l+feoc^^L^l+O.lfeoc,(a) =100 + 50 (fl)> Для системы третьего порядка, характеристический полином которой приведен к виду (3), формулы для коэффициента затухания £ и частоты колебаний и можно записать в виде [39] 6 2[Л2 + (Л, + 2£)21 » w с.ч2==__^*__б2 (5) ° Л, + 2| S* W Чтобы | и (о, полученные по этим формулам, определяли в основном переходный- процесс, должно выполняться
410 ГЛ. 14. ОЦЕНКА КАЧЕСТВА НЕЛИНЕЙНЫХ СИСТЕМ |ад условие Л,»16|. (6) Неравенство (6) определяет верхний и нижний пределы для значений %, которые должны подставляться в формулы (4) и (5). Рис. 279. Диаграмма качества переходного процесса к задаче 417. Подставим в (4) и (5) значения коэффициентов Аь ^2, А3, q(a) и полученные уравнения разрешим относительно к и со2. Получим /г = 5 + | + 0Д57с(10 + 5| + 0,8|2 + 0,04|3), (7) 2 12,7 50ft+ 250 б2 ' ,о\ ffl^-T 10 + | -%• <8> Задаемся различными постоянными значениями коэффициента затухания g и по формуле (7) строим кривые й = й(/г") (рис. 279). По формуле (8) строим кривые со = const (пунктирные прямые на рис. 279). С меньшей точностью, но гораздо быстрее эти кривые могут быть построены по формуле ' .2 ^ As со (9)
4181 * § 14.1. АНАЛИТИЧЕСКИЕ МЕТОДЫ 411 кх* -Ь О которая получается из (5) при выполнении условия (6). Из. рис. 279 следует, что на диаграмме, качества переходных процессов можно выделить две области: область устойчивого состояния равновесия при /г<5, где амплитуда колебаний затухает до нуля, и область автоколебаний при k>5. Эти данные полностью совпадают с данными, полученными при исследовании той же системы точным методом Ляпунова — Лурье (см. задачу 388). 418. -Для системы, рассмотренной в задаче 417, построить диаграмму качества переходного процесса, если нелинейное звено (см. рис. 278, а) имеет релейную характеристику с зоной нечувствительности (рис. 280). Рис. 280. Статическая характеристика нелинейного звена к задаче 418. Рис. 281. Диаграмма качества переходного процесса к задаче 418. Исходные данные те же, что и в задаче 41-7. Ширина зоны нечувствительности 6 = 1,0.'"
412 ГЛ. 14. ОЦЕНКА КАЧЕСТВА НЕЛИНЕЙНЫХ СИСТЕМ |41» Ответ. Диаграмма качества переходного процесса отображена на рис. 281. Из этой диаграммы видно, что наличие зоны нечувствительности в релейной характеристике приводит к некоторому расширению области устойчивого состояния равновесия, соответствующей k < 8,25. § 14.2. Исследование колебательных переходных процессов частотными методами 419. Для системы, структурная схема которой изображена на рис. 282, определить качество переходного процесса. а) х, А\ X к 4 -ъ с , i О •Кг ■К Ъ Рис. 282. Структурная схема системы и статическая характеристика нелинейного звена к задаче 419. Исходные данные: А: = 0,5, Ft = 0,1 сек, 6=1, с = 20, Решение. Коэффициент затухания | и частоту колебаний <о переходного процесса в нелинейной системе будем отыскивать путем решения гармонически линеаризованного уравнения WAl + i<»)-WAa)--l (D или WAI + М =='■ - ~^щ =-Z (a), (2) где №л(! + /ю) получается из передаточной функции линейной части системы W„ (/?) подстановкой р — | + /ю, а гармонически линеаризованная передаточная функция нелинейного звена W„(a) — подстановкой р — % + /<о в выражение (3) WAI, «) = ^(«) + £=^9'(«), ©
4i»l § US. ЧАСТОТНЫЕ МЕТОДЫ 413 в результате которой получаем WB (6, а) = q (a) + jcf (a) - Wa (a). (4) Уравнение (1) будем решать графически. Для этого в передаточной, функции линейной части системы W«{P) = WJTT) (5) произведем подстановку р. — | + /со. Получим W„ (t + /со)-Tz + j(u){l + Tll + j&Tl)- (6) Модуль, этой функции АЦ, со) = |ТГд(£ + Уа))| = -77=— * *• (7) У(|2 + со2)[(1 + 7-1|)2 + й)2Г?] и фаза ij)(g, со) =* - arctg-|- - arctg l" ^ . (8) Подставим в выражения (7) и (8) приведенные в исходных данных значения параметров и, задаваясь различными постоянными значениями показателя затухания |, построим серию кривых W„(%, /со) как функции от частоты колебаний со при | = const (рис. 283). На этом же графике нанесем обратную амплитудно- фазовую характеристику нелинейного звена ~-Z(a)^ ~ ~ "w (У ПРИ заданных параметрах звена b и с. Согласно приложению 28 для нелинейной характеристики релейного типа с гистерезисной петлей имеем Отсюда г, > 1 па -, / , Ьг . nb 1лп\ Мнимая часть выражения (10) не зависит от амплитуды е. Поэтому годограф —Z{a) представляет собой прямую, параллельную вещественной оси. Точки пересечения годографов №л (£ + /«>) и — Z{a) определяют решения уравнения (1) при различных
414 ГЛ. 14. ОЦЕНКА КАЧЕСТВА НЕЛИНЕЙНЫХ СИСТЕМ [ОД значениях \. Для каждого постоянного коэффициента затухания \ по годографу — Z (а) определяем соответствующее значение амплитуды колебаний а, а по годографу Wa(g + /co) — значение частоты колебаний о. Амплитуду и частоту периодического решения а = А и g> = Q находим в точке пересечения годографов U^, (g +/со) и — Z (а) при g = 0. Рис. 283. Частотные характеристики линейной части системы и нелинейного звена к задаче 41 &. Согласно рис. 283 при любом k > 0 в: системе существуют периодические решения, соответствующие автоколебанием. При & = 0,5 находим А~ 1,51, Q~8 сек'1. Из рис. 283 видно, что периодическое решение с этими параметрами действительно соответствует автоколебаниям, так как любое увеличение амплитуды колебаний приводит к g<0, а любое уменьшение ее — к g>0. Произведем грубую оценку основных качественных показателей переходного процесса. Пусть начальное значение амплитуды колебаний а —а0 = 2. Время переходного процесса, в течение которого амплитуда колебаний уменьшается от значения ае.= 2 до значения
4191 § 14.2". ЧАСТОТНЫЕ МЕТОДЫ 415 А =1,51, приближенно оценим по формуле 1 , А г-In — 5ср аа (11) В данном случае Л = 1,51, ао = 2, |ср= — 1. Подставим эти значения в (11). Получим tt «= — In —j- = 0,28 сек. Оценку величины перерегулирования произведем по формуле Sep йп ~ аф "tP , (12) где ап—-величина перерегулирования, соср — среднее lm № -0,12411 -Q,O8-0,O6-№-W О р( 1 -i 1 1 1;-- \0,02 0,02 0.04 Щ 0,08 0,1 RS i 4 h\ I ■„ I Рис. 284. Диаграмма качества переходного процесса к задаче 420. значение частоты колебаний, которое согласно рис. 283 равно 7 + 8 „ - _, = 7,5 сек . о, 'ср' 2 Подставим в (12) значения параметров. Найдем _ _1_ йп = 2е л?.3 =2 -0,65= 1,3.
416 ГЛ. 14. ОЦЕНКА КАЧЕСТВА НЕЛИНЕЙНЫХ СИСТЕМ [420 Для грубой оценки числа колебаний за время переходного процесса воспользуемся формулой «ср , А 7,5 1,51 т *» ■»-,.-■ п— *» —5— In —5— ~ 0,34. 2я|ср ав -2я 2 420. Для системы, рассмотренной в задаче 419, построить диаграмму качества переходного процесса и определить основные качественные показатели процесса", если А: = 0,2, а остальные параметры такиеже, как и в задаче 419. Начальное значение амплитуды колебаний ай = 2. Ответ. Диаграмма качества переходного процесса изображена на рис. 284. Амплитуда и частота автоколебаний соответственно равны А -1,54, Q«4,5. Время затухания переходного процесса от а0 — 2 до А = 1,54 ^^0,28 сек. Величина перерегулирования ап я* 0,88. Число колебаний за время переходного процесса m — 0,17.
ГЛАВА 15 ВЫНУЖДЕННЫЕ КОЛЕБАНИЯ В НЕЛИНЕЙНЫХ СИСТЕМАХ § 15.1. Отыскание симметричных одночастотных вынужденных колебаний графическим методом 421. Определить вынужденные колебания системы, структурная схема которой изображена на рис. 285, при синусоидальном внешнем воздействии f{t)-=Bsm<dBt. (1) Исходные данные: k— 10 сек"1, Тх = 0,1 сек, с =10, В = 20, юв = 10 сек-1. J7, о) ■Чй*. 1 г и., к Р(Т,р*П хг 0 0 с 1 с X Рис. 285. Структурная схема системы и статическая характеристика нелинейного звена к задаче 421. Решение. Составим дифференциальное уравнение замкнутой нелинейной системы. Согласно структурной 'схеме дифференциальное уравнение линейной части системы имеет вид (Tip+DpXz^kXi, (2) Нелинейное звено может быть описано уравнением ' jc, = F<*),"" . (3)
418 ГЛ. 15. НЕЛИНЕЙНЫЕ СИСТЕМЫ (421 где нелинейная функция F (х) задана статической характеристикой (рис. 285, б). Подставим (3) в (2) и одновременно учтем, что x2 = f(t)-x. Получим (TlP +\)px + kF (х) = (Г,р + 1) pf (t). (4) Решение для установившихся вынужденных колебаний в системе будем искать в форме х — ав sin (a>J + ф), (5) где «D и ф —искомые амплитуда и фаза вынужденных колебаний. Уравнение нелинейной системы (4) запишем в виде Q(p)x + R(p)F(x) = S(p)J(t), (6) где Q(p) = S(p) = (T1p+l)p, R(p) = k. В уравнении (6) переменную f(t) выразим через переменную х. Для этого запишем f(f) = B sin (ugt = В sin [(a>Bt + ф) — ф].■= = В cos ф sin (aBt + ф) — В sin ф cos (toB* + ф). (7; Из (5) найдем производную рх = ава>в cos (a>Bt + ф) (8) и, подставив (5) и (8) в выражение (7), получим окончательно Подставим значение функции f(i) (9) в уравнение (6). Получим [Q(p)-S(p)-^(cosV—^-p)]x + R(p)F(x) = 0. (10) Произведем гармоническую линеаризацию нелинейности F{x) = q{a)x, (И) где q(a) = коэффициент гармонической линеаризации для идеальной релейной характеристики (см. приложение 28).
4Я} * ■ '§• K.t- ГРАФИЧЕСКИЙ МЕТОД ■ i$19 В выражение (11) вместо амплитуды а подставим искомую амплитуду вынужденных колебаний ав. Получим F{x)^q(aB)x = ~^-x. (12) Из (10) и (12) получаем характеристическое уравнение для первого приближения ' Q (р) - S(p)-£ (cos Ф--^р) +Я(р) 9(с.) = 0. (13). Для отыскания синусоидального решения (5) подставим в (13) р = /юв. Получим Q (/сов) + R (/со„) д (ав) р .-«, п.ч "в S.(/oB) ct. • u^ При получении (14) учтено, что cos q> — / sin q> = e~№. Подставим в (14) выражение для Q(/g>b)> R(jaB) и S(jaB), полученные из (6), и значение коэффициента q {aB) (12): Ack fl-+^(i*rjmj"^w- (15) После подстановки численных значений параметров получим с„- 6,67- 6,67/ = 20е-/(Р. (16) На комплексной плоскости (рис 286) построим годограф Z (ав) = ав - 6,67 - 6,67/, (17) соответствующий левой части уравнения (16), и окружность радиуса В = 20, соответствующую правой части того же уравнения. Точка пересечения годографа Z(«B) и окружности определяет собой решение задачи. По отметкам амплитуды ав на годографе Z(aB) находим амплитуду вынужденных колебаний в системе йв = Лв = 25,2. Фазовый сдвиг ф = 20° определяется по дуге окружности. Следует иметь в виду, что отсчет положительных значений углов, ф в данном случае производится от положительной вещественной полуоси по часовой стрелке, так как в правой части (15) ф' входит с отрицательным знаком.
420 ГЛ. 15. НЕЛИНЕЙНЫЕ. СИСТЕМЫ (422 На рис. 286 видно, что в системе возникают одно- частотные колебания с частотой ©„== 10 сек"1 не при любых значениях амплитуды входного воздействия (1), а лишь при В>Впор. Для определения порогового значения амплитуды входного воздействия Впор проведем окружность, касающуюся годографа Z{aB). Радиус этой окружности и определяет ^пор — 6,67. 422. Для системы, структурная схема которой изображена на рис. 287, определить амплитуду Ав и фазу ф вынужденных колебаний и Z(a^) пороговое значение амплитуды входного воздействия. Исходные данные: k = 10 сек~\ 6=1, с =10, Рис. 286. Графическое построение для определения вынужденных колебаний к задаче 421. амплитуда и частота входного воздействия В = 8, юв = 20 сек'1. ' Ответ. Уравнение для отыскания амплитуды и фазы вынужденных колебаний имеет вид a0-j • 6,36 I /l - -V =8е-"Р. \ «в Графическое построение показано на рис. 288. Согласно этому рисунку Лв ~ 5, ф = 50°, Впор=1. 423. Определить амплитуду и фазу вынужденных колебаний, в системе, структурная схема которой изображена на рис. 285, при наличии временного запаздывания т. Исходные данные: k= \0 сек'1, Г, = 0,01 сек, т = = 0,01 сек, с =10,' частота и амплитуда внешнего воздействия соЕ = Ю сек~}, В = 20. Решение. Линейная часть системы описывается дифференциальным уравнением :*. [Tip+l)px2 = kxl. (1)
Ы) §:-Т5.Г. ГРАФИЧЕСКИЙ МЕТОД 421 С учетом временного запаздывания % Уравнение нелинейного звена запишем в виде Xj = Fx(x)-e-^F(x)t (2) где F (х) — нелинейная функция, заданная статической характеристикой (рис. 285, б). а). ■ «1 f(t) Л х # JCf р -6 0 Рис. 287. Структурная схема системы н статическая характеристика нелинейного эвена к задаче 422.. $ Re 2(Ц) Рис. 288. Графическое построе- Рис. 289. Графическое построение ние для определения выну- для определения вынужденных ждепных колебаний к за- . колебаний к задаче 423. даче 422. Из (1) и (2) определим дифференциальное уравнение замкнутой нелинейной системы (Г,р+1)р* + Л^(дс) = (Г,р+1)р/(0. О) . Вынужденные колебания входной величины нелинейного звена х будем искать в виде х = Д, sin (oe/ + f). (4)
422 ГЛ. .15.- -НЕЛИНЕЙНЫЕ СИСТЕМЫ 1428 Для отыскания синусоидального решения (4) запишем выражение 0в SOtoB) ~ое , (b) где ав и q> — искомые амплитуда и фаза вынужденных Ас колебаний, q (ав) — — коэффициент гармонической линеаризации для идеальной релейной характеристики о) 6) 1Х1 »-0 » 11 X, р(т,НЩА ж? -ь с ' 0 . с ъ -х Рис. .290. Структурная схема системы и статическая характеристика нелинейного звена к задаче 424. (см. приложение 28) при а = ав, a Q(ja>B), /?(/«>„) и S (/<»„) определяются из уравнения (3). Согласно уравнению. (3) имеем Q (/©в) = S (/сйв) = /сйв(1 4- Г,/сйв), R (/юв) = /г. (6) Подставим (6) и значение коэффициента q(aB) в (5). По^ лучим aB + i \kce /<ов(1 + 7,1/<ов) = Ве-1ч>. (7) После подстановки численных значений параметров и умножения числителя и знаменателя второго слагаемого левой части уравнения (7) на сопряженное выражение имеем ' ав - 2,5 - / • 12,3 = 20е~'Ч>. (8) На комплексной плоскости (рис. 289) строим годограф Z(aB) = aB-2,5-i-12,3
4251 • § 15.2. ЧАСТОТНЫЙ МЕТОД 423 и окружность радиуса 20. В точке пересечения по годографу Z(aB) определяем амплитуду вынужденных колебаний ав=Аз ~ 18,2, а по дуге окружности — фазу <р «= 38°. - Пороговое значение амплитуды входного воздействия 424- Для нелинейной- системы, структурная схема которой изображена на рис. 290, определить амплитуду Ав и фазу ф вынужденных колебаний. Исходные данные: k— 10 сек-1, Т{ — 0,01 сек, Т2 — = 0,02 сек, 6 = 4, с =10; амплитуда внешнего синусоидального воздействия В = 20, частота внешнего воздействия ©в= 10 сек-1. Ответ. Д,<*21, <p**_35°. § 15.2. Отыскание симметричных одночастотных вынужденных колебаний частотным методом 425. Для нелинейной системы,, структурная схема которой изображена на рис. 285, определить зависимость амплитуды вынужденных колебаний от амплитуды и частоты внешнего синусоидального воздействия. . Исходные данные: k= 10 сект1, Т\ = 0,1 сек, с=10. Решение. По структурной схеме (рис. 285, а) определяем передаточную функцию гармонически линеаризованной замкнутой нелинейной системы ^КР, а) р(р) i + и^л(р) U7H(р, а) ' V> где: Х(р) и F(p) — изображения по Лапласу ошибки системы x(t) и внешнего синусоидального воздействия f(t), Wji(p) — передаточная функция линейной части системы,' Wh (p. о) ~ передаточная функция гармонически линеаризованного нелинейного звена. Для рассматриваемой системы имеем ^) = ЯГ^П7' (2) Wa{a)~q(a), (3) Ас где q(a) = коэффициент гармонической линеаризации для идеальной релейной статической характеристики (см. приложение 28).
424 ГЛ. 15. НЕЛИНЕЙНЫЕ СИСТЕМЫ 1423 Передаточную функцию (1) запишем в виде Щр, а)- МАр) (4) Ma(p) + WAa) * где Мл(р) = -рг-т-г —обратная передаточная функция линейной части системы. Найдем обратную частотную передаточную функцию линейной части системы млцф i&(l+T,j&) = /ю(1+0,1/с>) к 10 (5) и передаточную функцию гармонически линеаризован ного нелинейного звена 4с 12,7 U«) = (6) а на комплексной плоскости ipuc. 291) по формуле (5) построим обратную амплитудно-фазовую характеристику Рис. 291. Обратная амплнтудио-фазовая характеристика линейной части системы н годограф нелинейного звена к задаче 425. линейной части системы, а по формуле (6) —годограф нелинейного звена — W„{a). По отметкам амплитуды на годографе — Wn(a) определяем амплитуду вынужденных колебаний ав (точка D на рис. 291), а по отметкам частоты на годографе Мя (/со) — частоту вынужденных колебаний, равную частоте внешнего синусоидального воздействия (точка Е
4йв! § 15.2. ЧАСТОТНЫЙ МЕТОД 425 на рис. 291). Из (4) находим «в I «Т» / .•„ „ \ I Щл (/(Ов) | _ -1 w у©,, «в> I ! Мл (/(йв) + Гн {вв) j 0£ £>£ • (Г) где 5-=-.амплитуда внешнего синусоидального воздействия. Выражение (7) определяет связь между амплитудой внешнего воздействия В и амплитудой вынужденных колебаний ав для любого фиксированного значения частоты а>в: £ = -§§«„< (8) Перемещая точку D на рис. 291 при фиксированном положении точки Е находим зависимость ав{В) при W/=C0n8t #=G0nSt со.сеп' 18 20 В '292. Зависимости амплитуды вынужденных колебаний от амплитуды и частоты внешнего воздействия к задаче 425. ©A —const (рис. 292), а перемещая точку F при фиксированном положении точки D, находим зависимость ЯвО»в) при В — const (пунктирные кривые на рис. 292). '426. Для нелинейной системы, рассмотренной в задаче 425, определить зависимость амплитуды вынужденных колебаний ав от амплитуды В и частоты а>в внешнего синусоидального воздействия, если передаточная
426 ГЛ.- 15. НЕЛИНЕЙНЫЕ СИСТЕМЫ 1420 функция линейной части системы ^л(Р)= р(Г1Р+1)(Г2р+1) ' где k = 10 сек~\ Г, == Г2 = 0,1 сек. Ответ. Обратная амплитудно-фазовая характеристика линейной части системы МЛЦ®) и годограф — Wu{a) / -4 '-} У^2 -/ о- ;/ 0,5 0 Re -~WM(a) ы=0 -0,5 :ч -1,5 -2 Рнс. 293. Обратная амплитудно-фазовая характеристика линейной части системы и годограф нелинейного звена к задаче 426. го 18 /б 14 12 10 8 В 4 г 0 а* ■ ■ • г V 4 <y=consX П. л. \ \Д У У у /Уу^ К v^. •в= const \ ^ /ff /5 20 ы.ш 6 8 Ю 12 14 # /<? £ Рис. 294. Зависимости амплитуды вынужденных колебаний от амплитуды и частоты внешнего воздействия к задаче 426. изображены на рис. 293. Кривые wB = const и В =const показаны на рис. 294. Частота сов= 10 сек"1, при которой
4271 ;; '§ 152: ЧАСТОТНЫЙ МЕТОД 42.7 пороговое значение амплитуды входного синусоидального воздействия Впор обращается в нуль, равна частоте автоколебаний в системе. 427, Определить амплитуду и фазу вынужденных колебаний скорости вращения электрического двигателя в нелинейной системе стабилизации, схема которой йзо: бражена на рис. 295. На схеме обозначено: Д — двигатель постоянного тока с независимым возбуждением. в) "I с РУ J2 дЩ* тг б) 1 -ь '■famax _ i |'Л \ рг 1 ил« Ъ б-тг "Рис. 295. Система стабилизации скорости вращения электродвигателя к задаче 427. ТГ — тахогенератор, РУ — релейный усилитель, статическая характеристика которого изображена на рис. 295, б. Исходные данные: ширина зоны нечувствительности релейной характеристики b = 1 е, установившаяся скорость двигателя при максимальном напряжении на выходе релейного усилителя ыд = ыДШах (рис. 295,6), Q0 = = 625 сек'1, пусковой момент двигателя М0 = 100 Г • см, момент инерции, приведенный к валу двигателя / = = 0,008 Г • см,' сек2, момент нагрузки на валу двигателя Л1Н = 0, коэффициент передачи тахогенератора /гтг = = 0,01 е • сек/pad. На вход системы подается синусоидальный входной сигнал щ (t) = В sin met,. амплитуда которого 5 = 8 е, а частота ав= 10 сек~1. Решение. Обычно при исследовании релейный исполнительный механизм, состоящий из двигателя постоянного тока с независимым возбуждением и реле, управляющего током якоря, представляется в виде двух
428 ГЛ. 15. НЕЛИНЕЙНЫЕ СИСТЕМЫ 1427 независимых звеньев. Одно из этих звеньев (двигатель) является линейным, а другое (релейный усилитель) — нелинейным. При этом двигатель описывается одним и тем же дифференциальным уравнением независимо от положения контактов реле. В действительности же при разомкнутых контактах реле ( — b<u<b) цепь якоря двигателя разомкнута, и он превращается в обычный маховик. Если не учитывать момент нагрузки и механические характеристики двигателя считать линейными, то при замкнутых контактах реле (и > | b \) движение электродвигателя описывается дифференциальным уравнением '! + Kfi = Mo' (1) где Q —угловая скорость вращения двигателя, Эо = 7Г" ~~ коэффициент наклона механической характеристики. При разомкнутых контактах реле дифференциальное уравнение движения примет вид Поэтому при исследовании нелинейных систем с релейными исполнительными механизмами двигатель и релейный . усилитель следует рассматривать как одно нелинейное звено. Введем в рассмотренное нормированное значение передаточной функции гармонически линеаризованного релейного исполнительного механизма W0{d, jz) W(d, jz) = &W0(d, jz) = -^-q0(d, z) + j ^-q'0{d, z). (3) В этом выражении d — относительная амплитуда d = f. (4) 2 —относительная частота z = co7\„ (5) Ти — постоянная времени двигателя, q0(d, z) и q'0(d, z) — коэффициенты гармонической линеаризации.
427) § Л5.2.--ЧАСТОТНЫЙ" МЕТОД " 429 Аналитические выражения для коэффициентов q0 (d, г) и q'(d, z) получаются слишком сложными. Поэтому для удобства расчетов в приложении 29 приведены амплитудно-фазовые характера- о стики нормированной частот- -!+&?*■ F(u,pu)-A k„\—£-£► ной передаточной функции Т I—'—I релейного исполнительного I механизма. рис 296 Структурная схема Для определения пара- системы стабилизации скоро- метров вынужденных коле- сти сращения электродвигателя баний системы составим к задаче 427. структурную схему (рис. 296) и запишем дифференциальное уравнение замкнутой нелинейной системы u + k„r{u,pu) =«,(/), (6) где F (и, ри) = q [d, z) и + ^~ ри. (7) Составим -равенство „ Q (/<ав) + R (/«в) [д (d*. zB) + /у (rfB. zB)] D . /r\ aB щ^ Be n, (8) где ав и ф — искомые амплитуда и фаза вынужденных колебаний, zB = a>BTK. Связь между амплитудой яв и относительной амплитудой вынужденных колебаний устанавливается форму-. лой (4) В данном частном случае согласно уравнению (6) имеем Q (/©„) = S (/юв) = 1, R (/юв) = k„. (10) Подставим (10) в (8). Получим aB + kTraB[q{dB,.zB) + jq'{dB,zB)] = Be-l<f. (11) Коэффициенты q{dB, zB) и (f{da,zB) определяем по графикам, приведенным в приложении 29. Для этого предварительно находим . гв = швГм = шв/^ = Ю-0,008-f|--0,5. (Ш)
43Э ГЛ. 15. НЕЛИНЕЙНЫЕ СИСТЕМЫ [427 Затем задаемся разными значениями амплитуды ав, для них по формуле (9) определяем значения относительной амплитуды dB, по амплитудно-фазовым 4 -? 1 -2 0 -2 -4 -6- -8 ilm '8 ■6 -^N#"* ■4 \ ■2 \ 2 4 § V? 10 Re \Jr J Рис. 297. Графическое построение для определения вынужденных колебаний к задаче 427. характеристикам (приложение 29) при гв=0,5 для различных dB определяем q0(d^, zB) и q'Q(dB, zB) и по выражению (3) находим q(dB, zB) = -f- q0(dB, zB) = 625q0(dB, zB), \ } q'{dB, zB) = fq'0(dB, zB) = 625^Bl zj. \ 03) Значения коэффициентов (13) подставляем в (11) и строим кривую Z (ав) = ав + k^cts [q (dD, zB) + \cf (dB, zB)], (14) соответствующую левой части равенства (11) (рис. 297). Проводим окружность радиуса В=8и б точке пересечения ее с кривой Z(aB) находим йв«1,Бе, <р *» 58°. Амплитуда вынужденных колебаний угловой скорости вращения двигателя равна Q. .ав 1,5 0,01 = 150 еек'К
ГЛАВА 16 ПРОХОЖДЕНИЕ СЛУЧАЙНЫХ ПРОЦЕССОВ ЧЕРЕЗ НЕЛИНЕЙНЫЕ СИСТЕМЫ § 16.1. Определение функций и моментов распределения случайного процесса на выходе нелинейной системы 428. На вход безынерционного усилителя, имеющего линейную с ограничением статическую характеристику (рис. 298, й), подается случайный сигнал в виде напряжения их. Плотность распределения этого сигнала описывается функцией w{th)= ~--. е > , (1) где щ — среднее значение входного напряжения ии <Tj — среднеквадратическое отклонение напряжения щ от среднего значения йу. В линейной зоне статической характеристики коэффициент усиления /г=105. В зоне насыщения выходное напряжение усилителя и2т = 100 е. Найти и представить в виде графика закон распре^ деления w (u2) на выходе усилителя при следующих исходных данных: среднее значение ы1 = 0е, среднеквадратическое значение 0[ = О,5- 10~3е. Решение. Так как усилитель безынерционный, то в линейной зоне u2 — kul. Поскольку kax = 105 • 0,5 • 10~3 = 50в,. то в этой области 02 = ^0,. Поэтому с учетом того, что «j = 0 в пределах линейности статической. характеристики, закон распределения выходного напряжения щ имеет вид - fflW^W = Ffer'H' (2)
432 ГЛ. 16. СЛУЧАЙНЫЕ ПРОЦЕССЫ В НЕЛИНЕЙНЫХ СИСТЕМАХ 1428 По условиям нормирования функций* распределения + О0 +<ю J ay(a,)rf«i= J w(u2)du2=l. Подставляя в Ч __]/ 7Г и№ «/ "г последнее выражение функцию найдем 2 (3) (2), 1 20 f /2S о) V2n J du, e 2 02 = 1. II Рис. 298. Преобразования функций распределения случайного сигнала линейным звеном с насыщением при и = 0. Таким образом, в линейной зоне нормированные плотности распределения для относительных переменных г} — —' z2 = — на входе и выходе 02 одинаковы (рис. 298, б, в). Протяженность линейной зоны статической характеристики усилителя по оси их (рис. 298, а) может быть определена из соотношения и2,п _ 100 Ю5 «I '1л к Следовательно, ее относительная величина = 10 е. *1=- 1(Г 0,5-ИГ Вероятность Р(ы2>100 в) появления сигнала и2> 100 в на выходе усилителя равна нулю, так как более чем 100 в усилитель дать не может согласно статической характеристике. ...
-«») 5 16.1- ОПРЕДЕЛЕНИЕ ФУНКЦИЙ И МОМЕНТОВ НА ВЫХОДЕ -433 Вероятность -же Р(и2 = 100 в) появления сигнала м2= 100 в, очевидно, равна вероятности Р (щ > щя = 10~3 в). Поэтому можно записать Р («2 = 100 в) = Р (н, > и1л) = Р (г, > г1л) = ?п -2 При zu — 2 согласно приложению 31 написаяный интеграл равен 0,477. Поэтому Р (м2= 100 е)=0,5 —0,477 = = 0,023. Таким образом, в пределах линейности усилителя функции w(u2) имеет такое же нормированное распределение, как и йу(ы,). На концах интервала линейности w(u2=\00 в) имеет вид дельта-функций с площадью, равной 0,023 (рис. 298, бив). 429. Для усилителя предыдущей задачи найти функции w(u2) распределения выходного напряжения, если на его вход подается напряжение «,, имеющее случайную величину, распределенную по нормальному закону (Ы|-Й|)' 2а? V 2я а, при й, = 0,75 • \0~' в, ох = 0,5 • 10~3 в. Решение. Так же, как и в предыдущем случае, в линейной области статической характеристики, ограниченной диапазоном — и1л<и1<ци, нормированные законы распределения w {zi) и w (z2) одинаковы. При этом относительные величины zx и z2 Удобно ввести в виде (2) *\ =■ II, — М| __ и2 ~ »2 _ ц2 ~ Щ /п\ 2 а2 ka, ' ._ ^ ' Вероятности появления сигналов \и2\>.\00 в равны нулю. Вероятности же сигналов щ=±.\00 в могут быть
434 ГЛ. 16. СЛУЧАЙНЫЕ ПРОЦЕССЫ В НЕЛИНЕЙНЫХ СИСТЕМАХ 142» определены по формулам Р(«2=100в) = Р(«1>«1л) = Р(г1><л) = 0,5 У 2я г1л е 2 dz, (4) Р(«2= - 100 e)^P(Ul< -«1л) = Р(г, < - 2-) = w 1 ==°'5-?t 1^^ (5) <1л Подставив в (2) значения й,", и1л и — и1л, найдем о.' "м-"| _ 1-КГ3-0,75-1 (Г3 пс (Т, ■ 0,5- 10 л "1л •«,л.-«1 _ (-1 -0,75)- 10 ,-3 '1л ■--2,5. о, 0,5 ■ 10~3 Воспользовавшись приложением 31 для модулей |г[л[ 0,006т) м ' \rnem 400 -50 0 50 75 Ю0 Рис. 299. Ф>йкци5[ распределения сигнала па выходе линейного звена.с насыщением при ифО. и IzJ^I, получим следующие значения вероятностей: Р (и2 = 100 в) = 0,5 - 0,192 = 0,308, Р (и2 = - 100 в) = 0,5 - 0,494 = 0,006. Таким образом, на концах интервала линейности функция w (и2) имеет вид дельта-функций с площадями, соответственно равными 0,308 и 0,006 (рис. 299).
431] §..16.1. ОПРЕДЕЛЕНИЕ. ФУНКЦИИ И-МОМЕНТОВ. НА ВЫХОДЕ 435 430. Найти функцию распределения щ(и2) выходного напряжения и2 безынерционного усилителя.с релейной статической характеристикой, изображенной на рис. 300, а, если на вход его подается напряжение иъ случайная величина которого распределена так же, как и в задаче 429. Задачу решить для двух случаев: 1) о, = 2 «1 — 0 в, 2) Uy = 1,8 ■ 10 в, а, = 2 Ю-3 в; З:»/ 0 (i.rI.SW3d ' 10"3 в. в) щ(иг) ^P(v,41)... ■\w(uz)duf r^M Срабатывание релейного усилителя происходит при напряжении и1л = = 1,9 ■ 10"3 е. Выходное напряжение и2т =100 е. Ответ. В том и другом случае функция w (и2) представляет собой дельта-функции при зна- ! чениях ы2 = 0 и «2= ± ЮО е-(рис. 300, б). Площади этих дельта-функций, равные вероятностям появления соответствующего, выходного сигнала и2, имеют следующие величины: -№ -50 0 50 6) Рис. 300. Функция распределения (б) сигнала на выходе элемента с идеальной релейной характеристикой (а). 1) 2) Р ("2> = Р («а): 0,658 0,171 0 0,5019 0,498 0,0001 0 при при при при при при при м2 = 0, а2= ±100 в, прочих и2; м2 — 0, «2=+ 100 в, и2= — 100 в, прочих и2. 431. Найти математическое ожидание my{t) и корреляционную функцию Ru(ti, t2) выходной величины y(t)
436 ГЛ. 16. СЛУЧАЙНЫЕ ПРОЦЕССЫ В НЕЛИНЕЙНЫХ СИСТЕМАХ (431 нелинейной системы, структурная схема которой изображена на рис. 301. Уравнения, описывающие динамику работы системы, имеют вид (Tp+l)y = F(x), x = g-y, где нелинейная зависимость F(x) описывается функцией F {х) = kiX + ft2-v3. (2) Входной сигнал g(t) представляет собой стационарный случайный процесс с нулевым математическим —н2ь— Рис. 301. Структурная схема нелинейной системы. ожиданием {mg=0) и корреляционной функцией /?Л^р^)=°^:«^>. (3) Моменты более высоких порядков во входном сиг* нале отсутствуют. Постоянные, входящие в уравнения (I) —(3), имеют следующие значения: Г = 0,125 сек, kt = 8, А>2 = 0,4, Og. = 6, a =10 сек'1. Решение. Поскольку нелинейная зависимость (2) непрерывна и дифференцируема, то для определения моментов выходной величины y{t) можно воспользоваться ее разложением по моментам входной величины g(t). Поэтому решение для y(t) будем искать в виде степенного функционального ряда у(0=2«р»(0г"(0. (4) Подставляя выражение (4) в (1) и (2) и приравнивая члены с одинаковой степенью g, получим линейные дифференциальные уравнения для последовательного F(jo) Tp+t ~~ff(t)
SMI § 16.1. ОПРЕДЕЛЕНИЕ ФУНКЦИЙ И. МОМЕНТОВ НА ВЫХОДЕ 437 нахождения необходимых функций у2М (/ = 0, 1, 2, ...) ' (Тр+1)Ча+1=.Сам (/ = 0,1,2,...). ' (5)' Значения членов с2М, входящих в зти уравнения, для / == 0 и / = 1 имеют вид: сх = kb сзТй2(1+Зф,-Зф«-фЗ). (6) '. При заданном виде нелинейности F{g, у) и моментов распределения входного случайного сигнала g(t) все функции ф2н-1 (/==1, 2, ...) с четными номерами равны нулю. Определив в результате решения системы уравнений (5) функции фгй-1 (0. можно найти интересующие нас моменты распределения выходного сигнала у it), через моменты входного сигнала g{t). Для этого необходимо воспользоваться разложением y(t) в ряд (4) по степеням g(t). В частности, моменты первого и второго порядков определяются согласно выражениям v оо ту=Иф2ж(0иг 40, " (7) 1=0 2М-» оо fyttT, '*)= 2 Ф2М ('.)%.,+. (*2)M[£a+1('i), g2n+i{tS)l (8) где М [g'2M (t{), g2n+l {t2)\■ — соответствующий смешанный момент распределения входной величины. Так как по условию задачи, задана только корреляционная функция входного сигнала (момент второго порядка) ад. 4)-л*[ям, g(h)i то для решения задачи достаточно определить лишь функцию ф[(0- Для этого согласно (5) и (6) необходимо решить уравнение (Гр + 1)ф, = *,. Решение его дает ...... . . 91уу=ь(1^е~у): ' . ....
438 ГЛ. 16. СЛУЧАЙНЫЕ ПРОЦЕССЫ В НЕЛИНЕЙНЫХ СИСТЕМАХ [432- С учетом характера входного сигнала получаем Щ = Ф1 С) tn.g{t), Rv{h, *2) = <Pi{'i)«Pi(/-2)/?*<*i, 'a) = = ft? (1 - e~^j (1 - e~^) a2ge~a <*»-*> = . Обозначим т = /2 — ?i- Тогда Подставляя значения всех входящих величин, окончательно найдем т.у — О, Ry Ci, т) = 2304 (1 - е-«>) (1-е-8 <т+<>>) е"101. Таким образом, корреляционная функция выходного сигнала y(t) зависит не только от интервала т, но и от начала отсчета tx по оси времени. § 16.2. Расчет нелинейных систем с помощью статистической линеаризации 432. Методом статистической линеаризации найти математическое ожидание, дисперсию и корреляционную функцию на выходе нелинейного безынерционного усилителя, если на вход его поступает случайный сигнал их (/), имеющий статистические характеристики mUl(0 = ««,. (т = t2 - f,). Статическая характеристика усилителя представлена на рис. 302. Необходимые для расчета величины имеют следующие числовые значения: mu,= l,6-10~3 в, 0Ы. = 2-1О"3 в, Гыи|= 1 • 10~3 в, и2т= 100 в, а = 2 сек~\
432} .§, 16.2. СТАТИСТИЧЕСКАЯ ЛИНЕАРИЗАЦИЯ 439 Решение. Согласно методу статистической линеаризации для безынерционного нелинейного звена искомые статистические характеристики выходного напряжения определяются с помощью следующих формул: ти, —k0{mUl, oUl)mUl, (1) где kQ, k\, k" — коэффициенты статистической, линеаризации, Для того чтобы воспользоваться коэффициентами статистической линеаризации, приведенными в приложении 30, обозначим ~°/ь л/ / Л *я игт Рис. 302. Статическая характеристика линейного звена с насыщением. В соответствии с заданными числовыми значениями сг т с и, «1. = 1,6, О Ц| о 1 «1л 100 т„ 1,6 -10-3 6,25- Ю4, -£- = —^-;-= 5- 10*. '«I 2-10" Подставляя эти величины в формулы (П.ЗЗ)-=-(П.35) приложения 30, получим k0 « 6,25 • 10* • 0,6 = 3,65 • 104, ; #' = 5- 104- 0,72 = 3,6 • 10*. k'{ = 5- 104 • 0,76 = 3,8 • 104, /г, =3,7- 104.
440 ГЛ. 16. СЛУЧАЙНЫЕ ПРОЦЕССЫ В НЕЛИНЕЙНЫХ СИСТЕМАХ [433 . • С учетом этих значений т«2 = 58,4е, а„2 = 74в, Ru2{tv ^) = 5328e-2V, Ruvu2{tv /2) = 0,152e2. 433. На вход безынерционного электронного трех- позиционного реле подается напряжение и\ — HJ0 + н1сл> где «ш = 4 • 10~3в —постоянная составляющая, и1сл — стационарная случайная составляющая со спектральной плотностью D -о? =25-10-6в2, а = 2 се/с"1, "i ui Напряжение срабатывания и Отпускания электронного реле «1сР= ± 2 • 10_3е. Выходное напряжение-реле и2 = = ± 10е. ■ Применяя метод статистической линеаризации, найти постоянную составляющую выходного напряжения и20, его автокорреляционную функцию RU]l(ti, t2) и взаимную корреляционную функцию Ruv u2(t\, t2). Ответ. и20 = 5,4в, ' /?«,,'«,('!, У = 2,87-1 (TV. 434. Найти математическое ожидание та и дисперсию о^- скорости вращения оси двигателя системы, состоящей из безынерционного линейного усилителя, двигателя и тахогенератора с ограниченной линейной характеристикой. Структурная схема системы представлена на рис. 303. На вход системы подается напряжение щ (t), представляющее собой сумму постоянного напря-
43*1 § i6.2. СГАТИСТИЧКСКАЯ ЛИНЕАРИЗАЦИЯ 441 ткения «ю и стационарного случайного процесса с нулевым математическим ожиданием (ти = 0) и корреляционной функцией' Ru (т) = а\ е-ат (а = 12,5 car1). Коэффициент усиления усилителя по напряжению ky=600. f-«v • и "ч % % \ -"1 -<2 Кд1 Гр+1 * Si Рис.' 303. Структурная схема следящей , системы с нелинейной обратной связью. При напряжении щ — 30 в вал двигателя вращается со_скоростью п = 3000 об/мин, а тахогенератор развивает напряжение Мтг = 0,1 е. Электромеханическая постоянная времени двигателя Т — 0,04 сек. Статическая характеристика тахогенератора имеет вид, представленный на рис. 302. Линейная зона характеристики ограничивается скоростью вращения вала двигателя 3000 об/мин. ^ Задачу решить для трех значений дисперсии входного напряжения: а% =6,25- 10-4в2,25- 10-4в2,100. 10~4e2 при «ю = 5•• 10~2в. Решение. Передаточная функция линейной части системы kvkaB k k ^^ = 7Fm=-7^T|=TW' (1) где С[р) = Тр+1. Коэффициент передачи двигателя в соответствии с заданными условиями задачи имеет значение 'С ли "■уш 60 3000-6,28 1Г1 с / „„ -\-1 зо-бо =Ю,5 (а-cg«) - Следовательно, k = 6 • ДО2 г 10,5 = 6,3 ■ Ю3 (в ■ сек)-\
442 ГЛ. 16. СЛУЧАЙНЫЕ ПРОЦЕССЫ Б НЕЛИНЕЙНЫХ СИСТЕМАХ [431 При измерении скорости вращения й вала двигателя в рад/сек зона линейности статической характеристики тахогенератора равна Ь = &л= "л^2я =314 ceiC1. При этом тахогенератор развивает напряжение июп = = 0,1 в. Можно предполагать, что при заданном виде передаточной функции и нелинейности автоколебания в системе отсутствуют. Поэтому для решения поставленной задачи можно применить метод расчета неколебательных стационарных систем с одной нелинейностью с использованием статистической линеаризации. Согласно приложению 30 для заданной нелинейности коэффициенты статистической линеаризациа вычисляются с помощью следующих формул: *--5Г{о+™л*га-«-«.>»(-^)+ +^=[е~л *) „е-2\ о, ;jjf (2) о. (l-m.) ~ \-ЕГ) _ 0,(1+/»,) -у {-ёГ) I /оч ^ = ^о1[ф[1^1,) + Ф^)], (4) где /п, = -у , С! = -у-. Для расчета примем Для вычисления этих статистических коэффициентов передачи нужно знать mQ и <rQ на выходе двигателя, которые неизвестны. Поэтому необходимо еще воспользоваться формулами для расчета этих величиг.
4341 § 16.2. СТАТИСТИЧЕСКАЯ ЛИНЕАРИЗАЦИЯ 443 Применительно к рассматриваемой схеме можно написать *("*«,,+яю) -л—" '— (5) (6) тп = a C<O) + fefeo(mQ,0o)• k •&-tJ C(i®) + kkx(mQl<yQ) SUi{m)d(i>, где С(0) = С(р)р.о, C(/<o) = C(p)p./B. Подставляя значения /reu , и1сл, С(р), найдем fe»io ш. a l+fefto(mQ,o0) ' °£-|-J <« T2a* + [l + ££, (,«Q, <rQ)]2 a2 + cos do. (7) (8) При написании au здесь учтено, что заданной корреляционной функции входного сигнала Ни (т) соответствует спектральная плотность 2ао2и Su (со) = —5-:—г • Пользуясь приложением 17, находим значение интеграла (8): ' - 2 *Ч, CTs ~ [1 + **, (ни,, ое)] [1 + fefc, (mo, o0) + аТ]" (9) Таким образом, для того чтобы найти значения аа и та, необходимо решить систему уравнений (7), (8), "используя также уравнения (2)-ь(4). Подставим в уравнения (7) н (9) числовые значения заданных величин • 315 ma~ l+6,3-l03ft0(/nQ,aa) ' 40- 10е -а2и Ч ~ [1 + 6,3 ■ I03fej («а, оа)] [1 + Ь,3 ■ №3fc, (otq, ou) + 0,5] ' (10) (11)
444 ГЛ. 16. СЛУЧАЙНЫЕ ПРОЦЕССЫ В НЕЛИНЕЙНЫХ СИСТЕМАХ [434 Решение задачи для всех заданных от будем вести методом последовательных приближений • 1. aU)=2,5- 10'2 в. В качестве первого приближения возьмем значения коэффициентов k0 и k{, равные Коэффициенту передачи тахогенератора в линейной зоне: fc(0 = £<i> = Ihm. = _М_ = з,2 -Ю-" в ■ сек. Подставляя эти значения, а также аи , в (10) и (11), получим первое приближение: ' mi" =105 сек'1, off = 48,9 сек'1. Вычислим теперь значения тх и -ot для следующего приближения: т„ 105 aQ 48,9 ■. Используя формулы (2) —(4), найдем второе приближение: - -. £02> = 3,2 • Ю-4 в ■ сек, /г(2) = 3,2- 10~4 в-сек. Таким образом, второе приближение совпало с первым. Следовательно, при аи, = 2,5 - 10~2 в и «ю =.5 ■ 10~2 в ша=105 сект1, aQ = 48,9 сек'1 или т„=1000 об/мин, an = 566 об/мин. 2. с„, = 5 ■ 10~2 в, «ю= 5 • Ю-2 в. В качестве первого приближения берем прежние значения: /го" = k? = 3,2 ■ ИГ4 <? - сек. В. результате расчета по формулам (10) и (11) получаем т"»>=-105 cei<r\ cj}> = 97,8 сект1'.
4.841 § 16,2. СТАТИСТИЧЕСКАЯ ЛИНЕАРИЗАЦИЯ 445 Это соответствует т<'> = 0.333, of»-0,312. Вычисление по формулам (2)—(4) дает k? = 3,15 • 10~4 в • сек, k?2) = 0,94 • 10-4 в • сек, k"i3) = 3,14 • 10-4 в • сек, kf = 2,04 • 10"4 в • сек. Далее в результате расчета находим га£2)= 106 сек~\ о$ = 125,4 сек~1. Расчет по аналогичной методике дает третье приближение: ftg9 = 3,12 • 10"4 в • сек, k\& = 1,17 • 10~4 в ■ сек, k"i3) = 3,05 • 10~4 в ■ сек, k? = 2,11 • 10"" в • сек, mi = 106 сек'1, аЧ = 124,8 сек'1. Этот результат можно считать окончательным, так как он мало отличается от второго приближения. 3. оги, = 0,1 в, «10 = 5- 10*2 е. - Первое приближение: ■ /#> = ft}1* = 3,2 • 10~4 в • сек, п№ = 105 сек~\ ai° = 195 сек"1, mi" = 0,333, а[1) = 0,62. Второе приближение: ftg9 - 2,56 • 10~4 в • te/c, fcf9 = 1,96- 10~4 в • сек, та2>=123 сек"1, aL2) = 234 сек\ т(:2) = 0,39, of = 0,744. Третье приближение kf = 2,54 • 10~4 в • сек, М3) = 1,97 • 10~4 в • се/с, • т$ = 123,4 сек"1, о(£) = 233,9 сек"1. Это приближение можно считать окончательным, ак как оно практически не отличается от второго.
448 гл. 16. случайные'процессы в Нелинейных системах изз 435. Найти дисперсию о\ угла поворота рамы / вокруг оси у простейшей гировертикали с маятниковой коррекцией (рис. 304), если маятник 2 поворачивается вокруг этой же оси на угол где me —const— математическое ожидание, бел у— случайная центрированная составляющая с корреляционной функцией Маятниковый корректор с усилителем 3 и датчиком момента 4 имеет нелинейную статическую характеристику, представленную на рис. 305. Максимальный ь мм № мт '%-* Рис. 304. Гировертикаль с маятниковой коррекцией. Рис. 305. Статическая характеристика маятникового корректора. момент коррекции М,„=17,5 Г • см. Кинетический момент гироскопа Н = 20000 Г-см-сек. Задачу решить для следующих значений дисперсии ое =10,20,30 и 40 угл-мин, т е = 0, а = 0,8 сект1. Указание. Поскольку структурная схема системы имеет вид, изображенный на рис. 306, сначала необходимо найти математическое ожидание и дисперсию ошибки ■& на входе нелинейного элемента. Для этого
43S1 § 16.2. СТАТИСТИЧЕСКАЯ ЛИНЕАРИЗАЦИЯ используются формулы ;т* - С (0) + kk0 (1Щ, оц)' а*~ я Не ,.,,,,, . и Se (и) rf©, 447 0) (2) где С (0) = С (р)Р»о — многочлен знаменателя передаточной функции линейной части (в рассматриваемом случае С (р) = р), С (/со) = С (р)р=/<о, 5е (ю) — спектраль ная ^ — 0 Рис. 306. Структурная схема гировертикали с нелинейной маятниковой коррекцией. плотность, соответствующая заданной корреляционной функции Re (т), ko и. k\ — коэффициенты статистической линеаризации нелинейности (см. рис. 305), которые определяются методом последовательных приближений при решении уравнений (1) и (2) совместно с форму*- лами приложения 30. После нахождения k0 и k} математическое ожидание тв и дисперсия сг| определяются по формулам kkomeu me C(Q)+kk0 ' оо ае п J С (/со) + kxk 5е (co)rffi). Ответ. а| (угл. мин) У о| (угл. мин)2 100 .94 400 304 900 567 1600 849
РАЗДЕЛ III ОПТИМАЛЬНЫЕ, ЦИФРОВЫЕ, САМОНАСТРАИВАЮЩИЕСЯ СИСТЕМЫ И МОДЕЛИРОВАНИЕ ГЛАВА 17 СИНТЕЗ ОПТИМАЛЬНЫХ СИСТЕМ УПРАВЛЕНИЯ § 17.1. Синтез оптимальных систем с использованием принципа максимума 436. Произвести синтез регулятора, обеспечивающего оптимальное по быстродействию управление движением спутника вокруг центра масс по одной из осей. Функциональная схема системы управления представлена на рис. 307. •ъ*. /) ■■'■ 1 Регулятор К-(л. Реактивные двигатели М Рис. 307. Функциональная- схема системы управления угловым положением спутника. Момент инерции спутника / = 200 кГ • м-сек2. Исполнительными органами системы управления являются газовые реактивные двигатели с регулируемой тягой, развивающие максимальный момент Мт==4кГ • м. При оптимальном управлении найти время, необходимое для прихода спутника в установившееся нулевое состояние, если в начальный момент его отклонение составляло 1°46', а угловая скорость 28,65 epadjcetc. Возмущения отсутствуют. Задачу решить, используя принцип максимума Л. С. Понтрягина.
486) -,,.... . 5.17.1» ПРИНЦИП.МАКСИМУМА 449 Решение. Уравнения движения спутника при отсутствии возмущений имеют вид По условию задачи момент М должен быть сформирован так, чтобы спутник переходил из любого отклоненного положения в нормально ориентированное за минимальное время. Поскольку тяга реактивных исполнительных двигателей конечна, то управляющий момент М ограничен М < Мтах - Мт. Для решения задачи преобразуем уравнение (1). Для этого обозначим *, ==■■0, х2 = —jt-*= ft, & =—-р-. Тогда уравнение (1) запишется в виде ■£- = ** ^ = ^ (2) где ц — нормированная функция управления, модуль которой | ц | ^ 1. Сформируем функцию [2,20] 2 Н= Htyifiixx, хъ ц). (3) i=i Для системы (2) Н = ^iX2 + \^k[i. (4) Максимум этой функции с учетом ограничения (2) на управляющий сигнал ц и обеспечивает оптимальность системы по быстродействию. Очевидно, что при наложенных ограничениях максимум И имеет место, если управляющий сигнал \i формируется согласно закону [20] ц = sign 'ф,. (5) Таким образом, оптимальное по быстродействию управление будет осуществляться в том случае, если регулятор будет переключать исполнительное устройство по релейному закону в соответствии со знаком вспомогательной функции ф2. Для того чтобы найти %,
450 ГЛ. 17. СИНТЕЗ ОПТИМАЛЬНЫХ СИСТЕМ УПРАВЛЕНИЯ запишем [20] dtyt дН dt d^2 dt дхх дН = 0, дх2 ! — *1- [436 (6) Интегрируя эти уравнения, получим где с\, с2 — постоянные интегрирования. Чтобы наглядно- представить, как происходит переключение, изобразим процесс движения на фазовой плоскости. Исключим из уравнений (2) dt. При этом для ц = == ±1 получим xzdx2— ± kdxu ± kxi + с. (7) Уравнениям (7) соответствуют параболы, симметричные относительно оси xt (рис. 308,а). Спутник придет и бу- °) б) Рис. 308. Фазовые диаграммы углового движения спутника • при ц±1 (а) и при оптимальном управлении (б). дет оставаться в исходном положении, если изображающая точка на фазовой плоскости попадет в начало коорди- / db \ нат 1*1 = 0 = 0, x2==-^rSH0J. Для этого при произвольном начальном отклонении f)o, Ьо изображающая точка М
4l6I; § I7.I. ПРИНЦИП МАКСИМУМА 451 должна перейти сначала в точку D (рис. 308, б), а затем уже по линии переключения АО — в начальную точку.. Переход на траекторию АО достигается изменением знака управляющей функции ц с « — 1» на «+1». Таким образом, управляющий сигнал [г должен менять знак при выходе изображающей точки на линию переключения. В соответствии с рис. 308, б !*(*)■ + 1 при — 1 при соответствии с рис. duo, о d® . I db \ db (db\ .n db . / db \ db tdb\ ^ n *8' Уравнение линии переключения находится из выражения (7) и рис. 308, б (9) (10) ft„=(4f)„=-K2Msignfl. Подставляя числовые значения, получим *пт(^г)Л= -0.2VU sign a. Следовательно, в регуляторе должно быть устройство, дифференцирующее ошибку Ф по времени и вычисляющее скорость у-тА на линии переключения по формуле (10),а также логическое-устройство, осуществляющее переключение реактивных двигателей на полную •/IL=i/ Рис. 309. Функциональная схема оптимального регулятора. тягу по закону, выраженному формулами (8). Функциональная схема регулятора представлена на рис. 309. ^ д Вычислитель 'о Мидхреренци- рующее устройство *" , & Логическое устрой- стдо
452 ГЛ. 17. СИНТЕЗ ОПТИМАЛЬНЫХ 'СИСТЕМ УПРАВЛЕНИЯ [438- Оптимальное время перехода спутника из заданного положения в нулевое может быть' определено следую4 щим образом. Из уравнения (2) можно написать' ■ Поэтому db = k\xdt. В результате интегрирования по^- лучаем ft2-#.=M'2-'l). (11) Поскольку переход совершается в два этапа и значения Ь в начале (■& = Фо = 28,65 град/сек) и в конце ('&2 = 0) заданы, то для вычисления времени на каждом этапе согласно выражению (II) необходимо найти Ф = #[ в точке D. Эта точка лежит на пересечении кривых, описываемых уравнениями Поэтому «, = ±Vcoft- (13) с0 определим из начальных условий. Для этого предварительно преобразуем ■Й'о и Ь0 в радианную меру ■&о = 1° 46' = 0,02 рад, 60 = 28,65 град/сек = 0,5 сек'1. Тогда для начальных условий имеем с0 = (0.02)2 + -£|£- •-= 0,129 рад. Подстановка с0 и k в (13) дает Ьл = V0.129 = 0,345 сек'1. Для расчета необходимо взять знак ( —), так как переключение происходит в области отрицательных значений ■&. Подставляя fy в (11), находим время движения на первом (Ml = tl — /0) и втором (Л»2==^2 — *i) участках ,, , , *i-*2 0.845 лпп д/1==^_/0 = _л__я==^__ = 42,3 сек, л/ * i *2-* 0,345 ,„ о Af2 = f2-*, = -?-_— = -^—=17,3 сек. Общее время Ы: = Д^, + At2 == 59,6 сек.
-.., ■ .§ 17,1. ПРИНЦИП МАКСИМУМА '-- 453 . 437. Спутник, рассмотренный в задаче 436, подвешен на торсионной подвеске в вакуумной испытательной камере. Жесткость торсиона равна 2 кГ ■ м/рад. Силы сопротивления, пропорциональные скорости вращения f}, отсутствуют. Спутник сбалансирован и поэтому моменты от сил тяжести пренебрежимо малы. Найти оптимальный по быстродействию закон управления, уравнение фазовых траекторий линий переключения и уравнение для расчета времени движения на участках между переключениями. Ответ. Уравнение закона переключения [20] ц = sign г|>2 = sign [с, sin (0,It + с2)]; С] и Сг определяются по начальным условиям. Фазовые траектории являются эллипсами, описывае-. мыми уравнениями 100в2 + ф ±"2)2 = с; с определяются по начальным условиям. -V^Vf-N 8 * \ )). . - Рис. 310. Фазовая диаграмма оптимального движения спутника при наличии момента, пропорционального углу - • отклонения. Линия переключения сострит из полуэллипсов, примыкающих к оси. т> (рис. -310) [20]. Уравнение линии переключения для n-го полуэллипса г>пп = _ У(р - 4я)2 + 4^г> - 4/г) sign f> _ (/г = 0, 1, ...). Уравнение для расчета времени движения изображающей точки от момента tx до момента ближайшего
454 ГЛ. 17. СИНТЕЗ ОПТИМАЛЬНЫХ СИСТЕМ .УПРАВЛЕНИЯ 1438 переключения t2 равно . Ья - f>, = 0,1 с\ [cos (О, U2 + с'2) - cos (О, U1 + с2)] ± 0,02 (t2 - *,). с\ и с2 — постоянные для данной траектории, вычисленные по известным значениям Ф и Ф в моменты ti. Логическое устройство должно вырабатывать управляющие сигналы в соответствии с формулами (8) задачи 436. 438. Найти оптимальный по быстродействию закон управления электрическим приводом с двигателем неза- . • висимого возбуждения. а— *~1 , (рис. 311), уравнения фа- S. , зовых траекторий и урав- г~^ в нение линии переключения. Суммарный момент инерции на валу двига- Рис зи. Электрический привод теля / = 50 Г-см-сек2. Пу- с двигателем постоянного тока. сковой момент, развиваемый двигателем при иу = = иут = 30 в, Мш = 0,785 кГ • м. При этом напряжении скорость холостого хода Qx. x = 3000 об/мин. В качестве выходной величины рассматривать угол 6 поворота вала на выходе редуктора, имеющего коэффициент редукции /СР =10 . Пренебречь индуктивностью цепи якоря. Учесть ограничение иу^иут = 30 в. Ответ. Закон управления [20, 28] ц = sign ty2 = sign [ct + с211 - е т )\. Уравнение фазовых траекторий е = е0 + 0,2 (ё0 - ё) - 6,28. io~V in .р-32? . р. — 32G0 Уравнение линии переключения 6П =- [0,2|в| + 6,28 • 10~3In(1+32|в|)]sign 6. 80 и 60 — начальные значения угла и скорости его изменения да рассматриваемом участке оптимального про- : цесса.
4891' § 17:2. МЕТОД ДИНАМИЧЕСКОГО ПРОГРАММИРОВАНИЯ 455 § 17.2. Синтез оптимальных систем методом динамического программирования и классического вариационного исчисления 439. Электрический привод с двигателем постоянного тока независимого возбуждения (рис. 312) нагружен моментом вязкого трения Мн = k^Q, большой величины и работает в режиме, при котором падение напряжения щ = i (гд + гя) на сопротивлении г — гА + гя значительно больше обратной электродвижущей силы е — eeQ: Определить закон управления электродвигателем, при котором суммарная энергия потерь, затрачиваемая на преодоление момента вязкого трения и на нагрев, будет минимальна. Влиянием индуктивности в цепи якоря пренебречь. Момент инерции якоря с объектом / = 0,2 Г • см -сек2, коэффициенты пропорциональности двигателя по э. д. с. се = 0,096 в • сек и по моменту си — 30 Г • см/а, kx = ГО Г • ем • сек, гд + гя = 5 ом. Решение. Уравнение моментов двигателя имеет вид Рис. 312. Электрический привод с двигателем постоянного тока, нагруженный моментомвяз!- кого трения. . J^ + hQ- Сы1. (1) По условиям задачи индуктивность цепи якоря мала. Поэтому, в соответствии с законом Кирхгофа, ir + ceQ — и ц, откуда следует г Подставляя это выражение в (1), найдем dQ tiy I сесы \ (2)
456 ГЛ. 17,. СИНТЕЗ. ОПТИМАЛЬНЫХ .СИСТЕМ ...УПРАВЛЕНИЯ М39 СеСщ 0,096 - 30 Поскольку —~'= ~~1—i—- = 0.52 <С k{ = 10, пренебрежем первым членом в скобках. Поэтому приближенно уравнение динамики примет вид dQ ■ ■ см Подставив численные значения и упростив, получим —ц£ = bQ + mUy, (3) где ft = — 50 сек-1, m = 30 в7*1 • сек"2. Нам необходимо определить иу как функцию Q. По условиям задачи двигатель работает в режиме, при котором ceQ <C ir. Следовательно, приближенно i~X (4) г Мощность электрических потерь вычисляется по формуле. и2 P. = iuy = -f-: (5) Мощность -потерь на вязкое трение, если момент М„ выразить в Г • см, составляет величину Рм = 9,81> 10т5УИп. fi-9,81 • 10_5/г,й2. Таким образом, минимизируемый функционал, представляющий суммарную энергию потерь, имеет вид /=| (9,8Ь lO^kiff + уul)dt. С учетом числовых значений получим оо /= J'(a,Q2+'a0«2)^, (6) о .■:...:. aj = 0,98.1. • 10"3 dm-сек, а0^0,2 v* , '.
§ 17.2. 'МЕТОД ДИНАМИЧЕСКОГО ПРОГРАММИРОВАНИЯ 457 Задачу отыскания оптимального управления, обеспечивающего минимум интеграла (6), будем решать методом динамического программирования. Тогда для рассматриваемой системы уравнения динамического программирования запишутся a,Q2 + a0«2 + (Ш + тии) -Ц- = О, 2а0иу + т -Ц- = О, (7) где я]) — вспомогательная функция, определяемая уравнением dt -v, V — подынтегральная функция минимизируемого функционала. Решив второе уравнение системы (7), найдем dty _ _ 2а0 dQ ~ тиг Рис. 313. Функция оптимален /пч ного управления. а* двь Подстановка этого значения ние (7) дает тафу + 2a0bQuy — a,mQz = О ■ В результате решения получаем иу= — Ш, в первое уравне- (9) где Чг+ Используя числовые значения, находим ■■ 0,87 • 10"3 в • сек. Е 30 + '50^2 .30/ V (Ш +■ 0,981 ■ 10" 0,2 Таким образом, искомый оптимальный в смысле минимума потерь закон-, управления является линейным (рис. 313).
458 ГЛ. 17. СИНТЕЗ ОПТИМАЛЬНЫХ СИСТЕМ УПРАВЛЕНИЯ [440 Следует иметь в виду, что в силу сделанного ранее допущения ceQ -С ir, полученный закон справедлив в об- tly ласти малых значении Q <С —. се 440. Решить задачу 439 для следующих двух случаев: a) &i = 0,l Г-см-сек, б) гя + гд=100 ом, при неизменных остальных исходных данных. Ответ. а) k — 0,088 в • сек; б) /г = 0,442- 10~4 в-сек. 441. Решить задачу 439, применив классические вариационные методы [2, 20]. Решение. Уравнение динамики системы имеет вид -^ = ЬО. + mUy. Необходимо найти закон-изменения uv = uu{Q), минимизирующий интеграл потерь I — \ {щО? + aQul) dt. Для решения поставленной задачи необходимо составить функцию [2] где F=Jjofc4+«0^ — подынтегральная функция минимизируемого функционала; г1^~йГ~ (b{lXl + • • • + bniX„ + m{uy) = 0 — функция, представляющая уравнение первого порядка по t'-й переменной; Я,-— произвольный множитель.
441! § 17.2. МЕТОД ДИНАМИЧЕСКОГО ПРОГРАММИРОВАНИЯ Дифференцируя функцию Н, получаем dH dx, dH du II ч r, an ._ Vi , Г = °' ~dT = 2ao"f _ 2j т'Я /■ /=i (2) ;=i Отсюда уравнения вариационной задачи имеют вид ,м " "" " -jf = — 2,16/^/ + 2а'*" ° — 2йо"г/ — 2j ш'Л/' ^ В рассматриваемом случае z =~jif — (b& +Упиу) = О, На основе (2) и (3), получим dt "'"l ' Л">оа' | О = 2а0иу — тЯ,. J (4) (5) (6) Решая эту систему относительно переменных 1Л и Q, после исключения функции времени из полученных решений и- алгебраических преобразований, получим После подстановки численных значений находим : ft = 0,87- 10~3 в -сек.
ГЛАВА 18 СИСТЕМЫ С ЦИФРОВЫМИ ВЫЧИСЛИТЕЛЬНЫМИ МАШИНАМИ (ЦВМ) § 18.1. Передаточные функции систем с ЦВМ при учете квантования по времени 442. Система регулирования имеет в своем контуре цифровую вычислительную машину (ЦВМ). Структурная схема системы показана на рис. 314. Найти z—передаточные функции разомкнутой системы в предположении, что запаздывание в ЦВМ отсутствует и можно а) 6} 9 ЦВМ ■ У Щр) s> , ~"1 ~*"t >-z^. \/-е-ъ/> рь \ р Щр) У, Рис. 314. а) Блок-схема системы с ЦВМ; 6) эквивалентная структурная схема. пренебречь влиянием квантования по уровню, .т. е. можно рассматривать линейную задачу. :• Передаточная функция непрерывной части W(p)- К Р (1 + Г,р) (1) Численные значения коэффициентов: общий коэффициент усиления. /(=.10 секг\ постоянная времени Ji = 0,05 сек 'и период дискретности ЦВМ Т0 = 0,1 сек.
4421 § 18.1. ПЕРЕДАТОЧНЫЕ ФУНКЦИИ СИСТЕМ С ЦВМ 461 Решение. Передаточная функция W(z) разомкнутой системы может быть найдена согласно [35, 38] оо . W (г) = ^± £ h {пТ0) г-» =~-F (г), (2) где h(nГ0) — переходная функция непрерывной части в дискретные моменты времени (п = 0, 1, 2, ...), a F (z) представляет собой z-преобразование этой функции. Переходная функция для (1) имеет вид AW = ^['-7i(l-eT)l (3) По таблице z-преобразований находим *M-4j^-T4*-m*-*>]' (4) -JJL где d = e т =е~2 = 0,135. Далее из (2) находим передаточную функцию разомкнутой системы _ WW-KIyzj- z_d J = (*-i)(*-d) ■ (5) Поскольку переходная функция h(t) является преобразованием Лапласа от передаточной функции непрерывной части W{p), деленной на р, т. е. то часто используют символическую запись формулы (2) H7(z) = ^-z{^-}. (6) При этом дискретная передаточная функция может определяться в следующей последовательности:
462 гл- 18- СИСТЕМЫ С ЦВМ [443 z-преобразование каждого слагаемого в правой части (7) нетрудно определить, воспользовавшись таблицей z-npe- образования (приложение 2), -К{-т1 + т=т+ z_d j- ^r0[2-d--J-(l-d)(2-l)] (z-l)(z-d) (8) Окончательное выражение (8), естественно, совпадает с (5). Подстановка численных значений дает „,, ч 0,568z + 0,297 W(z)= (z_I){z_d). Передаточная функция замкнутой системы W(z\ 0,5682+0,297 Ф(е) = 1 + W (z) z2 - 0,567г + 0,432 443. Решить предыдущую задачу, если передаточная функция нелрерывной части Ответ. где dx = е т • и d2 = е Tl. 444. Для системы с ЦВМ (см. рис. 314) определить дискретные передаточные функции, используя модифицированное z-преобразование. Передаточная функция непрерывной части Решение. W (*>) = -£
446) § 18.1. ПЕРЕДАТОЧНЫЕ ФУНКЦИИ СИСТЕМ С ЦВМ 463 где символ Z„ означает модифицированное г-преобразо- вание; а — относительное (безразмерное) время 0^а<1, означает, что рассматривается смещенная решетчатая функция для моментов времени t = пТ0 + оТ0. Обращаясь к таблице модифицированного 2-прерб- разования, получим W{z, o) = KT0az+^a, (2) ф(*. °)-тШу (3) где W (z) — дискретная передаточная функция разомкнутой системы при а = 0. у Окончательно имеем ф( )==KTB(az+l-a) 445. Решить предыдущую задачу, если передаточная функция непрерывной части Ответ. W(z,o) = K- j3rf » гДе d = e T, ФС? rr\ K[{\-<?)z+d°-d\ . W(z, a)- z_d + K_Kd • 446. Для системы с ЦВМ (см. рис. 314) определить дискретные передаточные функции разомкнутой системы, используя z-преобразование и ш-преобразование. Передаточная функция непрерывной части p{l + TlP) Решение. W{z) 2-1 ( к \ z-\ i.K КТ\ л л \ ?-'7f-* KTi \ ^(z-ljsin-^- W-fly- . „ т/1 2-2zcos-^ + l
464 :ГЛ. -\В, СИСТЕМЫ'С ЦВМ,. [«7 Для перехода к ш-преобразованию используем под- 1 + W . становку г = j—^, откуда K-f(l-wf\l + W(w)- l+tiiiM tg: 2Г, w l 1 + lg2 —- ё 2Г, 447. Для системы регулирования при учете временного запаздывания т, вносимого ЦВМ (рис. 315), определить дискретные передаточные функции разомкнутой и замкнутой системы.' 3 л у /-ёт°Р Щр) У Рис. 315. Структурная схема системы с ЦВМ при учете временного запаздывания. Передаточная функция непрерывной части где х = гТ0 — временное запаздывание, 0<е<11, £>(z)= 1. Решение. Дискретная передаточная функция разомкнутой системы при наличии чистого запаздывания определяется как или ^(г) = аг,^(г,о)|11ри0.1_е. (2) Таким образом, учитывая результат решения задачи 445, получим HMz) = z-'JCr0o* + 17ol * 1 1(т= 1 -е
«81 5 18.1, ПЕРЕДАТОЧНЫЕ ФУ-НКЦИИ'СИСТЕМ С ЦВМ "465 ИЛИ W(z)- КГ„[(1-е)гН-е] KT0 2(2-1) ■ (1-е) 2 + e 22 + [КП (1-е) - [\z + KTBe (3) (4) 448. Для автоматической системы (рис. 316), в которой ЦВМ осуществляет дискретную коррекцию, определить передаточные функции разомкнутой и замкнутой системы. ■ У \ >; В(г) —- ■ »; /-ёт"Р Р Щр) У Рис. 316. Структурная схема цифровой системы с дискретной коррекцией. Передаточная функции непрерывной части (1) Закон управления, реализуемый ЦВМ, описывается разностным уравнением вида (см. приложение 23) и {пТ0) = а0х (пТ0) — ахх [(п — 1) Т0]. (2) Решение. Дискретная передаточная функция разомкнутой скорректированной системы — WcAz) = D(z)W{z), (3) где 2-1 f W(p) Л 2-1 ( W{p) 1 2(2-1)2 — дискретная передаточная функция, соответствующая приведенной непрерывной части.системы; D (г) — передаточная функция ЦВМ, выполняющей роль дискретного корректирующего устройства. Для определения D(z) проведем z-преобразование обеих частей равенства (2) U(z) = {ai)-alz'l)X(z): (4)
466 гл- J8- СИСТЕМЫ С ЦВМ [449 Отсюда й(г) = Щ = а0-а^ = ^^-. (5) Таким образом, окончательно имеем *ofMg+i)(-g-g-i) W™ № ~ 2z(z-l)2 "' ф ы ireW ^,(«+i)(f.-i) Фск(г): ~ 1 + WCK (г) 2z (2 - l)2 + KTlax (z + 1) (-g. г - l) ' § 18.2. Устойчивость и оценка качества 449. Передаточная функция разомкнутой системы регулирования с ЦВМ имеет вид _ ' Найти условие устойчивости замкнутой системы и построить переходный процесс при подаче на вход системы единичной ступенчатой функции g(t)=\{t) для КТ0-1, /СГ„ = 0,5 и /О1,, = 1,5. Решение. Передаточная функция замкнутой системы , КТ0 Ф(г). z-1+KTo Для определения условия устойчивости воспользуемся алгебраическим критерием устойчивости. Рассмотрим характеристическое уравнение замкнутой системы z-l+KT0 = 0. Условие устойчивости КТ0<2, (I) Перейдем к построению переходных процессов путем разложения изображений входной величины в ряд Лорана.
449] § 18.2. УСТОЙЧИВОСТЬ И ОЦЕНКА КАЧЕСТВА 467 Рассмотрим случай /СГ0=1. При этом передаточная функция замкнутой системы будет равна Ф(г) = - кт0 1 г-\+КТй г' Изображение входной величины (см. приложение 13) (2) Z{l(0} = G(z) = - Изображение выходной величины ( 1 KW^WOW-f^-^. (3) (4) Разложим последнее выражение в ряд Лорана посредством деления числителя на знаменатель: . I ==i + _L+_L + X+ 2 ^ г2 ^ г3 ^ г4 ^ (5) Это дает следующие значения функции # на выходе в дискретные моменты времени: при t = 0 у = 0; при t^Tf, у= 1; при t = 2T0 у— 1; при * = ЗГ0 У= 1 и далее #=1 при всех значениях t = nT0. График этой функции изображен на рис. 317 (кривая /). t,cen Рис. 317. Переходные процессы к задаче 449. Между дискретными значениями функции времени на выходе проведены прямые линии, так как передаточная функция (1) соответствует идеальному интегрирующему звену, переходная функция которого представляет собой прямую линию.
4gg ..и, is. системы с цвм 146ft Аналогичным образом получен ряд Лорана . для КТ0 = 0,5: ... ,:• . У (г) = 0,5 i- + 0,75 ± + 0,875 ~ + 0,9375 ±- + '... График переходной функции изображен на рис. 317 (кривая 2). Для случая КТ0 = 1,5 получаем У(г) = 1,51 ± 0,75^+1,125 ^ + 0,8375^ + ... График переходной функции для этого случая изображен на рис. 317 (кривая 3). 450. Определить, устойчива ли система регулирования, структурная схема которой соответствует рис. 314. Передаточная функция непрерывной части W(-P)= р(1+Г|Р)(1+7» ' где К = 2 сек'1, Г, = 0,1 сек, Г2 = 0,05 сек. Период дискретности Т0 = 0,2 сек. Ответ. Система устойчива. .451. Передаточная функция непрерывной части системы с ЦВМ имеет вид W(p) = f, (1) где К. — 50 сек'1 — общий коэффициент усиления. Определить допустимое значение периода дискретности Т0 для ЦВМ, при котором показатель колебательности замкнутой системы не будет превышать М = 1,5. Запаздывание в ЦВМ равно нулю и влиянием квантования по уровню можно пренебречь. Решение. Передаточная функция разомкнутой системы с ЦВМ может быть найдена согласно ^(?)-ТГТ- Р) Построим амплитудно-фазовую характеристику разомкнутой системы по выражению (3). Выполним
Ш § 18.2. УСТОЙЧИВОСТЬ И ОЦЕНКА'КАЧЕСТВА 469 подстановку z = e /«Го. ■■ cos «7*0 + / si" ю^о- В результате получаем частотную передаточную функцию W{em°)- кт0 cos (оТв — 1 + / sin co7"o ^-/^ctg^ Нетрудно видеть, что амплитудно-фазовая характеристика представляет собой прямую, параллельную мни- мой оси и отстоящую от нее на расстоянии -~ (рис. 318). Для того чтобы показатель колебательности не превышал заданного значения, амплитудно-фазовая характеристика не должна заходить в окружность, представляющую собой запретную зону. Она изображена на рис. 318 пунктиром. Отсюда получаем условие I КТ0 < м (4) ' ■ 2 -^Af+1 * Допустимый период дис кретности 2 М %%%** Г0< ^ м М+1 К М+-1 ! Для заданных числовых значений То< 1,5 КГ„ Im п О 50 1,5+1 = 0,024 сек. CJ=ff Рис. 318. А. ф. х. к задаче 451. 452. Для системы регулирования, рассмотренной в задаче 447, определить условия устойчивости и построить область устойчивости в плоскости двух параметров, КТ0 и е = -=г-. ' с Решение. Воспользуемся результатом (4), полученным в задаче 447. Тогда характеристическое уравнение замкнутой системы можно записать в виде
470 ГЛ. 18. СИСТЕМЫ С ЦВМ HS2 ИЛИ где г2 + Аг + £ = 0, (1) Л = /СУо(1-е)-1. В = КТ0г. (2) Условия устойчивости для уравнений второй степени имеют вид 1+Л + В>0, 1-Л + ВХ), (3) В<1. Используя (2), получим следующие условия устойчивости: --" кт0>о, КТ0(1-2е)<2, КТ0г<1.) Первое условие (4) выполняется тождественно. Совместное рассмотрение двух последних неравенств (4) (4) иг он о.б о,8 w Рис. 319. Область устойчивости к задаче 452. Рис. 320. Область устойчивости к задаче 453. дает возможность записать условие устойчивости системы в окончательном виде так: /СЗГо- 1-2е * \_ е ' если -- ^~ 4 если -т- ^ е <^ 1. (б)
да] § 18.3. СИНТЕЗ СИСТЕМ С ЦВМ 471 Область устойчивости в плоскости параметров КТ0, е, построенная по уравнению (5), показана на рис. 319. 453. Для системы регулирования, рассмотренной1 в задаче 446, построить область устойчивости в пло- т скости параметров КТ0, -—. Ответ. Область устойчивости в плоскости параметров показана на рис. 320. § 18.3. Синтез систем с ЦВМ 454. Передаточная функция непрерывной части системы регулирования с ЦВМ имеет вид w{p) = I(1+IpL) (1) где К = ЮО сек"2 — общий коэффициент усиления разомкнутой цепи регулирования, а т — постоянная времени корректирующего устройства. Определить допустимое значение периода дискретности Т0 ЦВМ и требуемое значение постоянной времени корректирующего устройства, чтобы показатель колебательности не превышал значения М=1,3, если запаздывание в ЦВМ равно нулю, а влиянием квантования по уровню можно пренебречь. Решение. Определим передаточную функцию ра? зомкнутой системы совместно с ЦВМ: В соответствии с приложением 13 Л* . Kr\ KT^jz+l) t KT0rz l"p5" + "]prJ 2(z-l)3 "l"(2-l)2" K°> Далее из (2) находим ". *<*-%£?-+?$<• w Перейдем к ^-преобразованию, использовав подстановку ." • * = Т^Г- (5)
472 гл- 18- СИСТЕМЫ С ЦВМ ' [45& В результате получаем кт% (l+2~w)[l-w) ц^н^г1 V—-■ (в) Получим теперь частотную передаточную функцию посредством подстановки w = j^-K ' (7) где А представляет собой абсолютную псевдочастоту. Используя подстановку (7), имеем из (6) K(1+At)(i-A^-) *</*)- -^ ^-. (8) Модуль частотной передаточной функции разомкнутой системы равен KV\ + iW у 1 + -j- Я,2 • I W ЦК) | = -£- ^ , (9) а- фаза ф (А) = - 180° + arctg тА - arctg •££-. (10) По выражению (9) на рис. 321 построены л. а. х. По виду фазовой характеристики (10) этот случай сводится к л", а. х. типа "2 — 1—2 (см. приложения 24). В результате получаем следующие формулы для расчета: базовая псевдочастота л. а. х. равна 1о= \П( = 10 сек~1, требуемое значение постоянной времени корректирующего устройства т = Т0 У ТТЛ"= То У Тз^Г = °'21 сек- требуемая протяженность участка л. а. х. с наклоном 20 дб/дек , М+1 __ 1,3+1 „ М-1 ~ 1,3-1 ~ ''''
4SS1 § 18.3., СИНТЕЗ СИСТЕМ С ЦВМ .Щ допустимое значение периода дискретности откуда 7"0^ 0,054 сек. Рис' 321. Л. а. х. и л. ф. х. к задаче 454. 455. Произвести расчет следящей системы с аста- тизмом первого порядка, содержащей в своем контуре ЦВМ. Исходные данные: максимальная входная скорость Qmax — 20 град/сек; максимальное входное ускорение етах = 10 град/сек2; максимальная допустимая ошибка Фшах^^'; непрерывная часть содержит постоянные времени 7", = 0,01 сек, Г2 = 0,0,02 сек и Т3 = 0,001 сек; допустимый показатель колебательности М = 1,5; запаздывание в ЦВМ равно нулю. Требуется определить параметры последовательного корректирующего звена, включаемого в непрерывную часть, допустимый период повторения Т0 для ЦВМ и построить переходный процесс при входном воздействии типа единичной ступенчатой функции. Влиянием квантования по уровню можно пренебречь. Решение. Левее частоты среза л. а. х. системы с,ЦВМ. совпадает с л. а. х. непрерывной части, а абсо- лютная псевдочастота л = -т^- (см. задачу 454) совпало дает с реальной частотой; Поэтому формирование л. а. х
474 ГЛ. 18. СИСТЕМЫ С ЦВМ - И55 левее частоты среза можно произвести обычными приемами. Построим запретную зону для л. а, х. из условий точности (рис. 322). Контрольная частота Лк = - = 20 = 0,5 сек'К Модуль передаточной функции разомкнутой системы при К = К равен QJLv - гс^-бо I W (Ак) | = -Н2£- = -йкт- = 600 = 55.6 дб. 10-4 По этим данным на рис. 322 построены контрольная точка Ак и запретная зона, сформированная из прямых с наклоном 20 дб/дек и 40 дб/дек (наклоны / и 2). Рис. 322. Л. а. х. к задаче 455. Желаемая л. а. х. в низкочастотной области формируется так, чтобы она проходила выше точки Ак на 3 дб, что соответствует увеличению коэффициента усиления в у 2 раз. Она состоит из отрезков прямых с наклонами 1 — 2 — 1. В низкочастотной области частотная передаточная функция разомкнутой системы будет иметь вид
455] § 18.3. СИНТЕЗ СИСТЕМ С ЦВМ 475 Определим параметры желаемой л. а. х. передаточной функции разомкнутой системы в низкочастотной части. Базовая частота л. а. х. равна Ло = -|//2-|^-= 1,19|Л^-= 14,5 с'ек-К Постоянная времени корректирующего звена, формирующая первый излом л. а. х., равна Для получения заданного показателя колебательности должно выполняться неравенство ±4<ь/ м-\ м Отсюда получаем минимальное значение второй постоянной времени корректирующего звена т2 = г = / = 0,112 сек. » -i/"^-' . 1 |К1/ 1,5-1 , 1 Передаточная функция последовательного корректирующего звена равна и соответствует пассивному звену интегрирующего типа. Далее определяем необходимое значение общего коэффициента усиления К = V2 -§=**- = 1,41 ^^- = 420 сек~1 Г «'так 4 и частоту среза л. а. х. В соответствии с требованиями к л. а. х. в высокочастотной области имеем ■ -#- + 7,i + ^2 + 7V<-£7-¥4T' ■
476 ■ гл- 18. СИСТЕМЫ С ЦВМ 1Щ откуда получаем..допустимое значение периода дискретности ЦВМ .' ' ~ = 2 [-^ ■ уЛу - 0,01 - 0,002 - 0,00l] = 0,012 сек. Переходный процесс при единичном ступенчатом воздействии изображен на рис. 323. 456. Произвести расчет системы с ЦВМ по исходным данным, приведенным в предыдущей задаче, за ' исключением 7\ = 0,05 сек, Т2= г /"X =0,003 сек, Т3 = 0,001 сек. / -S^ .. Кроме того, в отличие от пре- 7 дыдущей задачи, задан период f дискретности Г0 = 0,02 сек. Требуется определить вид i i __i 1 и параметры последователь- О 0,1 0,2 03 0,4 t,cen ного корректирующего устрой- Рис. 323. Переходный про- ства, которое должно быть . цесс к задаче 455. введено в непрерывную часть системы, а также необходимое значение общего коэффициента усиления К- Ответ. Общий коэффициент усиления К = 420 сек'1. В непрерывную часть системы должно быть введено пассивное интегро-дифференцирующее звено с передаточной функцией 117 f„\— (} +Т2кР) С +Тзкр) Гкэ(Р>~(.1 + ГкР)(1 + 7-„р)' где Ты = 2 сек, Г2к = 0,12 сек, Т3к = 0,05 сек, TiK = 0,01 сек. ' 457. Передаточная функция непрерывной части системы с ЦВМ W{p) = -$r. Общий коэффициент усиления разомкнутой цепи регулирования К = 1 сект*, период дискретности Т0 = 1 сек. Определить закон управления, реализуемый при помощи ЦВМ (см. рис. 316), который бы обеспечил оптимальное в смысле минимума времени протекания про-
4581 § 18.3. СИНТЕЗ СИСТЕМ С ЦВМ 477'' цессов регулирование при отработке" ступенчатых воздействий, т. е. обеспечил бы системе конечное время; регулирования. Решение. Дискретная передаточная функция не-' прерывной части системы W{z) = 0,5 't' , =0,5- *~, + z~2 z2-2z+l "'" l-2z-' + z-2" Желаемую передаточную функцию замкнутой системы возьмем в виде Ф (z) = О.бг-1 + 0,5z-2. При этом переходные процессы в системе будут заканчиваться за два периода дискретности У(г) = Ф(г)-^т=^±^ = 0,52-'+г--2 + г-3+ ... Дискретная передаточиая функция ЦВМ равна -,, Ф(г) 1 ^ z2-2z+l _ l-.2z-'+z-2 D\z) 1 т- ф (z) ' Г (z) z2 - 0,5z - 0,5 1 - 0,5z-i - 0,5zr2 " Отсюда закон управления, реализуемый ЦВМ, может быть записан окончательно в виде рекуррентного соотношения и {пТ0) = х (пТ0) - 2х [(я - 1) Т0] + + х [(о - 2) Т0] + 0,5ы [(п - 1) Т0] + 0,5м \{п -2) Т0]. 458. Произвести расчет системы с ЦВМ, непрерывная часть которой имеет те же данные, что и в предыдущей задаче. Рассчитать дискретное последовательное кор- • ректирующее устройство D (z) и период дискретности Т0 таким образом, чтобы обеспечить запас устойчивости, оцениваемый показателем колебательности, не менее, чем М^ 1,5. Решение. Произведем расчет методом логарифмических частотных характеристик. Дискретная частотная передаточная функция исходной нескорректированной системы равна К М) rW- <w • (1>
478 ГЛ. 18. СИСТЕМЫ С ЦВМ [458 Желаемую дискретную передаточную функцию возьмем в виде где т> IV (A)2(l + A^) ' 1,73 сек, (2) м м-х h =* VK = 1 сек'1, г°<тгУ m+i °o,5Ce«, Го :0,25 сек. Передаточная функция дискретного корректирующего устройства примет вид Я(А)=- 1^ж (/Я) 1+/Ят Ц7(/Я) 1+/А То * (3) Соотретствующие логарифмические частотные характеристики построены на рис. 324. L ? , дБ \ град Рис. 324. Логарифмические частотные характеристики к задаче 458. Перейдем к «-преобразованию, используя подстановку
4591 § 18.3. СИНТЕЗ СИСТЕМ С ЦВМ 479 где &о = —^- = 3,96, &, = —^-=-2,96, откуда закон управления, реализуемый при помощи ЦВМ, имеет вид > и (пТ0) = Ь0х (пТ0) + biX [(и - 1) Т0], ' (5) т. е. используется управление по отклонению и первой разности (см. приложение 23). 459. Рассчитать дискретное последовательное корректирующее устройство D (z) и период дискретности Т0 из условия получения необходимого запаса устойчивости системы. Система с ЦВМ имеет передаточную функцию непрерывной части вида где /С = 90; 2^=10 сек2. Требуемый показатель колебательности М ^ 1,5. Ответ. D{z) = b0+blz~1, где b0 = 4, &i=—3, Т0 = *= 0,166 сек.
ГЛАВ А 19 ЭКСТРЕМАЛЬНЫЕ И САМОНАСТРАИВАЮЩИЕСЯ СИСТЕМЫ § 19.1. Составление структурных схем экстремальных и самонастраивающихся систем и исследование их устойчивости 460. Найти передаточную функцию и составить структурную схему системы автоматической подстройки частоты, изображенной на рис. 325. Система должна подстраивать колебательный контур, состоящий из индуктивности L и суммарной емкости С — С\ + С20, в резонанс с входным сигналом uy(t) = Uomsm2nfot. Подстройка контура в резонанс осуществляется изменением величины переменной емкости С\ е помощью двигателя постоянного тока Д\. Поиск резонанса обеспечивается синхронным детектированием входного сигнала с помощью синхронного детектора СД и переменного конденсатора С2 малой емкости, величина которой изменяется по закону С2 = С2о+ С2т sin ю^ вращением его пластин двигателем Д2 с постоянной угловой скоростью coi. Опорное напряжение частоты coi на синхронный детектор подается с генератора ГОН, приводимого во вращение двигателем Д2- С резонансного контура сигнал подается на безынерционный усилитель с коэффициентом усиления ky, на выходе которого имеется выпрямитель и фильтр Фи настроенный на частоту сигнала coi, с коэффициентом усиления &ф1 и полосой пропускания Af\ = 1000 гц. После синхронного детектора СД также имеется фильтр Фг с коэффициентом усиления &ф2 и полосой пропускания
«601 § 19.1. СОСТАВЛЕНИЕ СТРУКТУРНЫХ СХЕМ 481 С, ( ( I I L г h и, Ред Ч.У<У, V в А/2 = 20 гц. Фильтры Ф1 и Ф2 являются апериодическими звеньями первого порядка. Поиск экстремума осуществляется следующим образом. При расстройке резонансного контура на выходе синхронного детектора воз- R никает сигнал, пропорциональный производной 2т-. После сглаживания филы-, ром Фг он поступает на двигатель, вследствие чего емкость Ci изменяется до тех пор, пока производная ■ "2т не станет равной нулю. Это положение и будет соответствовать резонансу. Исходные данные для расчета передаточных функций: R=\03 ом, Сю=400пф, L = 0,5 гн, f0= Ю4 гц, С2= == 100+ 50sin coi/ пф, коэффициенты передачи фиЛЬТрОВ /2ф1 = /2ф2 = 0,8, действующее значение входного напряжения Ul = ~±r= 100 в. Двигатель Д4 развивает 3000 об/мин при напряжении 30 в. Редуктор (Ред) имеет коэффициент передачи /гред = 10~3. Емкость конденсатора С4 меняется на 80 пф при повороте его пластин на 1 рад. Входное сопротивление усилителя считать равным бесконечности. Решение. Структурная схема и передаточная функция данной системы могут быть представлены в двух вариантах в зависимости от выбора регулируемых величин. В качестве последних могут быть приняты частота входного сигнала /0 или емкость резонансного контура С = С\ + С2о.' Рассмотрим оба варианта. Если за регулируемую величину принять частоту coo = 2jt/o, то структурная схема системы будет иметь 4> <у Рис. 325. Система автоматической подстройки частоты.
ГЛ. 19. ЭКСТРЕМАЛЬНЫЕ И САМОНАСТРАИВ. СИСТЕМЫ f460 вид, представленный на. рис. 326. Передаточные функции входящих в систему элементов определяются следующим образом. 1- WKl{p)= AQ%„{ —передаточная функция, характеризующая, Айо (р) как изменяется действующее напряжение «ов. , &ы0 р—* i *ы(Р) "г ■ ■ > Щ(Р) \ Ь)щ * %№ WM(P) %1 Ас, %г(Р) %(р) % ее W Wped(P) ь Рис. 326. Структурная схема автоматической системы подстройки частоты с регулируемой величиной Ио- на выходе резонансного контура «2 при изменении ча« стоты сигнала /0. Будем ее искать в предположении, что сам контур является безынерционным звеном, так как работа происходит вблизи резонансной частоты. Фаза несущей частоты /0 не играет существенной роли ввиду наличия выпрямителя. С учетом этих допущений из схемы включения контура (см. рис. 325) следует и. t/.|zK(co)| (1) где гк{а>)- (a0L (4CL-1) тура. Учитывая это, получим |Д + 2к(«»)1 ' — сопротивление резонансного кон- и*=и C30L Отсюда dU0 1 Vr2(<*20lc~iY+«>Il V{R2L(\-a>40C2L2) д%- .[^(tfCL-lf + alL2]'1* в-сек. (2) (3) При резонансе (o^CL = 1 эта производная равна нулю. При отклонениях частоты <о0 от резонансной, завит симость -—■ от © в общем случае нелинейная. Поэтому
460] § 19.1. СОСТАВЛЕНИЕ СТРУКТУРНЫХ СХЕМ 483 передаточная функция WKX(p) в общем случае может быть представлена в виде №к1 (/>) = Ki (">)■ (4) Если частота входного сигнала отклонится на ±10% от /о = 104 гц, то [ -5,7- Ю-7 в -сек при /W0+0,lf0, fcKi (<*>)=] 0 при / = /о, I 7,5- 10-7 в -сек при f = f0 — 0,lf0- 2. Wy(p) = ky — передаточная функция усилителя. 3. №ф1 (р) = у—r^j— передаточная функция первого фильтра. Постоянная времени Г' = 1Щ" ~ 3,14 • Ю00 = 3>2 ' 10 сек- 4. №ф2(/?) = -f—-jr\— передаточная функция второго фильтра, где Г^^ = Й=0'016»" 5-WM ° 4!f)=р (гд!р+о ~ пеРедат°чна* фу"к- ция двигателя, где и _ А^х. ... Их'.х.л ___ 3000-3,14 .. , 1 ! дв_"^хТГ~ 60-t/x.x. — бо-зо —,u'De сек • 7\B = 0,04 сек. 6- ^ред (р) = *Ред — передаточная функция редуктора, &ред= !0_3- 7. 1^с (р) —-д~ТТ — передаточная функция, характеризующая изменение емкости С\ при повороте его оси. В соответствии с заданием, эта характеристика линейна. Крутизна характеристики *с = 80^—8.10-»#.
484 ГЛ. 19. ЭКСТРЕМАЛЬНЫЕ И САМОНАСТРАИВ. СИСТЕМЫ (460 8. WK2(p) = д"0/ ) — передаточная функция, характеризующая изменение резонансной частоты при изменении емкости С[. Для того чтобы определить ее при допущениях, сделанных в пункте «1», продифференцируем выражение (2) по С. В результате получим ЗУ, _ ' fi'L'mgO-mgCL) ' „ ас, - и' [R* (i _ ^lc)» + mgi»]V. в,ф- • w Разделив (5) на (3), найдем .3 сект1. (9) —_ = _—. /t» A . /**>*- 4 «С, н-и^С4 Следовательно, считая контур безынерционным, получим ^,й(р)=^к2(©). Если задающая частота /0 меняется в пределах ±10% от /о = 104 Щ> то *.,(»)-(б.з !|;!5)-ю13* _1 - сек *. Таким образом, передаточная функция системы в разомкнутом состоянии имеет вид Wfi.W- р(Г,р+1)(Г2р+1)(Гдвр+1) • l7> Подставляя числовые значения для частот f0, f0 Ч- O.lfo и /0 — 0,1/о, получим / = fo. WJp) = 0, (8) 2,28-10*%, f = fo + 0,1/* ^«.(P) = р(3^.10-*р+1)(0Л6р+1)(0,04р+1)' (9) ■2,08-10"% /~fo O.lfo. Г»-р(з,2.10Л + 1)(0.016р + 1)(0,04р+1У (10) Найдем теперь передаточную функцию, если в качестве регулируемой величины рассматривается емкость Су.
«°1 § 'S.T. СОСТАВЛЕНИЕ СТРУКТУРНЫХ СХЕМ 485 Структурная схема такой системы имеет вид, представленный на рис. 327. Входящие в эту систему передаточные функции Wy{p), W$i(p), W„(p), ^ред(р) и Wc{p) точно такие же, как и в предыдущем случае. w0a *»Ш Са -Л с 1—■ *Ш %(р) %№ Ар *V#--i ШР) ее Щ(р) Щь(р)^ 7ф2 Рис. 327. Структурная схема автоматической системы подстройки частоты с регулируемой величиной С\. Передаточная функция WK{p)= Д(Л%- характеризует нарастание напряжения U2 на выходе резонансного контура при изменении емкости С. При сделанных ранее замечаниях о характере выпрямления сигнала на выходе усилителя можно считать, что гк(р) = М«>), (П) где kK(со) — коэффициент, вычисляемый по формуле (5). В случае настройки контура в резонанс &к(со) = 0. . При f - /0 + 0,1/0 kK (со) = - 4,2 • 107 в/ф, при f = /о - 0,9/о kK (со) = + 3,6 ■ 107 в/ф. Таким образом, передаточная функция разомкнутой системы, охваченной обратной связью, имеет следующие значения: (12) (13) (И) при / = f0 ' Wc{p) = Q, ■при / = f0 + G,lfo Wc(p) = ■2,28.10_0fty (3,2 • 10~4p + l) (0,016p + l') (0,04p + 1) ' !ЙРИ / = /o-0,lfo Wc{p) = __ 2,08-%. ■■■:: ~ ■ (3,2■'■■ 10~4p + 1) (Q.016p + 1) (0,Q4jo + 1) *
486 ГЛ. 19. ЭКСТРЕМАЛЬНЫЕ И САМОНАСТРАИВ. СИСТЕМЫ [«1 Как видно из выражений (9), (10), (13) и (14), передаточные функции не зависят от выбора регулируемой величины. Формально это вытекает из того, что kK i (G>) • kK 2 (Ю)в4^-|тг = М«>) = 4^7 в/ф. дсзв dCi Перед замкнутым контуром системы необходимо включить элемент, характеризующий изменение величины емкости при резонансе от частоты f0 входного сигнала. Он может быть представлен безынерционным звеном с коэффициентом передани ., , ч дС 1 + a2LC , При /0 и /о ±Ю% величина 1,34 • 10~и ф • сек при f = f0 + 0, lf0, 1,62- 10~14 ф-еек при f = f0, 1,92 • I О"14 ф • сек при f = f0 - 0,1/0. 461. Составить структурную схему и определить передаточную функцию самонастраивающейся системы *;(»)=■ Ч и, в В* Ж?, снвт > - -0 и. АРУ Ъ> Ред =Ф 40 Рис. 328. Автоматическая самонастраивающаяся система построения вектора по двум составляющим. построения вектора по двум составляющим (рис.. 328). Составляющие вектора задаются в виде переменных напряжений Ux и Uy, прикладываемых к обмоткам ста-
'■4вИ -"- § W.I. СОСТАВЛЕНИЕ СТРУКТУРНЫХ СХЕМ 487 тора'синусно-косинусного трансформатора (СКВТ). Модуль построенного вектора представляется напряжением где кчэ = -^- — коэффициент трансформации СКВТ. Угол а поворота ротора определяет положение вектора t/2 в системе координат X, Y. Поворот ротора на угол а осуществляется следящей системой, состоящей из усилителя с автоматической регулировкой усиления, двигателя и редуктора. Самонастройка заключается в поддержании постоянного5 напряжения иъ на входе в усилитель при одном и том же рассогласовании Да в широком диапазоне изменения напряжения U{. Усилитель является апериодическим звеном, двигатель представляется в' виде произведения интегрирующего и апериодического звеньев. . Ull «яОПЩ асе ,(0 ц I к и/COS Act, СКВТ и <щ u,swAct АРУ КРУ У h V*/- - «-у »д« р(Тльр*П Ред *Р Рис. 329. Структурная схема автоматической самонастраивающейся системы построения вектора по двум составляющим. Ответ. Структурная схема изображена на рис. 329. Без автоматической регулировки усиления передаточная функция системы имеет вид Г(/>) = р(Гур + 1)(ГдвР+1) ' где- к =z Кц$к руКу/сдВ/2р. Если, подавая напряжение U2- на автоматический регулятор усиления, сделать коэффициент его передачи
488 " ' ГЛ.. 19. ЭКСТРЕМАЛЬНЫЕ И САМОНАСТРАИВ. СИСТЕМЫ 1*2 k' равным -у,—, то получи м W^~ р(Гур+1)(Ттр+1) • где k = ktiJi'vykykMikv. 462. В системе,1 рассмотренной в задаче 460, определить коэффициент усиления ky усилителя, обеспечивающий устойчивость управления в пределах отклонения частоты fQ на ±20%. Отв?г. /гу<1,71 ■ 107. 463. В системе, рассмотренной в задаче 461, найти значение коэффициента k' обеспечивающего устойчивую работу системы при следующих исходных данных: кчэ = 0,5, ky — 104, kAB = 12 в~ • сек" , kp = 1,3 • 10" , -Гу = = 0,025 сек, ГДБ = 0,045 сек. Ответ. £рУ<0,8. 464. Найти передаточную функцию Ф(р) замкнутой самонастраивающейся системы с эталонным фильтром Ф0(р), структурная схема которой изображена дШ *ъ*Ю ' \ г /<ф v+/ к, Т,рЧ lJ~ «3 1 ' Кг р(Ггр*1) ~уШ Рис. 330. Структурная схема самонастраивающейся системы с эталонным фильтром. на рис. 330. Определить, будет ли система устойчива при следующих числовых значениях, входящих в передаточные функции величин йф = й2 = *1 = 1. &з= 100сек-1, Тф = 0,04 сек, 7Y- 0,001 сек, Г2 = 0,2 сек.
466| § 1Э.2. КАЧЕСТВО СИСТЕМ ЭКСТРЕМАЛЬНОГО УПРАВЛЕНИЯ 489 Ответ. ., W' ~ (ТфР+\)[.р{Т1Р+1){Т2р+\)+к3к2(Т1Р+1) + к1к2] ' Если учесть, что k3 много больше других членов, то можно записать Ф(/>)~Т^~Г = Фо(р)- При заданных значениях величин система устойчива. 465. Система, рассмотренная в предыдущей задаче, предназначена для сохранения качества регулирования при изменении передаточной функции выходного элемента \W{p) = —.... 2, .. I и значений входящих в нее V Р (' гР + I) ! величин. Определить, при каком предельном значении k2 система будет устойчива. Для сравнения найти также предельный коэффициент k'2 в случае отсутствия, контура самонастройки (k3 = кф — 0). Ответ. k2<2140 сек'1, k'2< 1005 сек'1. § 19.2. Качество систем экстремального управления, 466. В системе автоматической подстройки частоты, структурная схема которой представлена на рис. 327, найти переходный процесс, пренебрегая постоянными времени фильтров Г, и Т2. При расчете принять следующие значения входящих величии Ткв = 0,04 сек, /гдв = = 10,5 в~1 • сект1, 6ф1 = 6ф2 = 0,8, ky = Ю7, £ред = 10~3» Дгс = 8 - 10_ ф. Коэффициент kK{a>) рассчитать по формуле , при R = 1000 ом, U1 = 100 в, С = 500 пф, 1 = 0,5 гн, pQ = 2зх/0 = 6,28 • 103 сек~1. '.[" Начальное отклонение от резонансной частоты составляет 0,2 (й0 при частоте ш0, равной резонансной частоте. Внешние возмущения отсутствуют. Начальная скорость изменения частоты равна нулю.
490 ГЛ. 19. ЭКСТРЕМАЛЬНЫЕ И САМОНАСТРАИВ. СИСТЕМЫ [466 Решение. Структурная схема системы при сделанных допущениях имеет вид, представленный на рис. 331. Здесь Дифференциальные уравнения системы имеют вид и2 = ккШСя-Су). Разрешая эту систему, получим rf2Cv ДВ dt2 + rfCv dt ■■ kkK (со) (Сд — Су). (1) Так как при заданных числовых значениях вблизи резонанса первый ч:лен знаменателя производной -^г- , КК,М к Р(ГИР*1) Кнг(со) - ы Рис. 331. Структурная схема упрощенной автоматической системы подстройки частоты. значительно меньше второго, то можно написать Mcd)~^^(1'-g>2CL). (2) В формуле (1) Су представляет собой суммарную емкость Су == С0 4- С2/ Для исследования же устойчивости важно знать отклонение ее от резонансного значения С0 = —J—. Учитывая это, можно записать <auL А. (со) = ^ [1 - (йс2(С0 + С2) L\=UyR2«>lC.z. Подставляя это в уравнение (1), получим
487] § 19.2. КАЧЕСТВО СИСТЕМ ЭКСТРЕМАЛЬНОГО УПРАВЛЕНИЯ 491 Малость отклонений С2 от резонанса позволяет линеаризовать это уравнение по С2: ' т» -?г+^г+^i*42CiA = ки^со1с0са. Подставляя числовые значения, получим 0,04-^- + ^ + 2,13С2 = 2,13Сд. Корни характеристического уравнения р.,, 2 = 12,5 ± / /28,2 = — 12,5 ± /5,3. Следовательно, общее решение С2 = /),е(-12.5+/5.3) t + /)2е(-12.5-/5.3) tф (3) Для нахождения постоянных интегрирования найдем, как изменится емкость С при изменении частоты со на 20% ю0. Воспользуемся для этого выражением (2), считая, что изменение частоты со должно компенсироваться изменением емкости. В соответствии с этим можно записать -^- [1 - (со0 + An? C0L] = ^f- [I — «ag(C0 + AC) L]. Отсюда следует ДС = 2-^Дю. Если Дсо = 0,2(о0, то АС = 0,4С0 = 200 пф. Подставляя начальные условия С20 = 200 пф и С2о=0 в (3), получим С2 = 200 [1 - е-12-5* (cos 5,3/ + 2,36 sin 5,31)\. 467. Найти переходный процесс в экстремальной системе стабилизации угловым положением спутника, блок-схема которой изображена на рис. 332. Уравнения, описывающие динамику работы системы, следующие: 1. Уравнение спутника j dp m.
492. . ГЛ- TS.- ЭКСТРЕМАЛЬНЫЕ. И САМОИЛСТРАИВ. СИСТЕМЫ И 68. , , 2. Уравнение стабилизирующего двигателя с маховиком М = kJJ2. 3. Уравнение логического звена 4. Уравнение экстремальной характеристики U{ = — kfi2. Исходные данные для расчета: / = 100 кГ • м • сек2, ki — 60 в/рад2, 7?2= 100, &м = 0,01 кГ • м • е-1, начальные отклонения 60=10°, 60 = 15 град/сек. в Страстной t в / гироскоп Ч \9 0 Ч Управляемый усилитель "г Цдигатепь -тхабик М Рис. 332. Блок-схема экстремальной системы стабилизации спутника.. : Указание. Общее уравнение линеаризовать в точке 6=15°. Ответ. 6 = 29° sin (0,56* + 20° 30'). 468. Найти переходный процесс и построить его фазовый портрет в системе, рассмотренной в задаче 467, если уравнение логического звена имеет вид U2 = k2UtsignQ — k3e, /?э = 4,3 • 103 в • сек. Остальные исходные данные и допущения принять равными приведенным в задаче 467. Ответ. 6 - 29°e-°-056' sin (0,558/ + 20° 30'). Фазовый портрет изображен на рис. 333. 469. Определить потери на поиск в системе автоматической подстройки частоты, рассмотренной в задаче
469] § 19.2. КАЧЕСТВО СИСТЕМ ЭКСТРЕМАЛЬНОГО УПРАВЛЕНИЯ 493 460. В основу исследования положить структурную схему, изображенную на рис. 326. За исходные данные для расчета принять: /гу=107, /гф1 = /гф2 = 0,8, &ред = 10~3, kc = 8- Ю-11 ф, k№ = 10,5 в'1 • сек"1, Гдв = 0,04 сек, 7\„,= = 0,016 сек, Гф, = 3,2 • 10~3 сек, i?=1000 сш, С, = 400 пф, Сг= Сго + C2m sin a{t = 100 + 50 sin 40^ пф, ю0 = 6,28- 103 сек-1. L = 0,5 г/{, зв-я-ш] Рис. 333. Фазовый портрет переходного процесса стабилизации, углового положения спутника. Решение. Под потерями на поиск понимается средняя ошибка x{t), возникающая в'экстремальной системе в установившемся режиме, в результате поискового движения. Если F {у) представляет собой функциональную зависимость, характеризующую экстремальные свойства некоторого элемента при изменении регулируемой величины у, то потери на поиск 1 d2F F-F.-^ dy* x2(t), где F3 — экстремальное значение. При синусоидальном поиске с амплитудой Ai -2 At Рассмотрим потери на поиск, принимая за регулируемую величину емкость Сд к этому потери на поиск Сд-С 1 ■to-T-g'k С20 + С\- Применительно дС2 (ДС)2, (I)
494 'ГЛ. 19. ЭКСТРЕМАЛЬНЫЕ И САМОВАСТРАИВ. СИСТЕМЫ [488 где к = kyk$ i кф 2 kRBkpkc = 5,35 • 10" ф/в — общий коэффициент передачи без резонансного контура с переда- точной функцией WK2 {р) = -^- — kKz (©о). Вторую производную -^™" найдем дифференцирова- нием выражения -г^-, приведенного в задаче 460, ею ее2 ~ VitCL«m» {-[Я2 (1 - co2CL)2 + to'Z,']3/2 + 3ff (1 - co2CL)2 [R' (1 —aKLf + со2!,2]1 Z2} U?2 (1 - co2CL)2 + co2Z.2p U,R2Hafi {-[F* (1 - e>2C£)2 + co2L21 + ЗЦг (1 - co2CL)2} [^(l-c^CZ^ + trti2]5/2 Расчет по этой формуле дает _ •g- = 4.105 e/ф2. (2) Амплитуду ошибки АС можно найти по формуле ACi»Ч ФхО«0 U, • Сш = [ 1 + ^(/(й) | • С2т, (3) где F0"ffl)= ***tfo>. при /э = ]щ. Р {TRBp+ 1) (Гф1р + 1) (Гф2р + 1) Следовательно, J Фх (/<*>) Uo, = «1 [(Г>? + 1)(r|i°>i + 0 (гФй°>1 + 0] '/2 X X { К2 Ы + ГдаГф^вХ - (Гдв + Гф, + Гф2) С?]2 + + ">?[» -ИКГДВГФ1 + ^ДВ^Ф2 + ГФ1ГФ2)]2}_,/2- Подставляя числовые значения с линеаризацией члена &к2 (юо) в точке, соответствующей отклонению емкости С на 25 пф от резонансной (на половине амплитуды С2т), получим |ФЛ/«)и, = 0,98. Подставляя это выражение в (3), имеем АСШ = 0,98С2т = 0,98 • 50 = 49 пф = 4,9 • 10"" ф.
470] § 19.2. КАЧЕСТВО СИСТЕМ ЭКСТРЕМАЛЬНОГО УПРАВЛЕНИЯ 495 Таким образом, потери на поиск по емкости равны Г -Г - l h&H (АСш>2 ... ^ л '-'до ^2 дС2 ' 2 = ~ 5,35 • 10"" • 4 • 105 • (4,9)2 • Ю-22 = 1,28 - Ю-21 ф. 470. На вход следящей системы, изображенной на рис. 334, поступает управляющий сигнал ug(t) и г- 1 1 Щ 1 1 Экстремаль-1 J I иый [_» \ / \ регулятор J V et=i/q(t)+un(t) Рис. 334. Следящая система, самонастраивающаяся на минимум квадратичной ошибки. случайная помеха и„ (t), представляющая «белый шум» со спектральной плотностью Sn(со) = N = const. Система самонастройки, воздействующая на коэффициенты усиления усилителей в цепи якоря (kyi) и в цепи возбужде. ния (ky2) двигателя Д, обеспечивает минимум общей квадрэтической ошибки на выходе. Необходимо определить функциональные зависимости коэффициентов усиления kyi и ку2 от скорости изменения входного сигнала при установившемся режиме работы и изобразить структурную схему с самонастройкой. Коэффициент демпфирования системы | должен быть равен 0,5. Система уравнений, описывающих динамику работы системы без самонастройки, имеет вид _ , , » Т dia . _ f dQ V-2 — Kyj \US — Uy), Ья -jjp -jr 1ЯГ — U2 — Ce -j£ , И = Яред о, Uy^KitBt С» Се = llBky2-
496 ГЛ. Т9. ЭКСТРЕМАЛЬНЫЕ И САМОНАСТРАИВ. СИСТЕМЫ [«№ Числовые данные для расчета: / = 0,2 Г • см • сек2, см ■= 70 Г • см/а, г = 2 ом, ыв = 30в, /?р=10~4, /?лвт = = 0,5 в/град, N = 0,01 в2. Решение. После приведения подобных членов и упрощения уравнений получим передаточную функцию системы в разомкнутом состоянии где т = /f = о-4 CuUvHyi 210fty2 ' fty,ftpfc.nBT -5 %1 _, /? =—г =9,6-10 -г—сек1. Передаточная функция замкнутой системы относительно ошибки равна - ^Ар) P(Tp+\) + k-- yz> Передаточная функция замкнутой системы Ф(Р) = >, \ • (3) Поскольку коэффициент демпфирования задан, то Управляющий сигнал ug{f) = ut и помеха ып(0 некор- релированы. Поэтому квадратичная ошибка е2 = е2 + е2, где е2 —квадрат ошибки от управляющего сигнала ug(t) и е2 — дисперсия на выходе от помехи. С помощью коэффициентов ошибок в типовых режимах находим 2 _ ц2 е*г ~ ft» ' Дисперсия от помехи 4-со — СО 4- со 2я J I W . Nk -|- (/«>)* + -g" t/€B) + Г
4701- § 19.2. КАЧЕСТВО СИСТЕМ ЭКСТРЕМАЛЬНОГО УПРАВЛЕНИЯ 497 Следовательно, < = ■ т 2/,2 агк, у2 Nkyl ■ 9,6 • КГ6 2/г„ 1? 2 (9,6-10-5)24, . .Ку2 . Дифференцируя это выражение по /гу1 и /?у2, получим условие минимума квадратичной ошибки 1(9,( 2й\ У2 N ■ 9,6 ■ 10" -](*yi-< £у2) = 0. ;6.ю-5)2^ >у2 |V~yl 'vyS7~u- (5) Второе уравнение для определения /гу1 и /гу2 получается из (4) в результате подстановки значения k и возведения в квадрат ( -5 kyi I CKUBky2 9,6 • 10" куг Ч2Т 4\Чг После подстановки численных значений получаем 9,6- 10"5|^-=5250fe /гу2 у2. (6) В результате совместного решения уравнений (5) и (6) находим заданные функциональные зависимости мстре- **•* Кцгюкъ Экстремальный KvfV-itn р(ГРЧ) К„гШй \n р(ТрЧ) о) б) Рис. 335. Структурные схемы самонастраивающейся системы: а) с измерителем скорости входного сигнала; б) с измерителем скорости входного сигнала и спектральной плотности шумов. оптимальных значений коэффициентов kyV и kyY от ско рости изменения входного сигнала И. (ОПТ ?у2 : 1,4- 10~3ы2/3 .сек, &v7=107u4/3. (7) (8)
498 ГЛ. 19. ЭКСТРЕМАЛЬНЫЕ И САМОНАСТРАИВ. СИСТЕМЫ [470 Таким образом, для оптимального регулирования с минимумом квадратичной ошибки &\ при заданной спектральной плотности N помехи в виде «белого» шума и переменной скорости и входного сигнала, экстремальная система должна содержать измерительное устройство й и автоматические регуляторы усиления в усилителях У, и У2, реализующие зависимости (7) и (8). Структурная схема такой системы представлена на рис. 335, а. Если измеряется и спектральная плотность N шумов, то оптимальные коэффициенты усиления k°T и &$>т дрлжны регулироваться в соответствии с выражениями ^Т = 3,02.10-4|Г^, Структурная схема системы представлена на рис. 335, б.
ГЛАВА 20 СОСТАВЛЕНИЕ СХЕМ ДЛЯ МОДЕЛИРОВАНИЯ АВТОМАТИЧЕСКИХ СИСТЕМ НА НЕПРЕРЫВНЫХ ВЫЧИСЛИТЕЛЬНЫХ МАШИНАХ § 20.1. Составление схем для моделирования элементов автоматических систем на электронных вычислительных машинах - ' 471. Составить схему модели двигателя постоянного тока с независимым возбуждением, если параметры якоря: индуктивность L„ = 0,1 гн, сопротивление Rs = 10 ом; скорость холостого хода при ывх = 24 в ftx. x = 3600 об/тин, пусковой момент Ма = 2 кГ • см, момент инерции /= 1,06 • Ю" кГ • см' сек и напряжение входного 'сигнала ивх ^ 24 е. В качестве выходной величины двигателя принять угловую скорость Q. Влиянием сил сухого трения пренебречь; не учитывать индуктивность и внутреннее сопротивление оконечного каскада усилителя, питающего двигатель. Решение. Воспользуемся уравнением двигателя (TJ„P2 + Тир + 1) Q (0 = Мвх (t), (1) полученным в задаче 15. Найдем постоянные времени Тя, Ты и коэффициент передачи /гд. Тя = LRRl =0,1-10" ■=» = 0,01 сек; Q*. х = ял*. хЗО-1 = 3,14 • 3600 • 30"' = 377 сек'1; Тм = /Qx. хМп1 = 1,06 • 10"3 • 377 • 2"1 = 0,2 сек; К = Q°. х («ex)"' = 377 • 24"1 = 15,7 в"л • сек"1.
500 " ГЛ. 20. СОСТАВЛЕНИЕ СХПЖДЛЯ МОДЕЛИРОВАНИЯ [471 (3) Подставляя найденные параметры в (1), получим уравнение движения двигателя (0,002р2 + 0,2р + I)Q(f)= 15,7uBX(0. (2) Дифференциальное уравнение (2) принадлежит апериодическому звену второго порядка, схема модели которого приведена в приложении 33. Для перехода к переменным модели воспользуемся уравнениями преобразования переменных: - ивх{1) = тищ, t = mttM, где mQ — масштаб представления угловой скорости двигателя в модели в виде напряжения и./, т„ — масштабный коэффициент представления входной величины "двигателя ы„х в виде входной величины модели ии mt — масштаб времени, tu — независимая переменная модели (или машинное время) и / — натуральное время. Выберем натуральный масштаб времени, т. е. mt= 1. Для большинства операционных усилителен непрерывных вычислительных машин линейный диапазон изменения напряжений равен ± 100 е. Поэтому при решении задачи выходные напряжения операционных усилителей не должны превышать 100 в. Однако меньшая погрешность решения обеспечивается, когда моделирование происходит при наибольшем допустимом уровне напряжений, что также необходимо учитывать при выборе масштабных коэффициентов модели. Исходя из этого, выбираем '"" ^ "Too" = Too = 3'77 в~Х рад ' сек1' «вх max _£*_ q 94 " 100 100 * ' Для перехода к машинным переменным подставим (3) в (1), в результате чего получим уравнение модели в общем виде (Т,Тар2 + Тар+1)и2У) = кА-^щУ). (5)
472] .. § 20Л-. МОДЕЛИРОВАНИЕ ЭЛЕМЕНТОВ СИСТЕМ 501 ИЛИ (0,002р2 + 0,2/7 + 1) щ (() = щ (/). (6) Для определения числовых значений сопротивлений и емкостей резисторов и конденсаторов модели, воспользуемся- формулами приложения 33, откуда следует к — д ^ = 1, 12 = ^ = U.0U2 селг, r _ R4R1R3C1 _ n 9 /.«к- При выборе /?] = /?3 = 100 ксш, /?2 == 20 ком, /?4 = #6= = 1 мои, R5= 10 ком и Ci = C2=l лк0 схема модели (п. 5 приложения 33) будет соответствовать двигателю постоянного тока с независимым возбуждением с параметрами, заданными в условии задачи. .472. Составить схему модели двигателя постоянного тока с независимым возбуждением и с параметрами* Рис. 336. Схема модели двигателя. заданными в условии предыдущей задачи, но в качестве выходной величины принять не скорость Q, а угол поворота вала двигателя а. Влиянием сил сухого трения пренебречь. ' Ответ. См. рис. 336, где гаа = ft-1 • tha = 3,77 в~ • рад — масштабный коэффициент по углу а. Здесь k = 1 сек'1 — коэффициент передачи интегратора.
Б02 ГЛ. 2Q. СОСТАВЛЕНИЕ СХЕМ ДЛЯ МОДЕЛИРОВАНИЯ ■4*73 473. Составить схему модели двигателя с независимым возбуждением по условиям задачи 471, но с учетом влияния момента нагрузки от сил сухого трения Мн = = Ж?, sign О, (рис.. 337, б), причем М°а = 0,4 кГ • см. ■а Si ■и: Рис. 337. Схема модели к задаче 473. Решение. Составим уравнение равновесия напря жений цепи якоря ^ВХ Дя'я Т 1^Я £1 Т Сеа (1) и уравнение равновесия моментов на валу двигателя J ЧГ = М~•Мн = см/Я-МнsignQ. (2) Решив их совместно относительно угловой скорости Q, получим уравнение движения двигателя с учетом момента нагрузки Мн {ТяТшр + Тыр +1)0 = knuBX — кшМ\ sign fi, (3) где Ад 1 К. X * То » йм: R. U°m ■ ■"" с.с„ М„ ■■ р —коэффициенты передачи двигателя по напряжению (кл) и по моменту нагрузки {kj. Коэффициенты сы, ся, р, Тя, Тш определены в задаче 15. ' Моделирование статической характеристики сухого трения (рис. 337, б) производится по схеме п. 2 приложения 34. При этом величина напряжения Us= C/4= UH
4731 § 20.1. МОДЕЛИРОВАНИЕ ЭЛЕМЕНТОВ СИСТЕМ 503 подбирается такой, чтобы диоды были надежно закрыты. Выберем UB = 20 е. Тогда масштабный коэффициент ml o,4 mM = -^ = -go- = 0,02 кГ • см • в"1. (4) Для перехода к машинным переменным воспользуемся масштабными коэффициентами, полученными в задаче 471, та, ти (mt — 1) и коэффициентом тм (4). После замены переменных уравнение (3) принимает вид {ТяТыр2 + Тмр + 1) u2(f) = 1щ (t) - ktUH sign «2, (5) где k = k -^ = 157.-^=1 k,~k -^- = iZI.^?=i где к йд ma - id,/ 3J7 - i, я, -дм ma 2 3J7- i. Подставив числовые значения в (5), получим уравнение модели (0,002р2.+ 0,2р + 1) и2 (t) = и, (Q - 20 sign u2. (6) Уравнение (6) отличается от уравнения (6) задачи 471 дополнительным нелинейным членом 20 sign щ, который реализуется с помощью схемы, изображенной в пп. 1, 2 приложения 34 при R0 = oo. При этом из уравнений (3) и (6) видно, что действие момента от сил сухого трения 'равносильно действию нелинейной отрицательной обратной связи. Поэтому схема модели, отображающая дифференциальное уравнение движения двигателя с независимым возбуждением с учетом момента от сил сухого трения принимает вид, изображенный на рис. 337, а. Уравнение модели, записанное с помощью параметров схемы рис. 337, а, принимает вид (IiM± CiC2P2 +-M^ ciP + i)«s(/) = -Ш^-Ш"-*1**"*®- (7) где RB = 100 ком, а величины сопротивлений и емкостей резисторов Ri — R6 и конденсаторов Сь С2 получены в задаче 471. Сопротивление резистора R-, (рис. 337, а), не вошедшее в уравнение (7), может иметь произвольное конечное значение,- но на порядок или несколько порядков превосходящее прямые сопротивления диодов и введенных сопротивлений потенциометров (см. п. 2
J504 ГЛ.' 20. СОСТАВЛЕНИЕ СХЕМ ДЛЯ МОДЕЛИРОВАНИЯ [tli приложения 34), включенных в обратную связь операционного усилителя. 474. Составить в общем виде схему модели двигателя с независимым возбуждением с учетом момента нагрузки от сил сухого трения Ми = М* sign Q (рис. 337, б), если влиянием электромагнитной постоянной времени цепи якоря Тл можно пренебречь. Выходной величиной принять угловую скорость вращения вала Q, а входной—напряжение икх, подаваемое на обмотку якоря.. Все переменные и параметры двигателя с независимым возбуждением определены в предыдущих задачах. Решение. Уравнение движения двигателя в машинных переменных (см. предьщущую задачу) при Тя = 0 ■принимает вид (Тир + 1) и2 (t) = kA -^ щ (0 - К ~г U» sien "2 (0- Схема модели двигателя, согласно полученному уравнению, приведена на рис. 338. Параметры модели Рис. 338. Схема модели.двигателя к задаче 474. (сопротивления резисторов R\ — R& и емкость конденсатора С) определяются на основании следующих формул: " #•» д _ т RsRe и '"и RsRe *, »'м СК6-1 „,- R.R..- йд :jno , ,R^ - ftu -mQ . Все масштабные коэффициенты определены при решении предыдущих задач. 475. Найти уравнение модели, схема которой представлена на рис. 339, где напряжение обратной связи и3
47в| ' § 20.1. МОДЕЛИРОВАНИЕ ЭЛЕМЕНТОВ СИСТЕМ 505 является частью, выходного напряжения и2 и снимается с помощью потенциометра (блока делителя напряжения) R3; Za(p), zi(p)~операторные сопротивления. Решение. Отношение переменного сопротивления Ra, с которого снимается напряжение и3, к сопро- D тивлению R3, -~ = а, назы- *<3 вают коэффициентом делителя напряжения! Тогда (1) *,'[ ■=>> MP) г,(Р)\ -СГЗ+ и3 = <ш2. Уравнение модели «2 = > £ /О LL — Z0\P) [zi(p) "1_^ /?2"3_Г Рис- 339- Схема модели к за- ,п\ Даче 475. После подстановки (1) в (2) и преобразований получим искомое уравнение Г—Чт + ! 1«»(0 = -^ —^-г «1 (0- 476. Составить схему модели гидравлического исполнительного устройства (рис. 2, а), дифференциальное уравнение и передаточная функция которого получены в задаче 2. Параметры гидравлического исполнительного устройства следующие: ky~50 сек"1, а = 14 см, 6 = 7 см, с{ = 0,05 кГ • слГ1 ■ сек, с2 — 0,5 кГ • см~\ с3 = = 2 кГ • см-1 • сек. Решение. Найдем постоянные времени и коэффициенты передачи (см. задачу 2): fe-= 14 a+b 14 + 7 = 0,667, /г3 *.„ ^ =05 a 14 ' * «4 = = 7ГГ ^ 4 Се/С, 4 с2 0,5 .-.* feife. 50 ■ 0,667 l+fe,Mu .1+50-0,5-4 ct 0,05 = 0,33 сек4, Т = J о. с 'о. с с. 0,5 0,1 -Л+Мз^Ч .1+50-.0.5-4 = 0,1 сек, 0,001 сек.
508 ГЛ. 20. СОСТАВЛЕНИЕ СХЕМ ДЛЯ МОДЕЛИРОВАНИЯ {«Ю Выражение передаточной функции гидравлического исполнительного устройства wW—Xip)— ptTp + i) • I1' где Y(p) изображение выходной величины — перемещения заслонки у, Х{р) — изображение входной величины — перемещения муфты к. Представим передаточную функцию (1) в виде произведения W{p) = W.l{p)Wa{p), (2) W*(P> TipJ— тР+\ J w* — ~pTW~ 7* () На основании передаточной функции Wy (p) запишем дифференциальное уравнение (Тр + 1) у (г) = * (Тоср +1)х (0, (4) где y{t) = py = ^r. Умножим обе части уравнения (4) на Гос и к левой части прибавим Ту — Ту Ту — Ту + (ТТоср + Тос) y = k {Тоср + 1) Тосх, {Тос -Т)у + (Тоср + l)Ty = k (Тоср + 1.) Тосх, * ос t i • t. ■* ос ■* ■ y=R—X— Tocf}+l У. Ho k^f- = k,k2, ^Ы*,Ц. Тогда Для перехода к машинным переменным (щ, и2) воспользуемся масштабными коэффициентами (см. задачу 471), у = т6и2, х = тхщ, t = mttM, mt = 1. (6) Подставив (6) в (5), получим
476] § 26Л. МОДЕЛИРОВАНИЕ ЭЛЕМЕНТОВ СИСТЕМ ( 507 (8) На основании дифференциального уравнения (7) составляем схему модели (рис. 340), уравнение которой имеет вид "2~ RtUl RjttiCRtp+i) и* Из сравнения уравнений (7). и (8) находим -It—^.ft.fe- -^50- 0,667, S=A,^4 = 50 • 0,5 - 4 = 100, К2К3 CR5 = Г^ 0,1 сел. (9) Выбрав масштабные коэффициенты ту = 66,7 см • в~ь- •сект1, mx=\G см-в'1, получим ^4 Я, 50 • 0,667 = 10 66,7 50 • 0,667 = 5. (10) На основании соотношений (9) и (10) выбираем Ry = 20 ком, R2 = Rz=z Ю ком, R4 = R5 =100 ком, С = — 1 мкф. *, Я* Т /,о хс 1.0, /?, Рис. 340. Схема модели к задаче 476. Подсоединив к выходу схемы модели, изображен-^ ной на рис. 340, интегратор (п. 8 приложения 33) с коэффициентом перейачи А5,= 1 сек'1, получим искомую
508 ГЛ. 20. СОСТАВЛЕНИЕ СХЕМ ДЛЯ МОДЕЛИРОВАНИЯ [477 схему модели гидравлического исполнительного устройства. 477. Составить дифференциальное уравнение модели, схема которой изображена на рис. 341, где R — потенциометры, представляющие собой однотипные блоки делителей напряжения с коэффициентами ait а2 (см. задачу 475). Выходной величиной принять напряжение щ, а входной — щ. Рис. 341. Схема модели к задаче 477. Решение. Обозначим выходные напряжения каждого операционного усилителя {щ — и5). Составим систему уравнений N M2=-*7(f + ^ + -^-), (1) "5=-#9-|-. (4) Исключим переменные и2, и3, и5. Для этого выразим «з через щ из уравнения Щ и.подставим в уравнения
4891 § 2ГМ. МОДЕЛИРОВАНИЕ ЭЛЕМЕНТОВ СИСТЕМ 509 (2) и (4). В результате получим н2 = С,С2#2#3р2"4 + С2 Ф риА, Ъ-Ъ-ЩЬ-рщ. Подставив полученные выражения для и2 и и5 в уравнение (1) и сделав преобразования, получим чискомое дифференциальное уравнение модели (а0р2 + щр + а2)иА(t) = -Ь0щ (О, где а0 = С,Сгадз, а, = C2(ffi- + "»%%;*9), а2 = /?7 &о = 478. Найти установившееся значение выходной величины «4 С) модели, схема которой изображена на рис. 341, а дифференциальное уравнение получено в предыдущей задаче, если входная величина u{(t) = = 10-1(0 в, #1 = 1 мои, #6=100 ком, а, = 0,425. Ответ. ы2(00)= —2,36 в. 479. Составить схему модели идеального интегрирующего звена с передаточной функцией W (р) = —, k = = 14 сек'1. Решение. Это звено набирается по схеме рис. 342. Составленное уравнение модели (см. решение задачи 477) R и2 = — откуда * = RCp а Ж' и, А. -ft с > ь Рис. 342. Схема модели идеального интегрирующего звена. Пусть С=\мкф и #=0,02 мом, тогда а = kRC — = 14-0,02-1=0,28. 480. Составить схему модели апериодического звена первого порядка с передаточной функцией W(p) = =ТТТ' й = 29, Г =; 0,24 сек. '
610 ГЛ. 20. СОСТАВЛЕНИЕ СХЕМ ДЛЯ МОДЕЛИРОВАНИЯ [48! Решение. Схема модели изображена на рис. 343. Составленное уравнение модели (см. решение задачи 477) имеет вид откуда (О Соотношения (1) позволяют выбрать параметры схемы на рис. 343: /?i = i?2 = 0,1 мом, /?3 = 0,02 мом, #4=1 мом, С=1 мкф, щ = 0,417, <х2 = 0,242. Ъ #1[V > >" ^ у иг Рис. 343. Схема модели апериодического звена первого порядка. 481. Составить схему модели апериодического звена первого порядка с передаточной функцией W(р) = = -^,—т-г, k = 0,83, Т = 9 сек. Ответ. Схема моде'ли изображена на рис. 343; R( = f= #2 = #3 = 1 мом, R4 = 0,1 мом, С = 1 JWK0, a! = 0,111, a2 = 0,92 (см. задачу 480). 482. Составить схему модели консервативного звена с передаточной функцией W(p)= J2 2 ^ ,k = 7,T=* = 0,34 сек. Решение. Схема модели изображена на рис. 344. Составленное уравнение модели (см. решение задачи 477)
483] § ZQ.I. МОДЕЛИРОВАНИЕ ЭЛЕМЕНТОВ СИСТЕМ 511 имеет вид ^ (™£ССг р2+л) мо = - -^г-мо, (и откуда Г = У ~~R^ k=-~R7 (2) Соотношения (2) позволяют выбрать параметры схемы на рис. 344: Ry = RA = 0,1 мом, R2 = Rs = Rs= 1 мом, C2 = C, = 1 лш£, ttj = 0,700, ct2 = 0,863. Рис. 344. Схема модели консервативного звена. 483. Составить схему модели консервативного звена с передаточной функцией W(p)= Тг 2+l » k—l, Т = = 0,007 сек. Решение. Схема модели изображена на рис. 344 (см. задачу 482). Так как угловая частота свободных колебаний Ч~у~~Кпт = 143 се/с-1 большая, то темп решения задачи необходимо замедлить, т. е. выбрать масштаб времени mt < 1. Для перехода от натурального времени t к машинному tM и обратно служит выражение t = mttu. > При этом в уравнении модели (1) предыдущей задачи вместо натурального времени t будет машинное время , . d tM, а вместо символа производной P^-jr ~ символ производной /?„= -^-.
512 ГЛ. 20. СОСТАВЛЕНИЕ СХЕМ ДЛЯ МОДЕЛИРОВАНИЯ [484 Тогда, подставив в уравнение модели (1) предыдущей задачи рм = tnt dt ■■т,р, получим (RaRf£lC'm*p*.+ l) «„(/„) = ~ -^-MU, откуда Г^^/g^AC, ft=-^ Так как ft = 1, то выбираем /?! = R2 = 1 -иси, а, = 1, т. е. в схеме рис. 344 можно отключить делитель напряжения щ. Для выбора масштаба времени из первого выражения (1) найдем (1) / т, \2 R2R3RtC,C2 а2-\-Т) R-s * (2) Так как коэффициент а2 должен быть меньше единицы, то при R5 = R2—l мом и Ci = C2=l мкф выражение См г—Г Рис. 345. Схема модели колебательного звена или апериодического звена второго порядка. (2) позволяет выбрать mt = 0,1, /?3 = 0,1 мом, RA = 0,02 мом, <х2 = 0,408 или же т, = 0,01, /?3 = 0,1 мом, /?4=1 мом, <х2 = 0,204. 484. Составить схему модели колебательного звена к с передаточной функцией W (р) = у2 2 , 2^т + t , k = 8, Г = 0,167 сек, | = 0,56.
4861 § 20.2. МОДЕЛИРОВАНИЕ УРАВНЕНИИ СИСТЕМ 513 Решение. Схема модели изображена на рис. 345. Составленное уравнение модели (см. решение задачи 477) имеет вид откуда находим Т . -| / R2R3R5C1C2 г- азТ и 1 ~ V «2Я6 ' 2R^C 2 а,#2 Л. "»' «1 ' (1) Соотношения (1) позволяют выбрать параметры схемы на рис. 345: /?, = R3 = R4 = /?5 = 0,1 лож, /?2 = /?6= = 1 лож, С, = С2 = 1 жкф, а, = 0,800, Oj = 0,371,03 = 0,670. 485. Составить схему модели апериодического звена второго порядка с передаточной функцией W(p)= T2p2 + 2lTp + x ' & = 2,14, Л = 4,3, Г =1,2 сек. Ответ. Схема модели изображена на рис. 345; /?, = = /?4 = 0,1 мом, R2 = i?3 = #5 = ^6 = * лож, С1! = С2 = 1мкф, щ = 0,214, а2 = 0,695, аз = 0,357 (см. предыдущую задачу). § 20.2. Составление схем для моделирования дифференциальных уравнений и автоматических систем на электронных вычислительных машинах 486. Составить схему модели для решения линейного дифференциального уравнения {а0р5 + GiP4 + a2ps + а3р2 + а4р + а5) у (t) = b0g (t) в натуральном масштабе времени и при нулевых начальных условиях. Решение. Преобразуем переменные y{t) и g(t) исходного уравнения в соответствующие переменные модели с помощью формул y = mytis, | . g = tngu. j
514 ГЛ. 20. СОСТАВЛЕНИЕ СХЕМ ДЛЯ МОДЕЛИРОВАНИЯ И86 Подставив (1) в исходное уравнение, получим (оор5 + ахр4 + a2ps + а3р2 + а^р + а5) и5 {t) = -f- b0u (t). (2) Разрешим уравнение (2) относительно старшей производной то Ьп По уравнению (3) составляем схему модели (рис. 346).- Схема модели составляется на основании следующего. >,--С±Н- ~><,--С±М- ">,--С±Й- >,--CZDJ- T>/---f р% I—II—I 1 Рис. 346. Схема модели к задаче 486. Согласно уравнению (3) высшая (пятая). производная переменной и5 равна сумме членов, пропорциональных низшим производным и самой переменной ы5, взятых с обратными знаками, и входной величине и. Предположим, что в точке б или в точке суммирования всех переменных имеем высшую производную {р5и5). Тогда, включив последовательно интеграторы 1—5, на выходах последних будем получать напряжения, пропорциональные соответствующим низшим производным и самой переменной. Учитывая, что операционные усилители изменяют знак входного сигнала, необходимо включить сумматор 6 для изменения знака напряжений, пропорциональных положительным производным (р3м5
486] § 20.2. МОДЕЛИРОВАНИЕ УРАВНЕНИЙ СИСТЕМ 515 и ри5), снимаемых с четных (2 и 4) интеграторов. Интегратор 1 одновременно выполняет и функцию суммирования. Для определения параметров модели (сопротивлений резисторов R\—R\2 и емкостей конденсаторов С{ — С5) найдем уравнение модели. Для этого составим систему уравнений: щ = «з = ClP 1 С*р 1 С3р 1 С4р 1 \r; и, «2 * «2 Дз ' «3 ««' «4 \Rl Rll Я|0 «8 Д»Г (4) Решив систему уравнений (4) относительно искомой переменной ы5, получим уравнение модели в виде (С1С2С3С4С5/?8/?2/?з/?4/?5Р5 + С2С3С4С5/?2/?3/?4/?5Р4 + + |^-§f С3С4С5/?з/?4/?5Р3 + "f^ C4C5R4R5p2 + Минус в правой части уравнения (5) появился в виду того, что при моделировании использовано нечетное число операционных усилителей (пять), включенных в прямую цепь. Чтобы уравнение (5) полностью соответствовало исходному,, входную величину —напряжение и — следует подать на вход модели через инвертор, схема которого приведена в приложении 33. Сравнив коэффициенты при переменных и их производных уравнения (5) с соответствующими
516 ГЛ. 20. СОСТАВЛЕНИЕ СХЕМ ДЛЯ МОДЕЛИРОВАНИЯ Н87 коэффициентами уравнения (2), С 1С2СзС4С5/?8 А2^?3^4^5 ^ 2^ 3^ 4^ 5^2 R ЗД 4 R 5 ~Rt~RfC3CiC^R4R5-- "5 ЦЧдЛ К|0 ^8 ^12 /-> Г> . «8 Ли «8 Я| получим = <2о. = с„ = а2> = Оз» = с4» = а5» _ "% ъ ту °* После выбора масштабных коэффициентов, уравнения (6) позволяют (произвольным образом) выбрать параметры модели при известных значениях коэффициентов исходного уравнения. 487. Как изменятся параметры модели на рис. 346, если дифференциальное уравнение из условия предыдущей задачи решать в измененном масштабе времени, т. е. решать в замедленном или ускоренном темпе? Решение. Для перехода от натурального времени t к машинному tM и обратно служит уравнение t = mttM, (1) где tnt — масштаб времени. При tnt > 1 темп решения задачи ускоренный, а при mt < 1 — замедленный. При выборе tnt ф 1 темп протекания процессов в модели должен отличаться от темпа процессов, описываемых исходным дифференциальным уравнением, т. е. в уравнении модели (5) предыдущей задачи вместо натурального времени t будет машинное время tM, а вме- d сто символа производной р = — символ производной d
488] § 20.2. МОДЕЛИРОВАНИЕ УРАВНЕНИЙ СИСТЕМ 517 Тогда, подставив в уравнение (5) предыдущей задачи получим (C^C^C^R^R^mtp* + С2С3С,С5Я2Я,Я^5т^ + + fBlg- СзС&Я^ту + Jfc C4C5R4R5my +. + |а-^.С8Лвт<р + |^)«Б('-)=-^«(и. «i Из сравнения коэффициентов при переменных и их производных полученного уравнения с соответствующими коэффициентами уравнения (2) предыдущей задачи имеем CiC&CfisRtRiR&Rs = а0т? щ C2Cf4C6R2R3RAR5 = altn7\ ~rT ~fa~ С3С4Сй^4^5 ~ а2т, ■3 i > t^c4cs#A = w2. (2) «и °5' Из полученных соотношений видно, что при решении задачи в замедленном темпе (mt < 1) величины сопротивлений резисторов R{ — Rl2 и емкостей конденсаторов Су — С5 модели на рис. 346 необходимо увеличивать, а при ускоренном темпе (mt>\)~уменьшать. 488. Составить в общем виде схему модели следящей системы, принципиальная и структурная схемы которой изображены на рис. 22, а параметры и уравнения отдельных звеньев заданы в условии задачи 38. Решение. Воспользуемся передаточными функциями отдельных звеньев и структурной схемой следящей системы из решения задачи 38.
518 ГЛ. 20. СОСТАВЛЕНИЕ СХЕМ ДЛЯ МОДЕЛИРОВАНИЯ [488 Преобразуем структурную схему следящей системы (рис. 22,6). Перенесем сумматор / на вход предыдущего звена и преобразуем два последних звена прямой цепи, в результате чего получим структурную схему, приведенную на рис. 347, а. А в -*—®—- и к, и Л* llfi*l к* и /^ и, If Г„Р*1 рос Kfi Р а) Рис. 347. Структурная схема (а) и схема модели (б) следящей системы. В этой следящей системе выделим три физические величины: угол поворота, момент нагрузки и напряжение. Масштабы для времени mt и для напряжения та примем равными единице. Масштабы по углу и моменту будем выбирать из соотношений Мн 4в=тмиа, 1 ft = тт. .1 О) Для перехода от структурной схемы следящей системы (рис. 347, а) к схеме модели воспользуемся моделями элементов структурных схем автоматических систем (приложение 33). В результате замены элементов структурной схемы (рис. 347, а) соответствующими
490] § 20.2. МОДЕЛИРОВАНИЕ УРАВНЕНИЙ СИСТЕМ 519 элементами модели (приложение 33) получим схему модели следящей системы (рис. 347, б). На основании принятых масштабов обозначим на схеме модели соответствующие структурной схеме следящей системы величины напряжений * 1 1 I U. = ■&■, Uo = Фг» U„ — Ми. 1 -ть " J ть * н тш н' иъ = и, и\ = и\, и\ = и\, иь = — ра. (2) Согласно формулам приложения 33 находим i (3) При известных числовых значениях параметров следящей системы и при выборе масштабных коэффициентов формулы (3) позволяют выбрать параметры модели (сопротивления резисторов Rl — Rg и емкости конденсаторов Су — С3), при этом изменением величины масштабных коэффициентов в допустимых пределах облегчается задача выбора параметров модели. 489. Составить уравнение модели, изображенной на рис. 347, б относительно выходной величины и2 по входным м* и ин, с учетом масштабных коэффициентов, принятых при решении предыдущей задачи. Ответ. [(ClR7p+l)(C2Rsp+l)p + -w^^-]u2(t) = 490. Составить в общем виде схему модели релейной следящей системы, отличающейся от следящей системы, схема которой изображена на рис. 22, тем, что вместо линейного усилителя взят релейный со статической характеристикой, приведенной на рис. 348, а. Параметры и уравнения всех остальных линейных звеньев заданы в условии задачи 38.
520 ГЛ. 20. СОСТАВЛЕНИЕ СХЕМ ДЛЯ МОДЕЛИРОВАНИЯ 1490 Рис. 348. Статическая характеристика (а) и схема модели (б) к задаче 490. Рис. 349. Схема модели к задаче 491.
J] § 20.2. МОДЕЛИРОВАНИЕ УРАВНЕНИЙ СИСТЕМ 521. Ответ. Схема модели релейной следящей системы представлена на рис. 348, б. Схема модели релейного усилителя взята из приложения 34, при этом принято ил = Ui = 11%, £/.-, = Ur, = U°. Выбор масштабных коэффициентов и параметров линейных элементов модели приведен в решении предыдущей задачи. 491. Составить в общем виде схему модели нелинейной следящей системы, отличающейся от следящей системы, схема которой изображена на рис. 22, тем, что вместо линейного усилителя взят релейный со статической характеристикой, изображенной на рис. 348, а. Кроме того, необходимо учесть приведенный к валу двигателя следящей системы момент нагрузки Ми от сил сухого трения {М„ = М„ sign 12; см. рис. 337, б). Параметры и уравнения линейных звеньев заданы в' условии задачи 38. Ответ. Схема модели нелинейной следящей системы с релейным усилителем и с учетом влияния момента нагрузки от сил сухого трения представлена на рис. 349. См. задачи 474 и 490.
ПРИЛОЖЕНИЯ 1. Изображения по Лапласу и Карсону — Хевисаиду функций времени Таблица П.1 в 1 о з 4 5 g 7 8 Оригинал 1(0 1" -e~at JL (i _ e-«*> Р-а (б-а) е-а'~(б~Р)е-в' Р-а 1 Ре-^-ае-Р* ар ар(а-р) 6+ б~« е-«. «Р т а(а-р) ^ + б_Р е* Р(р-а)' Изображение Лапласа Р- п! рп+1 1 р + а I р (р + а) 1 (р + а)(р + р) р + б (р + а) (р + Р) 1 Р(р + а)(р + Р) р + б р(р + а)(Р + Р) Изображение Карсона — Хевисайда 1 Р" Р р + а 1 р + а Р (р + а)(р + р) р(р + б) (р+а)(р + Р) 1 fp + a)(p + p) р + б (Р + «)(Р + Р)
и ПРИЛОЖЕНИЕ Б23 Продолжение табл. П.1 Оригинал Изображение Лапласа Изображение Карсона — Хевисайда 10 11 12 ■ sin %t V6l+/2 sIn(M + i» ф = arctg -г- -г- е ^ sin A/ Т-К(6-у)2+Я2Х X e~^ sin (Я/ + ф) 13 l l X V2 + As X J/ y2 + A2 X e~v' sin (M - ty) A, 14 i]> = arctg 1 -V X Y2 + A.2 hV-f + %2 X V(b-\)2 + h2 e_v' sin (W +1 1 p2 + A2 p + 6 Р2 + Л2 1 (p + Y)2 + ^2 p+6 (p + Y)2 + A2 1 p[(p + v)2 + *2] p + 6 p[(p + Y>2 + *2] P2 + A2 P(p+6) P2 + A2 (p+Y)2 + A2 p(p + 6) (P + V)2 + *2 1 (p + Y)2 + A2 p + 6 (p + Y)2+*s
2. г-преобразования функций времени Та б ли па П.2 Оригинал ft?) Преобразование Лапласа FA (p) -преобразование Р(г) Модифицированное г-преобразованне Р (г, о) КО t I" -at 1-е -at sinp< cos p/ e at sin I e~at cos p/ P I 1 i p + a a P(p + a) P p2 + P2 P • p2 + p2 P (p + a)3 + pz p + a (p + a)2 + p2 г-.l / q2 (z-U* 7|z(z+l) 2 (г-If Z d />-ar» 2~d'd~C (l-d)z a (z~l)(z-d)' a z sin P^o 22 - 2г cos pr0 + 1 Z2 — 2 COS РГ0 z2-22CosP7, + l г d sin P^o z2~22dcosp7,„-f d1 z*~zd cos pr0 22-22dcospy0 + d2 2-1 T.qZ oTnz (z-1)* 7-2, (z-1)' (1 + 2о)Г^г {aT0fz -H — ——h- 2(£- -I)2 2(г-1) -d • 2d0 2-1 6=1 Z-d z2 sin c$T0 + г sin брГр г2 - 2г cos РГ0 + I ' г2 cos о-рГр — 2 cos брГ0 г2 - 22 cos pr0 + 1 „ ,cr I z sin р-рУр -f d sin брГр \ ' I 22-22dcospr0 + d2 j zd p0 <[ 2 COS cos оРГр — d cos 22d cos рГ0 +d^;
*l ПРИЛОЖЕНИЕ 625 3. Нормирования я логарифмическая частотная фазовая Характеристика апериодического звена первого порядка (рис. 350) ф(шЦерад \ -80 ' '70 -60 -50 -40 -во ^2.0 ■ю п " „ ' 0.01 ОМ '0,1 02 OAOJB I 2 4 0 20 40 60 100 сиГ Рис. 350. 4. Нормированные логарифмические амплитудная и фазовая характеристики колебательного звена (рис. 351а и рис. 3516) ШТ), го -w -3D -40 Й ! (=№ ^\^X\WZ7 N^N ч % \ /?/ В.2 СЗ 0.1 С.-" Ц6 1.0 Рис. 351а. з 4 в е юш1
526 ПРИЛОЖЕНИЕ [б -180 -WO -J40 -130 -WO -ВО -60 ■-to -го Р=т 0,4^ #f 0£ 0,8^ <^ с**< ш й i^' /Z^ ш \ш уг П £=005 Щ — ею ■ 1Г — 0,№~~ 2^~~Ш> '&& ^J-"' 5-М 0.1 0.2 0,3 0.4 0,6 ОД 1.0 Рис. 3516. 3 4 6 8 ЮиГ Б. Отклонение асимптотической л. а. х. колебательного звена от точной (рис. 352) M(wT),dS 0,1 ' 0J 03 0.4 0,6 08 1.0 Рис. 352. Z v 3 4 6 8
e] ПРИЛОЖЕНИЕ 527 6. Критерий устойчивости Гурвица Линейная САУ, характеристический многочлен, который равен Z)(p)-fl0pn + flipn~' +fl2p"~2+ ... +fl/l_|P2 + fl/2_i!p + fln я flo > 0, устойчива, если положительны п главных определителей матрицы fl! S «3 flo Яг 0 с, 0 fl0 ... * •. 0 0 0 0 Ai-I Да"* Дз=* A„_i = \ at а» д2 . . 0 0 ! -1 а,| >0 я, я0 «о 0 «1 «0 0 0 0 Дл' я3 at а$ «2 01 я3 а2 oi Оо 0 -о„ ... ... * • « ■ • ■ >С fls а, о3 as я4 а2 0 •Дл ... ... ... • • • * * • • • • , >о, ... 0 ... о ... 0 ... 0 «•* Qf\ -!>(). 0 0 0 0 о Яп-1 А«1-2 -1 0 0 0 0 0 0 an >о, Последнее выражение сводится к неравенству аа > 0, так как предыдущий определитель Д,г_| > 0. Частные случаи критерия устойчивости Гурвица 1) D(p) — a0p + a,. Условия устойчивости ав > 0, а\ > 0. 2) £>(р)«=аоР2 + а,р + аг. Условия устойчивости а0 > 0, at > 0, аг > 0. 3) D (р) - а0р3 + а,р2 + а2р + аз- Условия устойчивости я0 > О» Я| > 0, а2 > 0, а3 > 0. а^г — аоЯз > 0. 4) D (р) ■» я„р* + й|Р3 + я2рг + а3р + я4.
528 ПРИЛОЖЕНИЕ tf Условия устойчивости . .-- «0>'0, Я! > 0, а2>0, а3>0, а» > О, а3 {аха2 — «o«;t) — а&\ > О- 5) D (р) = о0р5 + а^ + а2ръ + а3р2 + atp + обусловил устойчивости «0>0, о,>0, а2>0, а3>0, о, > 0, ' й5 > О, «|П2-йоаз > О, {«^(г - а„«3) (о;,«4 - c2Og) - («!«+ - а0а5)2 :> 0. 6) D (р) = а0рс + а ,р5 + «sp4 + а3р3 + а4р2 + аьр + а6. Условия устойчивости «0>0, «1>0, а2>0, й3>0, «4>0, а5 >0, ав > О» а3 (а^г — a„as) — а^ (а^а, — аф£) > О, (»i«2 - "о"з) f «5<а4а3 - а2«б) +.Яб.(2а105 - а|)] + . + (о,«4 - а0а5) [е,о3а:6 - а5 (а,«4 - а0о,-)] - а\а% > а 7. Диаграмма Вышнеградского с линиями равного затухания в процентах за один период (рис. 353) Рис. 353.
•I ПРИЛОЖЕНИЕ 529 8. Диаграмма Вышиеградского с линиями равной нормированной степени устойчивости ft„ = ft У «г <?ис- 3S4> 9. Эквивалентные начальные условия в системе регулирования после воздействия па нее единичной ступенчатой функции Дифференциальное уравнение системы (а&п + alPn~l + ... +all)ij{i)^{baPm + blpm-1 + ... +b„)s(t). где у (0 — пыкодная величина, g (1) — входное воздействие; y_fl> у' ..., у^-1) — начальные условии, имеющие место непосречствеино перед приложением единичной ступенчатой функция; //+(), ,/.,„.... • •■. уЙТ1* —начальные условия, имеющие место непосредственно
Бза ПРИЛОЖЕНИЕ по после приложения единичной ступенчатой фунинии: J/+0 ** У-О' У+0 = У-0> ■ • • > У+0 У-О » , (n-m) _ Хп-т) , _£о_ . а0 - „(n-m+1) _ „(n^-m+l) , 6| «о Г (п-т) _ Хп-тЦ [У+0 У-О J> а, ,,(«-!) _ ,,(«-!) . "т-\ . _ ат-1 \.Хп-т) _ гХп-тЦ. f/+0 -У-о +-^--1 ^[У+0 -V-0 J- .^L[„(n-2)_,.(n-2)l [f/+0 2/-0 J- 10. Решения однородных дифференциальных уравнений перного, второго и третьего порядков Таблица П.З 1! с >, Вещественные корни Комплексные корни Х=Хф °'* а2хр + оо Л; = Л2 = а2 —Oj а!— а2 х = (В cos U + С sin М) e~V £"=Хр - уХр+Рр X = A ie~a'' + Л2е_а2* + Л3е-аз< , Иг«з*Д + (а2 + Сз) Рр + £р 1=* <а2-а,)(а3-а,) а^зХр + (о. + п3) р„ + е0 x = Ae~a'f + ф cos U + С sin Ы) e~V (V2 + *-2> Хо + 2у«о + ео Д2 = - (a,-aj(a3-aa) CiCa^o + (Ct, + а£ y„ + ер (a,-aa)ia2-a3) B = (y-a,)2 + ^ a, (fc, — 2y) x0 + 2yti0 - e0 Ы-aJ' + K3 ~2 .,2 C = ai(fc2:-y2 + Ya,)*0+ (a2_V2+?"2)o() лКУ-а,)г + ^ (a, - y) gp MlY-a.f+^J Примечание. Здесь a,, Ог, Оз —абсолютные значения вещественных некратных корней, у и ?. —абсолютные значения вещественной и мнимой частей комплексных корней, я0 — начальное значение исследуемой функции, п0 = л'(0) и £(,=* к" (0) — начальные значения скорости и ускорении изменения исследуемой функции.
12] ПРИЛОЖЕНИЕ ^ 531 11. Номограмма для построения вещественной частотной характеристики замкнутой системы по амплитудно-фазовой характеристике разомкнутой системы (вещественная круговая диаграмма) (рис. 355) ' - ^ Рис. 355. 12. Кривые для определения времени переходного процесса и перерегулирования по коэффициенту наклона трапецеидальной вещественной частотной характеристики (рис. 356) toct„ 1Z в 4 б.% го 16 12 - в 4 Wet» \ ГЗ-^""" *" —"^ I 1 1 1 1 а о.г ofi o,6 ojb 1.о Рис. 356/
532 ПРИЛОЖЕНИЕ gts 13. Кривые для определения времени переходного процесса В перерегулирования для вещественной частотной характеристики, имеющей максимум (рис. 357) б,% so 40 ■зо го w п act„ - 5п 4к ■Зп ^^ .Zn^^ - тг а>сСп ■ / Уб "шах ) i » 10 И U 1.3 Рис. 357. 1.4 If 14. Зависимость требуемого запаса по фазе от модуля децибелах при различных показателях колебательности (рис. 358) -6 -i -Z О Z 4 6 % 70 72 14 16 16 20 22 Модуль в вйцибелал Рис. 358.
1*1 ПРИЛОЖЕНИЕ .533 15. Номограмма для построения вещественной частотной характеристики по заданным качественным показателям (рис. 359) ж =0.7 AR.% 80 50 10 30 ZO 10 О 10 ™d * 0,7 *,'■&* Q6 го 30 40 50 воу 70 Z 4 Ц2\0,4 0,6 0,8.1,0 1.2 1,4 Pl:lbl«o) S о,г\ ал ав ав i,o гг ifi Pmwm
534 ' ПРИЛОЖЕНИЕ 116 16. Нахождение квадратичной интегральной оценки Г х2 at, где х = у — Уоо — отклонение регулируемой величины от установившегося значения #«>■ Изображение регулируемой величины по Лапласу представляется в виде Пр) = bu + bjp+ ... +Ьтрт 1 ao + aip+ ... +апр" р (и > т). Тогда интегральная оценка может быть вычислена из выражения 1 2СпД (В0Л0 + В,Д,+ ... + BOTAOT-2Vi&)- Определитель А находится следующим образом: а0 0 0 0 0 —аа а, ~ай 0 0 «4 -аз а2 ~а\ 0 ~аь . а6 . — «4 • а3 . 0 . . 0 . 0 . 0 . 0 • ап Av (v = 0, I,..., т) — определитель, получающийся из А заменой (v + l)-ro столбца столбцом аи ай, 0,.... 0. Коэффициенты Вх, ..., Вт вычисляются следующим образом: Во"" ьо> Bl=-b2i-2b0b2> Bk - Ь\ - 26ft_,^+I + ... + 2 (-1 )* b0b2k, Bm-bt*
«81 ^ ПРИЛОЖЕНИЕ 535 17. Формулы для интегрирования спектральной плотности Искомый интеграф представляется в виде + 00 'п~-£Г J Л (/а) Л (-/со) т< где Л (/о) = а0 (/«)" + а, (/со)"-1 + ... + ап, G (/«) = Ь0 (/ш)2""2 + 6, (усо)2"-'1 + ... + *«_,. Многочлен G (/«) содержит только четные, степени /ш. Многочлен Л (/со) должен иметь корни в верхней полуплоскости, что соответствует устойчивой системе. Для п => 1 4-оо 1 2л J h |a0jco + a,|2 2a0ai* —oo Для re = 2 +°° _. , Г IboUay + bJda J | c00«)2 + c,/«) + c2|2 2я J | а„(;"«>)2 + а,/«> + а2|2 2a0cfi Для п = 3 + oo 3 2я J |a00«>)s + aiO'<»)!! + «»/0> + fl»l1! -a2b0 + a0bi 2a0 (aaa3 — a^) Для n = 4 + O0 J_ f [6D (/o)6 + 6, (jaY + b2 (ftp)2 + b3] da 4 = 2я J | a0 (/<o)4 + а, (ую)3 + a2 (;«)2 + a3ja + c412 = —oo &o {—ata, + a2a3) — a0a3&[ + aoai62 + -^- (aoa3 — аха2) = : 2Л • 2a0 (a0af + afo - й^йд) • 18. Нормированные стандартные передаточные функции разомкнутой системы Передаточные функции для различных степеней п дифференциального уравнения приведены в табл. П.4.
ПРИЛОЖЕНИЕ И8' Таблица П.4 Степень встатизмл J 2 .,,:. .:.,: п 2 3 4 4* 3 4 5 6 о, и 5 8 10 10 10 10 10 10 К ©о 1,4 щ 2' Щ 2,6 «5 о 5,1 О % 16 ®0 38 "о 73~ U?(P> «0 р2 + 1,4со0р «0 ря + 2со0р2 + 2«^р ®о р4 + 2,6сйср3 + 3,4с0цр2 + 2,6« ^р 2,5сй0р + «о-, Р2 б.Зсо^р + coq р3 + 5,1со0р2 12coi?p + c0o р4 + 7,2сй0р3+16ИоР2 Ificojp + Год р5 + 9со0р4 + 29со£р3 + 38со£р2 25со£р + «о р6 + Нсуэ5 + 43«У + 83«$р3 + 73ю1р2
m ПРИЛОЖЕНИЕ"'' 537; В них введен параметр соо> определяющий быстродействие системы. Переходные характеристики, соответствующие этим передаточным функциям, приведены на рис. 360. Перерегулирование о% приведено в таблице. . tl(t) iz 1,0 QB 0,6 0А аг г/1 131 Г . ttlgt 1 1 1 В W w v Рис. 360. Переходные характеристики, соответствующие стандартным передаточным функциям. & tcv0t) 1.1 12 10 0.8 ае Q" т — J / 7/ 7 / f t / i L i У 7\ и // н А *f~ п ._ —■*, —— "Л Ч'! **г О. /-1 ^ Рис. 361. Переходные функции для типовых л. а. х. симметричного вида. - - - ■ - -
ПРИЛОЖЕНИЕ JIB 19. Типовые л. а. х., соответствующие Симметрич Передаточная функция разомкнутой системы Вид л. а. х. Kd + TsP) (l+T0p){l+TlP)l[(l+tip) 1=3 ft(l + Тгр) Р (1+Г,р)П (1 +TiP) 1=3 KU + T&) i=3 М — показатель колебательности. Для типовых симметричных
161 ПРИЛОЖЕНИЕ заданному запасу устойчивости ные л. а. х. 539 Таблица П.5 Тип л. а. х. Базовая частота Частота среза Постоянные времени в области "средних и высоких частот связь с базовой частотой связь с частотой среза 0-1-2-1-2-3. 2-1-2-3. V г0г, 1-2-1-2-3... У1 КТ2 TJi кт2 г, -±У м М-1 т,> 1 М «е М — 1 Ук КТ2 < 1 Ум (м-1) 2* < 1 м Щ М + 1 Юс М+1 л. а. х. оп равен М = г, где т- т— 1 2г* i-3
Несимметричные л. а. х. Таблица П.6 Порядок аста- тизма 0 ; 1 Передаточная функция разомкнутой системы К (I + ТоР) П (» + Г^> 1=1 к п pJ\(l + Tip) ■ Вид л. а г\/ ! 1> / / Z 7 1 \ X. г--** V C^«fi> Тип л. а. х. 0-1-2-3... 1-2-3... Базовая частота, Щ К к Постоянные времени в области средних и высоких частот п i — l ^ 1 м2 + м У м2 ~ i ^ «о 2 . Приведенная здесь формула для 2 Г/ является приближенной; она дает.достаточную точность пра \ ^ '■ - • . . г'=1 М^ 1>3. Эта" формула становится точной при М— 1 и любом п, либо при п— 1 и любом /VI.
201" ПРИЛОЖЕНИЕ 641 , 20. Нормированные кривые переходных процессов для типовых л. а. х. А. Кривые переходных процессов при входном воздействии в виде ступенчатой функции g (t) = g0 • 1 (2). 1) Случай симметричных л. а. х. (приложение 19, табл. П.5), рис. 361. Кривые рассчитаны для л. а. х. типа 2—1—2, но с большой точностью отображают переходные процессы в случае всех типов л. а. х., приведенных в табл. П.5.' 2) Случай несимметричных л. а. х. (приложение 19, табл. П.6), рис. 362. J 1 м л Ц f М-ч < 1 5 Af=/ $. 3 * 1 -М=1.7 0 I234S67S9H> Рис. 362. Переходные процессы для типовых л. а. х. несимметричного вида. Кривые рассчитаны для л. а. х. типа 1—2, но практически пригодны для л. а. х. всех типов, приведенных в табл. П.6. Б. Кривые ошибки воспроизведения линейного задающего воздействия g(l)=at • 1 (/) 1) Случай симметричных л. а. х. (приложение 19, табл. П.5) рис, 363. ■ Кривые рассчитаны для л. а. х. типа 2—1—2, но практически пригодны и для л. а. х. остальных типов, приведенных в табл. П.5). 2) Случай несимметричных л. а. х. (приложение 19, табл. П.6). рис. 364. :. Кривые рассчитаны для л. а. х. типа 1—2, но практически пригодны и для л. а. х." остальных типов, приведенных в табл. П.6.
£42 ПРИЛОЖЕНИЕ |21 21. Корректирующие звенья Схема корректирующего звена Передаточная функция корректирующего звека П С, »—LcZD « я, о, Г. 1 + Г.р 1+Тф RlCl 1 + Г,р 1 + ГгР «|С, \Г 1 + ТгР 1 + Г,р ('♦*): К, «I С» 1 + Гар 1 + Г,р + л0«,+я,яг)сгх
sir постоянного тока l+- ПРИЛОЖЕНИЕ Таблица П.7 Асимптотическая л. а. х. •♦* 1 Т, ШдСц Ro + Щ ■г, «„ + «,+«, L \ 1 1 1 р-р .%& | ^ " t ! - ад «аСг ц + и,+л, в I 7. 71
.544 ПРИЛОЖЕНИЕ i (St Схема корректирующего - • звена■ Передаточная функции корректирующего звепа (1 + Г,р)(1 + Гяр) + |г.(|+-|^) + Гг]р + 7',У2Ря R.C, 0„ Л=- (1 + Т,р) () + Ггр) (1 + Ар + Врг) RiCi №+*о) С1С1^Й) + (¥^У|«|«;) с» 22. Корректирующее звено переменного Схема ' Передаточные функции звена Двойное Т- образное эбена Ь <И= К ft % Параметрическая U, В(р) ' Мр)-Т,ТгГгр* + + 1\ (S, + ТУ рГ + (Г. + S.) p+U В (p)~rI7'2J'ep-|-7'1 (S8-(-7'»)pI+ + r!!(7'1-l-S1+r!,)p'+ H-(7\+S, + 7yb7s+Si) P + l. По огибающей U, (/й) I + Г./Q -■ Q„ У, (/й) о l + t/й
«1 И»ИЛ&ЖЕВД8 545 Продолжение табл. П-7 Асимптотическая л. а. к. Я-.С2 ft>Cs «„+■■■«■, + «-• в = К0 + й, + ^ г «■ tf ■t 1 ' 1 7, 7г .1 1 1 1 1 1 1 i — ft> тока (двойное Г-образное звено) Таблица П.8 ' Связь менаду параметрическими постоянными времени и лостояи? ними временя по сгибающей 'Условии осуществимости звена r, = G, Tf тж.'" ' 3 *»>„ г V'h =-/«-AS< <■$ < ■ J_ vA y,-A,ft
Б46 приложение QS & 02 С ~0,2 Рис. 363. Ошибка воспроизведения линейного закона для типовых л. а. х. симметричного вида. 2,0 -х а IS 1.0 0.5 О / 2 3 4 S 6 7 8 9 Ю ll" Рис. 364. Ошибка воспроизведения линейного закона для типовых л. а. х. несимметричного вида.
23] ПРИЛОЖЕНИЕ Выбор параметров звена ■54,7 Таблица П;9 ширина полосы частот, гц а 2,5 ~2.5 0,758 0.226 5.0 4 0,553 0,110 ■7,5 ~7,5 0,486 0.077 10 "~ 10 0,453 0,052 15 15 0,420 0.034 20 - ~20 0,403 0,025 30 ~зо- 0,386 0,016 40 ~40 0,378 0,012 50 ~50 0,374 0.009 60 ~60 0,370 0,008 оо резонирует 0.353 0 При составлении этой таблицы принято, что С, =С2 = С3 = С. Коэффициент передачи на несущей частоте С0. Сопротивления под- считываются по формулам Я2 = #з = <оиС 1 2<рнаС ' 1 (П.1) Несущая частота юн = 2я/н. 23. Приведение дифференциальных уравнений к канонической форме А. Для уравнений нелинейной системы п-го порядка, заданных в нормальной форме Т)2 = «21^1 + «22^2+ ••• +«2, n-l4tl-l + Ь2%, Чп-1 = аП-Ь J*lt + an-2, 2%+ ••• + «И-h fj-l4n-l + *>n-ll> о = с1г\1+с2Цг+'... + c„_|ii„_, -r|, Ы(о), каноническая форма записи имеет следующий вид: Х|=Л|*1 + Г(а), Ж2 = ^2*2 + ^ (°^' ха-\ — hn-iXn + / (а), а = р,^ + p2*2 + ••• +Pn-iXn-i-'rf(o). (xn^o), где Х\, X2f-> xn-u о ss xn~канонические переменные. (П.2)
6*8.. ПРИЛОЖЕНИЕ (гз Для перехода к каноническим уравнениям (17.2) необходимо, а) вычислить определитель ■J £>(Я)> att —Л, ai2. a21> «22 — к, «2> n-l an-l, !r «И-1, 2' • • •• an-l, П-1 ~ ^ (П.З) б) вычислить корни характеристического уравнения 1>(Я)—О, Ль Аг. • • • > ^п-й в) вычислить коэффициенты Р/ по формулам Р«==- с1Л/1(Я/) + с3Л/г(Я<)+ ... + сп-,/У„., (Я,,-) О'.(Яг). (П.4) Выражения М/г (Я) получаются из определителя D (Я) (П.З) заменой £-го столбца на столбец 1>п-1 (П.5) Выражение , D'(Я) представляет собой производную.от D (Я) по Я. £. Для уравнений нелинейной системы и-го порядка, заданных в нормальной форме ;• .'Ti! = a11T)I + Oi2iij+ ... + amr\n+b\f (a), ' ■f)2 = a21T), + a22r|2+ ... + a2nt\n + Ьг[(о), Пп = o„iT), + o„,*)2 + ...-.+ cwiv, + bnl (a), -0=C1T)1+C2T)2+ ... + C,lXln канонические: уравнения относительно переменных jtt, ж2, принимают вид I -Yi = A.i*i + / (с) > . :. л-2 = Я2л:2 + / (а) . | (П.6) Хц (П.7) А'п=^цЖп + И0г) где Я'17 Я2,'...,■ Я>1 —корни характеристического многочлена D (Я). Он — к, д,2, :.., й|„ й21> «22 — ^ «2л £>(Я) = «ш. ■ - • > . «т —■ Я ■'и •■ Если в уравнении £> (Я) = 0 имеется нулевой корень, например An = 0, то кайопнчсские уравнения для- системы уравнений (П.6)
23] ПРИЛОЖЕНИЕ Б«' записываются в форме, совпадающей с (П.2) *, = A,*, + f(or), *j — Я2аг2 + f (a). ,*„_, =• Я„_,а:„_, 4- f (a). 6 = 0^,4-02*2 + ... + flnXn — rf (О), (&SSX„). Но здесь где г— — (с,6, + ее&2 + + с«Ы. Pfc - -A*Y*P - -*■* J c« ^rgff (* = L 2. • •., «)• /=i frk) В формуле (П. 10) (П.8) (П.9) (П.10) (П.П) где Z)/ft (Я) обозначает алгебраическое дополнение элемента i-й строки и fe-ro столбца определителя D (Я). В. Для уравнений нелинейной системы и-го порядка, заданных в виде V"-» +алц{"-2) + ... + ап-т = -dl, o = c0r\ + c\j) + c2ri+ ... + с„_г*)('7~2' — r|. l-f(or). канонические уравнения записываются в виде Xi = Я,.¥, + / (О), *2 = ^2*2 + / («0. (П. 12) •*fl-t = K-rfn-l + f (О).' о = Pi*i + Рг*2 +....+ fin~\Xn~i~rf(a), где Я|, Я2, .... Я,г_, — корни многочлена />(Я) = Я"-|+а,Яп-2+а2Яп-34- ... +«„_, А (Ял) Э*--<*- (fe=l, 2..... и-I)- D' (U) В формуле (П.15) Д(Я) = г0+с,Я + с2Л1!+ ... +с„_гЯ"-Е, Я'(Я)=^ЩЛ). ■ : (ПЛЗ) (П.14) (П. 15) (П. 15)
550 ПРИЛОЖЕНИЕ [24 Если в многочлене. (П.14) имеется один нулевой, корень, например, Л„_1 = 0, то в выражении (П. 13) Р»--ч*.А(У ч (fc-1. 2, .... л-2), Wiih) fa-i--rf- Со (П.17) аП-2 где £),(Л) = Л"-г + а,Яи-,+ ... +ап. 24. Достаточные условия устойчивости" нелинейных систем второго, третьего и четвертого порядков Считаем, что нелинейная система содержит одно нелинейное звено, статическая характеристика которого .имеет любую форму, но она обязательно однозначная, нечетная и симметричная относи- тельчо начала координат. Производная от функции Ляпунова будет знакоопределенной для систем третьего порядка, уравдения которых приводятся к канонической форме (П.2), (П.8) или (П.13); ' для систем четвертого порядка, уравнения которых приводятся к той же форТие (П.2) или (П. 13), но при наличии одного нулевого корня в уравнении D(X) = 0; для систем четвертого порядка с уравнениями типа (П.2), (П.8) или (П.13) без нулевого корня,- но при условии, что одному из вещественных корней уравнения D (А,) = 0 соответствует отрицательная постоянная Р; . ■ ■ при выполнении условий где Г>0 при «■>--£-, (П.18) T>2V~^W-f2 при ©<-j, (П.19) Г* = Т- + Т- + г, (П.20) Aj Л; 2 ^_ г (Л,-X,)»+(P,-fc) (*,,--*,) 4А,,А,2 (П.21) Для систем второго порядка, уравнения которых приводятся к- форме (П.7), справедливы условия (П.18) и (П.19) при г = 0, если в формулах (П.20) и (П.21) заменить Pi н рг на vi и у2.
125 ПРИЛОЖЕНИЕ 651. 25. Переход от л. а. х. вида 20 lg| 1 + W (/©) | к л. а. х. вида 20 lg | W (/«) | (рис. 365, 366, 367) 1.66 15 1 S 6,66 IS И ю 5 -5 -10 1.66 Л1 15 Ю -5 -10 С.дб 75 IV -70 1 1 wiff\/*mjm\ „66 "дек < „„Яб , гоЩ,*' и>0 MgV'IfOart 1 Ч **оё а>0 SOlfi/'WtjmK " i <Ча ZOW+W(Jwl\ im t?nffi гЩ^г Щ, I I w 1/сек 1 w 7/сеп -\^< ,У'мй ' т; fjSa ш 7/сек i w щ 7/сеп 1 1 tw - 1ё \1a 1 L66 yr r L,W ■;, Ш — Ю V< \!u 2oieWrM\ 1!35~ i 7П&-. - ^дён,* **Щ ?Mlq\Mju/>\ , — *Zi j">o w l/cex „66 ■ WB№/ „66 j бек^ | "*" I i -Mo 2t 1 Wl?W<j6>)\ ti-i n ** _,_ л -"o "o ZGWWfJcoi\ - UK ~(a Mm tllgW, \ ^-foo'r CO „s I/cat -»'^# -if"fa/rae со 7/Cff/f ' *-■'- to wr 7/cen J | Рис. 365.
552 приложбш-ш i.d6\zei!/\r*Mj«»\ ю p -я -w - lift IS w w s -IB Ft),5 "№ ^кШ=о. ®E« \~$$ ioiff\f*mo>>\ 4> '"£/. s Г =3 £_ IfiSi л ~>'+щ& w 3 Г/CfJft г i V- £_ £#7'' £*&•' H»'" -фФ W£.^»^ ^ш л=аз t\' УЫ5 ■ I> ' 't w i},~ZSa)/j 1/ceit Lffi\ mgW(Mi\ \5~Z7u ~W'*£k '.'■}^~ Щ/Ша, £■* + - --#?£ &.'".'! •* ш m.ff\w<jm\ f ,ЛпМ i Jlj%&. "Я/ _.i. Wfy^ r 4 -+■■ 4- 4- 4 4 J/a#^ \'\**^ ft' 'y'' <r Г i (til/сёк (-0.Z 0.5 -1' f 1 ftl / 'flSbhZ&ib l/cex r ■ ■ 4 t i . . i |l I I i' 1 il 1 . id 20lg\frW(jw)\ —> ' hJ^i w Г" " ft> 7/csk gOliAI+WOanS % 'S^ \ »fc 1 ar. 1BK to f/свл L.66 4c г Ш -- # •I r j0lOWtj(D>\ ~Ш\Щ I I .-,. ^ [ ГТ S£* ■ & /ВД1 $ 1 ч '.~Ч ,' It I .1..Ч1 1 | _| ,D6 m ■ ?t's-^ ,1 ir*tw. 1 И" III- <y 35 w ■ .-■ ' «a ; T/ce/t III I1 '"№ Ml TJ] HI IJJ 111 ту jljll 1 1 Ш II Рис. 366.
ПРИЛОЖЕНИЕ 553 _. _ % гош*юоан\ i 11 -РП36 Ьхш ч К 1 1 ZOl<A7*Wico)\ to, '*$ ' Ч 20l(/\JfWt 1 И А„Д? \?°де,; \ 1, з"т i?/\ 1 " \— ""' !Ш 1Ш 1' 1 w 7/сек аб Л iex to , We/) _.и , Ш)\ 'о Л'0,5 20Ш*МС/о»\ -го i -fa тй? дек ^>i ШШ0\; (О 7/сек V Ы ■;:" 7/сек ш С.дб —6 Ltd6 -\ю -7? ~ 20 '-М --15 mg\mjto>\ ' Г-- П -?n&> ^22 °fc ШрМГОшЛ f«!/JW,\ Р- .. лш ~&ъ ~ ~ттз& со 7/сея со ^Ь^-/»^ //«ЭТ Z{?lg\W(Ja>)\ *t >э, V<* _LL \J~unM- ■\~"й?/г * iX Ш№\£ T fs? -i. — /*ь Очч к ■■-. |_ ~^ <87 sO.!!\ it°J Йч^ jS^ < ' s Ъ*"о ■ * "s^ >о w gk ->пдб Vobh г^сЧсж т 4 Х!*^ Ч. > ■ч,к ■^0.5. \ -\ lqW(ftu)\ т I S. "О <*0,3-- ^-W2\ SI. &?*.. со I 1/сек L ■ I I Р\ & —1- ек 1 Рис. 367.
554 ПРИЛОЖЕНИЕ [26 26. Типовые л. а. х. систем с ЦВМ Степень астатизма Типовые л. а. х. Дискретная частотная передаточная функция разомкнутой системы ■Чи\ ' ■гт. Afli? }S. г/)о^ Kb (i+A*i) (i-A 4) 11 + '^ (-Т-ге)] А(1+Аг,)(1+/^-^) £й? Ф.град i i г *е (1+А*0 (l-Д £■) [i+A (-у-Ге)] CW>(i+/*-£) Выбор ^ илн Кс производится в соответствии с требованиями, предъявляе
261 ПРИЛОЖЕНИЕ при учете квантования по времени 555 Таблица П. 10 Л. а. х. непрерывной части системы "*Я& Ш Передаточная функция непрерывной части системы К(1+т,р) 0+V)(1+V)-(1+W 1..дбЖгрп<)L I Kq (l+t,p) P(l+V)(l+r3P)...(l+V). US ф, град , ,, ' i Lit Т, T,lL /Се (1+т1Р) Р2(1 + Г8р)...(1 + Ггр). Расчетные соотношения Т^ТГ^+Т4+-+Тп или ti> М-1 К' м-\ 1=3 fc 1 Ум (м-р М+1 или -|-+ 2 7г<х ,2 юГ„ I М М+1 мыми к точности или быстродействию системы с ЦВМ.
27. Типовые последовательные дискретные корректирующие звенья Таблица П.П Наименование звена Передаточная функция непрерывного аналога Дискретная передаточная функция D (г) Частотная передаточная функция Переходная характеристика Дифференцирующее 1+Тр z 0<а<1 1 + /^Я G0=(l-c), х- 1+а 1-а 5ГТ Аналог пассивного дифференцирующего Go 1 + Г,р 1+Г2р -I» °~ Г, ' Ti>T2 0<а<1, 0<6< 1, а> 6 _ 1 + /т,Я 1 + /т2Я 1+а Г0 1-а " 2 т2 = 1-а = 1-6 1+6 Интегрирующее Р 1 Тр г-1 1—j -5s- Л а 2 1* иг
Изодром- ное 1 + Тр 1 + Гр Тр 1+- а ~Т^ 1 + /тЯ ■<М) Л Аналог пассивного интегрирующего 1 + 7> 1"+Г,р' (1 + g)z+l -a (l + 6)z+l-6 а > I, 6 > 1, а< 6 1 + ?т2Я _ Г0 + 2т2 -1 + /т,Я ': ° Г0 + 2т, ' # 2 Интегро- дифферен- цируюшее 1 + -=- + тр 1Р az г — 1 + - г — '1 г а< 1 t+/ar0X + (l+-j)r«.y*)« /г0л(1 + /^л)"- Uf-T ^ГГТ -^ ц» 1. В таблице использована псевдочастота Я, = -=- tg —jp*- -1 2w . 2 'о 1 Го ' Т0 z+l ■ 2. Переходные характеристики построены для дискретного фильтра с запбминанием на период повторения.
28. Коэффициенты гармонической линеаризации основных нелинейных характеристик Таблица П.12 № п.п. 1 2 Статические характеристики нелинейного звена Идеальная 0 релейная х, \ Релейная с зоной ~~У тч/встеалитавт \ * IE. а з, Q(a> Ac эта па V a2 при а~^Ъ i q' (a) 0 0 -
Релейная с гостерезватй петлей •% а b с X, зга г а2 при а~^Ъ . АсЪ яа2 при а~^Ъ Релейная общего вода хе\ -jy 1 г* *, 2с_ па при а^ b (l-m) 2с6 яаг при а~^Ъ С насыщением хг у/ у<а\ 0 Ь х, K=tgu Ik { . b b , /, 62 \ —-1 arcsm — + — 1/ 1 в-) я V a a V a* J при а^Ь
Продолжение табл. П.12 §»■ _ о Статические характеристики нелинейного звеиа q' (о) 6 7 С зоной нечуее/яви/пелы/ости и насыщением ~ /Г /° K*tga без tfoct ~А щеиия Лес К" iff a -'" 2k ( .62 ■".- bx — 1 arcsm — — arcsm Ъ я \ а. а а у ■ а1 а V а2/ прн а~^Ъ2. , 2k 1 . Ь Ь -./ , Ь«\ к --arcsin 1 1/ 1 Т-] 31 \ а а У а21 при а^Ъ 0 0
Типа люфта им/ зазора li'tga С насыщением и зистерезисной петлей 3)г 1 г ft 7 ^ с X, k-tffa Сухого трения Г? рх7 к Г я , . (. 2Ь\ , -|_- + arcSm[l-—) + при а^ Ъ k\ . c + kb , : c~kb — arcsm —; !- arcsin —; \- zi L ka ka , c + kb .,/, (c + kb)' , + fax" У ' № + . c-kb _/ (c-fefe)2] T- fca Г fc2a2 J ^ c + kb при a ^ — AF v — амплитуда скорости па \ а } при a 5= 6 _ 6с па2 ^ c + kb при п^ — 0
Б62 ПРИЛОЖЕНИЕ [ев. 29. Нормированные амплитудно-фазовые частотные характеристики релейных исполнительных механизмов Релейный исполнительный механизм, состоящий из реле и двигателя постоянного тока с независимым возбуждением (рис. 368, а), рассматривается как одно нелинейное звено автоматической системы. о) *) Реле X г—£_ I ШЕ LTZ1 I и„ Г: -ь о «ЕГ Ш, Рис. 368. Релейный исполнительный механизм. Статическая характеристика реле изображена на рис. 368,6. Гармонически линеаризованная нормированная передаточная функция- релейного исполнительного механизма W0 {d, jz) = q0 (d, z) + jq'0 {d, z), где d — -r — относительная амплитуда, z = сзГм — относительная ча<- стота, Тм — электромеханическая постоянная времени двигателя. Нормированные амплитудно-фазовые характеристики построены на рис. 369 и.370 для различных значений коэффициента v. учитывающего влияние статического момента нагрузки Ма 1 - Л/о' где М0 — пусковой момент двигателя. На рис. 369 пунктиром показаны нормированные амплитудно- фазовые характеристики, полученные в результате перемножения нормированной гармонически линеаризованной передаточной, функт- ции реле (см. приложение 28) Wn(d,jz)- л V d* и нормированной амплитудно-фазовой характеристики двигателя остояниого тока с независимым возбуждением И7д(/г)=- 1 1 + \г • Сравнение этих характеристик показывает, что при небольших значениях относительной амплитуды {d < 10) раздельное рассмот- ение реле и двигателя является недопустимым.
ПРИЛОЖЕНИЕ О 0,1 02 03 • 0.4 _Q5 0,6 Рнс. 369. Нормированные а. ф. х. для случая Afa=>0.
- - П РИЭТ9ЖЕ«Н£- • О 0.1 О/ W 0,5 0,6 /7=// Рис. 370. Нормированные а. ф. х. для случая ЛТщ^О.
з»1 • ПРИЛОЖЕНИЕ 565 30. Коэффициенты статистической линеаризации некоторых типовых иелинейностей Если на вход нелинейного звена со статической характеристикой y = F(x) действует сигнал, представляющий собой случайный процесс •■ х (/) = тх (0 + х° (0, где mx(t) — его математическое ожидание, x°{t)—случайная центрированная составляющая, то процесс на выходе нелинейного звена приближенно может быть представлен в виде 1 </(0 -k0mx(l) + kax°(i): . При заданной одномерной плотности вероятности w (x)' коэффициенты статистической линеаризации &„ и ка вычисляются но формулам *о = - тх kt=- Ох Г Г (тх + х°) w (х) dx + F (тх) -оо I xw (х) dx —оо ~ +оо F2 (тх + х°) w (x) dx — т2у х2ш (х) dx £,= /fjry (0) «л- (0) Г /=■ (mv + -*;0) xw (к) dx x2w (х) dx *a=ir.(fei + ^2). Ниже приводятся значения этих коэффициентов и их графики для некоторых типовых иелинейностей при действии нормально.., распределенного входного сигнала, имеющего нормированную плох» иость вероятности ш (г) = —^=г е
ЙРГОГОЖЕЙЙЕ У , 0 'л с в) / ■ 0,8 0,4 0J 6) Рис. 371. Графики,для идеальной релейной характеристики.
80] ПРИЛОЖЕНИЕ ^ 567 ЕЕ сГ а) с 0.8 0.6 0,4 . бгО 0,3 К/\ \^ ^ м I f) 0.8 0.6 0,4 02 . б, = оо- Д5- J&2. \^~. с 0,8 0.6 0.4 02 J У /Л N б,. ^ ч ОО' \/ ^ 3 2 ~~-^ в) з т& о ь е) Рис. 372. Графики для однозначной релейной характеристики с зо» ной нечувствительности.
•568 ПРИЛОЖЕНИЕ ''ISO и интеграл вгербятности ■..■■■'■ -. "г3 Ф (»)■=- ,— \ е 2 dz 1) Идеальная релейная характеристика (рис. 371, а) о, /2Я Графики коэффициентов приведены на рис. 371,6, е.' (П.22) (П.23) (IL24) •i \ -с ?1 с Ъ ' X -& -А JL Ч +4 JT, Рис. 373. Рис. 374. 2) Однозначная релейная характеристика с зоной нечувствительности (рис. 372, а): *-£И^)-(^)]- *-чг{-^-(^)-(^)Г. <" (П.2Б) 26) Г 1 /I+miV I /1-miYl ffx V2n m, mx 0i=- o, •'- ь ' --6 Графики коэффициентов приведены на рис, 372, б, в, г. (П.27) (П.28)
:М1 .ПРИЛОЖЕНИЕ , 5S9 3) Релейная характеристика с гистерезисом (рис. 373).;':: ... , Коэффициенты k0 и 1г2 определяются соответственно форму- лама (П.25) н (П.27) fel=="57i,"^J • (П29) 4) Неоднозначная релейная характеристика с зоной нечувствительности (рис. 374) ' ' ' ■■■■.•■ (П.30) с Г khnl 1 Г / 1 + гаЛ /l-m.N *,=——-4=гх " ': " ах 2V 2п Г I /ItmA' 1 / ;-ш, V I / лч-m, у. I / v-m, \'1 х[е 2 v а,;+е su, ; +е 2 v o,j.+e 2 v о, ;jf (П.32) 5) Линейная характеристика с насыщением (рис. 376, а) +_fL,U 2Л о, ; _е .2 v а, ;ji . (пзз) ^{■-^«♦«-оК^Ь^)]- Pi (t-Wi) с 2 V о, J ff|(1+j?t|) g 2 \ a, M ^ (П.34) Графики коэффициентов представлены's ha рис: 375, б, в, -е.
570 ПРИЛОЖЕНИЕ ISO Рис. 375. Графики для линейной характеристики с насыщением. 1 ,-ь / У у А*. ь Рис. 376.
«1 ПРИЛОЖЕНИЕ 571 { ис* ™?ейнаЯ хаРактеРис™ш с зоной нечувствительности ^ = 4,~7^[(,+^ф(-^)-(1--.)ф(-Ц^)] + ,_,{.+^1-«-['*(^Л*(^г)- <7,Т2Я J (17.37)- (П.38) /=tga. SI. Нормированный интеграл плотности вероятности нормального закона распределения Значении функции Ф (и) = /2л и 2л J da Таблица П. 13 и 0.00 0,02 0,04 0,06 0,08 <Х>(ы) 0,000 0,008 0,016 0,024 0,032 и 0,66 0,68 0,70 0,72 0,74 Ф(и) 0,245 0,252 0,258 0,264 0,270 и 1,32 1,34 1,36 1,38 1,40 <Х>(ы) 0,407 0,410 0,413 0,416 0,419 и 1,98 2,00 2,04 2,08 2,12 Ф(«) 0,476 0,477 0,479 0,481 0,483
672 ПРИЛОЖЕНИЕ ;ЦЯ Продолжение табл. П. 13 и 0,10. 0,12 0,14 0,16" 0,18 0,20 0,22 0,24 0,26 0,28 0,30 0,32 0,34 0,36 0,38 0,40 0,42 0,44 0,46 0,48 0,50 0,52 0,54 .0,56 0,58 Ф<!<) 0,040 0,048 0,056 0,064 0,071 0,079 0,087 0,095 0,103 0,110 0,118 0,126 0,133 0,141 0,148 0,155 0,163 0,170 0,177 0,184 0,192 0,199 0,205 0,212 0,219 и ■- о,7ё 0,78 ; 0,80 0,82 0,84 0,86 0,88 0,90 0,92 0,94 0.96 0,98 1,00 1,02 1,04 1,06 1,08 1,10 1,12 1,14. 1,16 1,18 1,20 1,22 , 1-?=* Ф(н) 0,276 0,282 0,288 ■ 0,294 0,300 0,305 0,311 ' 0,316 0,321 0,326 0,332 0,337 0,341 0,346 0,351 0,355 0,360 0,364 0,369 0,373 0,377 0,381 0,385 0,389 0,393 и 1,42 1,44 1,46 1,48 1,50 1,52 1,54 1,56 1,58 1,60 1,62 1,64 1,66 1,68 1,70 1.72 1,74 1,76 1,78 1,80 1,82 1,84 1,86 1,88 1,90 и «£>) 0,422 0,425 0,428 0,431 0,433 0,436 0,438 0,441 0,443 0,445 0,447 0,450 0,452 0,454 0,455 0,457 0,459 0,461 0,463 0,464 0,466 0,467 0,469 0,470 0,471 и ' 2,16 2,20 2,24 2,28 2,32 2,36 2,40 2,44 2,48 2,52 2,56 2,60 2,64 2,68 2,72 2,76 2,80 2,84 2,88 2,92 2,96 3,00 3,20 3,40 3,60 ;Ф<«) 0,485 0,486 0,488 0,489 .0,490 0,491 0,492 0,493 0,493 0,494 0,495 0,495 0,496 0,496 0,497 0,497 0,497 0,498 0,498 0,498 0,499 0,499 0,4993 0,4997 0,49984
'-ПРИЛОЖЕНИЕ . 573 Продолжение табл. П. 13 и 0,60 0,62 0,64 Ф(и> 0,226 0,232 0,239 и 1,26 1,28 1,30 Ф(и) 0,396 0,400 0,403 и 1,92 : 1,94 . 1,96 Ф (") 0,473 . 0,474 0,475 и 3,80 4,00 4,50 5,00 Ф (и) 0,49993 0,49997 0,499997 0,49999997 32. Нормированная плотность вероятности нормального закона распределения Значения функции w (z) = VW Таблица. П.14 0,0 0,2 0,4 0,6 0,8 1,0 1,2 1,4 1,6 1,8 2,0 2,2 2,4 2,6 2,8 3,0 ,. 0 0,399 391 368 333 290 0,242 194 150 111 079 0,054 036 022 014 008 004 l 399 390 367 331 287 240 192 148 109 078 053 035 022 013 008 004 2 399 389 365 329 285 237 189 146 107 076 052 034. 021 013 008 004 3 399 389 364 327 283 235 187 144 106 075 051 033 021 013 007 004 4 399 $88 362 325 280 232 185 142 104 073 050 033 020 012 007 004 5 398. 387 360 323 278 230 183 139 102 072 049 032 020 012 007 004 6' 398 386 359 321 276 228 180 137 101 071 048 031 019 012 007 004 7 398 385 357 319 273 225 178 135 099 069 047 030 019 011 007 004 8 398 384 356 317 271 223 176 133 097 068 046 030 018 011 006 00.4 9' 397 383 354 314 269 220 174 132 096 067 045 029 018 011 006 003
33. Моделирование элементов структурных схем автоматических систем на операционных усилителях 23 Таблица П. 15 Название элемента Модель элемента Передаточная функция или уравнение элемента Сумматор «п+1= i-i 2 3 ' Инвертор Безынерционное звено а, -сУ- <г£ь > JL ■ "г «2=-—s^wi^— щ, Яо=-Я| — «2 =■ kll\, k ~ "75—
Апериодическое звено первого порядка См. также рис, 343 Апериодическое звено второго порядка с, > > См. Также рис. 345 Щр)=- £-= — Я. ' Tp + l' T = R0C W(p)- ry + rlP+i R\Rb * Rf> R<R2RSC Г,= R*R* i, Тг>2Тя.
Продолжение табл. П.15 Название элемента Модель элемента Передаточная функция или уравнение элемента Колебательное звено Схема совпадает со схемой набора апериодического звеиа второго порядка (позиция 5). См. также рнс. 343, 345 - Консервативное звено '(/>) = 7V + 2g7>+l ' * I _ / R4R2R3C1 *. , Г(р) = - -,А_ 7V + 1 ' RiRs' У Rb /?5 = 00 (отключено), | = 0. Идеальное интегрирующее звено (интегратор) X ^^ X ^w-j.*—-йг-
Инерционное интегрирующее звено Изодромное звено я. Ыи и, L k = р(Тр+1) Rs CtRtR2 , r = /?3c2 — it- *— £ (1 + Гр) "Же , Г = Я2С
Продолжение табл. П. 15 2J ее Название элемента Модель элемента Передаточная функция или уравнение элемента Инерционное днф- фер енцнрующее звено W(p)- kp Tp+l k = R2C, R&C R5Re Примечание. Апериодическое звено второго порядка можно получить с помощью двух последовательно соединенных моделей апериодического звена первого порядка.
34. Моделирование нелинейных статических характеристик на операционных усилителях Таблица П.16 Статическая характеристика Схема, моделирующая статическую характеристику Уравнение или аналитическая запись статической характеристики Линейная с ограничением или насыще ■ ннем Идеальная релейная У"г \ J>, -0з_ Uo —kuu -и» kuu - 0< при k «1 и, < < > и, ^3 k' и, к ' <о, и2-- Ut прн ui< y> = -jr при Гдобр = °°; Гдпр, Н и г4 «С Rv щ = Г- U4 8 ПРИ Щ > О, при «] < 0. #о = °°. '"добр = °°: /дпр, гз н г4 < Яг
" Продолжение табл. П. 16 Статическая характеристика Линейная ' с зоной чувствительности X. b'i и, "X Схема, моделирующая статическую характеристику Уравнение или аналитическая запись статической характеристики ( —k (и\ — U() при «, ^ Uit «2 = V к{щ + U3) при щ < — tAj,- I 0, — £/s < щ < f/4. k = ~- при" г добр — w; Оици гз и г4 -С /?i. Релейная с зоной не чувствительности. "г -!&± " ''С L 1^ v 1 'Ч / £L > Оэ j" —1/5 при и, > t/4, «2=1 ^6 При Mi < — Uiy { 0, —1/8 < u, < t/4- гдобр = °°; ^пр ~ О, г3. г*, г5 и г6 «С Я0.
Релейная с гистерезис- ной петлей 1 ~ut \ > .-"«. иг В Щ Р. о, -Us при Ht —Е/8<(Ц dU] -£Aj При -Mi-— £^a>0, J при .4/..: >0, U4 при ы, + Ut>0,\ dui -£/, при U] + t/4<0j при~5Г<0 Люфт или зазор ^ JL ■ "' U"2 — к(щ-ил при ~>0, *(«,+'£/,) при ^-<0, const при — UA^.—^- — к, < £/3 А =
ш и и ьо ~ _— р "сл "о сл Ъ "сл "о сл 0,0 И5 "сл 9,0 8,5 8,0 7,5 7,0 6,5 1° 5,5 5,0 4,5 л. "о со со ьз ьэ »—,•■* сл "о сл "о "сл "о. р р OOOOOOOOOOOOOOOOOi—' ^' *—' ^' *—' *—t (О со "со <D b "(О Ъ "со '"со to "со со со "со "со "со' "со "со оо "со "оо Vj "en "сл "*» — — слслслсл>******сосоьо>— ►-©ооооооосл — сл^^**г-сло ООООС0^05СОЬО*»СОО^*.*->(>.ОСЛСО^*>.СЛ>)^ЮСООООО ооорроррр "en ~т ">ь. Ъэ — о оооооооооооо w^wj-wwwwwwwj^OOpPPPPppppOpOpP toco^4o^^oto^^^^to^to"co"coto^"oo'coVjVj'biT^'co"^— "о COnoCOOOCOCOOO^J^JffiifJiejlJ^J^J^J^J^COK-irnoltOOCOOitOOiO со о 5 . .- _ — - yj yj yj yj yj yj wj CD CO 00 CO 00 00 00 ОЮЮООЯЯИВМСЛО-ИЛЮМОСвМО •J CD СП СЛ О H-»-_j-h- — — ^-,-,-ppopoppopoopppooopp о "о о о "о "о "о "о "о о Ъ ю "ю "ю Ь t> "(о b b о в Ь s о "*. "и - о ►— 1— — — — р— — ооюсомш^мсвш^свсоюьасомюл^о >"ИВСЛО)01и!ОЛЧ(0(ЛОЮОВО)ШОВ»»(ОЯ*.0010 «-*- — ►-*- — — м^»£_».н»1— »*».».» _ оорроооорр о о о о о Ъ "о о о о о о , о о "о о "о "о to "со "со оо Vi "го сл ы ~~- о »»ювмюммю«>-ооод»«-ю^1ос««)!л»ся1Х10 S(£lBOiStDO»01M0100010l8uiOM4'*I»U"-ui01C»^0 ►-►-►-►-►-►-►-*___ --j->-j->-j->-^-i— I— popo О"~0 О О b "о "о Ь b "о "в b "о "о "о о Ь "о Ь Ь "о "о "о b "в Ь Ь Ъ и !о "►• "о и >- >- В М U СО .СО (3 В В В » м»со£*.М001ЮОВИ-)(00 йСлюАВ-со-юсл-оойОЧсамюсосооввсо'-во ►_._►-►-►-_— _j-j- ►-£-£-„- _„^-„^-*__р pop ОООО b b "о "о b b "о "о "о b'b о b "о "о b "о "о "о b b "e » s b V - о 00----ЮЬОЬО(ОЬОЮЮЮЬОСО>^СЛ05СЙСЛСОСОЬОС000550СОО ОСЛОСЛ>-СЛОО-3^1СЛЬЭ.1*СЛСЛСО.(»^1^1^1СО>Й.ООСОСООа5СОО OOO — "- — «-►-«-"-«-«-«-'—"-«-"-«-■— — «-OOOOOpo "со "со 'со Ъ о о о о о о о "о о Ъ о о "о "о "о "о о "со "оо Vj "сл ~& "to "о (Ов(00»-»миммВМиСЛ01В1Яя5В!Л01ВЯООО OililOit'OuSlOlOOB'-ol^OCIWNOOiliB-COO'-SO ороро«- ►-►-«- j- >—j-«-«-»-j-j-j-«-lj-^"ppppopp b b b b b о "о "о "о "о "о "о о о "о о "о TS— "— о о "со "оо JVj Ь "*> Ь "о оососососооооооо*-»-со>б*^соросослоо co-Wen о •-*•-* о MfflO(>ioBoioiaBs»s со^со о.сосоооо^а — ►» w s w о о op dooooopppj-i— j-»-j-*»j-»-j«;-pop©pp "со V Ь Ъ "со "(О Ъ "со "со "со Ъ "со о о "о "о о V ">- ">- о "о b "s b "^ "в "о СОЯЯСО(0(0(ОСО(ОЮЮ!ООЙ*>0)!0>»>-ОЧ>--0>-СОМО ^ сл с» м - ииисовюслслиисослвсл~)^ис1>а> ,^j to со о ° ° р Р Р Р Р ° Р Р ° Р Р Г* Г* J" ■Г* Г* S" Г* .Г* Г* Р Р Р Р Р Р "со Ъ Ь Ъ Ъ Ъ Ь b Ъ b'fo Ъ Ъ Ъ Ъ Ъ "о •- ^ ^* Ъ Ъ Ъ со *fe ^ N3 о ааяяяо(омя^^.яяоий(ом1ою(5содм^л«о MSOJSlD>-'-41BMS'-(DCDWMS*4(Di^WroWOO)^'-0 И I
SSI ПРИЛОЖЕНИЕ 5S3 функций A (t0) -w.1 Таблица П.17 "0,50 0,000 0,240 0,461 ' 0,665 0,833 0,967 ; 1,061 1,115 J,-M2 1,138 1,118 1,092 1£5J 1,018 0,993 0,974 0,966 0*^966 0,970 0,975 Д982 0,987 0,993 | 0,997 ^0,997 0,997 0,997 0,998 0,55 0,000 0,248 6,476 0,685 0,856 0,985 1,082 1,132 1,152 1,134 1,115 1,083 1,037 1,001 0,975 0,958 0,951 0,949 0,960 0,972 0,985 0,996 1,002 1,006 1,006 1,006 1,006 1,006 0,60 0,000 0,255 0,490 0,706 0,878 1,010 1,100 1,145 1,158 1,134 1,107 1,070 1,021 0,982 0,957 0,944 0,941 0,944 0,961 0,980 0',993 1,007 1,014 1,017 1,019 1,018 1,014 1,010 0,65 0,000 0,259 0,505 0,722 0,899 1,031 1,117 1,158 1,159 1,138 1,098 1,050 1,003 0,965 0,941 0,926 0,935 0,948 0,966 0,987 1,006 1,017 1,027 1,029 1,026 1,019 1(012 1,005 0,70 0,000 0,267 0,519 0,740 0,919 1,042 1,130 1,161 1,160 1,132 1,084 1,032. 0,984 0,948 0,927 -0,922 0,932 0,951 0,976 1,000 1,020 1,033 1,039 1,037 1,027 1,017 1,005 0,995 0.75 0,000 0,275 0,534 0,758 0,938 1,060 1,142 1,166 1,161 1,127 1,069- 1,016 0,966 0,936 0,917 0,911 0,936 0,958 0,990 1,015 1,036 1,046 1,047 1,043 1,025 1,010 0,993 0,982 0,80 0,000 0,282 0,547 0,776' 0,956 1,078 1,154 1,171 1,156. 1,111 1,053 0,994: 0,949 0,920 0,911 0,920 0,944 0,974 1,006 1,033 1,049 1,054 1,048 1,034 1,015 0,995 0,980 0,968 0,85 0,000 0,290 0,562 0,794 0,974 1,098 1,164 1,174 1,149 1,099' 1,037. 0,979 0,934 0.910 0,908 0,927 0,955 0,990 1,023 1,048 1,059 1,058 1,044 1,024 1,000 0,979 0,964 . 0,958 0,90 0,000 0,297 0,575 0,813 0,986 1,113 1,172 1,175 1,141 1,085 1,019 0,962 0,922 0,903 0,909 0,934 0,970 1,006 1,039 1,059 1,063 1,055 1,034 1,010 0,984 0,965 0,955 0,954 0,95 0,000 0,304 0,593 0,832 1,003 1,125 1,176 1,175 1.131 1,071 1,001 0,951 0,914 0,903 0,915 0,946 0,986 1,023 1.053 1,066 1,062 1,048 1,021 0,994 0,969. 0,954 0,950 0,958 1,00^ 0,000 0,314 0,603 0,844 1,020 1,133 1,178 1,175 1,118 1,053 0,986 0,932 0,906 0,905 0,925 0,958 1,004 1,041 1,061 1,066 1,056 1,033 1,005 0,977 0,958 0,949 0,955 0,970
to ■<y> to СП ел "о to N5 fcS о о "ел "о со со "ел о йо "ел со о 'ел СП СП "ел "о ел pi л. ел "о ел со со -J -J ст ел о "со р р о "со "со 'со -О -<J ~J ел >*> со о ■со СП со о "со СП 00 о о ел се р р to се СП СП -J -J о "со СП СП о СО ст> ел р Р' со Ъ СП СП *■ — о р XD СО ел ел СО СП о СО ел о со ел о о о о .ел ел 9 8 ел 8 о о о о о о ел ел N5 о о о о о о ю to м о о о о ю о о о о о о о о в о о СО "со СО СО СО -J о о о "со "со "со SCO 00 О СО о о о о -О 00 о о 00 о о о — со о о о о о ~ ~ о о о о ю со £ ° >ь. ел о ел о ел о СП о о сп ел о о о о * м » - о Ъ СО СО О ■Jo со ел о "со со со о со со ел о о о о ■со со ел I § о ■со со СП о "со со СП о о о о о о (О U >9 S2 S2 0 "со "со "со со со со СП СП СП о о о о о о ю со со р to со ел со со ел 8 8 ■с» ел Р о "со "со СО СО *> ел 8 S СП 00 Р р со "со со со СП 00 о о о о 8 о СО 8 ел о о о о о о ел сп о о о о >£ь >ёь ел сп о о о о СП -J S 8 00 СО 1 о о со "со со со со со о ■со со со р р "со "со со со со со о о "со "со со со -о ел о "со со о СО со N5 р р со "со со со о со со р "со со ел р "со со ел о СО о "со со ел "со со ел о р "со "со со со СП -О — •— ,Г-*.7*,Г'.Г".Г*'-*.Г'Р000РРРРРР0РРРРР ^э'сЪ"осэ'сЪ'о'©'со^^'со'с©То^о'со'^'со"со'со"со'с©'ш'о"со'со OOOOOOOOOCDCDCOCOCOCOCDCDCOCOCOCDCOCOCOOO л^слелсллльоосо^спслслслслслл^со — сооо^оо — _-" "~ "Г*-"" "~ ■"."" "~ ** ",""Г" *■* .Г" .Г" Р Р Р Р ^ ' Р* £> ° "о "о 'о "о "о о о о "о "о о b "о "о "о "о "о Ъ 1с "о "ю "о "со о <о о о о о о о о *о ооооооооососососооооооооо MUit>O140SSO)CIl*.C0M-'--'-MCO4CJ»C0S&U О О .— «'--- ~ ►- w— — — —„>-„-' — — —' — — -- О О р О О "со io Э "о b b b "о "о "о b b b Ь b "о о "о "о о "в "в в "в Ь (осоооооооооооооооооез. осососооосв р р О О р © р о „— Г" .*" „-* Г* Г* 7" 1Г Г" jr f J"" .— Г* .— Р Р CCicDcOCOCDCDCOcOOOOOOOOOOOOOpOOcacD OCDCDCDCQCDCOCOOOOOOOOO — — .— ООООСЭСЭ Cl0)0!*JNC0©CDO« N-t— Ю^ОСООООСО^Л-а-ОСЯ — ® ш
ПРИЛОЖЕНИЕ 685 Продолжение табл. П.17 fl^o 1,000 1,002 1,005 1,008 1,011 1.0П 1,012 1,009 1,008 1,006 1,001 0,993 0,936 0,995 0,995 0,996 0,996 0,997 0,998 0,999 1,000 1,000 1,000 1,000 1,000 0.55 1,006 1,006 1,007 1,007 1,008 1,008 1,007 1,005 1,002 0,999 0,995 . 0,992 0,991 0,991 0,993 0,995 0,996 1,000 1,001 1,002 1,002 1,002 1,002 1,002 1,002 0,60 1,008 1,005 1,002 1,001 1,001 1,001 0,999 0,997 0,997 0,995 0,993 0,992 0,992 0,994 0,997 1,000 1,000 1,004 1,006 1,007 1,008 1,006 1,004 1,002 1,000 0,65- 0,999 0,994 0,993 0,993 0,994 0',996 0,997 0,998 0,998 0,998 0,997 0,996 0,995 0,996 0,996 0,995 0,997 1,000 1,001 1,002 1,003 1,003 1,003 1,002 1,001 0;70 0,987 0,983 0,983 0,985 0,990 0,995 0,999 1,002 1,004 1,003 1,004 1,003 1Д)3 1,001 0,999 0,998 0,997 0,996 0,997 0,998 0,999 1,000 1,001 1,002 1,002 0,75 0,974 0,970 0,976 0,984 0,993 1,001 1,008 1,012 1,014 1,012 1,009 1,005 1,001 0,996 0,993 0,992 0,991 0,992 0,994 0,997 1,000 1,002 1,003 1,004 1,004 0,80 0,965 0,969 0,978 0,991 1,003 1,014 1,020 1,023 1,020 1,014 1,006 0,998 0,991 0,986 0,983 0,986 0,991 0,998 1,002 1,007 1,008 1,008 1,005 1,004 • 1,002 0,85 0,961 0,971 0,987 1,003 1,018 1,027 1,030 1,027 1,018 1,007 0,995 0,985 0,979 0,976 0,975 0,988 0,997 1,008 1,015 1,017 1,017 1,014 1,008 1,001 0,987 0;90 «,965 0,981 1,001 1,019 1,031 1,036 1,032 1,023 1,008 0,993 0,981 0,973 0,972 0,974 0,981 0,997 1,012 1,022 1,025 1,023 1,015 1,005 0,991 0,986 0,984 0.95 0,976 0,997 1,017 1,032 1,039 1,038 1,027 1,013 0,993 0,978 0,969 0,967 0,974 0,990 1,002 1,013 1,024 1,028 1,027 1,023 1,012 0,995 0,985 0,978 0,977 1.00 0,990 1,010 1,030 1,040 1,039 1,028 1,012 0,988 0,979 0,969 0,967 0,973 0,985 1,001 1,016 1,024 1,029 1,026 1,016 1,002 0,988 0,979 0,975 0,977 0,983
ЛИТЕРАТУРА . . 1. А н и с и м о в В. И., В а в и л о в А. А., Фа т е е в А. В., Сборник примеров и задан по линейной теории автоматического регулирования. Госэнергоиздат,. 1959. 2. Б е с е к е р с к и й В. А., Попов Е. П., Теория систем автоматического регулирования, «Наука», 1966. > 3. Б е с е к е р с к н й В. А., В о с т о к о в С. Б., Ц е й т л и и Я. М, Электромеханические сглаживающие устройства, «Судостроение», 1964. 4. Б е с е к е р с к и й В. А., О р л о в В. П., По л о и с к а я Л. В„ Федоров С. М, Проектирование следящих систем малой мощности, Судпромгиз, 1958. 5. Б е с е к е р с к и й В. А., Ф а б р и к а и т Е. А., Динамический синтез систем гироскопической стабилизации, «Судостроение», 1968. 6. Васильев Д. В., Митрофанов Б. А. и др.,• Проектирование и расчет следящих систем, «Судостроение», 1964. 7. Васильев Д. В., Чуич В. Г., Системы автоматического управления, «Высшая школа», J967. 8. Воронов А. А., Основы теории автоматического управления, части 1 и 2, «Энергия», 1966. е 9. Джеймс X., Н и к о л ь с Н., Ф и л л и п с Р., Теория следящих систем, ГИИЛ, 1953. . 10. Егоров К. В., Основы теории автоматического регулирования, «Энергия», 1967. 11. Ивахненко А. Г., Техническая кибернетика, ГИТЛ УСС'Р, 1959. 12. Казаков И. Е., Доступов Б. Г., Статистическая ди-- намика нелинейных автоматических систем, Физматгиз, 1962. 13. К а з а м а р о в А. А., П а л а т и и к А. Т., Роднянский Л. О., Динамика двумерных систем автоматического регулирования «Наука», 1967. 14. Коган Б. Я-, Электронные моделирующие устройства, Физматгиз, 1963. 15. Кр а со в ский А. А., Поспелов Г. С, Основы автоматики и технической кибернетики, ГЭИ, 1961. 16. Кузин Л. Т., Расчет и проектирование дискретных систем- управления, Машгиз, 1962, 17. Кузовков Н. Т., Теория автоматического регулирования, основанная :на частотных методах, Оборонгиз, 1960. 18. Лет о в А. М., Устойчивость нелинейных регулируемых систем, Гостехиздат, 1955. - ' 19. Лурье А. И., Некоторые нелинейные задачи теории автоматического регулирования, Гостехиздат, 1951.
ЛИТЕРАТУРА S87 20. Павлов А. А., Синтез релейных систем оптимальных по быстродействию, «Наука», 1966. 21. Первозванский А: А., Случайные процессы в нелинейных автоматических системах, Физматгиз, 1962. 22. Под ред. Петрова Б. Н., Автоматическая оптимизация управляемых систем, ИЛ, 1960. 23. Попов Е. П., Динамика систем автоматического регулирования, ГИТТЛ, 1954. 24. Попов Е. П., Автоматическое регулирование и управление, Физматгиз, 1962. 25. Попов Е. П., П а л ь т о в И. П., Приближенные методы исследования нелинейных автоматических систем, Физматгиз, I960.- 26. Пугачев В. С, Теория случайных функций и ее применение к задачам автоматического управления, Физматгиз, 1960. 27. Пупков К. А., Статистический расчет нелинейных систем автоматического управления, «Машиностроение», 1965. 28. Скворцов С. В., Синтез корректирующих устройств судовых следящих систем, «Судостроение», 1968. 29. Солодов А. В., Линейные системы автоматического управления с переменными параметрами, Физматгиз, 1962. 30. Солодовников В. В., Статистическая динамика линейных систем автоматического управления, Физматгиз, 1960. 31. Солодовников В. В., Топчеев Ю. И., Крутиков а Г. В., Частотный метод- построения переходных процессов: с приложением таблиц и номограмм, Гостехиздат, 1955; 32. Под ред. Солодов пиков а В. В., Техническая кибернетика, книги 1 и 2, Теория автоматического регулирования, «Машиностроение», 1967. 33. Т р а к с е л Дж. Дж., Синтез систем автоматического регулирования, Машгиз, 1959. 34. Под. ред. Траксела Дж. Дж., Справочная книга по технике автоматического регулирования, ГЭИ, 1962. 35. Т у Ю. Т., Цифровые и импульсные системы автоматического управления, «Машиностроение», 1964. 36. Фатеев А. В., Основы линейной теории автоматического регулирования, Госэнергоиздат, 1954. 37. Ф е л ь д б а у м А. А., Вычислительные устройства в автоматических системах, Физматгиз, 1959. 38. Федоров С. М., Литвинов А. П., Автоматические системы с цифровыми управляющими машинами,. «Энергия», 1965. 39. Хлыпало Е. И., Нелинейные системы автоматического регулирования, «Энергия», 1967. .40. Цыпкин Я. 3., Теория линейных импульсных систем, Физматгиз, 1S63. 41. Цянь Сюэ-сень, Техническая кибернетика, ИЛ, 1956. 42. Ч е с т н а т Г., М а й е р Р., Проектирование и расчет следящих систем и систем регулирования, части 1 и 2, ГЭЙ, 1959. _ 43. Шаталов А. С, Структурные методы в теории управ- - лени я и электроавтоматике, Госэнергоиздат, 1962.